You are on page 1of 503

300 Solved Problems in

Soil / Rock Mechanics


and
Foundations Engineering
These notes are provided to you by Professor Prieto-Portar, and in exchange, he will
be grateful for your comments on how to improve them.

All problems are graded according to difficulty as follows:


* Easy; define general principles; typical of the PE examination questions;
** Slightly more difficult; typical of Master’s level problems;
*** Professional level design problems; typical of real-life problems.

by
Luis A. Prieto-Portar PhD, PE
Professor of Civil and Environmental Engineering
Florida International University

Telephone 305-348-2825 / Fax 305-348-2802


Website: http://web.eng.fiu.edu/prieto / E-mail: prietol@fiu.edu

© Copyright by L. Prieto-Portar, March 2010

ii
Table of Contents
Table of Contents ............................................................................................................... iii
Chapter 1 Soil Exploration.................................................................................................. 1
Symbols for Soil Exploration .......................................................................................... 1
*Exploration–01. Find the required number of borings and their depth. ....................... 2
*Exploration–02. Propose the number of samples from boring data. ............................. 3
* Exploration-03. What is a pressure cell? ...................................................................... 4
* Exploration-04. In-situ instrumentation used in the field. ............................................ 5
*Exploration–05. Which sampling method disturbs the soil the least? ........................... 6
*Exploration–06. Correcting the SPT for sampling and depth. ...................................... 7
*Exploration–07. Three methods used for SPT depth corrections. ................................. 8
*Exploration–08. Applying SPT corrections to a mat foundation. ................................. 9
*Exploration–09: The RQD versus fractured percentage of a rock sample. ................. 11
*Exploration–10. The Shear Vane Test determines the in-situ cohesion. ..................... 12
*Exploration–11. Using the SPT to estimate engineering parameters. ......................... 13
*Exploration–12. Reading a soil boring log. ................................................................ 14
*Exploration –13. Read a soil boring. ........................................................................... 15
*Exploration–14: Using a boring log to estimate soil engineering parameters. ............ 16
*Exploration–15. Find the shear strength of a soil from the CPT. ............................... 18
Chapter 2 Phase Relations of Soil..................................................................................... 21
Symbols for Phase Relations of soils ............................................................................ 21
Basic Concepts and Formulas for the Phases of Soils. .................................................. 22
*Phases of soils-01: Convert from metric units to SI and US units. ............................. 26
*Phases of soils–02: Compaction checked via the voids ratio e. ................................ 27
*Phases of soils–03: Finding the moisture of a soil. ..................................................... 28
*Phases of soils–04: Finding the wrong data. ............................................................... 29
*Phases of soils–05: Dry unit weight and degree of saturation. .................................... 30
*Phases of soils–06: Increasing the saturation of a soil. ............................................... 31
**Phases of soils – 07: Comparing two samples from a boring log.............................. 32
*Phases of soils–08: How much water is added to a truck to offset losses? ................ 34
*Phases of soils–09: Find γd, n, S and Ww. .................................................................. 35
*Phases of soils–10: Continued previous Phases-09 problem, adding water. ............. 36
iii
*Phases of soils–11: Use the block diagram to find the degree of saturation. .............. 37
*Phases of soils–12: Same as Phases-11 but setting the total volume V=1 m3. ............ 38
*Phases of soils–13: Same as Phases-06 but using a block diagram............................. 39
*Phases of soils–14: Block diagram for a saturated soil. .............................................. 40
*Phases of soils–15: Find the weight of water needed for saturation. .......................... 41
*Phases of soils–16: Identify the wrong piece of data. ................................................. 42
*Phases of soils–17: The apparent cheapest soil is not! ................................................ 43
*Phases of soils–18: Number of truck loads for Phases-17........................................... 44
*Phases of soils–19: How many truck loads are needed for a project? ......................... 45
*Phases of soils–20: Choose the cheapest fill supplier. ................................................ 46
*Phases of soils–21: Use a matrix to the find the missing data. .................................... 48
**Phases of soils–22: Find the voids ratio of “muck” (a highly organic soil). ............. 49
Chapter 3 Classification of Soils and Rocks ..................................................................... 50
Symbols for Classification of soils ................................................................................ 50
*Classify–01: Percentage of each of the four grain sizes (G, S, M & C). ..................... 51
*Classify–02: Coefficients of uniformity and curvature of granular soils. ................... 52
*Classify-03: Classify two soils using the USCS.......................................................... 53
*Classify-04: Manufacturing a “new” soil. ................................................................... 54
Classify – 05 .................................................................................................................. 56
Classify – 06 .................................................................................................................. 57
Classify – 07 .................................................................................................................. 58
Classify – 08 .................................................................................................................. 59
Classify – 09 .................................................................................................................. 60
Classify – 10 .................................................................................................................. 61
Classify – 11 .................................................................................................................. 62
Chapter 4 Compaction and Soil Improvement.................................................................. 63
Symbols for Compaction ............................................................................................... 63
*Compaction–01: Find the optimum moisture content (OMC). ................................... 64
*Compaction–02: Find maximum dry unit weight in SI units. ..................................... 65
*Compaction-03: What is the saturation S at the OMC? .............................................. 66
*Compaction-04: Number of truck loads required. ....................................................... 67
*Compaction-05: What is the saturation S at the OMC? .............................................. 68
*Compaction-06: Definition of the relative compaction (RC). ..................................... 69

iv
*Compaction-07: The relative compaction (RC) of a soil. ........................................... 70
*Compaction-08: Converting volumes from borrow pits and truck loads. ................... 71
**Compaction-09: Ranges of water and fill required for a road. .................................. 72
**Compaction-10: Find the family of saturation curves for compaction. ..................... 74
**Compaction-11: Water needed to reach maximum density in the field. ................... 77
**Compaction-12: Fill volumes and truck load requirements for a levee. ................... 79
**Compaction-13: Multiple choice compaction problem of a levee. ........................... 81
Chapter 5 Permeability of Soils ........................................................................................ 84
Symbols for Permeability .............................................................................................. 84
*Permeability–01: Types of permeability tests and common units. .............................. 85
*Permeability-02: Use of Hazen’s formula to estimate the k of an aquifer. ................. 86
*Permeability-03: Flow in a sand layer from a canal to a river. ................................... 87
*Permeability-04: Find the equivalent horizontal permeability of two layers. ............. 88
*Permeability-05: Equivalent vertical and horizontal permeabilities. .......................... 89
*Permeability-06: Ratio of horizontal to vertical permeabilities. ................................. 90
*Permeability–07: Do not confuse a horizontal with a vertical permeability. .............. 91
*Permeability-08: Permeability as a function of the voids ratio e. ............................... 92
*Permeability–09: Uplift pressures from vertical flows................................................ 93
*Permeability-10: Capillary rise in tubes of differing diameters. ................................. 94
*Permeability-11: Rise of the water table due to capillarity saturation. ....................... 96
*Permeability-12: Find the capillary rise hc in a silt stratum using Hazen. .................. 97
*Permeability-13: Back-hoe trench test to estimate the field permeability................... 98
**Permeability-14: Seepage loss from an impounding pond. ....................................... 99
Chapter 6 Seepage and Flow-nets ................................................................................... 103
Symbols for Seepage and Flow-nets ........................................................................... 103
*Flownets-01: Correcting flawed flow-nets. ............................................................... 104
*Flow-nets-02: A flow-net beneath a dam with a partial cutoff wall. ......................... 105
*Flow-nets-03: The velocity of the flow at any point under a dam. ........................... 106
*Flow-nets-04: Flow through an earth levee. .............................................................. 107
*Flow-nets-05: Finding the total, static and dynamic heads in a dam. ....................... 108
**Flow nets-06: Hydraulic gradient profile within an earth levee. ............................. 109
**Flow-net-07: Flow into a cofferdam and pump size................................................ 111
*Flow-nets-08: Drainage of deep excavations for buildings. ...................................... 113

v
Anchor............................................................................................................................. 113
Limestone ........................................................................................................................ 113
*Flow-nets-09: Dewatering a construction site. .......................................................... 115
*Flow-net-10: Dewatering in layered strata. ............................................................... 116
**Flownets-11: Flow through the clay core of an earth dam. ..................................... 118
Chapter 7 Effective Stresses and Pore Water Pressure ................................................... 122
Symbols for Effective Stresses and Pore Water Pressure............................................ 122
*Effective Stress–01: The concept of buoyancy. ........................................................ 123
*Effective Stress–02: The concept of effective stress. ................................................ 124
*Effective Stress–03: The concept of effective stress with multiple strata. ................ 125
Effective Stress-03B .................................................................................................... 126
Chapter 8 Dams and Levees ........................................................................................... 127
Symbols for Dams and Levees .................................................................................... 127
*Dams-01: Find the uplift pressure under a small concrete levee. ............................. 128
*Dams-02: Determine the uplift forces acting upon a concrete dam. ......................... 129
Chapter 9 Stresses in Soil Masses ................................................................................... 132
Symbols for Stresses in Soil Masses ........................................................................... 132
*Stress-01: Reading the Boussinesq charts. ................................................................ 141
*Stress-02: 2:1 Method to find the Stress below a Rectangular Footing. ................... 142
*Stress-03: Summation of vertical and line loads using Boussinesq. ........................ 143
*Stress-04: The influence of point loads on a culvert. ................................................ 144
*Stress–05: Stress increase at a point from several surface point loads. ..................... 146
*Stress-06: Find the stress under a rectangular footing. .............................................. 147
*Stress-07: The effect of the WT on the stress below a rectangular footing. .............. 148
*Stress–08: Finding the stress outside the footing area. .............................................. 149
*Stress-09: Stress below a footing at different points. ............................................... 150
*Stress-10: Stress increase from a surcharge load of limited width. ........................... 151
*Stress-11: Finding a stress increase from a surface load of limited width. ............... 152
**Stress-12: Stress increase as a function of depth. .................................................... 153
*Newmark–01: Stress beneath a tank at different depths............................................ 154
*Newmark-02: The stress below the center of the edge of a footing. ......................... 155
*Newmark-03: Stress at a point distant from the loaded footing. ............................... 156
*Newmark-04: Stresses coming from complex shaped foundations........................... 157

vi
*Newmark-05: Stress beneath a circular oil tank. ...................................................... 158
**Newmark-06: Use Newmark with a settlement problem. ....................................... 159
*Mohr-01: Find the angle of internal friction φ of sand. ............................................. 161
*Mohr-02: Simple transformation from principal to general stress state. ................... 162
*Mohr–03: Find the principal stresses and their orientation. ...................................... 163
*Mohr–04: Find the principal stresses and their orientation. ...................................... 164
*Mohr-05: Find the angle of internal friction. ............................................................ 165
*Mohr – 06: Normal and shear stress at a chosen plane. ............................................ 166
*Mohr–07: Find the maximum and minimum stresses on a given plane. ................... 167
**Mohr – 08: Back figure the failure angle ................................................................ 168
*Mohr – 09: find the Principle pressure using Mohr................................................... 169
*Mohr –10: Relation between θ and φ. ....................................................................... 170
*Mohr – 11: Normal and shear stresses on any plane. ................................................ 171
*Mohr–12: Derive the transformation from general to principal stresses. .................. 172
*Mohr – 13: Tensile stress from the Mohr-Coulomb failure envelope. ...................... 173
**Mohr–14: Maximum principal stress from triaxial test. .......................................... 174
*Mohr – 15: Derive the general formula for horizontal stress. ................................... 175
Chapter 10 Elastic Settlements ....................................................................................... 176
Symbols for Elastic Settlements .................................................................................. 176
*Elastic Settlement-01: Settlement (rutting) of a truck tire. ........................................ 177
*Elastic Settlement-02: Schmertmann method used for granular soils. ...................... 178
*Elastic Settlement-03: Schmertmann method used for a deeper footings. ................ 179
*Elastic Settlement-04: The 2:1 method to calculate settlement. ................................ 181
*Elastic Settlement-05: Differential settlement between two columns. ...................... 183
*Elastic Settlement-06: Compare Boussinesq, Westergaard, and the 2:1 methods. ... 184
*Elastic Settlement-07: Schmertmann versus the strain methods. .............................. 187
*Elastic Settlement-08: The Schmertmann method in multiple strata. ....................... 188
**Elastic Settlement-09: Settlement of a mat foundation. .......................................... 190
Chapter 11 Plastic Settlements........................................................................................ 192
Symbols for Plastic Settlements .................................................................................. 192
*Plastic Settlement–01: Porewater pressure in a compressible soil. ........................... 193
*Plastic Settlement-02: Total settlement of a single layer. ......................................... 195
*Plastic Settlement-03: Boussinesq to reduce the stress with depth. .......................... 196

vii
*Plastic Settlement -04: Surface loads with different units. ........................................ 198
*Plastic Settlement-05: Pre-consolidation pressure pc and index Cc. .......................... 199
*Plastic Settlement-06: Final voids ratio after consolidation. ..................................... 201
*Plastic Settlement-07: Settlement due to a lowered WT. .......................................... 202
*Plastic Settlement-08: The over-consolidation ratio (OCR)...................................... 203
**Plastic Settlement-09: Coefficient of consolidation Cv. .......................................... 204
*Plastic Settlement -10: Secondary rate of consolidation. .......................................... 206
*Plastic Settlement-11: Using the Time factor Tv. ...................................................... 207
*Plastic Settlement-12: The time rate of consolidation. .............................................. 208
*Plastic Settlement-13: Time of consolidation t.......................................................... 209
*Plastic Settlement-14: Laboratory versus field time rates of settlement. .................. 210
*Plastic Settlement-15: Different degrees of consolidation. ....................................... 211
**Plastic Settlement-16: Excavate to reduce the settlement. ...................................... 212
**Plastic Settlement-17: Lead time required for consolidation of surcharge. ............ 214
**Plastic Settlement-18: Settlement of a canal levee. ................................................. 216
**Plastic Settlement-19: Differential settlements under a levee. ................................ 218
***Plastic Settlement-20: Estimate of the coefficient of consolidation cv. ................. 219
**Plastic Settlement-21: The apparent optimum moisture content. ............................ 221
**Plastic Settlement-22: The differential settlement between two buildings. ............ 222
**Plastic Settlement-23: Settlement of a bridge pier. ................................................. 227
Chapter 12 Shear Strength of Soils ................................................................................. 229
Symbols for Shear Strength of Soils............................................................................ 229
*Shear strength–01: Maximum shear on the failure plane. ......................................... 230
*Shear strength–02: Why is the maximum shear not the failure shear? ..................... 231
*Shear strength–03: Find the maximum principal stress σ1. ....................................... 232
*Shear strength–04: Find the effective principal stress. .............................................. 233
*Shear strength–05: Using the p-q diagram. ............................................................... 234
**Shear strength–06: Consolidated-drained triaxial test. ............................................ 235
**Shear strength–07: Triaxial un-drained tests. .......................................................... 237
**Shear strength-08: Consolidated and drained triaxial test. ...................................... 239
***Shear strength-09: Plots of the progressive failure in a shear-box. ....................... 241
**Shear strength-10: Shear strength along a potential failure plane. .......................... 244
***Shear strength-11: Use of the Mohr-Coulomb failure envelope. .......................... 245

viii
***Shear strength-11b: Use of the Mohr-Coulomb failure envelope. ........................ 247
**Shear strength-12: Triaxial un-drained tests. ........................................................... 249
**Shear strength-12b: Triaxial un-drained tests. ......................................................... 250
**Shear strength-13: Determine the principal stresses of a sample. ........................... 251
**Shear strength-13b: Determine the principal stresses of a sample. ......................... 254
**Shear strength-14: Formula to find the maximum principal stress. ........................ 257
Chapter 13 Slope Stability .............................................................................................. 259
Symbols for Slope Stability ......................................................................................... 259
*Slope-01: Factor of Safety of a straight line slope failure. ........................................ 260
*Slope-02: Same as Slope-01 but with a raising WT. ................................................. 261
*Slope-03: Is a river embankment safe with a large crane? ........................................ 262
*Slope-04: Simple method of slices to find the FS. .................................................... 263
**Slope-05: Method of slices to find the factor of safety of a slope with a WT. ........ 264
**Slope-06: Swedish slip circle solution of a slope stability. ..................................... 266
Chapter 14 Statistical Analysis of Soils .......................................................................... 269
Symbols for the Statistical Analysis of Soils............................................................... 269
Chapter 15 Lateral Pressures from Soils ......................................................................... 270
Symbols for Lateral Pressures from Soils ................................................................... 270
*Lateral-01: A simple wall subjected to an active pressure condition. ....................... 274
*Lateral–02: Compare the Rankine and Coulomb lateral coefficients....................... 275
*Lateral-03: Passive pressures using the Rankine theory. .......................................... 276
*Lateral-04: The “at-rest” pressure upon an unyielding wall...................................... 277
*Lateral-05: The contribution of cohesion to reduce the force on the wall. ............... 278
**Lateral-06: The effect of a rising WT upon a wall’s stability. ................................ 279
*Lateral-07: The effects of soil-wall friction upon the lateral pressure. ..................... 281
*Lateral-08: What happens when the lower stratum is stronger? ............................... 282
*Lateral-09: Strata with different parameters. ............................................................. 283
*Lateral-10: The effects of a clay stratum at the surface. .......................................... 285
**Lateral-11: Anchoring to help support a wall. ......................................................... 287
**Lateral-12: The effect of five strata have upon a wall. ........................................... 289
**Lateral-13: The stability of a reinforced concrete wall. ......................................... 291
***Lateral-14: Derive a formula that provides K and σH as a function of σv. ............ 294
**Lateral-15: The magnitude and location of a seismic load upon a retaining wall. .. 297

ix
**Lateral-16: Seismic loading upon a retaining wall. ................................................. 299
Chapter 16 Braced Cuts for Excavations ........................................................................ 300
Symbols for Braced Cuts for Excavations ................................................................... 300
*Braced-cuts-01: Forces and moments in the struts of a shored trench. ..................... 301
**Braced cuts-02: A 5 m deep excavation with two struts for support. ...................... 306
*Braced cuts-03: Four-struts bracing a 12 m excavation in a soft clay. ..................... 310
Chapter 17 Bearing Capacity of Soils ............................................................................. 313
Symbols for the Bearing Capacity of Soils ................................................................. 313
*Bearing–01: Terzaghi’s bearing capacity formula for a square footing. ................... 316
*Bearing–02: Meyerhof’s bearing capacity formula for a square footing. ................. 317
*Bearing–03: Hansen’s bearing capacity formula for a square footing. ..................... 318
*Bearing–04: Same as #01 but requiring conversion from metric units. .................... 319
*Bearing–05: General versus local bearing capacity failures. .................................... 320
*Bearing–06: Comparing the Hansen and Meyerhof bearing capacities. ................... 321
*Bearing–07: Increase a footing’s width if the WT is expected to rise. .................... 322
**Bearing–08: The effect of the WT upon the bearing capacity. ............................... 324
*Bearing–09: Finding the gross load capacity. .......................................................... 326
**Bearing–10: The effect of an eccentric load upon bearing capacity. ...................... 328
**Bearing–11: The effect of an inclined load upon the bearing capacity. .................. 329
**Bearing-12: Interpretation of borings to estimate a bearing capacity. .................... 331
Chapter 18 Shallow Foundations .................................................................................... 333
Symbols for Shallow Foundations ............................................................................... 333
*Footings–01: Analyze a simple square footing. ........................................................ 335
*Footings–02: Add a moment to the load on a footing. .............................................. 339
*Footings–03: Find the thickness T and the As of the previous problem. ................... 341
*Footings–04: Find the dimensions B x L of a rectangular footing. ........................... 346
*Footings–05: Design the steel for the previous problem. .......................................... 348
*Footings–06: Design a continuous footing for a pre-cast warehouse wall. .............. 352
**Footings–07: Design the footings of a large billboard sign..................................... 357
Chapter 19 Combined Footings ...................................................................................... 361
Symbols for Combined Footings ................................................................................. 361
**Combined Footing-01: Rectangular footing with an eccentric column. ................. 362
**Combined Footing-02: Combined footing on the property-line. ............................. 369

x
**Combined Footing-03: Design a combined footing. ............................................... 373
**Combined Footing-04: Optimization via a trapezoidal footing............................... 380
**Combined Footing-05: Combined footing. ............................................................. 386
**Combined Footing–06: Strap footing (or grade beam) between two columns........ 388
Chapter 20 Mat Foundations ........................................................................................... 395
Symbols for Mat Foundations ..................................................................................... 395
*Mat Foundations–01: Ultimate bearing capacity in a pure cohesive soil. ................. 396
*Mat Foundations–02: Ultimate bearing capacity in a granular soil........................... 397
*Mat Foundations–03: Find the depth Df for a fully compensated mat. ..................... 398
*Mat Foundations–04: The consolidation settlement of a mat foundation. ................ 399
*Mat Foundations–05: Settlement of a rigid mat. ....................................................... 400
*Mat Foundations–06: Mat foundation carries a large transfer girder. ....................... 401
**Mat Foundations–07: Design a small mat for an office building. ........................... 402
Chapter 21 Deep Foundations - Single Piles .................................................................. 408
Symbols for Single Piles of Deep Foundations ........................................................... 408
*Single-Pile–01: Pile capacity in a cohesive soil. ....................................................... 409
*Single-Pile–02: Pile capacity in granular soils. ......................................................... 410
*Single-Pile–02b: The friction of a concrete pile in granular soils. ........................... 411
*Single-Pile–03: The capacity of a driven concrete pile. ............................................ 412
*Single-Pile–04: The driving criterion for a concrete pile. ......................................... 413
*Single-Pile–05: The Meyerhof and Vesic methods for pile capacity. ....................... 414
*Single-Pile–06: Tension or pull-out capacity in granular soils. ................................ 416
*Single-Pile–07: Tension (uplift) capacity of a pile in granular soils. ....................... 417
***Single-Pile–08: The settlement of a drilled shaft in cohesive soils. ..................... 418
***Single-Pile–09: The performance of steel pipe piles. ............................................ 422
Chapter 22 Deep Foundations - Pile Groups and Caps................................................... 426
Symbols for Pile Groups and Caps of Deep Foundations ........................................... 426
**Pile-caps–01: Design a pile cap for a 9-pile cluster. ............................................... 427
**Pile cap-02: Pile group efficiency and lateral load capacity. .................................. 429
***Pile caps–03: Settlement of the combined pile-cap and pile cluster. .................... 431
Chapter 23 Deep Foundations: Lateral Loads ................................................................ 435
Symbols for Lateral Loads on Deep Foundations ....................................................... 435
**Lateral loads on piles-01: Find the lateral load capacity of a steel pile. .................. 436

xi
**Lateral loads on piles-02: Find the lateral load capacity of a steel pile. (US Units) 441
**Lateral loads on piles-03: The Broms method for pile lateral loads. ...................... 445
***Lateral loads on piles-03: Lateral load capacity of a mooring bollard. ................. 449
Chapter 24 Reinforced Concrete Retaining Walls and Bridge Abutments..................... 453
Symbols for Reinforced Concrete Retaining Walls .................................................... 453
**RC Retaining Walls–01: Design a RC wall for a sloped backfill. .......................... 454
**RC Retaining Walls–02: Design a RC bridge abutment wall. ............................... 462
**RC Retaining Walls–02: Design a RC bridge abutment wall. ................................ 464
Chapter 25 Steel Sheet Pile Retaining Walls .................................................................. 468
Symbols for Steel Sheet Pile Retaining Walls............................................................. 468
**Sheet-pile Wall-01: Free-Earth for cantilevered walls in granular soils. ................ 469
**Sheet-pile Wall-02: Free-Earth for cantilevered walls in cohesive soils................. 474
** Sheet-pile Wall–03: Free-Earth Method f/anchored walls in granular soils. ......... 477
** Sheet-pile Wall–04: Free-Earth Method anchored walls in cohesive soils. ........... 483
Chapter 26 MSE (Mechanically Stabilized Earth) Walls ............................................... 485
Symbols for Mechanically Stabilized Earth Walls ...................................................... 485
**MSE Walls-01: Design the length L of geotextiles for a 16 ft wall. ....................... 486

xii
Conversion of Units
Multiplication Factor Prefix SI Symbol

1 000 000 000 giga G

1 000 000 mega M

1 000 kilo k

0.001 milli m

0.000 001 micro µ

0.000 000 001 Nano n

Base SI Units

Quantity Unit Symbol


length meter m
mass kilograms (mass) kgm
force Newton N
time second s

Derived SI Units
Quantity Derived SI Unit Name Symbol
area square meter m²
volume cubic meter m³
density kilogram per cubic meter kgm/m³
force kilogram-meter per square second Newtons N
moment of force Newton-meter N-m
pressure Newton per square meter Pascal Pa
stress Newton per square meter Pascal Pa or N/m²
work, energy Newton-meter joule J
power joule per second watt W

xiii
Conversion of SI Units to English Units

Lengths
Multiply by To

From inches feet yards miles

mm 3.94 x 10-2 3.28 x 10-3 1.09 x 10-3 6.22 x 10-7

cm 3.94 x 10-1 3.28 x 10-2 1.09 x 10-2 6.22 x 10-6

m 3.94 x 101 3.28 1.09 6.22 x 10-4

km 3.94 x 104 3.28 x 103 1.09 x 103 6.22 x 10-1


1 µm = 1 x 10-6 m
1 Å = 1 x 10-10 m = 3.28 x 10-10 feet

Area
Multiply by To

From square inches square feet square yards square miles

mm² 1.55 x 10-3 1.08 x 10-5 1.20 x 10-6 3.86 x 10-13


cm² 1.55 x 10-1 1.08 x 10-3 1.20 x 10-4 3.86 x 10-11
m² 1.55 x 103 1.08 x 101 1.20 3.86 x 10-7

km² 1.55 x 109 1.08 x 107 1.20 x 106 3.86 x 10-1


1 acre = 43,450 ft2 = 4,047 m2 = 0.4047 hectares

Volume
Multiply by To

From cubic inches cubic feet cubic yards quarts gallons

cm3 6.10 x 10-2 3.53 x 10-5 1.31 x 10-6 1.06 x 10-3 2.64 x 10-4
liter 6.10 x 101 3.53 x 10-2 1.31 x 10-3 1.06 2.64 x 10-1
m³ 6.10 x 104 3.53 x 101 1.31 1.06 x 103 2.64 x 102

xiv
Conversion of SI Units to English Units

Force
Multiply by To
From ounces pounds kips tons (short)

dynes 1.405 x 10-7 2.248 x 10-6 2.248 x 10-9 1.124 x 10-9

grams 3.527 x 10-2 2.205 x 10-3 2.205 x 10-6 1.102 x 10-6


kilograms 3.527 x 101 2.205 2.205 x 10-3 1.102 x 10-3
Newtons 3.597 2.248 x 10-1 2.248 x 10-4 1.124 x 10-4
kilo-Newtons 3.597 x 103 2.248 x 102 2.248 x 10-1 1.124 x 10-1
tons (metric) 3.527 x 104 2.205 x 103 2.205 1.102

Pressure (or stress) σ


Multiply by To

From lb/in² lb/ft² kips/ft² tons (short)/ft² feet of water atmosphere

gm/cm² 1.422 x 10-2 2.048 2.048 x 10-3 1.024 x 10-3 3.281 x 10-2 9.678 x 10-4

kg/cm² 1.422 x 101 2.048 x 103 2.048 1.024 3.281 x 101 9.678 x 10-1

kN / m² 1.450 x 10-1 2.090 x 101 2.088 x 10-2 1.044 x 10-2 3.346 x 10-1 9.869 x 10-3

ton (metric)/m² 1.422 2.048 x 102 2.048 x 10-1 1.024 x 10-1 3.281 9.678 x 10-2

Torque (or moment) T or M


Multiply by To
From lb-in lb-ft kips-ft

gm-cm 8.677 x 10-4 7.233 x 10-5 7.233 x 10-8


kg-m 8.677 7.233 7.233 x 10-3
kN-m 9.195 x 103 7.663 x 102 7.663 x 10-1

xv
Conversion of SI Units to English Units

Velocity v
Multiply by To
From ft / s ft / min mi / h
cm / s 3.281 x 10-2 1.9685 2.236 x 10-2
km / min 5.467 x 101 3.281 x 103 3.728 x 101
km / h 9.116 x 10-1 5.467 x 101 6.214 x 10-1
1 mile = 1,610 meters = 5,282.152 feet

Unit weight γ
Multiply by To
From lb / in³ lb / ft³
gm / cm³ 3.613 x 10-2 6.248 x 101
kg / m³ 3.613 x 10-5 6.248 x 10-2
kN / m³ 3.685 x 10-3 6.368 x 100
tons (metric ) / m³ 3.613 x 10-2 6.428 x 101

Power P

1 W = 1 J/sec = 1.1622 cal/hr = 3.41 Btu/hr = 0.0013 hp


1 hp = 745.7 W = 0.7457 kW
1 kW = 1.34 hp

xvi
Chapter 1
Soil Exploration

Symbols for Soil Exploration

CB → STP correction factor for the boreholes diameter.


CR → STP correction factor for the rod length.
CS → STP correction factor for the sampler type used.
cu → Soil’s un-drained cohesion.
Df → Depth of the foundation’s invert.
Em → The efficiency of the STP hammer.
N → The “raw” value of the STP (as obtained in the field).
po → The original vertical stress at a point of interest in the soil mass.
S → The number of stories of a building.
SPT→ Stands for “Standard Penetration Test”.
N60 →Corrected STP assuming 60% efficiency in the field.
N70 →Corrected STP assuming 70% efficiency in the field.
m→ Correction factor for the shear vane test using the clay’s Plasticity Index PI.

1
*Exploration–01. Find the required number of borings and their depth.
(Revised: Feb-109)
A four-story reinforced concrete frame office building will be built on a site where the soils are
expected to be of average quality and uniformity. The building will have a 100 ft x 120 ft footprint
and is expected to be supported on spread footing foundations located about 3 ft below the ground
surface. The site appears to be in its natural condition, with no evidence of previous grading.
Bedrock is 100-ft below the ground surface. Determine the required number and depth of the
borings.
Solution:
A reinforced concrete building is heavier than a steel framed building of the same size. Hence, the
design engineer will want soil conditions that are at least average or better. Table-1 below suggests one
boring for every 3,000 ft2 of building footprint area. Since the total footprint area is 100 ft x 120 ft
=12,000 ft2, use four borings.

From Table-2 the minimum depth required for the borings, 15S0.7 + D = 15(4)0.7 + 3 = 43 ft.
Most engineers want one of the borings to go to a slightly greater depth to check the next lower
stratum’s strength.

Table-1 Spacing exploratory borings for medium to heavy buildings.

Structural footprint Area for Each Boring


Subsurface Conditions
(m2) (ft2)
Poor quality and / or erratic 200 2,000

Average 300 3,000

High quality and uniform 700 7,000

Table-2 Depths of exploratory borings for buildings on shallow foundations.


Minimum Depth of Borings
(S = number of stories and D = the anticipated
Subsurface Conditions
depth of the foundation)
(m) (ft)
Poor and / or erratic 6S0.7 + D 20S0.7 + D

Average 5S0.7 + D 15S0.7 + D

High quality and uniform 3S0.7 + D 10S0.7 + D

2
*Exploration–02. Propose the number of samples from boring data.
(Revised: Feb-10)
Use the three borings shown below taken at parking lot to suggest the minimum number of
samples required to design the foundations of a light two-story office building. Suggest the
approximate depth of these samples.
Solution:
The legend identifies four distinct layers. Also notice that the water table (WT) is around
elevations -7.0’ to -7.5’, assuming that the ground elevation is at +0.0 feet. Casting spread footings
will be cheaper above the WT. Notice that the top Layer #4 (limestone) seems to have adequate
bearing capacities for lightly loaded spread footings since its SPTs range from 15 to 43. If three
borings are taken, then three (3) samples are the minimum required, at about elevation – 4 feet,
because it will permit casting spread footings 3-feet thick without constructing them below the
WT.
Prudence suggests taking samples of Layers #2 and #3 too, below the limestone but these extra
samples are above the minimum required.

3
* Exploration-03. What is a pressure cell?
(Revision: Feb-10)
a) What device is used to measure the total pressure within a soil mass?
b) (a) Diaphragm and hydraulic earth pressure cells.
c) (b) Flat plate dilatometers.
d) (c) The Iowa borehole shear test.
e) (d) Pressuremeter.

Solution:
The correct answer is (a).
a) Earth pressure cells are devices that are permanently embedded in the soil mass to attempt to
measure the total stress at that point. A pressure cell may be installed in a fill or an embankment
dam to determine (1) the distribution, (2) the magnitude, and (3) the direction of the stress. Other
structures that commonly use pressure cells are retaining walls, culverts, piles, slurry walls and
beneath shallow foundations. There are two types: (1) the diaphragm cells are a stiff circular
membrane that deflects with an external soil pressure. Inside the cell is a vibrating wire transducer
that detects the deformation via a change in the vibration rate of the steel string. (2) The hydraulic
cell is filled with a fluid under high pressure that detects a minor change in the outside pressure.
b) flat plate dilatometer is a device inserted into the ground by a truck mounted rig, to measure the
soil’s modulus of elasticity and lateral stress at the point of testing. It is then pushed down another
6” to 8” for another measurement. The device expands its cell through a gas pumped from the
surface, and is also known as the Marchetti dilatometer.
c) The Iowa borehole shear test is a device inserted into a 3”-diameter borehole to a depth slightly
below the point of interest. Then its serrated cylinder halves are expanded in the hole with gas
from the surface. The serrated cylinder is pulled and its load-displacements are measured. These
results yield the in-situ shear strength, and indirectly φ and c.
d) A pressuremeter (PMT) is a device similar to the Iowa borehole shear test, and it too measures the
undrained in-situ shear strength su of a soil and its static shear modulus G’ at a specific location in
a borehole. It is also known in the literature as the Ménard test.

4
* Exploration-04. In-situ instrumentation used in the field.
(Revision: Feb-10)
What instruments would you use to measure the deformation of a soil in the field?
(a) Settlement plate and piezometer.
(b) Penetrometer and CPT.
(c) Inclinometer and probe extensometer.
(d) Load cells and strain gages.

Solution:
The correct answer is (c). A definition of each term is as follows:
a) A settlement plate is a thick steel plate placed close to the surface in order to load it and measure
the load-settlement of the soil at that point. It is used to predict the behavior of a shallow footing at
that point by obtaining its soil modulus (stress-strain relationship).
b) A piezometer (also known as a pore pressure cell) is a device that is inserted into the ground and
sealed within the ground to measure the groundwater pressure at a specific point only. It is
different from an observation well because the latter has no subsurface seals and it creates a
vertical connection between strata.
c) A penetrometer is a solid steel tip and rod that is either pushed into the ground at a constant rate,
or driven by blows (like an SPT) in order to (indirectly) measure the shear strength of the soil.
There are many types, such as the Dutch cone, the electric friction, electric-piezo/friction and the
seismic cones. The CPT simply stands for cone penetration test, and is widely used in lieu of the
SPT for soft clays and silts and in fine sands.
d) An inclinometer is a device that monitors the deformation of a soil orthogonally to the axis of the
inclinometer’s pipe. A probe inside the pipe contains a gravity-sensing transducer designed to
measure the inclination with respect to the vertical.
e) A probe extensometer is a devise for monitoring the deformation or changing distance between
two or more points within a soil mass along a pipe. Inside the pipe is a probe that measures the
deformation either mechanically or electrically.
f) A load cell, also known as a dynamometer, is a device placed into a structure to measure the
amount of load that passes through the cell. They are used in piles, drilled shafts, and earth/rock
anchors.
g) A strain gage is used when a load cell cannot be interposed in the structure due to geometrical
reasons, and measures the load indirectly via the strain, provided the stress-strain relationship is
known.

5
*Exploration–05. Which sampling method disturbs the soil the least?
(Revised: Feb-10)
Which soil sampling method disturbs the soil the least?
a) The Standard Penetration Test (SPT) with a spilt spoon sampler.
b) A Standard Penetrometer Test.
c) A Cone Penetrometer Test (CPT).
d) A Shelby tube.

Solution:
The answer is (d) A Shelby tube (also known as a “thin tube sampler”).

Answer (a) is wrong. The most common soil and soft rock sampling tool is the Standard Split
Spoon, that splits longitudinally into halves and permit taking a soil or soft rock sample. The
tube’s outside diameter is Do = 50.8 mm (2 inches) and its inside diameter is Di = 34.9 mm (1 3/8
inches). This small size has the advantage of being relatively easy to drive into the ground.
However, it has the disadvantage of disturbing the natural texture of the soil. In soft rocks, such as
young limestone, it will destroy the rock to such a degree that the rock is often classified as “sand”.
A better sampler is the Shelby (or thin-tube sampler). It has the same outside diameter of 2 inches
(although the trend it to use the 3 inch sampler). Compare the degree of sample disturbance of a
Standard split-spoon sampler versus the two popular Shelby thin-tube samplers (2” and 3”
outside diameters) via their area ratio Ar which is a measure of sample’s disturbance.

The area ratio for a 2"-Standard split-spoon sampler is,

( 2.0 ) − (1.38) (100) = 110%


2 2
D2 − D2
Ar (%) = o 2 i (100) =
(1.38)
2
Di
The area ratio for a 2"-Shelby-tube sampler is,

( 2.0 ) − (1.875) (100) = 13.8%


2 2
D2 − D2
Ar (%) = o 2 i (100) =
(1.875)
2
Di
The area ratio for a 3"-Shelby-tube sampler is,

( 3.0 ) − ( 2.875) (100) = 8.9%


2 2
Do2 − Di2
Ar (%) = (100) =
( 2.875)
2
Di2

Clearly, the 3” O-D Shelby-tube sampler is the best tool to use.

Answers (b) and (c) are also wrong because penetrometers do not obtain soil samples.

6
*Exploration–06. Correcting the SPT for sampling and depth.
(Revision Sept-09)
A standard penetration test (SPT) has been conducted in a loose coarse sand stratum to a depth of
16 feet below the ground surface. The blow counts obtained in the field were as follows: 0 – 6 in =
4 blows; 6 -12 in = 6 blows; 12 -18 in = 8 blows. The tests were conducted using a US-style donut
hammer in a 6 inch diameter boring with a standard sampler and liner. The effective unit weight
of the loose sand stratum is about 93.8 pcf.
Determine the corrected SPT if the testing procedure is assumed to only be 60% efficient.
Solution: The raw SPT value is N = 6 + 8 = 14 (that is, only the last two sets of 6” penetrations).
The US-style donut hammer efficiency is Em = 0.45, and the other parameters are obtained from the
Table provided on the next page: CB = 1.05, CS = 1.00, CR = 0.85. With these values, the SPT corrected
to 60% efficiency can use Skempton’s relation,
E m C B C S C R N ( 0.45 )(1.05 )(1.00 )( 0.85 )(14 )
'
N 60 = = =9
0.60 0.60
Notice that the SPT value is always given as a whole number. That corrected SPT N’60 is then corrected
for depth. For example, using the Liao and Whitman method (1986),
2, 000 lb / ft 2 2, 000 lb / ft 2
N corrected − 60 = N 60
'
= (9 ) = 10
( depth )( effective unit weight ) (16 ft )( 93.8 pcf )

SPT Hammer Efficiencies (adapted from Clayton, 1990)


US Safety Hammers 2-turns on cat-head 0.60
US Donut Hammers 2-turns on cat-head 0.45
Correction Factors for the Boring Diameter, Sampling Method and Boring Rod Length
Correction Factor Equipment Variables Value
Borehole diameter factor CB 65 – 115 mm (2.5 – 4.5 in) 1.00
150 mm (6 in) 1.05
200 mm (8 in) 1.15
Sampling method factor CS Standard sampler 1.00
Sampler without liner
1.20
(not recommended)
Rod length factor, CR 3 – 4 m (10 – 13 ft) 0.75
4 – 6 m (13 – 20 ft) 0.85
6 – 10 (20 – 30 ft) 0.95
>10 m (>30 ft) 1.00

7
*Exploration–07. Three methods used for SPT depth corrections.
(Revision: Feb-10)
An N’60 = 40 was corrected for sampling errors at a depth of 20 feet in a sand stratum that has a
unit weight of 135 lb/ft3. Correct it for depth.

Solution:
Any of these three methods will provide acceptable answers. Notice how similar their results are from
each other:
1. Using the Bazaraa Method (1967):
4N '
N corrected = if p0 ≤ 1.5 kips / ft 2 and
1 + 2 po
4N '
N corrected = if p0 ≥ 1.5 kips / ft 2
3.25 + 0.5 po
(20 ft )(135 lb / ft 3 )
but p0 = = 2.70 kips / ft 2 > 1.5kips / ft 2
1000 lb / kip
4N ' 4(40)
therefore N corrected = = = 35
3.25 + 0.5 po 3.25 + 0.5(2.70 kips / ft 2 )
2. Using the Peck Method (1974):
20
N corrected = N 'CN where CN = 0.77 log10 if p0 is in tons / ft 2
p0
1915
or C N = 0.77 log10 if p0 is in kN / m 2
p0
(20 ft )(135 lb / ft 3 )
but p0 = = 1.35 tons / ft 2 = 2.70 kips / ft 2
2000 lb / ton
20
∴ CN = 0.77 log10 = 0.90 ∴ N corrected = (40)(0.90) = 36
1.35 tons / ft 2
3. The Liao-Whitman Method (1986), as used in Exploration-05,
100 2, 000 psf
N corrected = N ' with po in kN / m 2 or = N' with po in psf
po po
 96.1 kN / m 2 
but po = (1.35 tons / ft 2 )  2  = 129.7 kN / m
2

 1 ton / ft 
100 kN / m 2
∴ N corrected = 40 = 35
129.7 kN / m 2

8
*Exploration–08. Applying SPT corrections to a mat foundation.
(Revision: Feb-10)

Correct the SPT values shown below for a US-type safety hammer in a 2”-diameter boring.

9
Solution:
Skempton proposed in 1986 the following correction for the sampling methods to the raw SPT value,
assuming that only 60% of the energy of the hammer drives the sampler,
Em C B C S C R N
N 60 =
0.60
See the sampling corrections Table on page 15. The depth correction is,

2, 000 lb / ft 2
( N 1 )60 = N 60
( depth )( effective unit weight )
At depth of +5.2 feet:
2
(0.6)(1)(1)(0.75)(26) 2, 000 lb / ft
N 60 = = 20 and ( N )60 = ( 20 ) = 39
0.60 (8 ft )(127 − 62.4 pcf )
(0.6)(1)(1)(0.75)(25) 2, 000 lb / ft 2
At +4.1’ N 60 = = 19 and ( N )60 = (19 ) = 35
0.60 ( 9 ft )(127 − 62.4 pcf )
(0.6)(1)(1)(0.75)(24) 2, 000 lb / ft 2
At+2.0’ N 60 = = 18 and ( N )60 = (18 ) = 31
0.60 (11 ft )(125 − 62.4 pcf )
(0.6)(1)(1)(0.85)(30) 2, 000 lb / ft 2
At -1.0’ N 60 = = 26 and ( N )60 = ( 26 ) = 39
0.60 (14 ft )(126 − 62.4 pcf )
(0.6)(1)(1)(0.85)(31) 2, 000 lb / ft 2
At -5.0’ N 60 = = 26 and ( N )60 = ( 26 ) = 34
0.60 (18 ft )(126 − 62.4 pcf )
(0.6)(1)(1)(0.95)(30) 2, 000 lb / ft 2
At -10’ N 60 = = 29 and ( N )60 = ( 29 ) = 34
0.60 ( 23 ft )(126 − 62.4 pcf )
(0.6)(1)(1)(1)(43) 2, 000 lb / ft 2
At -21’ N 60 = = 43 and ( N )60 = ( 43) = 41
0.60 ( 33 ft )(130 − 62.4 pcf )
Notice that the effect of the depth correction decreases with depth.

10
*Exploration–09: The RQD versus fractured percentage of a rock sample.
(Revision: Feb-10)
What is the rock description of a sample that has a 40% fractured percentage?

Solution:
The SPTs of rock samples usually provide values of N > 100. The exception is in soft or partially
decomposed sandstone and limestone (grainstones). In these types of rock, it is necessary to use the
“rock quality designation” (RQD) as a measure of the quality of the rock. The rock quality is based on
the percentage of the sample recovered (what field crews call the “fractured percentage”). The RQD is
defined as,

RQD =
∑ ( length of recovered pieces equal or larger than 4" long )
Theoretical length of the rock core
A RQD = 1 would indicate it was an excellent quality intact rock mass, or in essence, equivalent to a
100%. The table below shows all the RQD quality descriptions.
In-Situ Rock Quality Designation (based on Deere, 1963)
RQD Description Unconf comp. strength qu-field / qlab
0 – 0.25 very poor 0.15
0.25 – 0.50 poor 0.20
0.50 – 0.75 fair 0.25
0.75 – 0.90 good 0.3 – 0.7
0.90 – 1.00 excellent 0.7 – 1.0

Based on this table, a RQD = 0.4 is considered a poor quality rock.

11
*Exploration–10. The Shear Vane Test determines the in-situ cohesion.
(Revision Sept.-08)
A shear vane tester is used to determine an approximate value of the shear strength of clay. The
tester has a blade diameter d = 3.625 inches and a blade height h = 7.25 inches. In a field test, the
vane required a torque of 17.0 ft-lb to shear the clay sample, which has a plasticity index of 47%
(PI = LL – PL). Determine the un-drained cohesion cu corrected for its plasticity.

47

T 17.0 ft ⋅ lb
cu = = = 168 psf
π (d h / 2) + (d / 6) 
2 3
 (0.3021 ft ) (0.6042 ft ) (0.3021 ft )3 
2
π + 
 2 6 
The plasticity index helps correct the raw shear vane test value (Bjerrum, 1974) through the graph
shown above. For a plasticity index of 47% read a correction factor µ = 0.80. Therefore,
cu − corrected = µ cu = (0.80)(168 psf ) = 134 psf

12
*Exploration–11. Using the SPT to estimate engineering parameters.
(Revision: Feb-10)
Estimate the angle of internal friction and the unit weight of a medium dense sand located at a depth
of 20 feet below the surface, where the SPT N’60 = 9?

Solution.
The corrected SPT N 60' = 9 is based on 60% efficiency. However, the Table provided
is based on 70% efficiency. Therefore, convert via the following relation,
( 0.60 ) = 9 ( 0.60 ) = 8
N 70' = N 60' ( ) (remember that the SPT values are whole integers)
( 0.70 ) ( 0.70 )
Correlation between SPT values and some Granular Soils Engineering Parameters
Description Very loose Loose Medium Dense Very dense

Dr Relative density 0 0.15 0.35 0.65 0.85

SPT Very loose - loose 1-2 3-6 7 - 15 16 - 30


N'70 Medium 2-3 4-7 8 - 20 21 - 40 > 40
Dense –Very Dense 3-6 5-9 10 - 25 26 - 45

φ Very loose - loose 26 - 28 28 - 30 30 - 34 33 - 38


Medium 27 - 28 30 - 32 32 - 36 36 - 42 < 50
Dense –Very Dense 28 - 30 30 - 34 33 - 40 40 - 50

γwet pcf 70 - 102 89 - 115 108 - 128 108 -140 128 -147
kN/m3 11 - 16 14 - 18 17 - 20 17 - 22 20 - 23
Note #1: These values are based on tests conducted at depths of about 6 m.
Note #2: Typical values of relative densities are about 0.3 to 0.7. Values of 0 or 1.0 do not exist in nature.
Note #3: The value of the angle of internal friction is based on Φ = 28º + 15ºDr.
Note #4: The typical value of an excavated soil ranges from 11 to 14 kN/m3.

Notice that an SPT N’70 = 8 for a medium dense sand permits estimating the angle of internal
friction φ = 32º and a moist unit weight of 108 pcf.

13
*Exploration–12. Reading a soil boring log.
(Revision Sept.-08)
Read the boring log shown below and determine, (1) the location of the phreatic surface, (2) the
depth of the boring and (3) the number of samples taken.

Solution:
(1) The phreatic surface (the water table) was not encountered in this boring and is noted at the bottom
of the report;
(2) The boring was terminated at 21 feet in depth; and
(3) Five samples were taken. Only one sample (#2) was used for laboratory tests (dry density and
moisture content). Samples #1 and #3 were complete split-spoon samples. Samples #4 and #5 were
incomplete split-spoon samples.

14
*Exploration –13. Read a soil boring.
(Revision: Aug.-09)
Read the boring shown below, and determine: (1) at what depth is the phreatic surface? (2) At
what depth was the boring terminated? (3) What type of sampler was used? (4) What is the lowest
SPT in the limestone?

(1) The depth of the phreatic surface (WT) is at -12 feet;


(1) The depth of the phreatic surface (WT) is at -12 feet; (2) The boring was terminated at a depth
of 15 feet below the surface; (3) Split spoon sampler; (4) N = 20.

15
*Exploration–14: Using a boring log to estimate soil engineering parameters.
(Revision Aug.-09)
Using the boring log and the SPT versus Soil Engineering Parameters Table to answer these four
questions:
(1) Correct the values of the SPT of Sample S-4 to a 70% sampling efficiency with a standard
sampling method and a US-donut hammer at elevation – 17 feet;
(2) Correct the same sample S-4 for depth assuming the unit weight is γ = 126 pcf;
(3) What are your estimates for the angle of internal friction and unit weight γ?
(4) What is the elevation (above sea level) of the groundwater and the elevation of the bottom of
the boring?

Solution:

(1) The log shows a value of N = 15 (Sample S-4) at elevation -16.5’; at elevation -17’ it has
dropped a small amount to N = 14. Notice that the “Legend” portion denotes that the sampler was
a 2” O.D. split spoon. Therefore, the sampling correction is,
EC B C S C R N ( 0.45 )(1.0 )(1.0 )( 0.85 )(14 )
N 70 = = ≈8
0.70 0.70
(2) Correct the same sample S-4 for depth.
2000 psf 2000 psf
( N 70 ) = N 70 = (8 ) ≈8
(γ h ) (126 )(17 ) psf

(3) What are your estimates for the angle of internal friction and unit weight γ?

The log identifies this level at -17’ as a “brown and grey fine to medium SAND”. Use the Table
provided in Exploration-10 to obtain an estimate of some of the engineering parameters for
granular soils. Read the SPT for medium sands; then go to the Medium column and read the value
of “N = 8” to obtain the values:

φ = 32º and γwet = 17 kN/m2.

(4) What are the elevations (above sea level) of the groundwater and of the bottom of the boring?

- The boring did not report finding a ground water table.

- The bottom of the boring was at -36.5’ from the surface, or 347.0’ – 36.5’ = +310.5’.

16
17
*Exploration–15. Find the shear strength of a soil from the CPT.
(Revision: Aug.-09)
Classify the soil at depth 11 m below the surface using the cone penetration test (CPT) data shown
below. The clay samples separately recovered at that depth had γ = 20 kN/m3 and Ip= 20. Estimate the
shear strength su, from the graphs, and compare to laboratory tests on the samples that showed shear
strength of 550 kPa.

Solution.

The CPT is a superior field test to determine soil engineering parameters because it provides continuous
readings, that discriminate between point resistance versus shaft friction. Its major disadvantage is that
samples are not obtained.

From the data, q s = 400 kPa and q c = 11 MPa which gives a fR ~ 3%

18
From the next chart, the soil appears to be silty clay.

19
At a depth of 11 m, po = γ z = (20 kN / m3 )(11 m) = 220 kPa
and from the N k versus I p graph, for I p = 20 yields an N k ~ 17.5.
The un-drained shear strength su is,
qc − po 11, 000 kPa − 220 kPa
su = = = 616 kPa
Nk 17.5
Compare with the laboratory tested value of 550 kPa (a 12% error).

20
Chapter 2
Phase Relations of Soil
Symbols for Phase Relations of soils

e→ Voids ratio.
GS → Specific gravity of the solids of a soil.
n→ Porosity.
S→ Degree of saturation.
V→ Total volume (solids + water + air).
Va → Volume of air.
VV → Volume of voids (water + air).
VS → Volume of solids.
VW → Volume of water.
w→ Water content (also known as the moisture content).
WS → Weight of solids.
WW → Weight of water.
g→ Unit weight of the soil.
gd → Dry unit weight of the soil.
gb → Buoyant unit weight of the soil (same as g’).
gSAT→ Unit weight of a saturated soil.
gW → Unit weight of water

21
Basic Concepts and Formulas for the Phases of Soils.

(A) Volumetric Relationships:

1. - Voids ratio e
VV
e= 2-1
VS

ranges from 0 to infinity.


Typical values of sands are: very dense 0.4 to very loose 1.0
Typical values for clays are: firm 0.3 to very soft 1.5.

2. - Porosity n
VV
n= (100% ) 2.2
V
ranges from 0% to 100%.

The porosity provides a measure of the permeability of a soil.

The interrelationship of the voids ratio and porosity are given by,
22
n e
e= and n= 2-3
1− n 1+ e

3. - Saturation S
VW
S= x100% 2-4
VV

ranges from 0% to 100%.

(B) Weight Relationships:

4. - Water content w
WW
w= x100% 2-5
WS

Values range from 0% to over 500%; also known as moisture content.

5. – Unit weight of a soil γ


W WS + WW
γ= = 2-6
V VS + VW + VA

The unit weight may range from being dry to being saturated.

Some engineers use “bulk density ρ” to refer to the ratio of mass of the solids and water
contained in a unit volume (in Mg/m3). Note that,

W m
γ= = ρg = g which is the equivalent of F = ma. 2-6
V V

6. - Dry unit weight γd


WS γ
γd = = 2-7
V 1+ w
The soil is perfectly dry (its moisture is zero).

7. - The unit weight of water γw


WW
γw = where γ = ρ g ( F = ma )
VW
γ w = 62.4 pcf = 1 g / ml = 1 kg / liter = 9.81 kN / m3

23
Note that the above is for fresh water. Salt water is 64 pcf, etc.

8. - Saturated unit weight of a soil γsat


WS + WW
γ SAT = 2-8
VS + VW + 0

9. - Buoyant unit weight of a soil γb


γ b = γ ' = γ SAT − γ w 2-9

10. - Specific gravity of the solids of a soil G


γS
GS = 2-10
γw
Typical Values for the Specific Gravity of Minerals in Soils and Rocks
Mineral Composition Absolute specific gravity Gs
Anhydrite CaSO4 2.90
Barites BaSO4 4.50
Calcite, chalk CaCO3 2.71
Feldspar KALSi3O8 2.60 to 2.70
Gypsum CaSO4 2H2O 2.30
Hematite Fe2O3 5.20
Kaolinite Al4Si4O10(OH)8 2.60
Magnetite Fe3O4 5.20
Lead Pb 11.34
Quartz (silica) SiO2 2.65
Peat Organic 1.0 or less
Diatomaceous earth Skeletons of plants 2.00

24
Other useful formulas dealing with phase relationships:
Se = wGS
γs
e= −1
γ dry
Unit weight relationships :
(1 + w)GS γ w (GS + Se)γ w (1 + w)GS γ w
γ= = = = GS γ w (1 − n)(1 + w)
1+ e 1+ e 1+
wGS
S
Saturated unit weights :
(GS + e)γ w  e  1 + w 
γ SAT = =   γ w
1+ e  w  1 + e 
 1+ w 
γ SAT = γ d + nγ w = (1 − n ) Gs + n  γ w =   Gsγ w
 1 + wGs 
γ SAT = γ '+ γ w

Dry unit weights :


γ GS γ w eS γ w eGsγ w
γd = = Gsγ w (1 − n ) = = =
1+ w 1 + e (1 + e) w ( S + wGs )
 e 
γ d = γ SAT − nγ w = γ SAT −  γ w
 1+ e 
.

25
*Phases of soils-01: Convert from metric units to SI and US units.
(Revision: Feb-10)
A cohesive soil sample was taken from an SPT and returned to the laboratory in a glass jar. It had
a mass of 140.5 grams. The sample was then placed in a container of V = 500 cm3 and 423 cm3 of
water were added to fill the container. From these data, what was the unit weight of the soil in
kN/m3 and pcf?

Solution.
Notice that the 140.5 grams is identified as a mass. The ratio of mass to volume is a density ρ,
m 140.5 g g
ρ= = = 1.82
V (500 − 423) cm 3
cm 3
3
   1 kg  m   1 kN   10 cm 
2
g kN
γ = ρ g =  1.82 3  3   9.806    = 17.9 3 ( SI units )
 cm   10 g  sec 2   10 3 N  1 m  m
3
 kN   1000 N   0.2248 lbs f   1 m 
γ =  17.9 3      = 114 pcf (U S units )
 m   1 kN   1N  3.28 08 f t 

26
*Phases of soils–02: Compaction checked via the voids ratio e.
(Revision: Feb-10)
A contractor has compacted the base course for a new road and found that the mean value of the
test samples shows w = 14.6%, GS = 2.81, and γ = 115.8 lb/ft3. The specifications require that e ≤
0.80.
Has the contractor complied with the specifications?

(a) Yes, the contractor has satisfied with more than required.
(b) Yes, barely satisfied.
(c) No, the contractor has not attained the required compaction.
(d) Not enough data.

Solution:
GS γ W (1 + w ) G γ (1 + w )
γ = ∴ 1+ e = S W
1+ e γ
 lb 
2.81  62.4 f3  (1 + 0.146 )
 ft 
1+ e = = 1.74
lb f
115.8 3
ft
e = 1.74 − 1 = 0.74

∴ e = 0.74 < 0.80 Yes, the contractor has complied; answer (a).

27
*Phases of soils–03: Finding the moisture of a soil.
(Revision: Feb-10)
A container has a sample of moist soil with a mass of 23.42 g. The soil is dried in an oven with its
original container. After drying, it is reweighed, and its mass is 19.81 g. If the container has a mass
of 1.73 g, what is the soil’s moisture content?

a) 18%

b) 20%

c) 30%

d) 85%

Solution:
mmoist = 23.42 g − 1.73 g = 21.69 g
mdry = 19.81g − 1.73 g = 18.08 g
∴ mwater = 3.61g
Ww mw 3.61g
w= = = = 19.96% ≈ 20% which is (b)
Ws mS 18.08 g

28
*Phases of soils–04: Finding the wrong data.
(Revision: Oct.-08)
A geotechnical laboratory reported these results of five samples taken from a single boring.
Determine which are not correctly reported, if any.
Sample #1: w = 30%, γd = 14.9 kN/m3, γs = 27 kN/m3; clay.
Sample #2: w = 20%, γd = 18 kN/m3, γs = 27 kN/m3; silt.
Sample #3: w = 10%, γd = 16 kN/m3, γs = 26 kN/m3; sand.
Sample #4: w = 22%, γd = 17.3 kN/m3, γs = 28 kN/m3; silt.
Sample #5: w = 22%, γd = 18 kN/m3, γs = 27 kN/m3; silt.

Solution:
e VV γ w VV γ w VS γ wVV V   V + VS − VS 
wsat = = = = = γw  V  = γw  V 
GS VS γ S VS 1 WS WS  WS   WS 
 V + VS VS   1 1 
wsat = γ w  V −  = γw  − 
 WS WS  γd γS 
The water content is in error if it is greater than the saturated moisture, that is,
 1 1 
∴ w ≤ wSAT = γ w  − 
γd γS 
 1 1 
1) wSAT = ( 9.81 kN / m3 )  −  = 30% = w = 30% GOOD
 14.9 27 
1 1 
2) wSAT = ( 9.81 kN / m3 )  −  = 18.5% v < w = 20% WRONG
 18 27 
 1 1 
3) wSAT = ( 9.81 kN / m3 )  −  = 24% > w = 10% GOOD
 16 26 
 1 1 
4) wSAT = ( 9.81 kN / m3 )  −  = 22.1% > w = 22% GOOD
 17.3 28 
1 1 
5) wSAT = ( 9.81 kN / m3 )  −  = 18.5% < w = 22% WRONG
 18 27 

29
*Phases of soils–05: Dry unit weight and degree of saturation.
(Revision: Aug.-09)
During compaction, the engineer needs to know the relationship between the saturation of the soil
and the ensuing dry density. As a rule, the maximum dry density is in the range of 70% to 85%
saturation. Express the theoretical dry unit weight as a function of the saturation S.

Solution:
The two basic equations of phase relationships are,

γd = S W (1) and
1+ e
Se = wGS ( 2)
Solving for e in equation (2), and substituting that value into (1), yields
Gγ SGS γ W
γd = S W or γ d =
1+
wGS S + wGS
S

30
*Phases of soils–06: Increasing the saturation of a soil.
(Revision: Aug.-09)
A soil sample has a unit weight of 105.7 pcf and a saturation of 50%. When its saturation is
increased to 75%, its unit weight raises to 112.7 pcf.
Determine the voids ratio e and the specific gravity Gs of this soil.

Solution:
γ W ( GS + Se )
γ =
1+ e
62.4(GS + 0.50e)
∴ 105.7 pcf = (1)
1+ e
62.4(GS + 0.75e)
and 112.7 pcf = (2)
1+ e

Solving exp licitely for Gs in equation (1),

Gs =
(105.7 )(1 + e ) − 0.50e
62.4

Replace Gs in equation (2) with the above relation from (1),


∴ (112.7 )(1 + e ) = (105.7 )(1 + e ) + ( 62.4 )( 0.25e )
∴ e = 0.814 and GS = 2.67

31
**Phases of soils – 07: Comparing two samples from a boring log.
(Revision Feb-10)
Two clay samples were taken from a boring to a laboratory and yielded the following:
Sample #1 Sample #2
Weight of the sample 0.48 N 0.68 N
-5 3
Volume 3 x 10 m 4.3 x 10-5 m3
Dry weight (at 105ºC) 0.30 N 0.40 N

Find: a) Their unit weights γ and moisture contents w;

b) Their voids ratios e if the unit weight of solids


in the lab was 27 kN/m3;

c) Their degrees of saturation S;

d) Their relative variation of volumes ∆V / V,


if before each sample was taken to the lab their
unit weight of solids in the field was
actually 27.5 kN/m3.

Solution.
a) Their unit weights and moisture contents:
W 0.48 N Ww W − WS ( 0.48 − 0.3) N
γ1 = 1 = = 16 kN / m 3
and w = = = (100% ) = 60%
V1 3 (10−5 ) m3
1
WS WS 0.3 N
W2 0.68 N W W − WS ( 0.68 − 0.4 ) N
γ2 = = = 15.8 kN / m3 and w2 = w = = (100% ) = 70%
V2 4.3 (10 ) m
−5 3
WS WS 0.4 N

b) Their voids ratios if the unit weight of solids in the lab was 27 kN/m3:

32
WS1 0.3 N γS 27 kN / m3
γd = = = 10, 000 N / m 3
= 10 kN / m 3
∴ e = − 1 = − 1 = 1.7
3 (10 −5 ) m3 γd
1
1
V1 10kN / m3
WS2 0.4 N γS 27 kN / m3
γd = = = 9,30 0 N / m 3
= 9.3 kN / m 3
∴ e = − 1 = − 1 = 1.9
4.3 (10−5 ) m3 γd
2
2
V2 9.3kN / m3

c) Their degrees of saturation:


w G ( 0.60 )( γ S / γ w ) ( 0.60 )( 27 / 9.81)
S1 = 1 s = = = 97%
e1 (1.7 ) (1.7 )
w2Gs ( 0.70 )( γ S / γ w ) ( 0.70 )( 27 / 9.81)
S2 = = = = 100%
e2 (1.9 ) (1.9 )
d) Relative variation of the volume:
 γ s −lab   27 kN / m3 
3 (
9.81kN / m3 ) (1 + 0.6 ) = 15.41kN / m3 =
W
γ 1− field = γ d (1 + w) = 
γ  ( γ w )(1 + w ) = 
 s − field   27.5kN / m  V1− field

∆V1 V1− field − V1 ( 3.334 − 3) (10 )


−5

= 3.114 (10 ) m and


W 0.48 N
∴ V1− field = = −5 3
= = (100% ) = 3.8%
γ 1− field 15.41kN / m3 V1 V1 3 (10−5 )
 γ s −lab   27kN / m3 
3 (
9.81kN / m3 ) (1 + 0.7 ) = 16.37 kN / m3 =
W
γ 2− field = γ d (1 + w) = 
γ (
 wγ )(1 + w ) = 
 s − field   27.5 kN / m  V2 − field

∆V2 V2− field − V2 ( 4.334 − 4.3) (10 )


−5

= 4.153 (10−5 ) m3 and


W 0.68 N
∴ V2− field = = = = (100% ) = 3.4%
γ 2− field 4.3 (10−5 )
3
16.37kN / m V2 V2

Notes: The relative variation of the volume of each sample before taking to the lab if its unit weight of solids in the
field was 27.5 kN/m3.
(γsolid)field = (γs)f = 27.5 kN/m3
[(γs)f] / γs = 27.5/27 = 0.981818181=> therefore a 1.82% loss of water
The 1.82% loss is from water loss, either through evaporation or leakage. Since the water content w is 60% and 70%
for samples #1 and #2 respectively, there is no net effect on the volume of the samples taken. The dry weight of solid is
independent of the amount of water since by definition it is the weight of the solid without any water and the volume
of solid is therefore unaffected. The volume of water however does change, but since the water content or moisture
content is 60% and 70% which falls within 0% < w < 500%, the relative total volume of the field sample should be the
same as the total volume of the lab sample – noting of course the weight differences due to the 1.82% loss of water.
The unit weights of the samples would need to be adjusted because of the total weight variation – in this case the lab
values of 0.48 N and the 0.68 N is only a percentage of the field values (slightly less than 100%.) A simple correction
of this data would yield closer to true results.

33
*Phases of soils–08: How much water is added to a truck to offset losses?
(Revision: Feb-10)
The weight of an empty truck is 3,000 lbs. It is filled with 1,000 lbs of soil. The original moisture of
the soil was 12%, but during transportation the moisture dropped to 11.5%.
1) How much water (in gallons) did the soil lose during transportation?
2) How much water (in gallons) must be added to the truck to arrive at the site with a 15%
moisture?

Solution:
For a moisture content w = 12%,
W W
w = W = 0.12 but W = 1, 000lbs = WS + WW = W + WW = 9.33WW
WS 0.12
1, 000lbs W 107.2lb
∴WW = = 107 lb and WS = W = = 893 lb
9.33 0.12 0.12
For the lowered moisture content of w = 11.5%,
W W
w = W = 0.115 but W = 1, 000lbs = WS + WW = W + WW = 9.696WW
WS 0.115
1, 000lbs
∴WW = = 103 lb
9.696
 1 ft 3   7.48 gallons 
∴ the truck lost 4 lbs of water = (4lb )    = 0.48 gallons
 62.4lb   1 ft 3 
To arrive at the site with a moisture content of w = 15%,
W
w = W = 0.15 ∴WW = 0.15WS = ( 0.15 )( 893lb ) = 134lb
WS
The truck load needs to add 134 -103 = 31 lbs of water ≡ 3.71gallons

34
*Phases of soils–09: Find γd, n, S and Ww.
(Revision: Feb-10)
The moist unit weight of a soil is 105 lb/ft3. Given that the w = 15% and Gs = 2.70, find:
a) Dry unit weight γd,
b) The porosity n,
c) The degree of saturation S, and

Solution:
γ 105 lb
a) γ d = = = 91.3 3
(1 + w) (1 + 0.15) ft
b) From the table of useful relationships,

γd =
Gs γ w Gγ
∴ 1+ e = s w =
( 2.70 )( 62.4 ) = 1.85 ∴ e = 0.85
1+ e γd ( 91.3)
e 0.85
n= = (100% ) = 46%
1 + e 1 + 0.85

wGs ( 0.15 )( 2.70 )


c ) Since Se = wGs ∴ S = = (100 ) = 48%
e ( 0.85 )

35
*Phases of soils–10: Continued previous Phases-09 problem, adding water.
(Revision: Feb-10)
Using the previous problem Phases-09, how much mass of water in lbm/ft3 must be added to reach
full saturation.

Solution:
The mass of water that must be added can be found from the relation,
γ
γ = ρg or ρ= which is the density of water (mass per unit volume)
g
However, this γ is the saturated unit weight, or
( G + e ) γ w = ( 2.70 + 0.85 ) ( 62.4lb /
γ sat = S
ft 3 )
= 119.7lb / ft 3
1+ e 1 + 0.85
γ  (119.7 - 62.4 ) lb / ft 
3
lbm
∴ ρ ( mass of water ) = =  2  = 1.78
g  32.2 ft / s  ft 3

36
*Phases of soils–11: Use the block diagram to find the degree of saturation.
(Revision: Feb-10)

A soil has an “in-situ” (in-place) voids ratio eo = 1.87, wN = 60%, and GS = 2.75 . What are the
γmoist and S? All soils are really “moist” except when dry, that is when w = 0%.

Solution: Set VS = 1 m3 (Note: this problem could also be solved by setting V = 1.0 m3).
VV 1.87
∴ eo = = = 1.87 ∴ V = VS + VV = 1 + 1.87 = 2.87 m 3
VS 1
Ww
The "natural" water content is wN = = 0.60 ∴ Ww = 0.60Ws
Ws
Ws
γs
∴ Ws = Vs ( GS γ w ) = (1 m 3 ) ( 2.75 ) ( 9.81 kN / m 3 ) = 26.98 kN
Vs
Gs = =
γw γw
Ww = 0.60 (Ws ) = ( 0.60 )( 26.98 ) = 16.18 kN
W = Ws + Ww = 26.98 + 16.19 = 43.16 kN
W 43.16 kN kN
∴ γ moist == 3
= 15.0 3
V 2.87 m m
Ww  16.19 
γ  
S= w = w =
V 9.81 
∴ = 88.1%
VV VV 1.87

37
*Phases of soils–12: Same as Phases-11 but setting the total volume V=1 m3.
(Revision: Feb-10)

A soil has an “in-situ” (in-place) voids ratio eo = 1.87, wN = 60%, and GS = 2.75 . What are the
γmoist and S?

Solution: Set V = 1 m3 (instead of Vs = 1 m3 used in Phases-11).


VV
but eo = = 1.87 ∴ but V = 1 m 3 = VS + VV = VS + 1.87VS = 2.87VS
VS
∴ VS = 0.348 and VV = 0.652
Ww
The "natural" water content is wN = = 0.60 ∴ Ww = 0.60Ws
Ws
Ws
γs
∴ Ws = Vs ( GS γ w ) = ( 0.348 m 3 ) ( 2.75 ) ( 9.81 kN / m 3 ) = 9.39 kN
Vs
Gs = =
γw γw
Ww = 0.60 (Ws ) = ( 0.60 )( 9.39 ) = 5.63 kN
W = Ws + Ww = 9.39 + 5.63 = 15.02 kN
W 15.0 kN kN
∴ γ moist = = 3
= 15.0 3
V 1m m
Ww  5.63 
Vw γ w  
=
9.8 
∴ S= = = 88.1%
VV VV 0.652

38
*Phases of soils–13: Same as Phases-06 but using a block diagram.
(Revision: Feb-10)
A soil sample has a unit weight of 105.7 pcf and a water content of 50%. When its saturation is
increased to 75 %, its unit weight raises to 112.7 pcf. Determine the voids ratio e and the specific
gravity Gs of the soil.

Solution:
S et V = 1 ft 3
γ 2 − γ 1 = 112.7 − 105.7 = 7.0 lbs are 25% of w ater
∴ 21.0 lbs are 75% of w ater
∴ W S = 112.7 − 20.8 = 91.9 lb
Ww 20.8 lb
Vw = = = 0.333 ft 3
γw 62.4 pcf
1
Va = V w = 0.111 ft 3 ∴ V v = V a + V w = 0.111 + 0.333 = 0.444
3
∴ V s = 1 − V v = 1 − 0.444 = 0.556
VV 0.444
e= = = 0.80
VS 0.55 6
γS WS 91.9 lb
and GS = = = = 2. 65
γ w V S γ w ( 0.556 ) (62 .4 )

39
*Phases of soils–14: Block diagram for a saturated soil.
(Revision: Feb-10)

A saturated soil sample has a unit weight of 122.5 pcf and Gs = 2.70. Find γ dry , e, n, and w.

Solution:
Set the total volume V = 1 ft 3 and rearranging terms,
1  WS 
V = VS + Vw =  + Ww  (1 )
γ w  GS 
W = W S + W w = 122.5 lb (2 )
1  122.5 − W w 
Combining equations (1) and (2) yields 1=  + Ww 
( 62.4 pcf )  2.70 
Ww 27.0 lb
∴ W w = 27.0 lb ∴ V w = = = 0.433 ft 3
γw 62.4 pcf
WS 95.5 lb
∴ W S = 95.5 lb ∴ V S = = = 0.567 ft 3
G S γ w ( 2.70 ) (62.4 pcf )
WS 95.5 lb
∴ γ dry = = = 95.5 pcf
V 1 ft 3
VV 0.433
∴ e= = = 0.764
VS 0.567
VV 0.433
∴ n= = = 0.433 n = 43.3%
V 1
Ww 27
∴ w= = = 0.2 83 w = 28 . 3 %
WS 9 5 .5

40
*Phases of soils–15: Find the weight of water needed for saturation.
(Revision: Sept.-08)
Determine the weight of water (in kN) that must be added to a cubic meter of soil to attain a 95 %
degree of saturation, if the dry unit weight is 17.5 kN/m3, its moisture is 4%, the specific gravity of
solids is 2.65 and the soil is entirely made up of a clean quartz sand.

Solution:
kN γ γ kN
γ = 1 7 .5= = ∴ γ = 1 8 .2
1+ w 1 + 0 .0 4
d 3
m m3
W = 1 8 .2 = W S + W w = W S + w W S = (1 .0 4 ) W S
∴ W S = 1 7 .5 k N , and W w = 0 .7 0 k N
WS 1 7 .5 k N 1 7 .5 k N
VS = = = = 0 .6 7 3 m 3

γ S G Sγ w ( 2 .6 5 ) ( 9 .8 1 k N / m 3 )
Ww 0 .7 0 k N
Vw = = = 0 .0 7 m 3
∴ V a = V − V s − V w = 0 .2 5 7 m 3

γ w ( 9 .8 1 k N / m 3 )
VV 0 . 0 7 + 0 .2 5 7
e = = = 0 .4 9
VS 0 .6 7 3
wGS ( 0 .0 4 ) ( 2 . 6 5 )
T h e e x is tin g S = = (1 0 0 ) = 2 1 . 6 %
e 0 .4 9
W e re q u ire a S = 9 5 % , th e re fo re ,

w =
Se
=
( 0 . 9 5 ) ( 0 .4 9 ) = 0 .1 7
GS 2 .6 5
W w = w W S = ( 0 . 1 7 ) (1 7 . 5 k N ) = 2 .9 8 k N
a lr e a d y h a v e W w = 0 .7 0 k N
∴ m u s t a d d w a te r = 2 .2 8 k N
3
Answer: Add 2.28 kN of water per m .

41
*Phases of soils–16: Identify the wrong piece of data.
(Revision: Feb-10)
A project engineer receives a laboratory report with tests performed on marine marl calcareous
silt). The engineer suspects that one of the measurements is in error. Are the engineer’s suspicions
correct? If so, which one of these values is wrong, and what should be its correct value?
kN
G iven γ = unit w eight of sam ple = 18.4
m3
kN
γ S = unit w eight of solids = 26.1
m3
w = w ater content = 40%
e = voids ratio = 1.12
S = degree of saturation = 95%

Solution:

Check the accuracy of 4 out of 5 of the variables using,


Se = wGS ∴ Se = ( 0.95 )(1.12 ) = 1.06
γS 26.1
wGS = ( w ) = ( 0.4 ) = 1.06 ∴ Therefore, these four are correct.
γw 9.81
The only possibly incorrect value is γ . Assume that V = 1 m3 .
V = 1 m3 = Va + Vw + VS (1)
VV
but e = = 1.12 ∴ 0 = − Va − Vw + 1.12VS ( 2 )
VS
∴ VS = 0.472 m3 , VV = 0.528 m3 but Vw = 0.95VV = 0.502 m3
∴ Va = 0.026 m3
 kN 
∴WS = γ SVS =  26.1 3  ( 0.472 m3 ) = 12.3 kN
 m 
 kN 
Ww = wWS = ( 0.40 ) 12.3 3  = 4.9 kN
 m 
W = 12.3 kN + 4.9 kN = 17.2 kN
Therefore, the actual unit weight of the soil is,
W 17.2 kN kN kN
∴γ = = 3
= 17.2 3 ≠ 18.4 3
V 1m m m

42
*Phases of soils–17: The apparent cheapest soil is not!
(Revision: Feb-10)
You are a Project Engineer on a large earth dam project that has a volume of 5x106 yd3 of select
fill, compacted such that the final voids ratio in the dam is 0.80. Your boss, the Project Manager
delegates to you the important decision of buying the earth fill from one of three suppliers. Which
one of the three suppliers is the most economical, and how much will you save?
Supplier A Sells fill at $ 5.28/ yd3 with e = 0.90
Supplier B Sells fill at $ 3.91/ yd3 with e = 2.00
Supplier C Sells fill at $ 5.19/ yd3 with e = 1.60

Solution: Without considering the voids ratio, it would appear that Supplier B is cheaper than
Supplier A by $1.37 per yd3. But,

Therefore: To put 1yd3 of solids in the dam you would need 1.8 yd3 of soil.
For 1yd3 of solids from A you would need 1.9 yd3 of fill.
For 1yd3 of solids from B you would need 3.0 yd3 of fill.
For 1yd3 of solids from C you would need 2.6 yd3 of fill.

The cost of the select fill from each supplier is (rounding off the numbers):

 5.28$ 
( 5 ) (10 6 yd 3 )  3  ≈ $ 27, 900, 000
1.9
A=
1.8  yd 
 3.91$ 
( 5 ) (10 6 yd 3 )  3  ≈ $ 32, 600, 000
3.0
B=
1.8  yd 
 5.19$ 
( 5 ) (10 6 yd 3 )  3  ≈ $ 37, 500, 000
2.6
C =
1.8  yd 
Therefore Supplier A is the cheapest by about $ 4.7 Million compared to Supplier B.

43
*Phases of soils–18: Number of truck loads for Phases-17.
(Revision: Feb-10)
Based on the previous problem data (Phases–17), if the fill dumped into the truck has an e = 1.2,
how many truck loads will you need to fill the dam? Assume each truck carries 20 yd3 of soil.

Solution:
VV VV
Set VS = 1 e = = = VV = 1.2 which means that there is 1 yd 3 of solids per 1.2 yd 3 of voids.
VS 1
∴ 2.2 yd 3 of soil for each 1 yd 3 of solids.
10 yd 3 of soil for each x yd 3 in a truck load
∴ x = 4.54 yd 3 of solids per truck trip.
The required volume of solids in the dam is,

Vsolids =
( 5x10 6
yd 3of soil )(1 yd 3of solids )
= 2.8 x106 yd 3 of solids
3
1.8 yd of soil
Therefore, (rounding off)

Number of Truck − trips =


( 2.8x10 6
yd 3of solids )
= 616,800
( 4.54 yd 3of solids / truck − trip )

44
*Phases of soils–19: How many truck loads are needed for a project?
(Revision: Feb-10)
You have been hired as the Project Engineer for a development company in South Florida to build
610 housing units surrounding four lakes. Since the original ground is low, you will use the
limestone excavated from the lake to fill the land in order to build roads and housing pads. Your
estimated fill requirements are 700,000 m3, with a dry density equivalent to a voids ratio e = 0.46.
The “in-situ” limestone extracted from the lakes has an e = 0.39, whereas the limestone dumped
into the trucks has an e = 0.71. How many truckloads will you need, if each truck carries 10 m3?

Solution:
VV VV
Assume: VS = 1 m3 ∴ e = = = VV = 0.46 m3 in the compacted fill
VS 1
The required 700,000 m3 of fill have 1.46 m 3 of voids per each 1 m3 of solids

Therefore, the 700,000 m3 of fill have 479,400 m3 of solids

Each truck carries 1.71 m3 of fill per 1 m3 solids

In order for the trucks to carry 479,000 m3 of solids they must carry 820,000 m3 of fill

Since each truck carries 10 m3 of fill,

820, 000 m3
∴ The number of truck-loads = = 82,000 truck-loads.
10 m3

 5.28$ 
( 5 ) (10 6 yd 3 )  3  ≈ $ 27, 900, 000
1.9
A=
1.8  yd 
 3.91$ 
( 5 ) (10 6 yd 3 )  3  ≈ $ 32, 600, 000
3.0
B=
1.8  yd 
 5.19$ 
( 5 ) (10 6 yd 3 )  3  ≈ $ 37, 500, 000
2.6
C =
1.8  yd 
Therefore Supplier A is the cheapest by about $ 4.7 Million compared to Supplier B.

45
*Phases of soils–20: Choose the cheapest fill supplier.
(Revised: Feb-10)
A large housing development requires the purchase and placement of the fill estimated to be
200,000 cubic yards of lime-rock compacted at 95% Standard Proctor with an OMC of 10%. Two
lime-rock suppliers offer to fill your order: Company A has a borrow material with an in-situ γ =
115 pcf, w = 25%, GS = 2.70; Standard Proctor yields a maximum γd = 112 pcf; at a cost of
$0.20/yd3 to excavate, and $ 0.30/yd3 to haul. Company B has a borrow material with an in-situ γ =
120 pcf, w = 20%, GS = 2.70; Standard Proctor yields a maximum γd = 115 pcf; a cost of $0.22/yd3
to excavate, and $ 0.38/yd3 to haul.
(1) What volume would you need from company A?
(2) What volume would you need from company B?
(3) Which would be the cheaper supplier?

Solution:

(1) The key idea: 1 yd3 of solids from the borrow pit supplies 1 yd3 of solids in the fill.

(2) Pit A: WS = 92 lb, WW = 23 lb VW = 0.369 ft3, VS = 0.546 ft3, Va = 0.085 ft3

VV 0.454
e= = = 0.83 ∴ 1.83 yd 3of soil contains 1.0 yd 3of solids.
VS 0.546

Pit B: WS = 100 lb, WW = 20 lb, VW = 0.321 ft3, VS = 0.594 ft3, Va = 0.08 ft3

VV 0.401
e= = = 0.68 ∴ 1.68 yd 3of soil contains 1.0 yd 3of solids.
VS 0.594

(3) Material needed for fill from company A:

γ = 0.95γ d (1 + w ) = 0.95 (112 )(1 + 0.10 ) = 117 pcf ∴ WS = 106.4 lb, Ww = 10.6 lb
VV 0.37
e= = = 0.59 ∴ 1.59 yd 3of soil contains 1.0 yd 3 of solids
VS 0.63
200, 000 yd 3 of fill
∴ Site A requires = 125,800 yd 3 of solids
1.59

Material needed for fill from company B:

γ = 0.95γ d (1 + w ) = 0.95 (115 )(1 + 0.10 ) = 120 pcf ∴ WS = 109.1 lb, Ww = 10.9 lb
VV 0.35
e= = = 0.54 ∴ 1.54 yd 3of soil contains 1.0 yd 3 of solids
VS 0.65
200, 000 yd 3 of fill
∴ Site B requires = 130, 000 yd 3 of solids
1.54

46
(4) a) Cost of using Company A:

 $0.50 
Cost A = (125,800 yd 3 ) (1.83)  3 
= $ 115,100
 yd 

Cost of using Company B:

 $0.60 
Cost B = (130, 000 yd 3 ) (1.68 )  3 
= $ 131,100
 yd 

Using Company A will save about $ 16,000.

47
*Phases of soils–21: Use a matrix to the find the missing data.
(Revision: Sept.-08)
A contractor obtains prices for 34,000 yd3 of compacted “borrow” material from three pits: Pit #3
is $11,000 cheaper than Pit #2 and $39,000 cheaper than Pit #1. The fill must be compacted down
to a voids ratio of 0.7. Pit #1 costs $ 6.00/yd3 and Pit #3 costs $ 5.50/yd3. Pits #2 and #3 reported
their voids ratios as 0.88 and 0.95 respectively. Use a matrix to find,
a) The missing unit cost C2 for Pit #2;
b) The missing voids ratio e for Pit #1;
c) The missing volume of fill V required from each pit; and
d) The amount paid by the contractor for each pit.

Solution:
A summary of the data provided above converted into matrix form, with missing data in red.

The volume of solids Vs contained in the total volume of fill V = 34,000 yd3 can be found from,
34, 000
V = VV + VS = 0.7VS + VS = VS ( 0.7 + 1) = 34, 000 yd 3 ∴ VS = = 20, 000 yd 3 of solids
1.7

= 1 + e3 ∴ V3 = VS (1 + e3 ) = ( 20, 000 yd 3 ) (1 + 0.95 ) = 39, 000 yd 3 of soil


V3
At Pit #3,
VS
The total cost of Pit #3 is TC3 = ( 39, 000 yd 3 )( $ 5.50 / yd 3 ) = $ 214,500

= 1 + e2 ∴V2 = VS (1 + e2 ) = ( 20, 000 yd 3 ) (1 + 0.88 ) = 37, 600 yd 3 of soil


V2
At Pit #2:
VS
But, the total cost of Pit #2 is TC2 − $ 11, 000 = TC3 = $ 214,500 ∴ TC2 = $ 225, 500
TC2 $ 225,500
The unit cost of Pit #2 C2 = = 3
= $ 6.00 / yd 3
V2 37, 600 yd

TC1 TC + 28, 000 225, 500 + 28, 000


At Pit #1: V1 = 3
= 2 = = 42, 250 yd 3 of soil
$ 6.00 / yd $ 6.00 / yd 3 $ 6.00 / yd 3
But, V1 = VS (1 + e1 ) = ( 20, 000 yd 3 ) (1 + e1 ) = 42, 250 yd 3 ∴ e1 = 1.11

48
**Phases of soils–22: Find the voids ratio of “muck” (a highly organic soil).
(Revision: Feb-10)
You have been retained by a local municipality to prepare a study of their “muck” soils. Assume
that you know the dry unit weight of the material (solids) γsm and the dry unit weight of the
organic solids γso. What is the unit weight γs of the combined dry organic mineral soil whose
organic content is M0? (The organic content is the percentage by weight of the dry organic
constituent of the total dry weight of the sample for a given volume.) What is the voids ratio e of
this soil if it is known that its water content is w and its degree of saturation is S?

Solution:
Ws 1
Set Ws = 1 unit and γs = =
Vs (Vso + Vsm )
(a) Assume Mo = Wo for a unit weight of the dry soil
Therefore 1 - Mo = Wm
Mo
= volume of organic Vso solids
γ so

(1 - M o )
= volume of mineral Vsm solids
γ sm

The total unit weight is the weight of a unit volume.

1  γ sm 
Therefore γ S = = γ so  
 Mo (1 - Mo )   M o ( γ sm - γ so ) + γ so ) 
 + 
 γ so γ sm 

 volume of water   weight of water   w ( weight of solids ) 


     
Vv  S   γ wS   γ wS 
(b) e = = = =
Vs Vs Vs Vs

 w 
 
 γ w S (1)  =
wγ sm γ so
Therefore e =
 Mo
+
(1 - M o )  γ w S  M o (γ sm − γ so ) + γ so 
 
 γ so γ sm 

49
Chapter 3
Classification of Soils and Rocks
Symbols for Classification of soils

Cc → Coefficient of gradation (also coefficient of curvature).


Cu → Coefficient of uniformity.
RC→ Relative compaction.
Dx → Diameter of the grains (at % finer by weight).
Dr → Relative density of a granular soil.
e→ Voids ratio.
emin → Minimum voids ratio.
emax → Maximum voids ratio.
IP → Index of plasticity (also referred to as PI).
K→ Constant of the yield value.
LL→ Liquid limit.
PL→ Plastic limit.
SL→ Shrinkage limit.
V→ Volume of the soil sample.
W→ Weight of the soil sample.
γd(min)→Dry unit weight in loosest condition (voids ratio emax).
γd → In-situ dry unit weight (voids ratio e).
γd(max)→ Dry unit in densest condition (voids ratio emin)

50
*Classify–01: Percentage of each of the four grain sizes (G, S, M & C).
(Revision: Sept.-08)
Determine the percentage of gravels (G), sands (S), silts (M) and clays (C) of soils A, B and C

shown below.

Solution:
Notice that the separation between gravels (G) and sands (S) is the #4 sieve which corresponds to a
particle size of 4.75 mm. The separation between sands (S) and silts (M) is the #200 sieve which
corresponds to a particle size of 0.075 mm. Finally, the separation between silts (M) and clays (C) is the
0.002 mm (or 2 micro-meters = 2 µm). These divisions are shown above through color differentiation.
Each soil A, B and C is now separated into the percentage of each:

Soil A: 2% G; 98% S; 0% M; 0%C. This soil is a uniform or poorly-graded sand (SP).


Soil B: 1% G; 61% S; 31% M; 7%C. This soil is a well-graded silty sand (SM).
Soil C: 0% G; 31% S; 57% M; 12%C. This soil is a well-graded sandy silt (M).

51
*Classify–02: Coefficients of uniformity and curvature of granular soils.
(Revision: Sept.-08)
Determine the uniformity coefficient Cu and the coefficient of gradation Cc for soil A.

Solution:
From the grain distribution curve, D60 = 1.4 mm, D30 = 0.95 mm and D10 = 0.50 mm, therefore the
coefficients are,

( 0.95) = 1.29
2
D 1.40 mm D302
CU = 60 = = 2.8 and CC = =
D10 0.50 mm D60 D10 (1.40 )( 0.50 )
A uniform soil has a coefficient of uniformity Cu less than 4, whereas a well-graded soil has a
uniformity coefficient greater than 4 for gravels and greater than 6 for sands. Since soil A has a low
value of 2.8, and it is sand, this corresponds to a poorly-graded sand (SP). Steep curves are uniform
soils (low Cu) whereas diagonal curves are well-graded soils (high Cu).
Smooth curved soils have coefficients of curvature Cc between 1 and 3, whereas irregular curves have
higher or lower values. Soils that are missing a type of soil (a gap) are called gap-graded (Cc will be less
than 1 or greater than 3 for gap-graded soils).
Therefore, this soil is classified as poorly-graded sand (or SP).
52
*Classify-03: Classify two soils using the USCS.
(Revision: Sept.-08)
Use the grain-size distribution curve shown below to classify soils A and B using the USCS. Soil
B’s Atterberg limits are LL = 49% and PL = 45%?

Solution:
Classify Soil A:
For soil A, the distribution is G = 2%, S = 98%, M = 0% and C = 0%.
D60 1.40 mm
CU = = = 2.8 , therefore, soil A is a poorly graded sand (SP).
D10 0.50 mm
Classify Soil B:
For soil B, the distribution is G = 0%, S = 61%, M = 35% and C = 4%.
D60 0.45 mm
CU = = = 90 , therefore, soil A is very well graded silty sand (SM).
D10 0.005 mm

53
*Classify-04: Manufacturing a “new” soil.
(Revision: Sept.-08)
A site has an unsuitable in-situ soil A that does not compact properly. In lieu of removing that soil
A, you have decided to improve it by mixing it with a borrow pit soil B to produce an improved
new soil C that will compact better.
You desire a coefficient of uniformity Cu of about 100 for the new soil C. Determine the relative
percentages of these two uniform soils A and B so that they will result in better graded soil C. Plot
your results.
The plots of soils A and B are as shown below,

Soil A is composed of 2% G, and 98% S: (6% coarse sand, 85% medium sand and 7% fine sand).
It is obviously a poorly graded sand (SP).

Soil B is composed of approximately 33% S, 55% M and 12% C. It is a well-graded sandy silt.

Consider several solutions as shown below with A/B ratios of 30/70, 35/65, 40/60 and 50/50. The
best is the 50/50 solution via D10 = 0.006 mm,
D60 D60
CU = = 100 = ∴ D60 = 0.6mm
D10 0.006mm
The best fit is a 50% of A plus 50% of B mix.

54
55
Classify – 05
(Revision: Sept.-09)
A sample of soil weights 1.5 N. Its clay fraction weighs 0.34 N. If its liquid limit is 60% and its
plastic limit is 26%, classify the clay.

Solution:
W = 1.5 N
Wclay = 0.34 N (or 23% of W)
Ip = PI = LL – PL = 60% – 26% = 34 %

IP 34%
A = = ≈ 1.5
% of clay fraction 23%

The activity number 1.5 falls above the U-line in Skempton’s diagram (see Classify-03). Therefore, this
is a CH clay, and is probably a member of the Montmorillonite family.

56
Classify – 06
(Revision: Sept.-09)
During a hydrometer analysis a soil with a Gs = 2.60 is immersed in a water suspension with a
temperature of 24°C. An R = 43 cm is obtained after 60 minutes of sedimentation. What is the
diameter D of the smallest-size particles that have settled during that time?

Solution:

Using the table below, for Gs = 2.60 and T= 24°C, K= 0.01321.

∴ L = 16.29 − ( 0.164 R ) = 16.29 − [0.164(43)] = 9.2 cm

L 9.2 cm
D= K = 0.01321 = 0.00517 mm = 5.2 x 10-3 mm ( a silt)
t 60 min

Table of constant K versus Temperature T (°C)

Temparature Gs
(°C) 2.45 2.50 2.55 2.60 2.65 2.70 2.75 2.80
16 0.01510 0.01505 0.01481 0.01457 0.01435 0.01414 0.01394 0.01374
17 0.01511 0.01486 0.01462 0.01439 0.01417 0.01396 0.01376 0.01356
18 0.01492 0.01467 0.01443 0.01421 0.01399 0.01378 0.01359 0.01339
19 0.01474 0.01449 0.01425 0.01403 0.01382 0.01361 0.01342 0.01323
20 0.01456 0.01431 0.01408 0.01386 0.01365 0.01344 0.01325 0.01307
21 0.01438 0.01414 0.01391 0.01369 0.01348 0.01328 0.01309 0.01291
22 0.01421 0.01397 0.01374 0.01353 0.01332 0.01312 0.01294 0.01276
23 0.01404 0.01391 0.01358 0.01337 0.01317 0.01297 0.01279 0.01261
24 0.01388 0.01365 0.01342 0.01321 0.01301 0.01282 0.01264 0.01246
25 0.01372 0.01349 0.01327 0.01306 0.01286 0.01267 0.01249 0.01232
26 0.01357 0.01334 0.01312 0.01291 0.01272 0.01253 0.01235 0.01218
27 0.01342 0.01319 0.01397 0.01277 0.01258 0.01239 0.01221 0.01204
28 0.01327 0.01304 0.01283 0.01264 0.01244 0.01225 0.01208 0.01191
29 0.01312 0.01290 0.01269 0.01249 0.01230 0.01212 0.01195 0.01178
30 0.01298 0.01276 0.01256 0.01236 0.01217 0.01199 0.01182 0.01169

57
Classify – 07
(Revision: Sept.-09)
The fines fraction of a soil to be used for a highway fill was subjected to a hydrometer analysis by
placing 20 grams of dry fines in a 1 liter solution of water (dynamic viscosity 0.01 Poise at 20
degrees centigrade). The specific gravity of the solids was 2.65.
a) Estimate the maximum diameter D of the particles found at a depth of 5 cm after a
sedimentation time of 4 hours has elapsed, if the solution’s concentration has reduced to 2
grams/ liter at the level.
At that moment,
b) What percentage of the sample would have a diameter smaller than D?
c) What type of soil is this?

Solution:
γs −γw 2 18η  L(cm) 
a) Using Stoke’s relation: v = d or d (mm) =  
18η γ s − γ w  t (min) 

 dyne ⋅ s 
where t = 4 hours = 14,400 sec, L = 5 cm, η = 10−2 Poise  i.e., 
 cm 2 
γs
but G s = ∴ γ s = G sγ w = ( 2.65 ) ( 9.81 dynes / cm 3 ) = 26 dynes / cm 3
γw

 dynes ⋅ sec 
18 x10 −2   ( 5 cm )
 cm 2 
d = = 0.020 mm
 dynes 
9.81  3 
(2.65 − 1.00)(14, 400 sec)
 cm 
b) The unit weight γ of the solution after 4 hours is,
weight of soil in solution 2 g + [1000cm3 − 2 g / 2.65] x 1 g / cm3
γ= = = 1.001 g / cm3
volume of solution 1000 cm3
The portion of soil having a diameter smaller than D is,

V γ γ 1000 cm 3  2.65 x 1 
Portion smaller = • s w (γ − 1) =   (1.001 − 1) = 0.08
W γs −γw 20 g  2.65 − 1 

∴ The remaining soil is only 8 % of the original sample.

c) The diameter d = 0.020 mm corresponds to a silt.

58
Classify – 08
(Revision: Sept.-09)

emax − e
The formula for the relative compaction Dr is, Dr =
emax − emin
Derive an equivalent equation as a function of dry unit weights, such that
(γ − γ d min ) (γ d max )
Dr =
d ( field )

(γ d max − γ d min ) (γ d ( field ) )

Solution:
γd (min) = dry unit weight in loosest condition (voids ratio emax)
γd = in-situ dry unit weight (voids ratio e)
γd (max) = dry unit in densest condition (voids ratio emin)

where
WS G S γ W
γd = = and Dr = 0 = loose, to 1= very dense
V 1+ e

1 1

γ d min γd  γ − γ d min   γ d max   γ d ( field ) − γ d min   γ d max 
Dr = = d  =  
1

1  γ d max − γ d min   γ d   γ d max − γ d min   γ d ( field ) 
γ d min γ d max

For example, what is the RC (relative density) of a sand in the field if it was tested to be at 98%
Standard Proctor, its maximum unit weight was 18.8 kN/m3 and its minimum unit weight was 14.0
kN/m3?

γ γ
RC = 98% = = ∴ γ = 1 8 .4 k N / m
d ( fie ld ) d ( fie ld ) 3

γ d ( S td . P r o c t .) 1 8 .8
d ( fie ld )

 γ − γ d min   γ d max  (18.4 − 14.0 )(18.8)


Dr =  d  = = 94%
 γ d max − γ d min   γ d  (18.8 − 14.0 )(18.4 )

59
Classify – 09
(Revision: Sept.-09)
The data obtained from relative density tests is shown below. Calculate the range of relative
densities.

Limiting γ Average γ in kN/m3


γ max 18.07 17.52
γ min 14.77 15.56
γ field 16.97

Solution:

 γ −γ  γ 
Dr =  n min  max 
 γ max − γ min  γ n 

 16.97 − 14.77  18.07 


range 1 (low γ min )(high γ max ) Dr =    = 0.71
 18.07 − 14.77  16.97 
 16.97 − 15.56  18.07 
range 2 (avg γ min )(high γ max ) Dr =    = 0.60
 18 .07 − 15 .56  16 . 97 

 16.97 − 14.77  17.52 


range 3 (low γ min )(avg γ max ) Dr =    = 0.83
 17 . 52 − 14 . 77  16 . 97 

 16.97 − 15.56  17.52 


range 4 (avg γ min )(avg γ max ) Dr =    = 0.74
 17.52 −15.56  16.97 

∴ 60% ≤ D ≤ 83%
r

60
Classify – 10
(Revision: Sept.-09)
South Florida has two types of sand, a quartzitic sand (γs2 = 165.5 pcf) and calcareous sand (γs1 =
146.3 pcf). At a particular site, their voids ratios were found to be:
for the quartzitic sand, еmax = 0.98 and emin = 0.53
for the calcareous sand, еmax = 0.89 and emin = 0.62
These voids ratios were measured by using a mold with a diameter of 4 inches and a height of 4.59
inches. The dry quartzitic sand weight was 3.254 lbs, and the dry calcareous sand was 2.868 lbs.
Find their relative densities and dry unit weights. Comment on these results.

Solution:
e max − e
By definition Dr =
e max − e min
πd 2 h P1 π (4) 2 (4.59) 2.868
− −( ) × 12 3
4 γ S1 4 146.3
For the calcareous sand, e1 = = = 0.70
P1 2.868 × 12 3
γ S1 146.3

πd 2 h P2 π (4) 2 (4.59) 3.254


− −( ) × 12 3
4 γ S2 4 165.6
For the quartzitic sand, e2 = = = 0.70
P2 3.254 × 12 3
γ S2 165.6
Notice that e1 = e2
0.89 − 0.70
For the calcareous sand, Dr1 = = 0.70 and
0.89 − 0.62

0.98 − 0.70
For the quartzitic sand, Dr2 = = 0.62
0.98 − 0.53
The two types of sand have different relative densities because the calcareous sand grains are
more tightly packed than the quartzitic sand grains.
γs 146.3
For the calcareous sand, γ d1 = 1
= = 86.1 pcf (but Dr1 = 0.70)
1 + e1 1 + 0 .7
γs 165.6
For the quartzitic sand, γ d 2 = 2
= = 97.4 pcf (but Dr21 = 0.62)
1 + e2 1 + 0 .7

As a result, the dry unit weight is greater for the soil with the lower relative density.
61
Classify – 11
(Revision: Sept.-09)
Prove that emin = 0.35.

Vtetr = 0.1179a 3 = 0.1179(2 R )3 = 0.943R 3


4
Vsphere = π R 3
3
The volume of the sphere occupied by the tetrahedron is:
60
∴ Vsphere ( tetr ) ≡ = 0.167 = 16.7%
360
V V− V
( )
3
2R VV −V 3
0.943R − 0.167 4 / 3 π R 3
∴ e= = = = 0.35
sphere

VS V sphere (
0.167 4 / 3πR 3 )

ALTERNATE METHOD:

d 2

( )
3
Volume of cube = d 2 = d32 2
 π d3  2 3
Volume of sphere = 4   = πd
 6  3
2 
2d 2 −  π d 3 
Vcube − Vsphere 3  = 0.35
emin = =
Vsphere 2 3
πd
3
e = 0.35 OK !!!
2d

62
Chapter 4
Compaction and Soil Improvement
Symbols for Compaction

e → Voids ratio.
GS→ Specific gravity of the solids of a soil.
n → Porosity of the soil.
OMC→ Optimum moisture content.
S → Degree of saturation.
−4 3
V → ( 1 30 ft ≡ 9.44*10 m Standard Proctor mold, ASTM D-698).
3

Va→ Volume of air.


VS→ Volume of solids.
VV → Volume of voids (water + air).
VW → Volume of water.
w→ Water content.
VS→ Volume of soil sample.
γ→ Unit weight of the soil.
γd → Dry unit weight.
γb → Buoyant unit weight of the soil.
γSAT →Saturated unit weight of the soil.
γS → Unit weight of the solid.
γW → Unit weight of water.
γd.field → Dry unit weight in the field.

63
*Compaction–01: Find the optimum moisture content (OMC).
(Revision: Aug-08)
A Standard Proctor test has yielded the values shown below. Determine:
a) The maximum dry unit weight and its OMC; remember V = 1/30 ft3.
b) The moisture range for 93% of maximum dry unit weight.
No Weight of wet soil (lb) Moisture %
1 3.26 8.24
2 4.15 10.20
3 4.67 12.30
4 4.02 14.60
5 3.36 16.80

Solution:
W γ
Formulas used for the calculations: γ = and γ d =
V 1+ w
W (lb) w(%) γ (lb/ft3) γd (lb/ft3)
3.26 8.24 97.8 90.35
4.15 10.20 124.5 113.0
4.67 12.30 140.1 124.8
4.02 14.60 120.6 105.2
3.36 16.80 100.8 86.30

130

γdmax = 124.8 pcf


125

120

Maximum dry unit weight = 124.8 pcf


115

OMC = 12.3 %
γd

110

105

100
γfield = (0.93)(124.8) = 116.1 pcf

95

90
7 8 9 10 11 12 13 14 15 16 17 18

w(%)

64
*Compaction–02: Find maximum dry unit weight in SI units.
(Revision: Aug-08)
Using the table shown below:
a) Estimate the maximum dry weight of a sample of road base material, tested under Standard
Proctor ASTM D-698 (all weights shown are in Newton).
1 3  1 m3  −4 3
b) Note that the volume V = ft   = 9.44 × 10 m
30  35.32 ft 3 
c) Find the OMC.
d) What is the appropriate moisture range when attaining 95% of Standard Proctor?
Trial No. 1 2 3 4 5
W(Newton) 14.5 15.6 16.3 16.4 16.1
ω (%) 20 24 28 33 37

Solution:
Trial No. 1 2 3 4 5

γ =
W kN
V m3
( ) 15.4 16.5 17.3 17.4 17.1

γd =
1+ ω
γ
(kN m ) 3 12.8 13.3 13.5 13.1 12.5

γd max = 13.5 kN/m3


γd max = 13.5 kN/m 3

OMC = 28 %
OMC = 28%
13.6

γd-field = 0.95(13.5) = 12.8 kN/m3


13.2
γ d (kN/m)

12.8

12.4

12
18 20 22 24 26 28 30 32 34 36 38

w(%)

65
*Compaction-03: What is the saturation S at the OMC?
(Revision: Sept.-08)
The results of a Standard Compaction test are shown in the table below:

ω (%) 6.2 8.1 9.8 11.5 12.3 13.2


γ (kN/m3) 16.9 18.7 19.5 20.5 20.4 20.1
γ 15.9 17.3 17.8 18.4 18.2 17.8
γd =
1+ ω

e) a) Determine the maximum dry unit weight and the OMC.


f) b) What is the dry unit weight and moisture range at 95% RC (Relative Compaction)?
g) c) Determine the degree of saturation at the maximum dry density if Gs = 2.70.

Solution:
a) γd max = 18.4 kN/m3, OMC = 11.5%
b) γd at 95% = (0.95)(18.4) = 17.5 kN/m3

The moisture range w for 95% RC is from 8.75% to 13.75%.

19.5

18.2
Dry Unit Weight

16.9

15.6

14.3

13
5 6.1 7.2 8.3 9.4 10.5 11.6 12.7 13.8 14.9 16

w%

γd max = 13.5 kN/m3 OMC = 11.5 %


c)
wGsγ d max ( 0.115 )( 2.70 )(18.4 )
S=
γw
=
( 9.8) = 0.71 Saturation S = 71%
γ  18.4 
Gs − d max 2.7 −  
γw  9.8 
66
*Compaction-04: Number of truck loads required.
(Revision: Sept.-08)
The in-situ moisture content of a soil is 18% and its moist unit weight is 105 pcf. The specific
gravity of the soil solids is 2.75. This soil is to be excavated and transported to a construction site,
and then compacted to a minimum dry weight of 103.5 pcf at a moisture content of 20%.
a) How many cubic yards of excavated soil are needed to produce 10,000 yd3 of compacted fill?
b) How many truckloads are needed to transport the excavated soil, if each truck can carry 20
tons?

Solution:

Wborrow site = Wconstruction site


Vborrow siteγ borrow site = Vconstruction siteγ construction site
γ
105 pcf
γ d (borrow site) = = 89 pcf versus γ d ( construction site ) = 103.5 pcf
=
1 + w 1 + 0.18
 103.5 pcf 
a ) Volume to be excavated = 10, 000 yd 3   = 11, 630 yd
3

 89 pcf 
 27 feet 3  105 lb 
(11, 630 yd )  yd 3  feet 3  3

b) Number of truck loads =    = 824 truck − loads


 20 ton  2, 000 lb 
  
 truck  ton 

67
*Compaction-05: What is the saturation S at the OMC?
(Revision: Sept.-08)
A Standard Proctor test was performed on a clayey gravel soil; the test results are shown below.
Find the degree of saturation at the optimum condition; assume that Gs = 2.60.
Test 1 2 3 4 5 6 7
w% 3.00 4.45 5.85 6.95 8.05 9.46 9.90
γd
γw 1.94 2.01 2.06 2.09 2.08 2.06 2.05

γd kN/m3 19.4 20.1 20.6 20.9 20.8 20.6 20.5


Use γw = 10 kN/m3 for simplicity.

Solution:

wGs γ  γ −γ
It is known that Se = wGs ∴ S = but e =  s − 1 = s d
e γd  γd
therefore, at the OMC the saturation is,
γ   20.9 
SOMC = wOMC Gs  d −OMC  = ( 0.0695 )( 2.60 )   = 0.74
 γs −γd   ( 2.60 )(10 ) − 20.9 
The degree of saturation at the OMC is 74%.

2.15

2.1

2.05
Dry Unit Weight

1.95

1.9

1.85

1.8
2 2.5 3 3.5 4 4.5 5 5.5 6 6.5 7 7.5 8 8.5 9 9.5 10 10.5 11 11.5

w%

68
*Compaction-06: Definition of the relative compaction (RC).
(Revision: Sept.-08)
The relative compaction (RC) of a sandy road base in the field is 90%. The maximum and
minimum dry unit weights of the sand are γd(max) = 20.4 kN/m3 and γd(min) = 13.9 kN/m3. Determine
the field values of:
a) The dry unit weight in the field;
b) Relative density (of compaction) Dr;
c) The moist unit weight γ when its moisture content is 15%.

Solution:
The relative compaction RC is the dry unit weight obtained in the field, as compared to the Standard
Proctor obtained in the laboratory.
a ) The relative compaction RC is,
γ d ( field ) γ d ( field )
RC = 0.90 = = ∴ γ d ( field ) = ( 0.90 )( 20.4 ) = 18.4 kN / m3
γ d (max) 20.4
b) The relative density Dr is,
γ d ( field ) − γ d (min) γ d (max) (18.4 − 13.9 )  20.4 
Dr = × = = 0.768 = 76.8%
γ d (max) − γ d (min) γ d ( field ) ( 20.4 − 13.9 )  18.4 
c) The moist unit weight γ is,
γ = γ d (1 + w ) = 18.4 (1 + 0.15 ) = 21.2 kN / m3

69
*Compaction-07: The relative compaction (RC) of a soil.
(Revision: Aug-08)
A Standard Proctor compaction test performed on a sample of crushed limestone (Gs = 2.70)
obtained a maximum dry unit weight of 90 pcf at OMC. A field compacted sample showed a
moisture of 28% and a unit weight of 103.7 pcf.
Find the relative compaction (RC).
Find the degree of saturation S of the field soil sample.

Solution:

γ moist
103.7
γ d ( field ) = =
= 81.0 pcf
1 + w 1 + 0.28
γ d ( field )  81.0 pcf 
a ) RC = = = 0.90
γ d max  90.0 pcf 
The Relative Compaction = 90%
Gγ Gγ
b) γ d = s w ∴ 1 + e = s w
1+ e γd
G γ  ( 2.70 )( 62.4 )
∴ e =  s w − 1 = − 1 = 1.08
 γd  ( 81.0 )
Se = wGs
wGs ( 0.28 )( 2.70 )
∴ S= = = 0.70 Saturation S = 70%
e (1.08)

70
*Compaction-08: Converting volumes from borrow pits and truck loads.
(Revision: Oct.-08)
An embankment for a highway 30 m wide and 1.5 m thick is to be constructed from a sandy soil,
trucked in from a borrow pit. The water content of the sandy soil in the borrow pit is 15% and its
voids ratio is 0.69. Specifications require the embankment to be compacted to a dry unit weight of
18 kN/m3. Determine, for 1 km length of embankment, the following:
a) The dry unit weight of sandy soil from the borrow pit required to construct the embankment,
assuming that GS = 2.70;
b) The number of 10 m3 truckloads of sandy soil required to construct the embankment;
c) The weight of water per truck load of sandy soil; and
d) The degree of saturation of the in-situ sandy soil.

Solution:
GSγ w 2.7 (9.8) kN
a) T he borrow pit's dry unit w eight γ d = = = 15. 7 3
1+ e 1 + 0.69 m
b) T he volum e of the finished em bankm ent V = 30 m (1.5 m )(1km lo n g ) = 45x10 3 m 3
γ d ( reqd )
(V ) = 
18 
V olum e of borrow pit soil requir ed =  ( 45 x10 ) m
3 3

γ d ( borrow pit )  15.7 


  45 x10 m 
 18
3 3
N um ber of truck trips =  3  = 5,160 truck-loa ds
 15.7
  10 m 
 kN 
c) W eight of dry soil in 1 truck-load W d = 10 m 3  15. 7 3  = 157 kN
 m 
W eight of w ater = w W d = (0.15)(1 5 7 kN ) = 23 .6 kN per truck load

d) D egree of satu ration S =


wGS
=
( 0.15 )( 2.70 ) = 0.59 = 59%
e 0.69

71
**Compaction-09: Ranges of water and fill required for a road.
(Revision: Octt.-08)
From the Standard Proctor compaction curve shown below:
Give two possible reasons that may cause a Proctor test to cross the ZAV curve?
What is the water content range (in gallons) needed to build a street 1,000 feet long of compacted
16” base at 98% Standard Proctor for two lanes, each 12 ft wide?

126

124

122

120 γd(max)
S
= 0.41 0.29
10
0%

118
98% S. Proctor 0.43 0.30
1 4 .3
9 .7
γd

116

P o ro s ity
V o id R a tio
0.46 0.32

114

0.49 0.33
112

0.52 0.34
110

108
5 6 7 8 9 10 11 12 13 14 15 16 17 18 19 20 21 22
w%

72
Solution :
Gsγ w γ
a) Since γ zav = and γ zav = wet
1+  s 
wG 1+ w
 S
The crossing of the ZAV (zero air voids) curve is due to an incorrect assumption of Gs ,
and/or a miscalculation of the water content w and γ wet .

b) The soil volume V required for the road is,


 1 ft 
V = (16inthick)   ( 24 ft wide)(1,000 ft long) = 32,000 ft
3

 12 in 
The peak dry density γ d max = 119.5 pcf at an OMC=12%, and the range of the water content
is from 9.7% to 14.3% for 98% of Standard Proctor or ( 98%) γ d max = ( 0.98)(119.5) = 117.1pcf
The total weight of soil in the pavement is W = Vγ = ( 32,000 ft3 ) (117.1pcf ) = 3.75x106 lbs

The weight of water Ww at the low end (9.7%) is,


 3.75x106 lbs 
( 0.097)  
Ww  1.097   7.45gal 
Vw = =   ≃ 40,000gallons
γw 62.4 pcf  ft 3

The weight of water Ww at the high end (14.3%) is,
 3.75x106 lbs 
( 0.143)  
Ww  1.143   7.45gal 
Vw = =   ≃ 56,000gallons
γw 62.4 pcf 3
 ft 
Therefore, the average volume of water is (1/2)( 56,000 + 40,000) = 48,000gallons
Therefore, the volume of water required = 48,000 ± 8,000 gallons.

73
**Compaction-10: Find the family of saturation curves for compaction.
(Revision: Oct.-08)
This problem expands Compaction-05: A Modified Proctor compaction test is performed on a
clayey gravel road base. The solids have a specific gravity of 2.65. The compaction data yielded the
following binomial values for γd/γw versus w %:

w(%) 3.00 4.45 5.85 6.95 8.05 9.46 9.90


γd / γw 1.94 2.01 2.06 2.09 2.08 2.06 2.05

h) Find the γd max and the OMC from the compaction curve;
i) Find the degree of saturation at the above conditions.
j) Calculate the percentage of air for a given porosity n and the saturation S.
k) Find the equation that describes the points of equal saturation.
l) Determine the equation for S = 100%
m) Discuss the characteristics of this last curve and equation.

Solution:
a)
22

21.5

γd max = 20.9 kN/m


3
21

20.5 OMC = 7.7%


γd (kN/m )
3

20

19.5

19

18.5

18
2 3 4 5 6 7 8 9 10 11
Water Content w(%)

74
wGs  γ d max   2.09 
Se = wGs ∴ S =   = ( 7.7%)( 2.65)   = 76%
b)
e  γ w −γ d max   2.65 − 2.09 

c)
A a
n

nS 1
W
1-n
S

VV VW VW VW
ns = = = = VW
V VV V 1
a = 1 − nS − (1 − n ) = 1 − nS − 1 + n
a = n(1 − S ) = Va

That is, the percentage of air a is equal to the porosity n times the factor (1 - S), where
S is the degree of saturation.

d) Consider a family of curves of equal saturation,

volume of water volume of water


S= = =
volume of air+volume of water 1 − volume of soil grains
γ 
w d 
γ w S  γ Sγ s
S =  w  ∴γ d  +  − S ∴ d =
γ 
1−  d   γw γs  γ w wγ s + Sγ w
 γs 

75
Curves can be plotted for varying values of a and saturation S.

12

10

γd = (1-a2)γs
γd

4
a=0 Asymptotes
a1

a2

2 a3

0
0 3 6 9 12 15 18 21 24 27
w

1.2

1 If w = 0 all curves pass thru γs

0.8
γd

0.6

0.4
S = 100%
S1

0.2 S2

0
0 10 20 30 40
w

These are hyperbolas with the w-axis as an asymptote.

76
**Compaction-11: Water needed to reach maximum density in the field.
(Revision: Aug-08)
A Standard Proctor test yields the values listed below for a soil with Gs = 2.71. Find:
n) The plot of the dry unit weight versus the water content;
o) The maximum dry unit weight;
p) The optimum moisture content;
q) The dry unit weight at 90% of Standard Proctor;
r) The moisture range for the 90% value;
s) The volume of water (in gallons) that must be added to obtain 1 cubic yard of soil at the
maximum density, if the soil was originally at 10% water content.

w (%) 10 13 16 18 20 22 25
γ (pcf) 98 106 119 125 129 128 123

γd (pcf) 89 94 102.6 105.9 107.5 104.9 98.4

Solution:
a) The plot of γd (dry unit weight) versus w (water content):

125

120

115
dry unit weight, γd

110 100% Proctor

105

100 90% Proctor

95

90 OMC=20%

85
0 10 20 w 30
moisture range

(b) From the plot, the maximum dry unit weight is γdmax = 107.5 pcf.
(c) From the plot, the optimum moisture content is OMC = 20%.
(d) The 90% value of the maximum dry unit weight γdmax = (0.9) (107.5) = 96.8 pcf
(e) The moisture range for the 90% value is approximately from 13% to 26%.

77
(f) Soil at 10% moisture,

W Ww=9
V = 1 ft3
W = 98 lb
S
Ws=89
lb

Ww
w = 0.10 = ∴ Ww = 0.10Ws
Ws
but Ws = 89 lb and Ww = 0.10Ws = ( 0.1)( 89 ) = 9 lb ∴ W = Ws + Ww = 89 + 9 = 98 lb

Soil at 20% moisture,

Ww=21.5
W
V = 1 ft3
W = 129 lb
S
Ws= 107.5

Ww
w = 0.20 = ∴ Ww = 0.20Ws
Ws
but Ws = 107.5 lb and Ww = 0.20Ws = ( 0.2 )(107.5 ) = 21.5 lb ∴W = Ws + Ww = 129 lb

Therefore, need to add the following water: (21.5 lb) – (9 lb) = 12.5 lb/ft3

12.5lb ft 3 7.48 gallons 27 ft 3


Added water = = 40 gallons / yd 3
∴ ft 3 62.4 lb ft 3 1 yd 3
Answer: Add 40 gallons of water per cubic yard of compacted soil.

78
**Compaction-12: Fill volumes and truck load requirements for a levee.
(Revision: Aug-08)
Your company has won a contract to provide and compact the fill material for an earth levee, with
the dimensions shown below. The levee fill is a silty clay soil to be compacted to at least 95% of
maximum Standard Proctor of γd = 106 pcf at an OMC of 18%. Your borrow pit has a silty clay
with an in-situ moist density of 112.1 pcf at 18%, and a Gs = 2.68. When the soil is excavated and
loaded on to your trucks, the voids ratio of the material is e = 1.47. Your trucks can haul 15 cubic
yards of material per trip.
a) Determine the volume of fill required for the levee;
b) Determine the volume required from the borrow pit;
c) Determine the required number of truckloads.

Solution:
 1  1   450 ft long 
a) Volume of levee =   ( 20')( 40') + ( 20')( 20') +   ( 20')( 60')  ( ft 2 )  3  = 23,300 cy
 2  2   27 ft / cy 
b) To find the volume required from the borrow pit consider that the weight of solids is the same
in both, Ws (borrow) = Ws ( levee )
Ws
But γ d = or Ws = γ dV or γ d (borrow)Vborrow = γ d (levee)Vlevee
V
γ 112.1
where γ d (borrow) = = = 95 pcf and γ d (levee ) = ( 0.95)(106 ) = 100.7 pcf
1 + w 1 + 0.18
γ 100.7 pcf 
∴ Vborrow = Vlevee d (levee) = ( 23,300 cy )   = 24,700cy
γ d (borrow)  95 pcf 

79
Number of truck-loads required is based on,
Ws ( hauled ) = Ws ( levee )
γ d ( hauled )Vhauled = γ d ( levee )Vlevee
 γ d ( levee ) 
∴ Vhauled = Vlevee  
γ
 d ( hauled ) 
GS γ w ( 2.68 )( 62.4 )
but γ d ( levee ) = 100.7 pcf and γ d ( hauled ) = = = 67.7 pcf
1 + e 1 + 1.47
 γ d ( levee )   100.7 pcf 
Vhauled = Vlevee   = ( 23,300 yd 3 )   = 34, 700 yd 3
γ   67.7 pcf 
 d ( hauled ) 
 Vhauled   34, 700 yd 3 
Number of truck-loads =  =
   = 2,314
 truck capacity   15 yd / truck − load 
3

80
**Compaction-13: Multiple choice compaction problem of a levee.
(Revision: Aug-08)
A town’s new reservoir will impound its fresh water with a small earth dam, rectangularly shaped
in plan. The perimeter of the dam will be 2,200 ft long by 1,750 ft wide, and its cross-section is
shown below in figure B. The dam is to be built with a silty clay soil, with a specific gravity of 2.70,
available from three different local sources. Specifications call for a compacted soil at the dam
with a dry unit weight of 97 pcf at an OMC of 31 percent. Assume all voids are totally devoid of
any gas (or air). The borrow suppliers quoted the following:
Pit Price ($/yd3) Gs S (%) w (%)
A 1.05 2.69 65 22
B 0.91 2.71 49 22
C 0.78 2.66 41 18

Questions:

1. What is the dam’s cross-sectional area?


a) 675 ft2
b) 2,100 ft2
c) 2,350 ft2
d) 2,700 ft2
e) 2,550 ft2

81
2. What is the approximate total volume V of soil required for the dam?
a) 2,550 ft3
b) 300,000 yd3
c) 900,000 yd3
d) 1.22 Myd3
e) 0.75 Myd3

3. What is the approximate volume of water impounded (stored) in the dam?


a) 6,300,000 ft3
b) 134 acre-ft
c) 7 Mft3
d) 154 acre-ft
e) 45 · 106 gallons

4. What is the unit weight γ of soil of the compacted earth dam?


a) 127 pcf
b) 97 pcf
c) 86 pcf
d) 98 pcf
e) 128 pcf

5. What is the designed voids ratio e of the compacted soil in the dam?
a) 1.1
b) 0.92
c) 0.84
d) 0.79
e) 1.2

6. What is the voids ratio e of the material in pit A?


a) 0.98
b) 0.91
c) 1.02
d) 1.1
e) 0.72

7. What is the voids ratio e of the material in pit B?


a) 0.93
b) 1.22
c) 0.81
d) 1.01
e) 1.00

82
8. What is the voids ratio e of the material in pit C?
a) 1.10
b) 1.12
c) 1.08
d) 1.05
e) 1.17

9. Assume that the voids ratio e of pit A is 0.98. What is the equivalent volume required of pit A to
place 1.2 million cubic yards of compacted soil in the dam?
a) 1.29 Myd3
b) 1.20 Myd3
c) 0.95 Myd3
d) 0.97 Myd3
e) 0.96 Myd3

10. Assume that the voids ratio e of pit B is 0.81, what is the equivalent volume required of pit B to
place 1.2 million cubic yards of compacted soil in the dam?
a) 1.00 Myd3
b) 1.02 Myd3
c) 1.18 Myd3
d) 1.05 Myd3
e) 1.07 Myd3

11. Assume that the voids ratio e of pit C is 1.10, what is the equivalent volume required of pit C to
place 1.2 million cubic yards of compacted soil in the dam?
a) 1.34 Myd3
b) 1.37 Myd3
c) 1.25 Myd3
d) 1.23 Myd3
e) 1.21 Myd3

12. Which pit offers the cheapest fill?


a) Pit A
b) Pit B
c) Pit C
d) Both A and C
e) Both B and C

83
Chapter 5
Permeability of Soils
Symbols for Permeability

A→ area of a seepage surface.


C→ Hazen’s coefficient.
d→ Diameter of a capillary tube
Dx → Diameter of a soil % finer (represents % finer by weight)
e →The voids ratio.
GS → Specific gravity of the solids of a soil.
h→ Thickness of the aquifer.
H→ Thickness of the soil layer.
hC → Height of the rising capillary
i→ Hydraulic gradient.
k→ coefficient of permeability in D’Arcy’s equation .
kH → Coefficient of horizontal permeability..
kV → Coefficient of vertical permeability
L→ Distance of the hydraulic head loss.

Ludgeon→ Standard unit for permeability ( 10 −4 mm ).


sec
Nf→ The number of flow channels in Forheimer’s equation.
Neq→ The number of equipotentials drops in Forheimer’s equation.
po →In-situ vertical pressure at any depth .
q→ Flow rate (ft3 per second per foot of width).
Q→ Total seepage (total flow).
Sc → Seepage capacity.
TS → Surface tension (typically given as 0.073 N per meter).
u → pore water pressure.
umax → Maximum pore water pressure.
γS →Unit weight of solids.
γW →Unit weight of water .
σV→ Vertical effective stress.

84
*Permeability–01: Types of permeability tests and common units.
(Revision: Aug-08)
(a) When is it appropriate to use a constant-head permeability test versus a falling-head
permeability test?
(b) What are the “standard” units of permeability versus the “common” unit?
Solution:

(a) The constant head test is performed for granular soils (gravels G, and sands S), whereas the falling
head test is used for fine-grained (cohesive) soils (silts M, and clays C).

(b) In Europe, the “standard” unit is the Ludgeon (10- 4 mm/sec), and in the USA the standard unit is the
Meinzer, which is the rate of flow in gallons per day through an area of 1 square foot under a hydraulic
gradient of unity (1 foot/foot).

The “common” unit of permeability is cm / sec.

85
*Permeability-02: Use of Hazen’s formula to estimate the k of an aquifer.
(Revision: Aug-08)
A test boring was performed at an elevation 955 feet MSL, and it found the phreatic surface
(water table) 5 feet below the ground surface. An aquifer stratum was identified, and a sample of
its soil showed the grain size distribution below. Estimate the permeability using Hazen’s formula
with the coefficient C = 12. A piezometer (measures the location of the WT) was installed 2500 feet
downstream from the boring, and showed its phreatic surface at elevation 942 feet MSL. If the
thickness of the aquifer was a uniform 12 feet between both points, estimate the quantity of flow
per foot of width in gallons/hour (1 ft 3 ≈ 7.45 gallons ).
Solution:
mm
Allen Hazen’s (1893) formula for the permeability k = C ( D10 ) 2 = (12 ) (0.16mm)2 = 0.31
s
The hydraulic head drop is (955 ft – 5 ft) – (942 ft) = 8 feet
Applying D’Arcy formula,
∆h  mm 3600s  1in  1ft  7.45gal  8 ft  gallons
q = k A= 0.31     3  (12 ft)(1ft) =1.05 per ft width
L  sec  hour  25.4mm12in  1ft  2,500 ft  hour

D10

D10 = 0.16 mm

GRAIN SIZE DISTRIBUTION DIAGRAM

86
*Permeability-03: Flow in a sand layer from a canal to a river.
(Revision: Aug-08)
A canal and a river run parallel an average of 250 feet apart. The elevation of the water surface in
the canal is at +1050 feet and in the river at +1021 feet. A stratum of sand intersects both the river
and the canal below their water levels. The sand is 6 feet thick, and is sandwiched between strata
ft 3
of impervious clay. Compute the seepage loss q from the canal in if the permeability of
day − mile
ft
the sand is 2 x10-3 .
sec

250’
ELEV. +1050’

ELEV. +1021’

CLAY

6’
SAND

CLAY

Solution:
D’Arcy’s formula for q yields,

∆h ft  86, 400 s   (1050 − 1021) ft   5, 280 ft 


q=k A = ( 2 ) (10) −3   ( 6.0 ft )  
L s  day   250 ft 
  mile 
ft 3
q = 635, 000
day − mile

87
*Permeability-04: Find the equivalent horizontal permeability of two layers.
(Revision: Aug-08)
The topmost layer is loose, clean sand, 1 meter thick. Its vertical permeability kV can be estimated
using Hazen’s formula with C = 1.5 (to over-estimate) and the sieve analysis shown here. Its kH is
known to be approximately 500% of the kV. Below the sand stratum is a marine marl, 3 meters
thick, with a kV = kH = 10-6 m/s. What is the combined kHcomb for the upper 4 m in cm/sec?

Sand 1m
.

Marl 3m

Solution:
Use Hazen’s formula to find the permeability, where k is in cm/s if C ranges from 0.8-1.5. D10 is in mm.
Use the grain-size distribution curve of Permeability-02.

k H1 = 5kV1 = 5(CD102 ) = (5)(1.5)(0.16 mm) 2 = 1.92 cm / s = 1,920(10-6 ) m / s

The formula for combining several horizontal layers is,


k1 H1 + k2 H 2 (1,920 x10 m / s ) (1 m ) + (10 m / s ) ( 3 m )
−6 −6

k Hcomb = = = 4.8 ×10−4 cm / s


H1 + H 2 1m + 3 m

88
*Permeability-05: Equivalent vertical and horizontal permeabilities.
(Revision: Aug-08)
The soil profile shown below is typical of Miami-Dade County. Estimate the equivalent
permeabilities kV(eq) and kH(eq) in cm/sec, and the ratio of kH(eq) / kV(eq)..

1m Fine Sand k = 1x10-1 cm/sec Pamlico Formation

5m Porous Limestone k = 2x10-3 cm/sec Miami Formation

3m Fine Sand k = 1x10-3 cm/sec Fort Thompson Formation

12 m Upper sandy Limestone k = 2x10- 4 cm/sec Fort Thompson Formation

Solution:
H (1+ 5 + 3 +12) m
kV (EQ) = =
H1 H2 H3 H4 1m 5m 3m 12 m
+ + + + + +
k1 k2 k3 k4 10−1cm / s 2×10−3 cm / s 10−3 cm / s 2×10−4 cm / s

=
( 2,100 cm) = 0.32×10−3 cm / s
3
(6,551x10 )s
1 1
kH ( EQ) = [ H1k1 + H2k2 + H3k3 + H4k4 ] = (1)(10−1) + (5)(2×10−3 ) + (3)(10−3 ) + (12)(2×10−4 )
H ( 21 m)
kH ( EQ) = 4.8×10−3 cm / s
kH 4.8×10−3 cm / s
Therefore = = 15
kV 0.32×10−3 cm / s
The horizontal permeability is 15 times larger than the vertical permeability.

89
*Permeability-06: Ratio of horizontal to vertical permeabilities.
(Revision: Aug-08)
Estimate the ratio of the horizontal to the vertical permeability of these four strata.

H1 = 3’
-3
k1 = 10 cm/sec

k2 = 2x10-4 cm/sec H2 = 3’

k3 = 10-5 cm/sec H3 = 3’

k4 = 2x10-3 cm/sec H4 = 3’

Solution:
The equivalent horizontal permeability of all four layers is:

( ft )( cm/ s)  3 10−3 + 3 2 10−4 + 3 10−5 + 3 2 10−3 


( ) ( ) ( )( ) ( ) ( ) ( ) ( )( ) ( )
1
kH(eq) = (k1H1 + k2H2 + k3H3 + k4H4 ) =
H (12 ft ) 
kH(eq) = 8x10−4 cm/sec
The equivalent vertical permeability of all four layers is:

H 12 ft
kVeq = = = 0.37 x10 − 4 cm / sec
H1 H 2 H 3 H 4 3' 3' 3' 3'
+ + + −3
+ −4
+ −5 +
kV1 kV2 kV3 kV4 10 2 x10 10 2 x10 − 3

Therefore the ratio of the horizontal to the vertical permeability is:

k H eq 8 x1 0 − 4 c m / s
= = 22
k V eq 0 .3 7 x 1 0 − 4 c m / s

90
*Permeability–07: Do not confuse a horizontal with a vertical permeability.
(Revision: Aug-08)
The soil layers below have a cross section of 100 mm x 100 mm each. The permeability of each soil
is: kA =10-2 cm/sec.; kB =3 x 10-3 cm/sec; kC = 4.9 x 10-4 cm/sec. Find the rate of water supply in
cm3/hr.

∆h=300 mm
hA hB

A B C v

H1=150 mm H2=150 mm H3=150 mm

Solution:

This is a trick drawing: it “looks” like a horizontal flow, but in reality it is a vertical flow because the
flow has to cross through every layer; it can not “bypass” any layer. Therefore, every soil layer has the
same flow v = v1 = v2 = v3 and the total head ∆h = ∆h1 + ∆h2 + ∆h3.

= 1.2 (10 −3 ) cm / sec


H 450mm
kV ( eq ) = =
H1 H 2 H 3 150mm 150mm 150mm
+ + + +
k1 k2 k3 1x10 cm / s 3 x10 cm / s 4.9 x10−4 cm / s
−2 −3

∆h  300 mm   3, 600sec 
∴ q = keq iA = keq A = (0.0012 cm / s )   (10 cm )(10 cm )   = 291 cm / hour
3

H  450 mm   1 hr 

91
*Permeability-08: Permeability as a function of the voids ratio e.
(Revision: Aug-08)
The coefficient of permeability of fine sand is 0.012 cm/sec at a voids ratio of 0.57. Estimate the
increased permeability using the Kozeny-Carman formula of this same sand when its voids ratio
has increased to 0.72.

Solution:
Using the Kozeny-Carman formula,

e3
k=
(1 + e )
e13 ( 0.57 )
3

1 + e1
= 1 + 0.573 = 0.544
k 0.012
∴ 1 = =
( 0.72 )
3
k2 k2 e2
1 + e2 1 + 0.72
0.012
∴ k2 = = 0.022 cm / s
0.544
Notice that since k1 = 0.012 cm/s the permeability has almost doubled. A 26% increase
of the voids ratio has effected a doubling the permeability.

92
*Permeability–09: Uplift pressures from vertical flows.
(Revision: Aug-08)

The soil below is a dense well-graded clayey sand with γd = 112 pcf and Gs = 2.63, a permeability k
= 240 mm/min at a voids ratio of e = 0.85; the cross-sectional area of the tank is 36 ft2. Find (a) the
seepage rate q in ft3/min., and (b) the direction of the flow.

∆h = 4 feet

H1=2’

H2=6’ Clayey sand

Solution:
a) The hydraulic gradient i and the voids ratio e are:
∆h 4 ft Gsγ w (2.63)(62.4 pcf )
i= = = 0.667 and e= −1 = − 1 = 0.465
H 2 6 ft γd 112 pcf
The Casagrande formula relates the known permeability k0.85 at e = 0.85 to an unknown
permeability k at any voids ratio e,

 mm   1 in  1 ft 
k = 1.4e 2 k0.85 = 1.4(0.465) 2  240    = 0.239 ft / min
 min   25.4 mm  12 in 
Therefore, the seepage rate q is:

 ft  ft 3
q = kiA =  0.238  (−0.667)(36 ft ) = −5.74
2

 min  min

b) The flow direction is UP.

93
*Permeability-10: Capillary rise in tubes of differing diameters.
(Revision: Sept.-2008)
Determine the different heights hc that water will raise in three different capillary tubes, with
diameters:
d1 = 0.00075 mm (corresponding to a fine clay sized particle),
d2 = 0.075 mm (corresponding to the smallest sand sized particle) and
d3 = 0.75 mm (corresponding to a medium sand sized particle).
Assume that the surface tension is 0.075 N/m with an angle α = 3o.

Solution:
The surface tension of water Ts ranges from about 0.064 to 0.075 N/m (0.0044 to 0.0051 lb/ft). In this
problem we have chosen the largest value. Notice that the negative sign indicates that the water has risen
due to the capillary tension.

94
-4 ( T s ) ( c o s α ) − 4 ( 0 .0 7 5 N / m ) ( c o s 3 ° )
hc = =
dγ w d ( 9 .8 1 k N / m 3 )
− 4 ( 7 .5 x 1 0 − 2 N / m ) (c o s 3 ° ) = - 4 1
h1 = m
( 7 .5 x 1 0 m ) ( 9 .8 1 k N / m )
−7 3

− 4 ( 7 .5 x 1 0 N / m ) ( c o s 3 ° )
−2

h2 = = - 0 .4 1 m = 410m m
( 7 .5 x 1 0 m ) ( 9 .8 1 k N / m )
−5 3

− 4 ( 7 .5 x 1 0 N / m ) ( c o s 3 ° )
−2

h3 = = - 0 .0 4 1 m = 41 m m
( 7 .5 x 1 0 m ) ( 9 .8 1 k N / m )
−4 3

95
*Permeability-11: Rise of the water table due to capillarity saturation.
(Revision: Sept.-08)
How much does the capillary water rise above the water table in a very fine sand (d = 0.1 mm)
if the surface tension force is To = 0.064 N/m with an α = 3º?

Solution:

−4To cos α −4 ( 0.064 N / m ) cos 3°


hc = = = −0.26 m
dγ w (10 −4
m )( 9. 81 kN / m 3
)

Discontinuous moisture zone


Vapor flow

Capillary fringe zone

Capillary flow
Capillary saturation zone
hc

100% Saturation zone


z

96
*Permeability-12: Find the capillary rise hc in a silt stratum using Hazen.
(Revision: Aug-08)
Another method of determining the capillary rise in a soil is to use Hazen’s capillary formula. The
3 m thick dense silt layer shown below is the top stratum of a construction site, has an effective
diameter of 0.01 mm. What is the approximate height of the capillary rise in that silt stratum?
What are the vertical effective stresses at depths of 3 m and 8 m below the surface? The “free
ground water” level is 8 meters below the ground surface, the γS =26.5 kN/m3, and the soil between
the ground surface and the capillary level is partially saturated to 50%.

Dense silt
h1 = 3 m

Clay h2 = 5 m

Solution:
0.0306 0.0306
1. The Hazen empirical formula for capillary rise is hC = = = 15.3 m
0.2 D10 ( 0.2 )( 0.01 mm )
In essence, the entire silt stratum is saturated through capillarity.
2. For full saturation, S = 100%,
Se Seγ ω (1)( 0.40 )( 9.81)
Se = wGS ∴ w = = = = 0.148
GS γS 26.5
 1 + w  ( 26.5 )(1.148 ) kN
∴ γ SAT = γ S  = = 21.8 3
 1+ e  1.40 m
For 50% saturation,

w=
Seγ ω
=
( 0.5 )( 0.40 )( 9.81) = 0.074
γS 26.5
 1 + w  ( 26.5 )(1.074 ) kN
∴γ = γ S  = = 20.3 3
 1+ e  1.40 m
Therefore,
∴ (σ V ')3 = h1γ = ( 3 )( 20.3 ) = 61 kPa
∴ (σ V ')8 = h1γ + ( h2 ) γ SAT = ( 3 )( 20.3 ) + ( 5 )( 21.8 ) = 170 kPa

97
*Permeability-13: Back-hoe trench test to estimate the field permeability.
(Revision: Sept.-08)
A common method of determining a site’s drainage capabilities is the constant-head trench
percolation test shown below. The trench is dug by a backhoe to roughly the dimensions shown.
The testing crew uses a water truck to fill the trench with water above the WT, and then they
attempt to maintain the head constant for about 10 minutes. The amount of water that has flowed
out during the test is Q (in gallons/minute). The seepage capacity Sc = Q/CL’H, where C is a units
conversion factor, L’ is the trench semi-perimeter (length plus width, in ft) and H is the head. The
units of the seepage capacity are commonly given in cfs/ft/ft. Based on the reported geometric
conditions shown below, and that the crew used 1,540 gallons during a 10 minutes test, what is the
surface seepage capacity of that site?

0.0’
1.8’

4.4’

8.0’

Solution:
1,540 gal
Q = = 154 gallons / minute
10 min
The perimeter of the trench is L ' = 2 ( 6 ft + 1.5 ft ) = 15 ft and H = ( 4.4 ft − 1.8 ft ) = 2.6 ft
The seepage capacity of the surface stratum is Sc ,
Q 154 gallons / min ( ft 3 ) ( min )
Sc = = = 8.8 x10−3 cfs / ft / ft
C ⋅ L '⋅ H (15 ft )( 2.6 ft ) ( 7.45 gallons )( 60 sec )

98
**Permeability-14: Seepage loss from an impounding pond.
(Revision: Aug-08)
Borings were taken at the site of an intended impoundment pond and the in-situ voids ratios at
various depths are shown in the figure below. A constant-head permeability test was performed on
sample #1 (which was 6” high and 2” in diameter) subjected to a pressure head of 27”: after 5
seconds, 50 grams of water were collected through the sample.

Impoundment pond area = 4.4 miles2

ELEV
0’

5’ e = 0.750 (Sample 1)

∆h

10’ e = 0.217 (Sample 2)

Geomembrane

15’ e = 0.048 (Sample 3)

L
20’ e = 0.015 (Sample 4)

25’ e = 0.006 (Sample 5)

d) Determine the permeability of sample #1;


A trial test of a vibroflotation (densification) probe was taken to a depth of 5 feet and showed a
densified voids ratio e = 0.55. If this densification ratio is attained to the full depth of 25 feet, at
99
what depth (to the nearest 5 feet) would you place the bottom of the pond in order to keep the total
seepage Q below 50,000 gal/min?
Assume seepage only through the bottom of the pond, and that the pond is kept filled.

Solution:

Step 1: Determine the permeability k of sample # 1.


q 4q
From Darcy; Q= k i A, therefore k= =
iA iπ d 2

Since 50 grams of water is equivalent to 50 cm3 (for γw = 1g/cm3)

50cm3 cm3 ∆h 27in


but , q = = 10 , and i= = = 4.5
5sec sec L 6in

( 4 ) 10
cm 
 (10mm / cm)
∴ ka =
4q
=  in 
= 0.122
mm
iπ d 2
2 cm 
2
sec
4.5π ( 6in )  2.54 
 in 

Step 2: Determine the new permeability of the sand due to vibroflotation densification.

Using Casagrande’s relation k = 1.4k0.85 e 2

( 0.55 ) = 0.066 mm / sec


2
ea2
kb = ka * 2 = 0.122
( 0.75 )
Therefore 2
eb

Step 3: The ratio of densified permeability to in-situ permeability

kb 0.066
= = 0.54 ∴ Vibroflotation has reduced the permeability by half.
ka 0.122

Step 4: Find the densified permeability at each sample depth,

100
k (0.122mm / sec)
k = 1 .4 k 0 .8 5 e 2 therefore k0.85 = 2 = = 0.155 mm / sec
1.4e 1.4(0.752 )

The corresponding densified permeability

k = 1.4 k 0.85 e 2 (x ratio)

Depth (feet) Original eo Original k (mm/sec) Densified k (mm/sec)


0. 0. 0. 0.
-5 0.750 0.122 0.066
-10 0.217 0.010 0.005
-4
-15 0.048 0.005 2.76x10
-5 -5
-20 0.015 5.08x10 2.74x10
-5 -5
-25 0.006 1.03x10 0.56x10

Step 5: Estimate the required depth h of the pond.

Q = 50,000 gallons/min (1ft3 / 7.48 gal) = 6,680 ft3/min.

Consider this rate to be constant.

k ∆h Q
From Darcy’s equation Q = kiA or ki = =
L A

 ft 3  1mm 
 6, 680 
min   3.281× 10 ft 
 − 3
Q 
= = 2.79 × 10−4 mm
( ) (
( 4.4mile2 ) 5280 ft mile (1min ) 60 sec min )
sec
2
A

101
Consider the depth -15 ft.

∆h 15  −4 mm  −4
k =  2.76 x10  = 4.14 x10 mm / sec
L 10  sec 
or at a depth of: -20 ft.

∆h 20  −5 m m  −4 m m
k =  0 .5 6 x1 0  = 0 .2 2 4 x1 0
L 5  se c  se c

Therefore, place the bottom of pond at -19 feet.

Q
A
(× 10 −4 mm
sec
)
0
0 1 2 3 4 5 6

-5

-10
Depth
ft
-15

18.7 ft
-20

2.76

-25

-30

102
Chapter 6
Seepage and Flow-nets
Symbols for Seepage and Flow-nets

103
*Flownets-01: Correcting flawed flow-nets.
(Revision: Aug-08)
Do you recognize something wrong with each of the following flow-nets?

a) b)

filter

c)

Equipotential Lines
Flow Lines

Well

Solution:
a) Incorrectly drawn mesh, because two equipotential lines intersect each other (equipotential lines
and flowlines must intersect orthogonally to each other).

b) Incorrectly drawn mesh, because two flow-lines intersect each other (same as above).

c) The well should be at the center of the net (a sink or a source point).

104
*Flow-nets-02: A flow-net beneath a dam with a partial cutoff wall.
(Revision: Aug-08)
The completed flow net for the dam shown below includes a steel sheet-pile cutoff wall located at
the head-water side of the dam in order to reduce the seepage loss. The dam is half a kilometer in
width (shore to shore) and the permeability of the silty sand stratum is 3.5 x 10-4 cm/s. Find, (a) the
total seepage loss under the dam in liters per year, and (b) would the dam be more stable if the
cutoff wall was placed under its tail-water side?

15 m

∆h = 6.0 m

2.0 m
ψ1
10.0 m

ψ2
17.0 m ψ3
IMPERVIOUS STRATUM (CLAY OR ROCK)

Solution:
(a) Notice that ∆h = 6.0 m, the number of flow channels Nf = 3 and the equipotentials Neq = 10.
Using Forcheimer’s equation,
Nf  cm   m   3
q = k ∆h =  3.5 × 10−4 
−6
 (6.0 m)   = 6.3 ×10 m / sec/ per m of dam width
2

N eq  sec   100 cm   10 
Since the dam is 500 meters wide (shore-to-shore) the total flow Q under the dam is,

 103 liters   sec  million liters


Q = Lq = 500 m  6.3 × 10−6 m3 / sec   3   31.5 × 10
6
 = 100
 1 m  year  year
b) No. Placing the cutoff wall at the toe would allow higher uplift hydrostatic pressures to develop
beneath the dam, thereby decreasing the dam’s stability against sliding toward the right (down-stream).

105
*Flow-nets-03: The velocity of the flow at any point under a dam.
(Revision: 12 Oct.-08)
Using the flow net shown below, (1) determine the seepage underneath the 1,000 foot wide
concrete dam, and (2) the velocity at point “a” in feet/hour, where the height of the net’s square is
19 feet. The soil has a GS = 2.67, D10 = 0.01 mm. Overestimate the flow by using Hazen’s coefficient
C = 15 to determine the permeability k.

∆h = 30 feet
feet 5 feet

“a” 19’

Solution:
Find the permeability k using Hazen's formula:
mm
k = C ( D10 ) 2 = 15(10 −2 mm) 2 = 0.0015
sec
Using Forheimer's equation with flow lines N f = 5 and equipotentials N eq = 12,
Nf  mm   1 in   1 ft   3, 600 sec   5 ft 3
q = k ∆h =  0.0015     ( 30 '− 5')   = 0.185
N eq  sec   25.4 mm  12 in   1 hour   12  hr − ft of dam
 ft 2  ft 3
∴ Q = Lq = (1, 000 ft )  0.185  = 185
 hr  hr
The velocity at "a" has a flow q in only that channel, or q/5,
 0.185 ft 3 
q  5 hr
 ft
v= =  ≈ 0.002
A  (19 ft high )(1 ft wide )  hr
 
 

106
*Flow-nets-04: Flow through an earth levee.
(Revision: Aug-08)
In western Miami-Dade County, the Everglades are contained with levees. Levee #111 runs
North-South about 2 kilometers west of Krome Avenue and its cross section is show below.
Laboratory tests indicate that the permeability of the 80-year old levee is 0.30 m/day. What is the
volume of water lost through the levee along each kilometer in m3/day?

50 m 12 m 50 m

WT
2m C B

23 m 2:1 2:1

D
A E DRAINAGE

112 m 2.7 m

Cross-section of levee looking north.


Solution:

Using Forheimer’s equation,

 N   m   3 m3
Q = Lq = L  k ∆h f
  = (1, 000 m )  0.3  ( 23 m )   = 2, 070
 N eq   day   10  day

107
*Flow-nets-05: Finding the total, static and dynamic heads in a dam.
(Revision: Aug-08)
Find the seepage through the earth dam shown below in gallons/day if the sieve analysis shows the
D10 to be 0.17 mm, and the dam is 1,200 feet wide. What is the pressure head at the top of the
aquiclude and at mid-dam (point A)?
Number of flow channels N f = 3
Number of equipotential drops N eq = 7
mm
Using Hazen's formula k = CD 102 = 15 D102 = 15(0.17 mm ) 2 = 0.43
sec
Note: 8 ≤ C ≤ 15 for D10 mm; to overestimate flows, use C =15.

40’ rock toe

7
1 2 A3 4 5 6
0

25’ Clay (an aquiclude)

Solution:

 N   mm   1inch  1 ft  7.5 gallons   3   86,400sec 


Q = Lq = L  k∆h f  = (1,200 ft )  0.43      ( 40 ft )    
 Neq   sec   25.4 mm  12inches  1 ft 3
  7   1day 

gallons
Q = 18.8×106
day
 4.4 
At point "A" the dynamic pressure head is   ( 40 ft ) = 25.1 feet
 7 
2
The static head at "A" is approximately ( 40 ft ) = 26.7 feet
3
Therefore, the total head = static + dynamic = 25.1 ft + 26.7 ft = 51.8 feet

108
**Flow nets-06: Hydraulic gradient profile within an earth levee.
(Revision: Aug-08)
The cross-section of an earth dam 5,000 feet wide is shown below. Determine (a) the seepage flow
through the dam, in ft3 / minute, (b) the hydraulic gradient in square I, and (c) the pore pressures
along a trial failure surface along the line ED.

109
Solution:

(a) From graph D10 = 0.04 mm. Using Hazen’s relation, with C = 15 to overestimate the permeability of
the dam,

k = C ( D10 ) = (15 )( 0.04 mm ) = 0.024 mm / sec


2 2

 Nf   mm   1inch   1 ft   86, 400 sec  3


Q = Lq = L  k ∆h
  = ( 5, 000 ft )  0.024     ( 40 ft )  
 N eq   sec   25.4 mm   12 inches   day  9
ft 3
Q = 454, 000
day
(b) The gradient in square I is,

∆h 40/9
iI = = = 0.40
lI 11.2

(c) The pore pressures along ED are approximately,

at pore pressure
E 0 0

40
1 u= 40' - 2.5 ( 9 ) 62.4 = 1803 psf = 1.80 ksf

40
2 u= 40' - 3 ( 9 ) 62.4 = 1664 psf = 1.66 ksf

3 40
u= 40' - 4 ( 9 ) 62.4 = 1387 psf = 1.39 ksf

40
4 u= 40' - 5 ( 9 ) 62.4 = 1109 psf = 1.11 ksf

40
5 u= 40' - 6 ( 9 ) 62.4 = 832 psf = 0.83 ksf

40
6 u= 40' - 7 ( 9 ) 62.4 = 555 psf = 0.55 ksf

40
D u= 40' - 8 ( 9 ) 62.4 = 277 psf = 0.28 ksf

110
**Flow-net-07: Flow into a cofferdam and pump size.
(Revision: Aug-08)
A cofferdam is to be built in the middle of a bay to place the foundations of a tall television tower.
A plan area of the cofferdam is 30 m long by 10 m wide. A sample taken from the bay bottom was
subjected to a hydrometer analysis: 20 grams of bay bottom dry fines were mixed with 1 liter of
water. The specific gravity of the solids was found to be 2.65. The dynamic viscosity of water is 10-
2
Poise (dynes-sec/cm2) at 20oC. After 1 hour of precipitation, the hydrometer dropped 16 cm. The
soil is uniform in size, with 80 % passing the # 200 sieve.
e) What type of soil was the sample?
f) Will a large 3 m3 per minute pump be adequate to maintain a 1 m draw down below the bay
bottom? Use FS > 2.

Pump

CL

7m

1m

15 m

Clay stratum

111
Solution:

a) Use Stoke’s formula to find diameter of the bay bottom particles:

18η L  18η   18 x10−2 dynes − sec x cm3 x 16cm 


D10 = d = . =  
γ S − γ W t  γ W ( GS − 1) t   cm 2 ( 2.65 − 1) ( 9.81dynes x 3600sec ) 
 
d = 0.070 mm
Therefore the soil is silt (0.075 mm to 0.002 mm).

b) Hazen’s formula permits us to estimate the permeability k of the soil:

mm mm
k = CD102 = 15(0.07 mm ) 2 = 0.074
sec sec

Use Forheimer’s formula to estimate the total flow Q into the cofferdam:

 − 2 mm   4  m 
3
Nf −5 m
q = k ∆h =  7.4 x10  ( 8m )    3  = 29.6 x10
N eq  sec   8   10 m m  m−s
 −5 m
3
60 s  m3
Q = ( perimeter )( q ) = 80 m  29.6 x10 .  = 1.42
 m − s min  min
3 m 3 / min
Pump FS = = 2.11 > 2 OKAY Pump is adeq uate !
1.42 m 3 / min

112
*Flow-nets-08: Drainage of deep excavations for buildings.
(Revision: Aug-08)
A new office building will require a two-level underground parking garage. The plan size of the
site is 100 x 80 meters. Some of the soil properties are shown below.
a) At what depth of the excavation will the limestone (shear strength = 0.1 MN/m2) have a
punching shear failure? Suggest using a 1m x 1m plug as a model.
b) What size pump do you need (m3/minute) with a factor of safety of 3?

Pump

Elevation +0 m

kN
Sand γ = 1 7
m3
Elevation –1.0 m

Anchor

Limestone
kN
γ = 19
m3

Nf = 3 A Neq = 8 Elevation –7 m
0 8
F2 F1
F3

Elevation –9 m
1

2 7
3

4 5 6
113 kN
Sandstone γ = 21 Elevation –11 m
m3
Solution:
a ) T h e u p li f t f o r c e a t p o i n t A i s f o u n d b y ,
∑ Fy = 0 F u p l if t - F s h e a r r e s is ta n c e = 0

( 6 m )  9 .8 1
kN 
F u p l if t = u A = (∆ hγ w ) A =  (1 m )=
2
59 kN
 m3 
MN kN
F s h e a r = τ A = 0 .1 (4 x ) m 2 = 4 0 0 x
m2 m
59
∴ x = = 0 .1 4 7 m w i t h F S = 7 ∴ x =1m
400
T h i s c o r r e s p o n d s t o e le v a t i o n - 6 m .

b ) D e t e r m i n e t h e f lo w q u a n t i t y w i t h F o r h e i m e r 's f o r m u la ,
 a  N 
Q = qL = Lk     ∆ h
f

 b   N q 
mm
k = C D 120 = (1 2 .5 ) ( 0 .0 9 5 m m ) = 0 .1 1 3
2
w here
sec
∆h = 5m

( 3 6 0 m )  0 .1 1 3
mm  3   60   m 
   (5 m ) 
−3
∴ Q =  10  = 4 .6 m / m i n ≈ 5 m / m i n
3 3

 s e c  
8  m  m m 
3
m
F o r a F S = 3 u se a 15 pum p.
m in

114
*Flow-nets-09: Dewatering a construction site.
(Revision: Aug-08)
The figure below shows a dewatering plan to build the foundations of an office building below the
water table and without sheet-piling. The plan area of the excavation is 400 m long by 100 m wide.
The soil has a D10 of 0.02 mm. What size pump do you need (gpm) with a Factor of Safety = 2?

Dewatering wells

Marshy soils

∆h = 2m Excavated site CL

∆h = 2m

1 ∆h = 2m
2
3 ∆h = 2m Lowered WT

Neq = 4
Nf = 3

Solution:
N otice that ∆ h = 8 m , N f = 3, N eq = 4 and L = perim eter = 1, 000 m .
T he perm eability k = C ( D10 ) = (15 )( 0.02 m m ) = 0.006 m m / s
2 2

 N   mm   3  m   60 s  m3
Q = L q = L k ∆h  f
N  (
= 200 + 800 ) 
m 0.006 ( 8 m )   3   = 2.16
 eq   s   4   10 m m   1 m in  m in
3
 m3   ft   7.45 gal  g a l lons
Q =  2.16    3  = 60 0
 m in   0 . 30 m   ft  m in
Therefore, for a factor of safety of 2 use at least a 1,200 gallons per minute pump or
two 600 gallons per minute pumps.

115
*Flow-net-10: Dewatering in layered strata.
(Revision: Aug-08)
The figure below shows the profile of a square excavation (in plan view) in a layered soil, where
the vertical permeability is 5 x 10-5 m/s and the horizontal permeability is roughly ten times higher
than the vertical. Estimate the dewatering capacity requirements, in m3/hour, to prevent the
excavation from flooding. The value of ∆h is to scale, but you may use 10 m.

CL
40 m 40 m
10m
10m 80 m

Phreatic surface condition is CL


fulfilled along the top flow-line

GWL Sheet-pile wall

Datum for h
(½ square) h=0
h=1(1/3)m

h=2(2/3)m
h=4m

h=5(1/3)m

h=6(2/3)m
(½ square)
h=8m

h=9(1/3)m
(½ square)

(½ square)

Assumed recharged boundary h = 10 4m


Scale

116
Solution:

∆h = 10m , N f = 4 , N eq = 8

k = kxk y = ( 5 x1 0 −4
m / s )( 5 x1 0 − 5 m / s ) = 1 .6 (1 0 − 4 m / s )
T h e p erim eter o f th e co fferd am p = ( 4 )( 8 0 m ) = 3 2 0 m
  Nf    −4 m   4 
Q = qp = k ∆h 
   p =  (1 .6 )  1 0  (1 0 m )    ( 3 2 0 m )
  N eq     s   8 
 m 3   3, 6 0 0 s  m3
Q = 0 .2 5 3    = 911
 s   hr  hr

117
**Flownets-11: Flow through the clay core of an earth dam.
(Revision: Aug-08)
An earth dam on a pervious but strong earth foundation has the cross-section shown in the figure
below. The core of the dam is sealed from the jointed rock foundation with a thin layer of grout.
g) State the function and properties of the core shell and drains;
h) What is the function of the grout between the core and foundation? Under what conditions is it
most important?
i) Calculate the seepage quantity per foot of length of the dam through the dam, through the
foundation, and the total seepage quantity.
j) What grading requirements should be specified for the inclined filter A?
k) What minimum permeability k is required in the horizontal drain B to prevent saturation
from rising into the random fill zone? Give the results of k in ft/day.

Reservoir surface
inclined filter A
H=100 ft

10 ft

shell dam permeability k1= 0.001 ft/day

core
L = 150 ft
grout

h1=5 ft
horizontal drain B

Note: The grain size of core = 100% passes 1”, 15% size = 1/8”, and 85% size = 0.001 in.
Foundation layer permeability k2 = 0.1 ft/day

Solution:
(a) The core is used to retain water within the dam, that is, to resist seepage. The material should be
relatively impermeable (clay) and should not shrink or swell excessively.
The shell provides the structural strength to support and protect the core. The material must be
more permeable than the core material, strong and durable.

118
The drains are provided to reduce the pore water pressures in the foundation and in the
embankment to increase stability. The drains also remove seepage water to reduce soil erosion.
The drain material must be permeable enough to permit drainage with a low head loss and yet
fine enough to keep the adjacent soil in place.
(b) The primary function of the grout between the core and foundation is to form an impervious
layer which prevents seepage along the contact surface. This becomes most important when the
ratio k2/k1 becomes large.
(c) Calculate the seepage Q by using the flow net shown in the figure.
1) Through the dam: Q = k1(∆h/L)b in ft3/day/ft where b is the normal distance between
streamlines. The flow net divides the core into 4 zones (#1 at the bottom, #4 at the top).
4 4

Q = k1 ∑ ij bj = k1 ∑ (∆h)j (b)j
j=1 j=1

In zone #1, the flow net is nearly rectangular, so (b)1 = 2; for zone # 4, (b )j = 1
ℓ ℓ
The average head loss hl across the core in each zone
(p) + Zu = constant = 100’ on the upstream face
γ
On down stream face of core (p) = 0 is assumed in the drain,
γ
so that ZL + hL = constant = 100’ on the downstream face

Using an average ZL for each zone by scaling,

Zone#1 ZL = 2’ hL = 98’ ∆hL = 98


6 6
Zone#2 ZL = 10’ hL = 90’ ∆hL = 90
5.5 5.5
Zone#3 ZL = 25’ hL = 75’ ∆hL = 75
4.5 4.5
Zone#4 ZL = 55’ hL = 45’ ∆hL = 45
3 3
4

Q1 = k1 ∑∆hj (b/L)j = 0.001 [98’(2) + 90’ + 75’ + 45’] = 0.081 ft3/day/ft of dam
j=1
6 5.5 4.5 3

119
2) Through the foundation Q2 = NF kh
Nd

where NF = number of flow paths, Nd = number of equipotential drops and


h = total head dissipated.

Q2 = 3 (0.1)(100) = 3.75 ft3 /day/ft of dam


8

3) The total seepage Q is therefore,


Q = Q1 + Q2 = 0.081 + 3.75 = 3.83 ft3/day /ft of dam

(d) The grading requirements for the inclined filter A,


(1) Free drainage, require D15 (filter) ≥ 4 D15 (soil)
D15 (filter) ≥ 4 (0.001 in)
≥ 0.004 in
(2) To prevent erosion of the core material requires D15 (filter) ≥ 4 D85 (soil)
≥ 4 (0.001 in)
≥ 0.004 in
So 85% of the filter material must be coarser than 0.01” to 0.2”. The filter grain size grading
curve should be parallel to or flatter than the core material grading curve. See the graph on the
next page for one possible grading curve, which gives
100% passes = 10 inch
15% passes = 2 inch
85% passes = 0.01 inch

(e) The drain B must carry the total seepage flow Q = 3.83 ft3/day/ft of dam calculated above. The
Dupuit formula for two-dimensional flow on a horizontal impervious boundary is

Q = k (h12-h22)
2L

where Q = 3.83ft3/day/ft, L = 150 ft, h1= 5 ft and h2 < 5 ft.

120
At what value of h2 will it minimize k? Clearly it is when h2 = 0, although this does seem unrealistic
since we are saying that the flow at the lower end of the drain has zero depth. Nevertheless, it gives us a
minimum value, which is:

kmin = 2LQ = 2(150)(3.83) = 46 ft/day


h1 2 52

121
Chapter 7
Effective Stresses and Pore Water Pressure
Symbols for Effective Stresses and Pore Water Pressure

122
*Effective Stress–01: The concept of buoyancy.
(Revision: Aug-08)
What force is required to hold an empty box that has a volume of 1 cubic foot, just below the
water surface?

Solution:

The volume of the displaced water is 1 ft3.

Therefore, the force is the weight of 1 ft3 of water = 62.4 lbs / ft3.

What is the force required to hold the same box 10 feet below the surface?

123
*Effective Stress–02: The concept of effective stress.
(Revision: Aug-08)
A sample was obtained from point A in the submerged clay layer shown below. It was determined
that it had a w = 54%, and a Gs = 2.78. What is the effective vertical stress at A?

hw = 25 m
water

Saturated clay
hs = 15 m
A

Solution:

The effective stress σ’ at the point A consists solely of the depth of the soil (not of the water)
multiplied by the soil buoyant unit weight.

σ ' = γ ' hsoil where γ ' = γ b = γ SAT − γ W


In order to find γ ' there are a number of derivations, such as this one,
 ( GS + e ) γ W 
γ' =  − γ W where the voids ratio e can be found through Se = wGS
 1 + e 
and noticing that S = 1 because the soil is 100% saturated, e = wGS = ( 0.54 ) (2.78)
  ( GS + e ) γ W     2.78 + ( 0.54 ) (2.78)  (9.81) 
σ ' = γ ' hsoil =  − γ
 W  soil 
h = − 9.81  (15 m )
  1 + e    1 + ( )
0.54 (2.78) 
σ ' = 105 kPa

124
*Effective Stress–03: The concept of effective stress with multiple strata.
(Revision: Aug-08)
The City of Houston, Texas has been experiencing a rapid lowering of its phreatic surface (draw-
downs) during the past 49 years due to large volumes of water pumped out of the ground by
industrial users.
a) What was the effective vertical stress at a depth of 15 m in 1960?
b) What is the effective stress at the same depth in 2009?
c) What happens to the ground surface as a result of the draw-downs?

Solution:

a)
σ V' = [γ h + γ ' h ' ]S A N D + [γ ' h ' ]S IL T + [γ ' h ' ]C L A Y w h ere γ ' = γ SAT − γ W
σ V' =  ( 2 0 .4 )(3) + (1 8 .8 − 9 .8 1)(3 )  + [ (1 4 .9 − 9 .8 1)(6 ) ] + [ (1 2 .6 − 9 .8 1)(3) ]
σ V' = 1 2 8 kP a
b) σ V' = (20.4)(6) + (16.5)(6 )  + [ (12.6 − 9.81)(3) ] = 230 kPa
This is an 80% increase in stress due solely to a dropping water table.

c) The ground surface has also been lowered, due to the decreasing thickness of the sand and the
silt strata due to their loss of the volume previously occupied by the water.

125
Effective Stress-03B
Revision
In the soil profile shown below, show a plot of the pore water pressure and the effective stress
along the right margin of the figure, with numerical values at each interface. Pay heed to the
capillarity in the upper clay. Assume S = 50%.

Solution:
∆h = (62.4) x (4) = -250
∆h = (62.4) x (6) = 562
∆h = (110.7) x (6) = 664
∆h = {[(110-62.4) x (4)] + 664} = 854
∆h = {[(117-62.4) x (9)] + 854} = 854
∆h = 664 – 250 = 414
∆h = 0 + 854 = 854
∆h = 562 + 1346 = 1908

Depth (ft) u + σ' σ


0 0 0 0
6 -250 664 414
10 0 854 854
19 562 1346 1908

126
Chapter 8
Dams and Levees
Symbols for Dams and Levees

127
*Dams-01: Find the uplift pressure under a small concrete levee.
(Revision: Sept.-08)
Calculate the uplift force at the base of the weir, per foot of width. Points A and B are at the
corners of the concrete levee.

Solution:
The dynamic head drop per equipotential is,
HA − HB 30 ft − 5 ft
∆ (∆h) = = = 1.8 ft / drop
N eq 14 drops
Pressure head at A = 30 ft + 8 ft - ∆ h (1.5) = 35.3 ft
Pressure head at B = 30 ft + 8 ft - ∆ h (10) = 20.1 ft
The uplifting force F is,
 p + pB   35.3 + 20.1  lb lb kip
F = L A γ w  = 98 '   ( 62.4 ) 3 = 169, 000 = 16 9
 2   2  ft ft ft
128
*Dams-02: Determine the uplift forces acting upon a concrete dam.
(Revision: Aug-08)
The uplift (hydrostatic) force under the concrete gravity dam shown below varies as a straight line
from 67% of the headwater pressure at the heel, to 100% of the tail-water at the toe. Assume γ
Concrete = 145 pcf

a) Determine the Factor of Safety against overturning; and


b) Determine the FS against sliding, if the sand that underlay the dam has ϕ = 3 7 .
o

129
Solution:

Step 1: Determine all forces on the dam

ΣFV = 0
 k  1 
V1 = weight of dam =  0.145 3  (300 ft )(40 ft ) +  (260 ft )(300 ft ) = 7,395 kips / ft
 ft  2 
∑M0 = 0
(1740 k )(280 ft ) + (5655 k ) 2 (260 ft )
x1 , from toe O = 3  = 198 ft left of toe
(1740 k + 5655 k )

k 1
V2 : vertical weight of water upon toe sec tion = 0.0624 3 
(60 ft )(52 ft ) = 97 kips / ft
ft  2 
x 2 , from toe O =
1
(52 ft ) = 17.3 ft left of toe
3
1 k 
H 1 : lateral force from headwater =  0.0624 3 (285 ft ) = 2,534 kips / ft
2

2 ft 

y1 , from toe O = (285 ft ) = 95 ft above the toe.


1
3
1 k 
H 2 : lateral force from tailwater =  0.0624 3 (60 ft ) = 112 kips / ft
2

 2 ft 

y 2 , from toe O = (60 ft ) = 20 ft above the toe.


1
3

Step 2: The hydrostatic uplift at the base of the dam.

1
V3 : uplift force = ( pLEFT + pRIGHT )( 300 ft ) = 2, 534 kips / ft
2

where the pressure pLEFT = ( 0.67 ) γ w h = 0.67 ( 0.0624 )( 285 ft ) = 11.9ksf

and the pressure pRIGHT = (1.00 ) γ w h = 1.0 ( 0.0624 )( 60 ft ) = 3.7 ksf

1 k
∴ V3 = (11.9 + 3.7 ) 2 ( 300 ft ) = 2,340 kips / ft
2 ft
 k   300 ft  1  k  2 
 3.7 2 (300 ft )  + 11.9 2 − 3.7 2 (300 ft ) (300 ft )
k
 2  2 3 
x3 , from toe O = 
ft  ft ft 
= 176.3 ft left of toe
 k  1 k 
 3.7 2 (300 ft ) + 11.9 2 − 3.7 2 (300 ft )
k
 ft  2 ft ft 

130
Step 3: The factor of safety (FS) against overturning (taken about the toe),

resisting moments V x + V2 x 2 + H 2 y 2
FS = = 1 1
overturning moments V3 x3 + H 1 y1

FSoverturning =
( 7395 )(198 ) + ( 97 )(17.3) + (112 )( 20 ) = 2.2 > 2 GOOD
( 2534 )( 95 ) + ( 2340 )(176.3)

Step 4: The F.S. against sliding,

resisting forces H 2 + (V1 + V2 − V3 ) tan φ


FS sliding = =
driving forces H1

FS sliding =
(112 ) + ( 7395 + 97 − 2340 ) tan 37° = 1.58 < 2 NOT GOOD ENOUGH
2534

131
Chapter 9
Stresses in Soil Masses
Symbols for Stresses in Soil Masses
θ→ The angle of the plane of interest angle with respect to the major principal stress (σ1).
σmax→ Maximum normal axial stress.
σmin→ Minimum normal axial stress.
σθ→ The normal stress on a plane with an angle θ with respect to the major principal stress plane (σ1).
τmax→ maximum shear stress.
τminx→ Minimum shear stress.
τθ→ The shear stress on aplane with an angle θ respect to the major principal stress plane (σ1).
ф → The angle of internal friction of the soil.
γ→ Unit weight of the soil.
τn→ Shear stress.
σn→ Normal stress.
qu→ Ultimate shear strength of a soil.
Symbols for Boussinesq Stresses
B→ Width of the loaded selected region.
GS→ Specific gravity of the solids of a soil.
L→ Length of the loaded selected region.
m→ The ratio (B/Z).
n→ The ratio (L/Z).
N→ Normal load carried by a foundation.
Dp→ Increased stress on the soil from a surface loaded area.
p→ Stress of the loaded area.
z→ Depth of the soil at the point of interest.
γ→ Unit weight of the soil.
Symbols for Newmark
IV → The influence value in the Newmark’ chart (for example, a chart divided into 100 areas, each is IV=0.01.
AB→ Scale to the depth of interest to determine the size of the surface structure graph the Newmark’s graph.
C→ consolidation.
M→ Nomber of squares (enclosed in the Newmark’ chart).
po → The effective stress at the point of interest.
q→ The load of the footing.
qu→ Ultimate shear strength of a soil.
σx→ stress at an specific point (x).
z→ Depth of the stratum stat of the soil.

132
133
134
Boussinesq Isobars for a Square Footing (left) versus a Strip Footing (right).

135
Boussinesq Influence factor I1 (the influence from a point load on a surface
upon a point inside a semi-infinite body).

 
P  3 1  P
∆pz = 2   = 2 I1
 2π ( r / z ) + 1  z
5/ 2
z 2

   

136
Vertical Stress caused by a Line Load.

Vertical Stress caused by a Strip Load.

137
Boussinesq Influence factor I4 (the influence of a rectangular loaded area upon a point
A beneath a corner of the rectangular area).

138
Boussinesq Influence factor I4 (alternative).

139
Vertical Stresses induced by a uniform load upon a circular area.

140
*Stress-01: Reading the Boussinesq charts.
(Revision: Feb-10)
Case-1: A single column with a load of 1,000 kips is supported by a square footing 10 feet by 10
feet in plan view.
Case 2: A 10 feet wide continuous (wall) footing that carries 10 kips per linear feet.
Find the stress increase in a soil mass at the following depths:
(a) 5 feet and 10 feet below the center of the footing,
(b) 5 feet and 10 feet below the edge of the footing, and
(c) 5 feet beyond the edge of the footing, both 5 and 10 feet below the ground surface:

Solution.
Use the charts on pages 202 through 204 to read the answers.
Case 1: The stress at the square footing’s invert is the column load divided by the footing area.
Q 1, 000 kips
σ= = = 10 ksf
B2 100 ft 2
This is 100% of the stress at the footing invert. It decreases as the stress spreads out in all directions
below the footing.
σ = ( 0.6 )(10 ksf ) = 6.0 ksf
a) The stress at 5 feet below the center of the footing is
σ = ( 0.32 )(10 ksf ) = 3.2 ksf
b) The stress at 10 feet below the center of the footing is
σ = ( 0.4 )(10 ksf ) = 4.0 ksf
c) The stress at 5 feet below the edge of the footing is The stress at 10
σ = ( 0.25 )(10 ksf ) = 2.5 ksf
feet below the edge of the footing is
d) The stress at 5 feet beyond the edge of the footing, and 5 feet below the surface is
σ = ( 0.07 )(10 ksf ) = 0.7 ksf
The stress at 5 feet beyond the edge of the footing, and 10 feet below
σ = ( 0.1)(10 ksf ) = 1.0 ksf
the surface is
Case 2: The stress at the footing’s invert is simply 1 ksf. Therefore,
σ = ( 0.82 )(1 ksf ) = 0.82 ksf
a) The stress at 5 feet below the center of the footing is The stress at 10
σ = ( 0.54 )(1 ksf ) = 0.54 ksf
feet below the center of the footing is
σ = ( 0.4 )(1 ksf ) = 0.4 ksf
b) The stress at 5 feet below the edge of the footing is The stress at 10 feet
σ = ( 0.4 )(1 ksf ) = 0.4 ksf
below the edge of the footing is
c) The stress at 5 feet beyond the edge of the footing, and 5 feet below the surface is
σ = ( 0.12 )(1 ksf ) = 0.12 ksf
d) The stress at 5 feet beyond the edge of the footing, and 10 feet below the surface is
σ = ( 0.18 )(1 ksf ) = 0.18 ksf

141
*Stress-02: 2:1 Method to find the Stress below a Rectangular Footing.
(Revision: Feb-10)
A rectangular footing carries a single column and measures B = 10 feet wide by L = 14 feet long.
The total load of the footing and column results in a uniform load of 10,500 pounds per square feet
at the footing’s invert.

Determine the stress increase in the soil mass from this footing at a depth of z = 25 feet below the
footing’s invert.

Solution.
The simple 2:1 method assumes that the stress is reduced from 100% at the footing’s invert to
decreasing values at increasing depths due to the spreading out of the stress over an increasing area,
based on the 2 vertical: 1 horizontal geometry.

The stress at z= 25 feet is given by,

∆σ z =
P
=
(10 ,500 psf )(10 ft )(14 ft )
( B + z )( L + z ) (10 ft + 25 ft )(14 ft + 25 ft )
= 1, 080 psf versus 10 , 500 psf at the footing' s invert .

142
*Stress-03: Summation of vertical and line loads using Boussinesq.
(Revision: Feb-10)
A rail gantry crane has been installed close to an existing water main (point A). A pressure cell
placed at point A measured a vertical pressure increase of 35 psf due to the two line loads and the
point load P = 50 kips. What is the magnitude of the line load q2 (lb/ft)?

P = 50 k Line load = q2
Line load = 750 lb/ft

?s z
A

Solution.
The pressure increase at “A” due to the concentrated load P1 is found by Boussinesq,

3P z 3 3 ( 50, 000 lb ) ( 3 ft )
3

∆σ 1 = = = 0.56 psf
2π L5 2π
( )
5
16 + 3
2 2

The pressure increase at “A” due to the line load q1 is also found by Boussinesq,

2 ( 750 lb / ft )( 3 ft )
3
2q1 z 3
∆σ 2 = = = 0.55 psf
π ( x2 + z 2 )
2 2
π (12 ft ) + ( 3 ft ) 
2 2
 
Similarly, the pressure increase at “A” due to the line load q2 is,

2q2 ( 3 ft )
3
2q2 z 3
∆σ 3 = = = 0.0275q2 psf
π ( x2 + z 2 )
2 2
π ( 4 ft ) + ( 3 ft ) 
2 2
 
Use the Law of Superposition to add these three pressures; they must equal 35 psf,
∆σ 1 + ∆σ 2 + ∆σ 3 = 0.56 + 0.55 + 0.0275q2 = 35 psf
∴ q2 = 1, 232 psf

143
*Stress-04: The influence of point loads on a culvert.
(Revision: Feb-10)
The truck parked over the drainage culvert box has the rearmost axle directly over the center of
the box. The next axle is 48 inches to the front of the rearmost axle. Each axle has four (4) tires
and weighs 18,000 lbs. Calculate the stress contributed by each tires of the truck.

Solution.

The vertical stress increase from each tire can be found using the Boussinesq point load formula and/or
its influence factor I1 (table on page 205). The formula is shown below, with the data from Tire #2 as an
example,

   
   
P 3 1  4, 500lb  3 1 
∆σ V-truck = 2 5/ 2 
=   = 95.0 psf
z  2π  r 2   ( 2 ft )  2π
2 5/ 2
 2 ft 2  
   + 1     + 1 
  z      2 ft   

144
All eight (8) tires will contribute an increase in the vertical stress, as shown in the following table,
Tire # x (ft) y (ft) r (ft) z (ft) r/z P (lbs)
1 1 0 1 2 0.50 4500 307.5
2 2 0 2 2 1.00 4500 95.0
3 8 0 8 2 4.00 4500 0.451
4 9 0 9 2 4.50 4500 0.258
5 1 4 4.123 2 2.06 4500 8.506
6 2 4 4.472 2 2.24 4500 6.092
7 8 4 8.944 2 4.47 4500 0.266
8 9 4 9.849 2 4.92 4500 0.168
Total stress 418 psf

The total vertical stress increase from the truck tires is 418 psf.

145
*Stress–05: Stress increase at a point from several surface point loads.
(Revision: Feb-10)
Point loads of 2000, 4000, and 6000 lbs act at points A, B and C respectively, as shown below.
Determine the increase in vertical stress at a depth of 10 feet below point D.

A 10 feet B

10 feet

C 5 feet D

Solution.

Using the Boussinesq (1883) table on page 202 for vertical point loads, the vertical increase in stress
contributed by each at a depth z =10 feet is found by,

 
P  3 1  P
∆pz = 2  5/ 2 
= 2 I1
z  2π  ( r / z ) +1  z
2

   
Increase in P
r (ft) z (ft) r/z I1
∆p
the load at: (lbs) (psf)
(102+52) 1/2 =
∆p from A 2,000 10 1.12 0.0626 1.25
11.18
(102+52) 1/2 =
∆p from B 4,000 10 1.12 0.0626 2.50
11.18

∆p from C 6,000 5 10 0.50 0.2733 16.40

Total = 20.2 psf

Therefore, the vertical stress increase at D from the three loads A, B and C is 20.2 psf.

146
*Stress-06: Find the stress under a rectangular footing.
(Revision: Feb-10)
Determine the vertical stress increase in a point at a depth of 6 m below the center of the invert of
a newly built spread footing, 3 m by 4 m in area, placed on the ground surface carrying a
columnar axial load of N = 2,000 kN.
Solution:

The Boussinesq solution for a rectangular loaded area only admits finding stresses below a corner
of the loaded area. Therefore, the footing must be cut so that the load is at a “corner” (shown as
the quarter of the area), where the reduced footing dimensions for the shaded area are B1 = 1.5 m
and L1 = 2.0 m.

N = 2,000 kN

B=3m

L=4m

Depth = 6 m

B1 1.5m L 2.0m
m= = = 0.25 and n = 1 = = 0.33
z 6.0m z 6.0m
Use the table and extrapolate and find I 4 = 0.0345
 N   2, 000 kN 
∆qz = qo (4 I 4 ) =   (4 I 4 ) =  
 ( 3 m )( 4 m )  ( 4 )( 0.0345 ) = 23 kN / m 2
 BL   

147
*Stress-07: The effect of the WT on the stress below a rectangular footing.
(Revision: Feb-10)
Find the effective stress increase in the soil at a depth of 4 m below the footing, and then find the
increase in the stress due to a drop of the WT from originally 1 m below the footing to 5 m below
the footing.

N = 4,500 kN

B=3m

WT

L=5m WT-1

Depth = 4 m
WT-2
Solution:
N 4,500 kN kN B 1.5 m L 2.5 m
q= = = 300 m= = = 0.375 and n = = = 0.625
A 15 m 2 m2 z 4m z 4m
∴ I 4 = 0.076
a) The total stress increase from the footing is,
∆po = q(4 I 4 ) = ( 300 )( 4 )( 0.076 ) = 91.2 kN / m 2
and the effective stress when the WT is 1 m below the footing is,
∆po' = ∆po − u = ( 91.2 ) − ( 3 m )( 9.8 ) = 61.8 kN / m 2

b) When the WT drops from -1 m to -5 m below the footing, the effective stress is
identical to the total stress. Therefore the effective stress increase is,
∆po = 91.2 kN / m 2 which is a 48% increase in stress.

148
*Stress–08: Finding the stress outside the footing area.
(Revision: Feb-10)
Find the vertical stress increase ∆p below the point A at a depth z = 4 m.

Solution:

The stress increase, ∆p , can be written as : ∆p = ∆p1 − ∆p2


where ∆p1 = stress increase due to the loaded area shown in (b).
∆p2 = stress increase due to the loaded area shown in (c).

For the loaded area shown in (b):


B 2 L 4
m = = = 0.5 and n = = = 1.0
Z 4 Z 4
∆p1 = qI 4 = (150)(0.12) = 18 kN / m 2
Similarly, for the loaded area show in (c):
B 1 L 2
m = = = 0.25 and n = = = 0.5
Z 4 Z 4
∆p2 = (150 )( 0.048) = 7.2 kN / m 2
Therefore, ∆p = ∆p1 − ∆p2 = 18 − 7.2 = 10.8 kN / m 2

149
*Stress-09: Stress below a footing at different points.
(Revision: Sept.-08)
A clay sanitary pipe is located at a point C below the footing shown below. Determine the increase
in the vertical stress ∆p at the depth of the pipe, which is z = 5 feet below the footing invert, and 3
feet away from its edge. The footing has a uniformly distributed load q = 1,800 psf.

q = 1,800

. .
psf

. . .
B A

.
5 ft 4 ft 5 ft

2 ft B
A C
C
10 ft 3 ft 10 ft 3 ft

PLAN VIEW SECTION


VIEW
Solution:
For the expanded 5’ x 13’ area,
B 5 L 13
m= = = 1 and n= = = 2.6 therefore, I 4 = 0.200
Z 5 Z 5
B 3 L 5
For virtual 3’ x 5’ area m= = = 0.6 and n= = = 1 therefore, I 4 = 0.136
Z 5 Z 5
The increase in stress at point C from the footing is therefore,
 lb 
∆p = q ( I 4 - I 4' ) =  1,800 2  (0.200 - 0.136) = 115 psf
 ft 

150
*Stress-10: Stress increase from a surcharge load of limited width.
(Revision: Aug-08)
Calculate the stress increase at the point A due to the new road embankment.
∆p2
25m

15 ft

1 1
∆p 1 1 ∆p 3
1

Z
15 ft

The contribution from the central portion of the fill is ∆p2 whereas the contribution from the left and right
hand slopes are ∆p1 and ∆p 3 respectively. Using Boussinesq,

Solution: The contribution from the central portion of the fill is ∆p2 , whereas the contribution from the
left and right hand slopes are ∆p1 and ∆p 3 respectively. Using Boussinesq,

2x1 2(15') 2z 2(15') ∆p1  lb 


∆p1 ⇒ = =2 ∴ = =2 = 0.25 ∴∆p1 = (0.25)(15') 120 3  = 450 psf
B1 15' B1 15' q  ft 
-------
2x2 2(−12.5') 2z 2(15') ∆p 2  lb 
∆p2 ⇒ = = −1 ∴ = = 1.2 = 0.47 ∴∆p 2 = (0.47)(15') 120 3  = 846 psf
B2 25' B2 25' q  ft 
--------
2x3 2(40') 2z 2(15') ∆p 3  lb 
∆p3 ⇒ = = 5.3 ∴ = =2 = 0.02 ∴∆p 3 = (0.02)(15') 120 3  = 306 psf
B3 15' B3 15' q  ft 
∆p = ∆p1 + ∆p 2 + ∆p 3 = 450 + 846 + 36 = 1, 332 psf

151
*Stress-11: Finding a stress increase from a surface load of limited width.
(Revision: Feb-10)
Determine the average stress increase below the center of the loaded area, between z = 3 m and z =
5 m.
q = 100 kN/m2

z L

.
.
B
1.5 m 1.5 m
3m 3m
5m A,

.
A
3m

SECTION PLAN VIEW


VIEW

Solution:
The stress increase between the required depths (below the corner of each rectangular area) can be given
as:
 ( H )( I ( H )) − ( H1 )( I 4 ( H1 ))   (5)( I 4 ( H 2 )) − (3)( I 4 ( H1 )) 
∆pavg ( H 2 / H1 ) = q  2 4 2  = 100  
 H 2 − H1   5−3

For I4(H2): m′ = B / H2 = 1.5 / 5 = 0.3


n′ = L / H2 = 1.5 / 5 = 0.3
For m′ = n′ = 0.3, I4(H2) = 0.038

For I4(H1): m′ = B / H1 = 1.5 / 3 = 0.5


n′ = L / H1 = 1.5 / 3 = 0.5
For m′ = n′ = 0.5, I4(H1) = 0.086
Therefore:
(5)(0.038) − (3)(0.086)
∆ p av(H 2 /H 1) = 100 × = 3 .4 kN /m 2
5−3
The stress increase between z = 3 m and z = 5 m below the center of the load area is equal to:
4∆pavg ( H 2 / H1 ) = (4)(3.4) = 13.6 kN / m 2

152
**Stress-12: Stress increase as a function of depth.
(Revision: Feb-10)
The vertical stress σv in a soil at any depth below the surface can be estimated as a function of the
soil unit weight γ by the equation,
Z 100
σv = ∫ γ (σ ) dz
0
v = ∫ (95 + 0.0007σ
0
v ) dz

If a particular stratum has a function γ = 95 + 0.0007 σv , where γ is in pcf and σv is in psf, find the
vertical stress at a depth of 100 feet below the surface.

Solution:
Rearranging, and integrating by parts,
σv
dσ v
100

∫0 ( 95 + 0.0007σ V )
= ∫ dz
0

1
ln ( 95 + 0.0007σ V )0 V = z100
σ
0
0.0007
σ v = 135,800 ( e0.0007 z − 1)
100
= 9,840 psf
0

At Z = 100 σ v = 135,800 (1.0725 − 1)

∴ σ v = 9,840 psf
(Revision: Sept.-08)

153
*Newmark–01: Stress beneath a tank at different depths.
(Revision: Feb-10)
A construction site has a surface layer of aeolic sand 2 m thick, underlain by a 10 m thick clay
stratum. The project involves placing a wastewater treatment tank, 10 m x 10 m in plan, with a
contact pressure po = 400 kN/m². Find the stress down the centerline of the tank at the top and the
bottom of the clay stratum using Newmark’s influence chart shown below.

Solution:

BB == 1100 m
m
B = 10 m for point #1
SSA
ANND
D 22 m
m
1

C
CLLA
AYY 1100 m
m

B = 10 m for point # 2
2
x
AB = 2 m A B
AB = 12 m

Influence Value (IV) = 1/200 = 0.005

The increase in the vertical stress is found from ∆p = po M (IV) where M is the number of “squares”
enclosed in the Newmark chart and (IV) is the influence value.

For ∆p1, AB = 2 m  ∆p1 = po M(IV) = (400 kN/m²) (190) (0.005)  ∆p1 = 380 kN/m²

For ∆p2, AB = 12 m  ∆p2 = po M(IV) = (400 kN/m²) (42) (0.005)  ∆p2 = 84 kN/m²

(This decrease in stress represents a 78% reduction in the vertical stress).

154
*Newmark-02: The stress below the center of the edge of a footing.
(Revision: Aug-08)
Find the stress at the point A shown below, at a depth of 3 m below the edge of the footing. The
plan of the square footing has been plotted on top of the Newmark graph to a scale of AB = 3m
and placed in such a way that point A falls directly over the center of the chart.

Solution:

The number of elements inside the outline of the plan is about M = 45. Hence,

 6 6 0 kN 
∆ p = q M ( IV ) =   ( 4 5 )( 0 .0 0 5 ) = 1 6 .5 k N / m
2

 ( 3 m )( 3 m ) 
The number of elements inside the outline of the plan is about M = 45. Hence,

 6 6 0 kN 
∆ p = q M ( IV ) =   ( 4 5 )( 0 .0 0 5 ) = 1 6 .5 k N / m
2

 ( 3 m )( 3 m ) 

155
*Newmark-03: Stress at a point distant from the loaded footing.
(Revision: Aug-08)
The footing shown below has a load q = 1.8 ksf. Find the stress at a depth of 5 feet below the
footing invert, at the point C.
Influence Value (IV) = 0.005 Depth Point = Z

Solution:

Set AB = 5 and draw the footing to that scale. The number of affected areas M =8, therefore
∆p = q M (IV) = (1,800 psf)(8)(0.005) = 72 psf

156
*Newmark-04: Stresses coming from complex shaped foundations.
(Revision: Aug-08)
A small but heavy utility building will be placed over a 2 m thick sand stratum. Below the sand is
a clay stratum 2 m thick. Find the stress at points A and B in the clay stratum directly below point
C at the surface.

Solution:

kN
Point A (top of clay stratum) q A = q M ( IV ) = (100 )(136 )( 0.005 ) = 68
m2
kN
Point B (bottom of clay) q B = q M ( IV ) = (100 )(100 )( 0.005 ) = 50 2
m
q + qB ( 70 + 50 ) kN
qaverage = A = = 59 2
2 2 m

157
*Newmark-05: Stress beneath a circular oil tank.
(Revision: Aug-08)
A circular oil storage tank will be built at the shore of Tampa Bay. It will be 20 m in diameter, and
15 m high. The tank sits upon a 2 m thick sand deposit that rests upon a clay stratum 16 m thick.
The water table is at practically at the surface. Find the stress increase from a fully loaded tank, at
mid-clay stratum, (a) directly under the center of the tank, and (b) at its outer edge, using the
Newmark influence chart shown below.

Set AB 10 m Influence Value (IV) = 0.001

Solution:
The contact stress is qo = (0.95)(9.81 kN/m3)(15m) = 140 kN / m 2
At mid-clay depth along the centerline of the tank (depth = 10 m) ∴OQ = 10 m

kN
σ v' = ( qo − γ w h ) M ( IV ) = 140 − ( 9.81)(10 )  ( 680 )( 0.001) = 28.5
m2

158
**Newmark-06: Use Newmark with a settlement problem.
(Revision: Aug-08)
A small but heavily loaded utility building has dimensions of 20 m x 20 m. It applies a uniform
load on its mat foundation of 100 kN/m2. Its mat foundation sits 1 m below the surface. The soil
profile consists of 3 m of a dry sand, with γ = 17.5 kN/m3 under laid by a 5 m thick clay layer with
a γ = 18.5 kN/m3, a moisture content of 22%, Cc = 0.30 and a Gs = 2.70. The clay stratum is under
laid by another sand stratum, and the phreatic surface coincides with the top of the clay stratum.
l) Using the Newmark method, what are the new stresses at the top and bottom of the clay
stratum due to the building’s loading?
m) What is the expected differential settlement between the building’s center and one of its
corners, in mm?
n) If a laboratory sample 4” thick of the field clay attained 50% consolidation in 5 hours, what
time will the clay layer in the field attain 60% consolidation?

q =100 kN/m2

1m SAND
3m
2m C E
• •
CLAY
γ = 18.5 kN/m3 w = 22 %
5m

•D •
F

Solution:

(1) Set AB = 2 m and observe that the building's foot-print covers the entire graph.
∴ ∆ σ E = 100 kN / m 2
Set AB = 7 m and the number of units M = 175
∴ ∆ σ F = qM ( IV ) = (100 )(175 )( 0.005 ) = 87.5 kN / m 2
The stress at point C has dropped to 50% of the stress at E,
∴ ∆ σ C = 50 kN / m 2 and ∴ ∆ σ D = ( 0.96 )( 50 ) = 48 kN / m 2
(100 + 87.5 ) kN
Average stress in the clay stratum beneath the center = = 93.8
2 m2

Average stress in the clay stratum beneath the corner =


( 50 + 48 ) = 49
kN
2 m2

159
(2) The differential settlement between the center and a corner of the building is,
Cc = 0.30 and GS = 2.70
wGs ( 0.22)( 2.70)
Seo = wGS ∴ eo = = = 0.59
S 1
The in-situ stress at mid-clay stratum before the building is built is,
kN
po = γ sand hsand + γ clay hat mid −clay = ( 3 m)(17.5) + ( 2.5 m)(18.5 − 9.8) = 74.3
m2
Therefore,
CC H  p + ∆σ  ( 0.30)( 5m)  ( 74.3) + ( 93.8) 
the settlement at the center ∆H = log  o = log   = 0.33 m
1+ e  po  1 + ( 0.59)  ( 74.3 ) 
CC H  p + ∆σ  ( 0.30)( 5m)  ( 74.3) + ( 49) 
the settlement at the corner ∆H = log  o = log   = 0.21m
1+ e  po  1+ ( 0.59)  ( 74.3) 
the differential settlement is ∆ ( ∆H ) = 0.33 m − 0.21 m = 0.12 m = 120 mm

(3) The time required to attain 60% consolidation in the field is,
( 0.26)( 2,500 mm) = ( 0.18)( 2x25.4 mm)
2 2
T H2
cv = σ drained =
t tF ( 5 hours )
Solving for the time of settlement in the field t f ,
( 0.26)( 2,500 mm) ( 5 hours )  1day  1 year  ≈ 2 years
2

tF =   
( 0.18)( 2x25.4 mm)
2
1  24 hours  365 days 

160
*Mohr-01: Find the angle of internal friction φ of sand.
(Revision: Jan-09)
The horizontal stress on a soil particle at a depth of 130 ft is ⅓ of its vertical stress. If the average
unit weight of the intervening soil mass above is 116.4 pcf, and the shear stress on the soil
particle’s failure plane is 30 psi, what is the soil’s average angle of friction?

Solution:
The vertical stress is the major principal stress σ1 and the horizontal stress is the minor principal stress
σ3. The shear stress is related to these two principal stresses via the angle θ, which is the angle to the
failure plane with respect to the principal stress σ1. These are related by the equation,

σ1 − σ 3
τ= sin 2θ
2
From this relation, the angle of internal friction is found from θ,

ϕ = 2θ − 90°
Therefore,

σ 1 = γh = (116.4lb / ft 3 )(130 ft )


 1 ft 2 
 = 105 psi
2 
 144in 

σ 3 = σ 1 = (0.33)(105 psi ) = 35 psi


1
3
σ1 − σ 2 105 psi − 35 psi
From τ = sin 2θ we have 30 psi = sin 2θ
2 2
∴ 2θ = 120
∴ ϕ = 2θ − 90 = (120 ) − 90 = 30°

161
*Mohr-02: Simple transformation from principal to general stress state.
(Revision: Feb-09)
A soil particle is found to be subjected to a maximum stress of 14.6 kN/m2, and a minimum stress
of – 4.18 kN/m2. Find the σ and τ on the plane of θ = 50° with respect to the major principal
stresses, and also find τmax.

(a) The graphical solution,

σ θ = 36kN / m 2
τ = 9 . 2 kN / m 2

σ 3 = −4.18kN / m 2 σ 1 = 146 kN / m 2

(b) The calculated solution,


 σ1 + σ 3   σ1 − σ 3   14.6 − 4.18   14.6 + 4.18  kN
σθ =  +  cos 2θ =  +  cos 2 ( 50° ) = 3.6 2
 2   2   2   2  m
 σ −σ   14.6 + 4.18  kN
τ θ =  1 3  sin 2θ =   sin 2 ( 50° ) = 9.2 2
 2   2  m
 σ1 − σ 3  kN
τ maximum =   = 9.4 2
 2  m

162
*Mohr–03: Find the principal stresses and their orientation.
(Revision: Sept.-08)
Equations for the principal stresses in the elastic half-space shown below for a uniformly loaded
strip footing are as follows: σ1 =
Β /2 Β /2 q/ π (α + sin α ) and σ3 = q/ π (α - sin
α).

The direction of the major


δ
principal stress bisects the angle α.
α Calculate the vertical stress σy, the
horizontal stress σx, and τxy at
point A if x = 0.75B and y = 0.5B
using Mohr’s diagram.

Solution:

σ =

σ1 + σ2
2 σ ,τ
σ =

Ο
β = 85.23
σ =
Ο
47.38

σ =
β
θ
σ =
Ο
2θ = 94.77

σ = σ1 + σ3
2

σ ,τ

τ =

α + δ = arc tan 2.5 = 68.20° 2θ + β = 180°

δ = arc tan 0.5 = 26.57° β = 85°

68.20° - 26.57° = 41.63°

if q = 432.5 kPa then σ1 = (σx + σy)/2 + R = 100 + 70.7 = 170.7 kPa

σ3 = (σx + σy)/2 - R = 100 - 70.7 = 29.3 kPa

163
*Mohr–04: Find the principal stresses and their orientation.
(Revision: Sept-09)
Given the general stresses at a point in a soil, determine the principal stresses and show them on a
properly oriented element.

164
*Mohr-05: Find the angle of internal friction.
(Revision: Sept-09)
A sample of clean sand was retrieved from 7 m below the surface. The sample had been under a
vertical load of 150 kN/m2, a horizontal load of 250 kN/m2, and a shear stress of 86.6 kN/m2. If the
angle θ between the vertical stress and the principal stress is 60°, what is the angle of internal
friction φ of this sample?

Solution:

165
*Mohr – 06: Normal and shear stress at a chosen plane.
(Revision: Sept-09)
Using a Mohr circle, determine the normal and shear stresses on the plane AB.
Solution:
2 σy = 90 lb/ft2
- 40 lb/ft

θ=30°

τn

σx = 125 lb/ft2
σn

τxy = - 40 lb/ft2

(a) (b)
σ y +σ x σ1 + σ 3 σ1 − σ 3
σ y −σx 
2
σn = + cos 2θ
σ1 = +   + τ xy
2
2 2
2  2  151 + 64 151 − 64
2 σn = + cos 60 = 129 psf
90 + 125  90 − 125  2 2
σ1 = +   + (−40)
2

2  2  σ −σ
τ n = 1 3 sin 2θ
σ 1 = 151 psf 2
151 − 64
σ y +σ x  σ y −σ x 
2 τn = sin 60 = 38 psf
σ3 = 2
−   + τ xy
2

2  2 
90 + 125  90 − 125 
2

σ3 = −   + (−40)
2

2  2 
σ 3 = 64 psf

166
*Mohr–07: Find the maximum and minimum stresses on a given plane.
(Revision: Feb-10)
Determine the normal and shear stresses on plane AB.

Solution:

σ1  σ X + σ Y σ Y − σ X 
2
400 + 750  400 − 750 
2
 σ 1 = 922 psf
 = ±  2  + τ 2
= ±   + ( − 300) 2
= 
σ3  2  
XY
2 2 σ 3 = 228 psf
Knowing the principal stresses permits to calculate the stresses on the normal plane,

σ1 + σ 3 σ1 − σ 3 922 + 228 922 − 228


σn = + cos 2θ = + cos 2 ( 45° ) = 575 psf
2 2 2 2
σ −σ 922 − 228
τ n = 1 3 sin 2θ = sin 2 ( 45° ) = 347 psf
2 2

167
**Mohr – 08: Back figure the failure angle
(Revised: Sept-09)
From the stress triangle shown below, find (a) the maximum and minimum principle stresses, (b)
the angle alpha, as shown, (c) the angle theta, and (d) the value for the maximum shear stress.

B
20 kN/m2 σ3 = 0
10 kN/m2

A β β C

A Graphical Solution: OR B Analytical Solution:

τ
τmax = 12.5
σ1 +σ1 σ1 +σ1
1 σ1 = + COS 2θ
2 2
σ1 σ1 40
20 = + COS 2θ ∴ σ1 =
2θ 2 2 1 + cos 2θ
σ1=25 1+cos2θ
σ3=0
σ1 −σ3
2 τn = SIN 2θ

2
2
σ1 20
(σ, τ) 10 = SIN 2θ ∴ ϑ1 =
2 SIN 2θ
(20, -10)

∴ 1 - 2 sin 2θ + cos 2θ = 0
(a) σ1 = 25 kN/m2 and σ3 = 0
…… θ = 63.4˚
(b) α in the figure is in the angle 2θ between
(σ, τ) and σ3.
20
….. σ1 = = 25 KN / m2 , σ3 =0
…… α = 2θ = 126.9˚ sin 126.90

(c) …… θ = 63.4˚

and τmax= 12.5 kN/m2 …… τmax= 12.5 kN/m2

168
*Mohr – 09: find the Principle pressure using Mohr
(Revised Sept-09)
The temporary excavation shown below is braced with a steel tube strut. Every morning, a
misguided foreman tightens the screw mechanism on the strut “just to be safe”. The stress on a
soil particle at point A, just behind the wall, has been measured with a pressure sensor installed by
the Engineer. It now measures 40 kN/m2. If the potential failure planes in the soil behind the wall
sustain 60° angles with respect to the vertical wall, estimate the normal and shear stresses at that
point A along a potential failure plane.

Solution:

At point A:

σv = hγ = (1.25 m) (16 kN/m3) = 20 kN/m2

∴ σv is the minor principal stress at A,

Since θ = 60° is with respect to the major principal stress (σ1) plane, then σv=σ 3

∴ σθ = (σ1+σ3)/2 + (σ1-σ3)/2 cos 2θ = (40+20)/2 + (40-20)/2 cos 120°

∴ σθ = 25 kN/m2

and τθ = (σ1-σ3)/2 sin 2θ = (40-20)/2 sin 120°

∴ τθ = 8.7 kN/m2

169
*Mohr –10: Relation between θ and φ.
(Revised Sept-09)

For a clean sand, prove that θ = 45°+ φ/2 using Mohr’s circle.

Solution:

For sand c=0.


By inspection in ∆OAB
(180°-2θ)+90°+φ=180°
∴ 2θ=90°+φ

∴ θ = 45° + φ/2

A failure test on a clean sand (i.e. c=0) shows that σ1=11.5 ksf and σ3=3.2 ksf at failure. Find
the angle φ for this sand.

170
*Mohr – 11: Normal and shear stresses on any plane.
(Revised Sept-09)
Determine the normal and shear stresses on the plane AB.

Solution.

001 = (300+125)/2 = 212.5 psf

0102 = (300-125)/2 = 87.5 psf

01B = 87.52 + 552 = 103 psf

σ1 = 212.5 + 103 = 315.5 psf

σ3 = 212.5 – 103 = 109.5 psf

πC0102 = tan-1 (55/87.5) = 32°

σn = 212.5 – 103 cos (32+40) = 181 psf

τn = 103 sin (32+40) = 98 psf

171
*Mohr–12: Derive the transformation from general to principal stresses.
(Revised Sept-09)
o) Derive the equation that transforms a general state of stress to the principal state of stress.
(Hint: Use Mohr’s circle for a graphical solution).
p) Determine the value of the major principal stress.
q) Determine the angle θ between the major principal stress and the state of stress shown in the
figure above.

172
*Mohr – 13: Tensile stress from the Mohr-Coulomb failure envelope.
(Revised Sept-09)
A soil sample has been tested and when plotted developed the Mohr-Coulomb envelope of failure
shown below. Find (1) axial stresses at failure, (2) the normal and shear stresses on the failure
plane, (3) the angle of failure with respect to the principal axis, and (4) the soil tensile strength.

(2.) σf = σθ

τf = τθ
(3.) θ
25 °
2θ = 115°
96 kN/m2
+σ σ1 +σ

(1.) σ1f = qu
(4.) qu

Solution:

σ1f = qu = 300 Pa

σf = σθ = 86 Pa

τf = τθ = 136 Pa

θ = 115°/2 = 57.5°

qu = -123 Pa

173
**Mohr–14: Maximum principal stress from triaxial test.
(Revised Sept-09)
A dry sample of sand was tested in a triaxial test. The angle of internal friction was found to be
36°. If the minor principal stress was 300 kPa, at what value of maximum principal stress will the
sample fail? The same test was then performed on a clay sample that had the same φ, and
cohesion of 12 kPa. What was the new maximum principal stress?

Solution:

τ τ
Φ = 36°

T
R
Φ Φ
σ σ
0 σ3 0
d

τ’

a) Failure will occur when the Mohr circle becomes tangent to the failure envelopes.

R
= sin φ
d
R (1 - sin φ )
σ3 = d − R = −R= R
sin φ sin φ
R (1+ sin φ )
σ1 = d + R = +R=R
sin φ sin φ
σ (1+ sin φ ) φ
∴ 1= = tan2 (45° + )
σ3 (1 − sin φ ) 2
36°
∴σ1 = (300 kPa) tan2 (45° + ) = 1160 kPa
2
b)
H = c(cot φ )
φ
σ1 + H = (σ 3 + H ) tan2 (45° + )
2
H = (12 kPa)(cot 36°) ≈ 17 kPa
36°
σ1 = (300 +17) tan2 (45° + ) −17 = 1,200kPa
2
174
*Mohr – 15: Derive the general formula for horizontal stress.
(Revised Sept-09)
Derive the general formula of the horizontal stress as a function of the vertical stress, cohesion and
the angle of internal friction.

Solution:
θ
θ = 45° + ∴ 2θ = 90° + φ
2

σ1 −σ3
ad =
2

ad
sinφ ≈ (1)
af

σ1 +σ3
af = af + af = (c) cotθ + (2)
2

c of
tan θ = ∴ = cot θ ∴ of = (c) cot θ (3)
of c

According to the Mohr’s circle properties:

σ1 -σ3 σ1 -σ3 σ1 -σ3


oa = σ 3 + ∴ oa = ∴ af = (c) cot φ + (4)From (1) & (4):
2 2 2
σ1 -σ3
 σ +σ3  σ1 -σ3
sin φ = 2 ∴ sinφ  (c )cot φ + 1 =
(c )cot φ + σ 1 + σ 3  2  2
2

(c ) sin φ cot φ + σ 1 + σ 3 sin φ = σ 1 - σ 3 ∴ (c) sin φ cot φ =


σ1 -σ3

σ1 +σ3
sin φ
2 2 2 2
∴ 2 ((c ) sin φ cot φ ) = σ 1 - (sin φ ( σ 1 ) - (σ 3 + sin φ ( σ 3 ) )

∴ 2  (c ) sin φ
cot φ 
 = σ 1 (1 - sin φ ) - σ 3 + (1 + sin φ ) ∴ 2
(c ) cos φ + σ 1 + sin φ
= σ1
sin φ  1 − sin φ 1 − sin φ
3

cos φ 1 + sin φ φ
Since
1 − sin φ
≈ tan 45 0 + φ( ) and
1 − sin φ

≈ tan 2  45 0 + 
 2

 φ  φ
σ 1 = σ 3 tan 2  45 0 +  + 2c tan 45 0 + 
 2  2

175
Chapter 10
Elastic Settlements
Symbols for Elastic Settlements

N → Raw value of the STP (obtained in the field).


qo → Contact pressure.
C1 → Embedment coefficient.
C2→ Creep correction factor.
ES → Soil elastic modulus.
Eeq→ Equivalent modulus.
∆(∆e) → Differential settlement between adjacent foundation.
∆Hi→ Elastic settlement.
I→ Influence factor; essentially equivalent to the strain ε in the soil.
IZ→ Simplifying influence factor.
ε→ Strain at mid stratum.
γ→ Unit weight of the soil.
v→ Poisson’s ratio.
MT→ Transverse moment.

176
*Elastic Settlement-01: Settlement (rutting) of a truck tire.
(Revision: Jan-08)
You are required to move a 60-ton truck-mounted crane onto your construction site. The front
wheels carry 20% of the load on tires inflated with 80 psi air pressure. Calculate the possible
rutting depth to your temporary jobsite road built from in-situ compacted medium sand. A
surface SPT test shows an N = 12 and the tire’s bearing area is roughly square. Use the
Schmertmann method to estimate the rutting.

Solution:

Each front tire has a square bearing area of BxB such that:

tire's load (0.5)(20%)(120, 000 lb)


B2 = = 2
= 150 in 2 ∴ B = 12.2 inches
tire pressure q o (80 lb / in )

A rough estimate of the soil’s elastic modulus is Es = 14N = 14(12) = 168 ksf.

Since the sand is compacted, it is a dense sand, and the influence factor Iz is equivalent to the strain ε.
The strain reaches a maximum value of 0.6. Therefore, the average value of the strain is about 0.3
throughout its depth to 2B = 2(12.2 inches) = 24.4 inches. Since the crane loads are on the surface and
only for a few days, it is permissible to assume that there is no creep and therefore C1 = C2 = 1.
Therefore, for the single layer of soil, the rutting is,

 ε  lb 0.30  144 in 2  k 
∆ = C1C2 qo   dz = (1)(1)(80 2 )( 2
)(24.4 in) ×  2  
 Es  in 168 k / ft  ft   1, 000 lb 
∴∆ ≈ 0.5 inches of rutting.

177
*Elastic Settlement-02: Schmertmann method used for granular soils.
(Revision: Aug-08)
Estimate the settlement of a square footing placed on a fine, medium dense sand, embedded 4 ft
below the ground surface, for long-term use. Use the Schmertmann method.
Es
Assume ≅ 14 where Es is in ksf; used for fine medium sands.
N

Solution:

I z ∆z
Layer ∆z Es Iz=ε
Es
(in) (ksf)
1 42 140 0.30 0.090
2 60 210 0.46 0.130
3 66 168 0.16 0.061
∑= 0.281

Q 200 k
The contact pressure on the soil is, qo =
2
− Df γ = − ( 4 ft ) (0.120 kcf ) = 3.60 ksf
B 49 ft 2
The coefficients for the Schmertmann method are C1 and C2 :
D γ   0.48 
Depth factor C1 = 1.0 − 0.5  f  = 1.0 − 0.5   = 0.93
 qo   3.60 
Creep factor C2 = 1.35 for a five year period .
The Schmertmann formula for the elastic settlement ∆ is,
2B
ε
∴∆ = C1C2 qo ∑ ∆z = (0.93)(1.35)(3.60)(0.281) = 1.27 inches
0 E

178
*Elastic Settlement-03: Schmertmann method used for a deeper footings.
(Revision: Aug-08)
Determine the elastic settlement of a deep spread footing after five years of the 3 ft. x 3 ft. footing
when it is placed on a uniform clean sand with γ = 110 pcf.

Solution:
Q 64 kips
The contact pressure on the soil is, q0 = = = 7.11 ksf
B 2 ( 3 ft )2

The SPT value indicates that the soil is a loose sand. The modulus E for loose sand can be calculated
using the following formula:

ES ≈ 10 ( N + 15 ) ksf

179
The following table summarizes the data and calculations:

Layer’s SPT
Soil’s Average
strain at
εZ∆Z/E
Layer elastic
thickness (average) mid
number modulus
∆Z (feet) N
E (ksf) stratum ε 3
(ft /kip)

1 1.5 6 210 0.35 0.0025


2 4.5 7 220 0.30 0.0061

Σ = 0.0086

The correction factors are as follows:


 0.5γ D f   ( 0.5 )( 0.110 )( 4 ) 
1. Depth factor, C1 = 1 −   = 1 −   = 0.97
 7.11 − ( 0.110 × 4 ) 
 q −γ D
 0 f 
Since C1 > 0.5, this is FINE.
 t yr   5 
2. Creep factor, C2 = 1 + 0.2log   = 1 + ( 0.2 ) log   = 1.34
 0.1   0.1 
The total elastic settlement is,

ε
1 2 ( qo −γ Df ) ∑ dz = ( 0.97)(1.34) 
2B
∆= CC 7.11−( 0.110)( 4)( 0.0086) = 0.07 ft = 0.84in
0 Es

180
*Elastic Settlement-04: The 2:1 method to calculate settlement.
(Revision: Aug-08)
Use the 2:1 method to find the average stress increase (∆q) due to the applied load Qu in the 5-foot
sand stratum directly beneath the footing. If ES = 400 ksf and v = 0.3, what is the expected
immediate settlement ∆Hi?

Qu=120 kips

Solution:

The settlement ∆H of an elastic media (the 5 foot thick sand stratum in this case) can be found from the
theory of elasticity as,

1 − v2
∆H = qo B Iw
Es
For square and flexible footings the influence factor is about IW = 0.95. The 2:1 method essentially
assumes that the stress reduces vertically by a vertical slope of 2 units vertically to 1 unit horizontally.
The stress increase can be found by integrating the above equation,
1 H2 Qu
∆q =
H ∫ ( B + Z ) dz
H1 2

where H1 = 0 feet (the footing’s invert) to H2 = 5 fee (bottom of the sand stratum).

181
5
1  Q  1  120kip 120kip 
∆q =  =  −  = 1.82ksf
H  B + z  0 5 ft  6 ft 11 ft 
but
1 − v2  120 kip   1 − 0.32   12 in 
∆H = qo B Iw =  2  ( 6 ft )   ( 0.95 )   = 0.52 inches
Es  3 6 ft   400 ksf   ft 

182
*Elastic Settlement-05: Differential settlement between two columns.
(Revision: Aug-08)
The allowable bearing capacity of a 30-ft thick, medium dense sand stratum (with φ = 36o and γ =
112 pcf) is 3 ksf. Column A has a design load of 430 kips and column B has a design load of 190
kips. Select footing sizes and determine the differential settlement ∆(∆H) between them. Is this
∆(∆H) acceptable for columns spaced 30 ft apart?

Solution:
Footing size for column A:

QA 430 kips
BA = = = 12 feet
qall 3 ksf

Footing size for column B:

QB 190 kips
BB = = = 8 feet
qall 3 ksf
∆H A 12 ft
A quick estimate of the ratio of settlement to the proportionality is = = 1.5
∆H B 8 ft
Therefore if the settlement at column B is ∆HB = 1 in., then the settlement at column A will be ∆HA =
1.5 in. Then,

∆ (∆H ) = 1.5 in − 1.0 in = 0.5 inches


and the rotation between the two columns is

θ=
∆ ( ∆H )
=
( 0.5 in )
= 0.0014
L  12in 
( 30 ft )  
 ft 

Both of these values [∆(∆H) and θ] are acceptable, since ∆(∆H) should be < 1" and θ < 0.0033.

183
*Elastic Settlement-06: Compare the settlements predicted by the Boussinesq,
Westergaard, and the 2:1 methods.
(Revision: Aug-08)
Compute the average stress ∆q at mid-clay stratum, for the values shown below, using: (a)
Boussinesq's method, (b) Westergaard's method, and (c) the 2:1 method.
Also, determine the size of a square spread footing, in order to limit total settlement ∆H = ∆Hi +
∆Hc + ∆HS to only 1.5 inches.

Estimate the initial settlement ∆ H i = 0.05 Z s where Zs is the thickness of the granular
stratum beneath the footing in feet, to give ∆Hi in inches.

Solution:
Assume an initial value of B = 10 feet.

Q = 240 kips

4’ γS= 110 pcf

5’ SAND WT
γ = 120 pcf
eo = 1.11
5’
CLAY
Cc = 0.42

ROCK

The contact pressure qo of the footing is:

Q 240 kips
qo = = = 2.4 ksf
AB 100 ft 2

(a) Stress at mid-clay stratum using Boussinesq’s method (use the charts on page 205) for a square
footing:

q
= 0.52 ∴ q = ( 0.52 )( 2.4ksf ) = 1.25 ksf
qo

184
(b) Stress at mid-clay stratum using Westergaard’s method for a square footing:

q
= 0.33 ∴ q = ( 0.33)( 2.4 ksf ) = 0.79 ksf
qo

(c) Stress at mid-clay stratum using the 2:1 method for a square footing:

The depth (Z) from the footing invert to mid-clay is 7.5 feet:

B = 10 ft.

z = 7.5 ft.

qo

B+z

Q 240 kips
q= = = 0.78 ksf
AB + Z (10 ft + 7.5 ft ) 2

Note that the Boussinesq method provides the highest predicted stress. Since this would predict faster
consolidation rates, it is the least conservative method. Therefore, for this problem, use the 2:1 method's
stress of 0.78 ksf. The instantaneous settlement (∆Hi):

∆H i = 0.05 Z s = 0.05 5 ft = 0.11 in

The in-situ effective stress qo' at mid-clay layer, before placing the footing is:
 kip   kip kip  
qo ' = γ s hs + γ c hc =  0.11 3
× 9 ft  +  0.120 3 − 0.0624 3  × 2.5 ft  = 1.13 ksf
 ft   ft ft  

Using q = 0.78 ksf from the 2:1 method, the total settlement ∆H is equal to the immediate ∆Hi, plus the
consolidation ∆Hc, and the secondary settlement ∆Hs, but limited to no more than 1.50 inches.

∆H = ∆H i + ∆H C + ∆H s = 1.50 inches

185
But ∆Hi = 0.11 in, and ∆Hs is negligible for this problem. Therefore the maximum permissible
consolidation settlement ∆Hc is limited to:

∆H c = 1.50 in − ∆H i = 1.50 in − 0.11 in = 1.39 in


or
 12 in 
( 0.42 )  5 ft ×
Cc H q + ∆q
'
 ft   1.13 ksf + ∆q 
∆H c = log o = 1.39 in = log   = 1.39 in
1 + eo qo ' 1 + 1.11  1.13 ksf 
Therefore ∆q = 0.39 ksf

Using the 2:1 method:

Q Q 240 kips
∆q = B+Z = = = 26.6 ft
AB + Z ∆q 0.34 ksf
Therefore B = 19 feet

Since the initial B = 10 feet, the new value of 19 feet should be used to re-iterate towards a better
solution that converges.

186
*Elastic Settlement-07: Schmertmann versus the strain methods.
(Revision: Aug-08)
Compute the immediate settlement ∆Hi using the Schmertmann formula using an average ∆q
value (qv1 = 233.3 kPa, qv2 = 163.3 kPa, qv3 = 77.0 kPa, qv4 =44.0 kPa and qv5 = 28.0). Es at point A is
Df
20,400 kPa, = 0.5 and C1 = C2 = 1. Compare the results with an alternate method
B
1− v2 
using ∆H = qo B  I w , where v = 0.3 and Iw = 0.95.
 E s 

Q = 2,100 kN

B = 3m x 3m

1.5 m

1.5 m
A

3m
B SAND

5m

ROCK

Solution:
The average stress from multiple layers is solved via this formula,
∆H  q1 + qn   1.5m  233.3kPa + 28.0kPa  
∆qv =   + q + q + … + q =   + 163.3kPa + 77.0kPa + 44.0kPa
H  2   6m  
2 3 n −1
  2  
∆qv = 104 kPa
The elastic settlement via Schmertmann is,
∆q  1000 mm   104 kPa 
∆H = C1C2 (0.6B) = (1)(1)( 0.6)( 3 m)    = 9.2 mm
Es  m   20, 400 kPa 
The alternative method from the theory of elasticity would yield,
 1 − v2   1 − 0.32   1000 mm 
∆H = qv1B   I w = ( 233.3 kPa )( 3 m)   ( 0.95)   = 10 mm
 Es   20, 400 kPa   m 

187
*Elastic Settlement-08: The Schmertmann method in multiple strata.
(Revision: Aug-08)
Determine the elastic settlement using the Schmertmann method of the 10'x 10' footing as shown
below. Estimate the elastic modulus using ES = 10(N + 15), where ES is in ksf and N is the
corrected SPT value.

Solution:
The data from these strata are placed into a table below.
Layer
Soil Modulus IZ = ε I Z ∆Z
Layer No. Thickness (ft/kp)
ES, (ksf) (average strain) E
∆Z, (feet)
1 5.0 360 0.35 0.00486
2 5.0 260 0.50 0.00962
3 2.5 300 0.35 0.00292
4 2.5 560 0.25 0.00112
5 5.0 190 0.10 0.00263
Σ = 2B = 20 ft Σ= 0.02115

188
The Schmertmann coefficients are,
γDf  0.5(0.1)(5) 
The depth coefficient C1 =1- [0.5( )]= 1-   = 0.75
q -γ D f  1.5 - (0.1)(5) 
The time coefficient C2 = 1.35
2B
ε
∆H i = C1C2 (q - γ D f )∑ dz = ( 0.88 )(1.35 )  2.5 − ( 0.1)( 5 )  ( 0.02115 ) = 0.042 ft = 0.05 in
0 E

189
**Elastic Settlement-09: Settlement of a mat foundation.
(Revision: Aug-08)
A mat foundation located 8 feet below grade supports a ten story building upon an area of 50 ft by
150 ft, and carries a uniform load of 6 ksf. For the soil profile conditions shown below, determine
the total settlement at the center and a corner of the foundation. The structure is of reinforced
concrete with column spacing at 25 ft. Is the calculated differential settlement acceptable?

Solution:
a) Using the Schmertmann method,
Layer ∆z (in) Es (ksi) Iz =ε (Iz/Es)∆z (in3/kip)

1 300 3.47 0.35 30.26


2 300 3.47 0.50 43.23
3 300 8.33 0.30 10.80
Σ = 84.29

γ Df  ( 0.100 )( 8 ) 
The depth factor C1 = 1.0 − 0.5 = 1.0 − 0.5   = 0.92
( q − γ D f )  6 − ( 0.100 )( 8 ) 
The time factor (creep ) C2 = 1.0
ε
∆ = C1C2 ( qo − γ D f ) ∑
2B
dz = ( 0.92 )(1.0 ) 6 − ( 0.100 )( 8 )  ( 84.29 ) = 3.78 in
0 Es

b) Consolidation settlement Method A (take each layer at a time),


Set eo1 = 1.00, and from e = 125/γ, eo2 = 0.96, eo3 = 0.89.
For clay 1: qo = (0.100)(50) + (0.110 - 0.0624)25 + (0.125 - 0.0624)12.5 = 5.95 ksf
At the mid clay stratum is at 87.5 ft below the surface,
(ie. 87.5/50=1.75 B = 3.5 B/2) ∆q = 0.12q = 0.12(6) = 0.72 ksf

CcH 1 q + ∆q 0.20(25x12) 5.95 +0.72


∆1 = log 10 o = log 10 = 1.48 in
1+ e o qo 1+1 5.95

At corner ∆q = 0.09q = 0.09(6) = 0.54

0.20(25x12) 5.95+0.54
∆1 = log 10 = 1.13 in
2 5.95

For clay 2: qo = 5.95 + ( 0.125 + 0.0624)12.5 + (0.130 - 0.0624)12.5 = 7.58 ksf


190
midlayer at 112 ft = 4.5 B/2, ∆q . 0.07q = 0.42

0.35(25x12) 7.58 +0.42


∆2 = log 10 = 1.25 in at the centerline
1+0.96 7.58

∆q . 0.05q = 0.30

0.35(25x12) 7.58 +0.30


∆2 = log 10 = 0.96in
1+0.96 7.58

For clay 3: qo = 7.58 + (0.130 + 0.0624)12.5 + (0.140 - 0.624)12.5 = 8.65 ksf


at midlayer (137.5' / 5.5 B/2) ∆q = 0.04q = 0.24 ksf

0.15(25x12) 8.65+0.24
∆3 = log 10 = 0.28in
1+0.89 8.65

at corner ∆q = 0.03q = 0.18 ksf

0.15(25x12) 8.65+0.18
∆3 = log 10 = 0.21in
1+0.89 8.65

Method B: the equivalent layer equation

∆ ∆e q o + ∆q
= where ∆e = C c log
H 1+ e o qo

The total settlement on the centerline is,


∆6 = 3.78 + 1.48 + 1.25 + 0.28 = 6.78 inches

and along the foundation edge,


∆edge = 3.78 + 1.13 + 0.96 + 0.21 = 6.08 inches

The differential settlement is ∆(∆) = 0.70 inches

The allowable for reinforced concrete buildings ∆(∆)< 0.003(span) = 0.003(25x12 inches)
= 0.90 inches
Therefore, the design is acceptable.

191
Chapter 11
Plastic Settlements
Symbols for Plastic Settlements

e→ Voids ratio.
ES → Soil elastic modulus.
CC→ Compression index.
GS→ Specific gravity of the solids of a soil.
OCR → Over-consolidation ratio (ratio of in-situ stress divided by the overburden stress).
H→ Depth of zone influence.
DHc→ Plastic settlement (also called primary consolidation).
DHtotal→ Total settlement of a structure.
DHs→ Second consolidation settlement.
Dp → Increasing pressure on the surface.
u→ Pore water pressure.
Pe → Pre-consolidation pressure of a specimen.
γSAT → Saturated unit weight of the soil.
γW → Unit weight of water.
U→ Degree of consolidation.

192
*Plastic Settlement–01: Porewater pressure in a compressible soil.
(Revision: Oct.-08)
a. How high will the water rise in the piezometer immediately after the application of the
surface load of 3 ksf?
b. What is the degree of consolidation from the 3 ksf at point A, when h =15 ft.?
c. Find h when the degree of consolidation at A is 60%.

∆ p = 3 ksf

Ground water table

20 feet

15 feet Sand

4 feet
Clay
10 feet
A

Rock

Solution:
a) Assume a uniform increase of the initial excess pore water pressure throughout the 10-foot
thickness of the clay layer:
∆p 3,000
The pore water pressure is u0 = ∆p = γ w h = 3,000 lb / ft 2 ∴ h = = = 48.1 feet
γw 62.4

193
b) The degree of consolidation at A is UA (%) when h = 15 feet:

 u   (15 ft )(62.4 pcf ) 


U A % = 1 − A 100 = 1 − 100 = 69%
 u 0   (48.1 ft )(62.4 pcf ) 

c) When UA = 60 %, what is the value of h?

 u   uA 
U A = 0.6 = 1 − A  =  1 − 
 u0   3, 000 psf 
∴ u A = (1 − 0.6 )( 3, 000 psf ) = 1, 200 psf

h=
uA
=
(1, 200 psf ) = 19.2 feet
γw ( 62.4 pcf )

194
*Plastic Settlement-02: Total settlement of a single layer.
(Revision: Aug-08)
A new building is planned upon the site shown below. Assume that the clay solids have a specific
gravity of 2.67. Find the primary consolidation settlement if the clay is normally consolidated.

∆p = 1 ksf

Sand
8 feet γ = 110 pcf,
Water Table

7 feet γsat = 115 pcf

Clay
17 feet w = 34%, LL = 50%, γsat = 120 pcf

Solution:

Skempton formula for the C c = 0.009( LL − 10) = 0.009(50 − 10) = 0.36


Se = wG s but S = 1 ∴ e = wG s = ( 0.34 )( 2.67 ) = 0.91
The stress of the clay at its mid − stratum before the building was built is ,
p o = γ d − sand H dry − sand + (γ sat − γ w ) H sat − sand + (γ sat − γ w ) H mid − clay
= (0.11kcf )(8 ft ) + (0.115 − 0.0624) kcf (7 ft ) + (0.120 − 0.0624) kcf (8.5 ft )
p o = 1.74 ksf

The consolidation settlement is ,


HCc p + ∆ p (17 )(12 ) (0.36) 1.74 + 1
∆H = log 10 o = log 10 = 7.6 i nches
1 + eo po 1 + 0.91 1.74

195
*Plastic Settlement-03: Boussinesq to reduce the stress with depth.
(Revision: Aug-08)
Calculate the settlement of the 10-foot thick clay layer shown below that will result from the
column’s load carried by a 5-foot square footing. The clay is normally consolidated. Apply
Boussinesq's formula to find the reduction of the vertical stress with depth.

200 kips

5’
10 feet 5’ x 5’ footing
Sand
γdry = 100 lb/ft3
Ground Water Table
γsat = 120 lb/ft3
5 feet

Clay
10 feet γsat =110 lb/ft3, e = 1.0, LL = 40

Solution:

Skempton formula Cc = 0.009(LL −10) = 0.009(40 −10) = 0.27


The stress of the clay at its mid − stratumbefore the building was built is,
po = γ d −sand Hdry−sand + (γ sat − γ w )Hsat −sand + (γ sat − γ w )Hmid −clay
po = (0.10kcf )(10 ft) + (0.120 − 0.0624)kcf (5 ft) + (0.110 − 0.0624)kcf (5 ft )
po = 1.53 ksf

The ∆pavg below the center of the footing between z1 = 15 feet to z2 = 25 feet is given as,

 H 2 I 4( H 2 ) − H1 I 4( H1 )   B L 
∆pavg = 4q   where I 4( H 2 ) = f  m′ = ,n' = 
 H 2 − H1   H2 H2 
5
B = L = = 2.5 feet
2

196
B 2.5 L 2.5
m′ = = = 0.167 and n′ = = = 0.167
H1 15 H1 15
From Boussinesq's Table for I 4
I 4( H 2 ) = 0.05 and I 4( H1 ) = 0.075
The average pressure at mid-clay layer is thus given by,
 H 2 I 4( H 2 ) − H1 I 4( H1 )   200 kips   (25)(0.05) − (15)(0.075) 
∆pavg = 4q   = 4  ′ ′    = 0.4 ksf = 400 psf
 H 2 − H 1   ( 5 )( 5 )   25′ − 15′
The consolidation settlement is,
HCc p + ∆pavg (10 )(12 ) (0.27) 1.53 + 0.40
∆H = log10 o = log10 = 1.6 inches
1 + eo po 1+1 1.53

197
*Plastic Settlement -04: Surface loads with different units.
(Revision: Aug-08)
Find the total settlement under a building that applies the load shown below.

q = 1.2 daN/cm2

4.6 m
Ground Water Table

Sand
6.0 m
γ = 17.6 kN/m3, γ’ = 10.4 kN/m3

Normally consolidated clay


7.6 m
Gs =2.78, w = 40%, PI = 15%, PL = 30%

Solution:

Notice that the data provided does not include a unit weight for the clay stratum. Therefore, this value
must be determined through the other information provided.

γ s = γ wGs = 27.8 kN m3 Se = (1)e = wGs = 0.40(2.78) = 1.11

Therefore, the clay unit weight is γ sat =


( Gs + eo ) γ w = [(2.78) + (1.11)] ( 9.81) = 18.1 kN m3
1 + eo 1 + (1.11)
The effective unit weight for the clay is,
γ ′ = γ sat − γ w = 18.1 − 9.81 = 8.3 kN m3
Also, the Skempton relation is, Cc = 0.009( wL − 10) = 0.009(45 − 10) = 0.32
The stress at the mid-clay stratum,
σ ′ = (4.6m)(17.6 kN m3 ) + (6.0m)(10.4 kN m3 ) + (7.6m 2)(8.3 kN m3 ) = 175kN / m 2
The consolidation settlement is,
HCc σ ′ + ∆σ (7.6m)(0.32) 175 + 120
∆H = log10 = log10 = 0.26 m = 260 mm
1 + eo σ o′ 1 + 1.11 175

198
*Plastic Settlement-05: Pre-consolidation pressure pc and index Cc.
(Revision: Aug-08)
The results of a laboratory consolidation test on a clay sample are given below:
Pressure, p
(kN/m2) Void ratio, e

23.94 1.112
47.88 1.105
95.76 1.080
191.52 0.985
383.04 0.850
766.08 0.731
r) a) Draw an e-log p plot.
s) b) Determine the pre-consolidation pressure, pc .
t) c) Find the compression index, Cc.

Solution:

b) Determine the pre-consolidation pressure, pc. From the e-log p plot,


199
p2 = 500 kN m 2 and e2 = 0.8 p1 = 300 kN m 2 and e1 = 0.9 ∴ pc = 117.5 kN / m 2

c) Find the compression index, Cc. From the slope of the graph,

e1 − e2 0.9 − 0.8
Cc = = = 0.451
 p2   500 
log   log  
 p1   300 

200
*Plastic Settlement-06: Final voids ratio after consolidation.
(Revision: Aug-08)
The clay stratum shown in the profile below has a total vertical stress of 200 kN/m2 at its mid-
height with a voids ratio of 0.98. When the vertical stress increases to 500 kN/m2 the voids ratio
decreases to 0.81. Find (a) the effective overburden pressure at mid-height of the compressible
clay layer, and (b) the voids ratio of the clay if the total pressure at its mid-height is 1000 kN/m2.

Elev. 760 ft.


Sand and Gravel
Unit Weight = 132 lb/ft3
Elev. 752 ft.
Ground Water Table
Sand and Gravel
Unit Weight = 132 lb/ft3
Elev. 732 ft.

Clay
Elev. 721 ft.
3
Unit Weight = 125.4 lb/ft
Elev. 710 ft.

Solution:

a ) po = [γ h ]dry sand − gravel + [γ ' h ]saturated sand − gravel + [γ ' h ]mid −clay stratum
po = (132 lb ft 3 )(8 ft ) + (132 − 62.4) lb ft 3 (20 ft ) + (125.4 − 62.4) lb ft 3 (11 ft )
po = 3.14 ksf
e1 − e2 0.98 − 0.81
b) Cc = = = 0.427
log( p2 p1 ) log(500 200)
0.98 − e2
0.427 = ∴ e2 = 0.68
log(1000 200)

201
*Plastic Settlement-07: Settlement due to a lowered WT.
(Revision: Aug-08)
Find the settlement due to lowering of the phreatic surface from elevation 349.5’ to 344.0’ using
the boring report shown below.

Solution:

po = ( 0.110 pcf )( 7 ft ) + ( 0.110 pcf )( 6 ft ) +


( 0.110 − 0.0624 pcf )( 3 ft ) + ( 0.110 − 0.0624 pcf )( 3 ft )
po = 1.71ksf
∆q
∆p = + ∆p due to the lowering WT
H
∆q2 = ∆p − ∆q1 = 1.853 − 1.16 = 0.693

c log po + ∆q1
C H
∆H1 =
1+ e po
0
log 3 + 1.16 = 0.032 ft
0.030(10)
∆H1 =
1 + 0.96 3
log 3 + 0.693 = 0.157 ft
0.034(10)
∆H 2 =
1 + 0.96 3
versus 0.083ft the additional settlement.
The rising WT may reduce settlement.

202
*Plastic Settlement-08: The over-consolidation ratio (OCR).
(Revision: Sept-08)
Oedometer (consolidation) tests of several samples from the clay stratum yields the consolidation
curve shown below. Given that Gs = 2.65, find (a) the value of po, (b) The value of pc and (c) the
OCR of the clay.

Solution:

1200 kN

1.5m
2m WT 4mx4m

4m
Sand γb = 9.1 kN/m3

Clay
3m γb = 9.2 kN/m3

Sand and Gravel


γb = 9.0 kN/m3

a) In order to find the in-situ stress po before the footing was built, we need to find the unit weight
of the sand stratum,
γ bGs (9.1kN / m3 )(2.65)
γd = = = 14.6 kN / m3
Gs −1 1.65
The stress po is found at mid-clay stratum,
po = [ h(γ d )]dry(sand ) + [ h(γ ′)]sand + [ h(γ ′)]clay
po = (2m)(14.6kN / m3 ) + (4m)(9.1kN / m3 ) + (1.5m)(9.2kN / m3 ) = 79.4 kPa

203
**Plastic Settlement-09: Coefficient of consolidation Cv.
(Revision: Aug-08)
An odometer test was performed on a peat soil sample from an FDOT project in the Homestead
area. The results are shown below. The initial sample thickness is 20 mm, with two-way drainage
through porous stones, simulating field conditions. The vertical stress increment is 10 kPa.
Estimate of the coefficient of consolidation cv as,
2
3H dr
Coefficient of consolidat ion(C v ) =
4t
Time (minutes) 0 0.32 0.64 1.28 2.40 4.80 9.60 16.0
Settlement 0 0.16 0.23 0.33 0.45 0.65 0.86 0.96
(mm.)
( 12 ) 0 0.57 0.8 1.13 1.55 2.19 3.10 4.0
time = (min)

a) Plot a graph of settlement against the square root of time.


b) Determine the value of the coefficient of consolidation cv (in m2/s).
c) Estimate what could be a good estimate of the elastic modulus of this soil Eo (kPa)
d) What sort of permeability k (in m/s) could you estimate for this soil (from Eo)?

Solution:
(1)

Time vs. Settlement

Time (min)^(1/2)
0 1 2 3 4 5
0

0.2
S ettlem en t (m m .)

0.4

0.6
3.40
0.8

1.2

(2)
204
vt x = 3.40 min1/ 2 and time(t x ) = 11.56 min
3d 2 3(10mm) 2 (1min)(1m) 2 −7 m
2
cv = = = 1.08 ×10
4t x 4(11.56 min)(60sec)(1000mm) 2 sec

(3)

1 mm
Voids − ratio (ev ) = = 0.05 ∴ ev = 5%
20 mm
σv 10 × 103 N
Eo = = 2
= 200 × 103 Pa = 200 kPa
ev 0.05 m

(4)

Cc  2.30σ z ' k  1 + e   σ z  k   k 
e≈ ∴ cv =    ∴ cv =     = Eo  
1 + eo  γw   Cc   ez   γ w  γw 

k=
cvγ w
=
(
1.08 × 10−7 m
2

s )(
9.81 kN 3 )
m = 5.3 × 10−9 m
Eo
(
200 kN 2
m ) sec

205
*Plastic Settlement -10: Secondary rate of consolidation.
(Revision: Aug-08)
An oedometer (consolidation) test is performed on a normally consolidated clay stratum that is 8.5
feet thick, and it found that the clay’s initial voids ratio was eo = 0.8 and its primary compression
index is Cc = 0.28. The in-situ stress at mid-clay layer is po = 2,650 psf, and the building exerts a
pressure through its mat foundation of 970 psf. The secondary compression index Cα = 0.02.
The time of completion of the primary settlement is approximately 18 months. What is the total
consolidation of the 8.5 foot clay stratum 5 years after the primary consolidation?

Solution:

The primary consolidation ∆H p is,


Cc H  po' + ∆p '  ( 0.28 )( 8.5 ft )(12in / ft )  2, 650 + 970 
∆H p = log  = log   = 2.15 inches
1 + eo  po
'
 1 + ( 0.8 )  2, 650 

The secondary consolidation ∆H s is,


Cα H t 
∆H s = log  2 
1 + ep  t1 
We must find e p first, by finding the change in the voids ratio during primary consolidation,
 po' + ∆p '   2, 650 + 970 
e p = eo − ∆e = eo − Cc log  '  = 0.8 − ( 0.28 ) log   = 0.76
 po   2, 650 
C H  t  ( 0.02 )( 8.5 − 0.18 )(12in / ft )  5 
∴ ∆H s = α log  2  = log   = 0.59 inches
1 + ep t
 1 1 + ( 0.76 )  1.5 

The total consolidation settlement is thus =∆H p + ∆H s = 2.15 + 0.59 = 2.74 inches

206
*Plastic Settlement-11: Using the Time factor Tv.
(Revision: Aug-08)
A 3-m thick, doubly-drained saturated stratum of clay is under a surcharge loading that
underwent 90% primary consolidation in 75 days. Find the coefficient of consolidation cv of this
clay in cm2/sec.

Solution:

The clay layer has two-way drainage, and Tv = 0.848 for 90% consolidation.

Tv H dr2 (0.848)(150 cm) 2


cv = = = 0.00294 cm2/sec
t (75 days × 24 × 60 × 60)

0
Percent consolidation

10
20
30
40
50
60
70
80
90
100
0 0.1 0.2 0.3 0.4 0.5 0.6 0.7 0.8 0.9 1 1.1 1.2
Time factor , Tv

207
*Plastic Settlement-12: The time rate of consolidation.
(Revision: Aug-08)
An oedometer (consolidation) test is performed on a 4” thick specimen, drained on top and
bottom. It was observed that 45 percent consolidation (Tv = 0.15) was attained in 78 hours.
Determine the time required to attain 70 percent consolidation (Tv = 0.40) in a job site where the
clay stratum is shown below.

Building’s load
q = 4 ksf

20’ WT
SAND

25’ CLAY

SAND

Solution:
TvH dr2
The coefficient of consolidation cv = is the same for the lab and field samples.
t
Tv H dr 2  Tv H dr 2   Tv H dr 2 
cv = =  =  
t  t laboratory  t  field
( 0.15) (2 in)
2
(0.40)(12.5 ft × 12 in / ft ) 2
∴ =
( 78 hours ) t field
 1day   1 year   0.40   (12.5 ft × 12in / ft ) 
2
∴ t field = ( 78 hours )     
 24hours   365days   0.15   (2in) 2 
∴ t field = 134 years to attain 70% consolidation

208
*Plastic Settlement-13: Time of consolidation t.
(Revision: Oct.-08)
Using the information derived from Problem 11, how long will it take a 30-mm thick undisturbed
clay sample obtained from the field to undergo 90% consolidation in the laboratory?

Solution:

The Time Factor Tv is the same 90% in the field as in the laboratory, therefore,

   
cv t90( field )  c (75days × 24 × 60 × 60)   cvt90 
T90 = = v  =
H dr2 ( field ) 3, 000mm 2 30mm 2 
 ( )  ( ) 
 2  field  2  laboratory
(75days × 24 × 60 × 60) (15mm )
2

∴ t90( lab ) = = 648 seconds =10 minutes


(1,500mm) 2

209
*Plastic Settlement-14: Laboratory versus field time rates of settlement.
(Revision: Aug-08)
Laboratory tests on a 25mm thick clay specimen drained at both the top and bottom show that
50% consolidation takes place in 8.5 minutes.

a) How long will it take for a similar clay layer in the field, 3.2 m thick, but drained at the top
only, to undergo 50% consolidation?
b) Find the time required for the clay layer in the field as described in part (a) above, to reach a
65% consolidation.

Solution:
(1)

tlab t( field )
=
H dr2 ( lab ) H dr2 ( field )
(3.2m) 2 (8.5 min)
∴ t( field ) = 2
= 557, 000 min = 387 days
 25mm 
 
 2 ×1000mm 
(2)

2
 25 
(0.197)  
cv =
Tv H dr2
=  2 ×1000  = 0.36 × 10−5
t50 (8.5 min)
T65 H dr2 (0.34)(3.2m) 2
∴ t( field 65) = = = 961, 400 min = 668 days
cv 0.36 × 10−5 m 2 / min

210
*Plastic Settlement-15: Different degrees of consolidation.
(Revision: Aug-08)
A clay layer 20 feet thick sitting on top of granite bedrock, experiences a primary consolidation of
8.9 inches. Find:
(d) The degree of consolidation when the settlement reaches 2 inches.
(e) The time to reach 50% settlement if cv is 0.002 cm2/sec.
(f) The time for 50% consolidation if the clay stratum is doubly-drained?

Solution:

 2 
(a ) U % =  100 = 22.5%
 8.9 
(0.197) ( 20 ft )( 30.48cm / ft ) 
2
Tv
(b) T50 = 0.197 and t= H 2
dr = = 424 days
cv
( 0.002 cm 2
/ sec )  360 sec 
 1hr  days 

24h 

(0.197) (10 ft )( 30.48cm / ft ) 


2
T
(c) t = v H dr2 = = 106 days
cv
( 0.002 cm 2
/ se c )  360 sec   24h 
 1hr   days 

0
Percent consolidation

10
20
30
40
50
60
70
80
90
100
0 0.1 0.2 0.3 0.4 0.5 0.6 0.7 0.8 0.9 1 1.1 1.2
Time factor , Tv
0
Percent consolidation

10
20
30
40
50
60
70
80
90
100
0 0.1 0.2 0.3 0.4 0.5 0.6 0.7 0.8 0.9 1 1.1 1.2
Time factor , Tv

211
**Plastic Settlement-16: Excavate to reduce the settlement.
(Revision: Oct.-08)
An oedometer test on a 1” thick, doubly-drained sample from the clay stratum (shown below)
attained 50% consolidation in 6.5 minutes. Find:
(a) The total differential settlement of the fully loaded tank.
(b) The time required for 75% consolidation in the field.
(c) The depth of excavation for minimal settlement.

Solution:
a) The surface load from the oil tank (neglecting the weight of the tank) is ∆p:
∆p = hγ oil = (40 ft )(60 pcf ) = 2.4 ksf
Using the Boussinesq pressure diagram (next two pages) for B = 75' , provides the stress levels at
any point in the soil mass, thus,
The stress at point A ⇒ 0.91∆p = 0.90 ( 2.4 ) = 2.2 ksf

point B ⇒ 0.43∆p = 0.43(2.4) = 1.0 ksf


The in-situ effective stress at point A was po, before the tank was built:
po = (10 ')( 0.120 − 0.062 ) + ( 28.5')( 0.110 − 0.062 ) = 1.95 ksf
The settlements at point A (below the center of the tank), and point B (at the edge of the tank) are,

212
Cc H  p '+ ∆p  0.4 ( 57 ' )  1.95 + 2.2 
∆H A = log  o = log   = 3.34 ft
1 + eo  po '  (1 + 1.27 )  1.95 
CH  p '+ ∆p  0.4 ( 57 ' )  1.95 + 1.0 
∆H B = c log  o = log   = 1.85 ft
1 + eo  po '  (1 + 1.27 )  1.95 

∴ The differential settlement between A and B is = 3.34 ft − 1.85 ft = 1.49 ft = 18inches


b) Since the time required for consolidation is,
Tv for 50% = 0.197, and
Tv for 75% = 0.480, using the relationship between field and lab conditions, through
the coefficient of consolidation,
2
 ft  
( 0.2 ) 0.5in  
( 0.480 )( 28.5 ft )
2
Tv field H 2 field Tvlab H 2 field   12in  
= ∴ cv = =
t75 tlab 6.5 min t 75
∴ t75 = 13.8 years for 75% consolidation.
c) The settlement can be reduced by excavating weight of soil equal to the weight of the
structure (note: total, not effective, since the water is also removed). Assume need to
excavate “x” feet from the clay layer.
σ total = (10 ')( 0.120 ) + x ( 0.110 ) = 2.4 ksf
Solving for x: ∴ x = 10.9 ft ≅ 11 feet of clay
Add the 11 feet of the clay to the 10 feet of the sand on top of the clay for a

∴ Total excavation depth = 11 ft + 10 ft = 21 feet

213
**Plastic Settlement-17: Lead time required for consolidation of surcharge.
(Revision: Aug-08)
A common method used to accelerate the consolidation of a clay stratum is a sand surcharge, as
shown below. The surcharge load will force the clay to attain a large part of its settlement before
the structure with the same load is built. This method minimizes the settlement of the structure.
An office building is planned to be built on the site shown below. The total weight of the building
is 136,000 kips, spread over a square foundation 200 ft by 200 ft. Field tests showed that the clay
stratum has a liquid limit of 28 percent, an initial void ratio of 0.95, a γ = 130 pcf and a
consolidation coefficient of 10-3 in2/second. The sand stratum has a CC = 0.01, a γ = 125 pcf and an
initial void ratio of 0.70. The sand surcharge has a γ = 115 pcf.
(d) Determine the total settlement at mid-clay under the center of the surcharge.
(e) The time required to attain 60% consolidation of the clay stratum (i.e. TV = 0.30). This is the
lead time required to place the surcharge before construction.
(f) The SPT in the sand stratum is N = 15.

h ft
Sand Surcharge
(Area = 200’ x 200’)
15 ft
WT 30 ft Sand

20 ft
Clay

Impermeable rock

Solution:
(a) The weight of the new building is estimated at Q = 136,000 kips. The surcharge will have to weigh
the same, spread over an area = 200’ x 200’ = 40,000 ft2, using a fill with a unit weight of γ = 115
pcf. The unit pressure of the surcharge γsurcharge is,
Q 136, 000kips
qsur = = = 3.4 ksf
A 40, 000 ft 2
lb
but , qsur = γ sur h = 115 3 (h) = 3.4 ksf ∴ h = 30 feet high
ft
Both the sand and clay strata contribute to the settlement. However, the settlement from the clay
stratum is a consolidation settlement, such that,

214
Using Skempton's formula Cc = 0.009( LL -10) = 0.009(28 -10) = 0.162
Also eo = 0.95 and H clay stratum = 20 ft = 240 in.
The initial stress at mid-clay stratum is,
p0 = γ s h + γ sb h = γ cb h = (0.125kcf )(15 ft ) + (0.125 − 0.0624)(15) + (0.130 − 0.0624)(10) = 3.5 ksf
The increased load from the surcharge ∆p is,
∆p = γ h = (0.115kcf )(30 ft ) = 3.45 ksf
The settlement created by the surcharge is,
HCc  p + ∆p  (240in)(0.162)  3.5 + 3.45 
∆H c = log10  0 = log10   = 5.94 inches
(1 + e0 )  p0  (1 + 0.95)  3.5 

The settlement from the sand stratum is an elastic settlement,


H sand = 30 ft = 360in ES ≈ 14 N = (14 )(15 ) = 210k / ft 2 ; depth factor C1 = 1 and creep factor C2 = 1
ε  0.30
∆ = C1C2 qo   dz = (1)(1)(3.4ksf )( )(360 in) = 1.75in
E 210 ksf
The total settlement = 5.94in + 1.75in = 7.69in

b) The lead time t required for the surcharge to accomplish its task is,
( )( )
2
T H dr2 (0.30)  20 ft 12in /1 ft   1min  1hr   1day 
t= v =  
    = 200 days for 60% consolidation
c
v (10 in / sec )
−3 2
 60sec  60 min   24hr 
The total settlement that has taken place at 200 days (60% consolidation) is,
Total Time = ∆H S + (60%) ∆H C = 1.73in + (0.6)(5.94in) = 5.3 inches

215
**Plastic Settlement-18: Settlement of a canal levee.
(Revision: Aug-08)
A uniform surcharge of sand 20 feet in height will be placed over the marl stratum as shown
below, in order to preconsolidate that layer for a future building. The in-situ voids ratio of the
marl is 0.59, and its index of compression can be found from a relation proposed by Sowers as Cc =
0.75 (eo – 0.30). Find the total settlement of the surcharge at its point A.

The coefficient of consolidation cv for the marl can be found from the relation,
k (1 + eo ) 2
cv = ft /day
av γ w
where the permeability k = 1.0 × 10 −5 cms / s , and a v = 2.9 × 10 −4 ft 2 / lb .
Find the time required (in days) for the marl to attain 50% consolidation.

0
10
Percent consolidation

20
30
40
50
60
70
80
90
100
0 0.1 0.2 0.3 0.4 0.5 0.6 0.7 0.8 0.9 1 1.1 1.2
Time factor , Tv

216
Solution.

cc = 0.75 ( eo − 0.30 ) = 0.75 ( 0.59 − 0.30 ) = 0.218

At midpoint of the marl, the in-situ stress σ o is,


 lb 
σ o = γ h = 120  ( 20 ft ) = 2, 400 psf
 ft 3 
The increase in stress due to the surcharge ∆σ is,
 lb 
∆σ = γ∆h =  125 3  ( 20 ft ) = 2,500 psf
 ft 
Therefore, the consolidation settlement of the marl is,
HCc  σ + ∆σ  ( 40 ft )(12in / ft )( 0.218 )  2.4 + 2.5 
∴∆ = log10  o = log10   = 20.4inches
1 + eo  σo  (1 + 0.59 )  2.4 
The time required for 50% of consolidation to take place is found through the coefficient of
consolidation cv ,
 1 + eo 
cv = k 
(1.0 ×10−5 cm / sec ) (1 + 0.59 )  1in   1 ft   86.4 ×103 sec 
=  = 2.5 ft / day
2
3   
 avγ w  ( 2.9 ×10 ft / lb )( 62.4lb / ft )  2.54cm   12in  
−4 2
day 
The time factor for 50% consolidation is Tv = 0.2, therefore the time t required is,
( 0.2 )( 40 ft )
2
Tv H dr2
t= = = 128 days
cv 2.5 ft 2 / day

217
**Plastic Settlement-19: Differential settlements under a levee.
(Revision: Aug-08)
An oedometer (consolidation) test was performed on a clay sample 3 cm high, drained on both
sides, and taken from mid-stratum shown below. Seventy percent consolidation was attained in
6.67 minutes. Find:
(b) The time required to attain 70% consolidation of the clay stratum.
(c) The magnitude of that settlement in that time.

Solution.
(a) Since the soil is the same clay in the laboratory and the field, and both are 70% consolidation,
Tv H dr 2  Tv H dr 2   Tv H dr 2  t field H 2field
cv = =  =  ∴ = 2
t  t  laboratory  t  field tlab H lab
2
t H 2  700cm 
or t field = lab 2F = ( 6.67 min )   = 1.45 × 10 min = 2.76 years
6

HL  1.5cm 
(b) The amount of settlement that takes place at 70% consolidation is,
 C H  p '+ ∆p '  
∆H 70% = ( 0.70 )  c log10  o 
1 + eo  po '  
The in-situ stress at mid-clay stratum before the surcharge was applied was,
∴ po ' = ∑ γ i hi = (18kN / m3 ) ( 2m ) + ( 20 − 9.81) kN / m3 ( 3.5m ) = 71.7kN / m 2
i

and ∆p ' = 72 kN / m 2
0.7Cc H  p '+ ∆p '  0.7 ( 0.20 )( 700cm )  71.7 + 72 
∴ 70% ∆H = log10  o = log10   = 14.8 cms
1 + eo  po '  (1 + 1)  71.7 

218
***Plastic Settlement-20: Estimate of the coefficient of consolidation cv.
(Revised Oct-09)
An oedometer test was performed on a peat soil sample from an FDOT project in the Homestead
area. The results are shown below. The initial sample thickness was 20 mm with two way
drainage through porous stones, simulating field conditions. The vertical stress increment is 10
kPa. Use an estimate of the coefficient of consolidation cv as,

3H dr2
cv =
4t
Given Data
Time(min) 0.00 0.32 0.64 1.28 2.40 4.80 9.60 16.00
Settlement (mm) 0.00 0.16 0.23 0.33 0.45 0.65 0.86 0.96

Time^1/2(min)^1/2 0.00 0.57 0.80 1.13 1.55 2.19 3.10 4.00

(a) Plot a graph of settlement against the square root of time.


m2
(b) Determine the value of cv ( ).
s
(c) Estimate what could be a good estimate of the elastic modulus of this soil, E o (in kPa).
m
(d) What sort of permeability k ( ) could you estimate for this soil (from E o )?
s
Solution:
e)

Settlement vs. Time^1/2

Time^1/2(min)^1/2
0.00 1.00 2.00 3.00 4.00 5.00
0.00

0.20
Settlement Delta H (mm)

0.40
y = 0.2503x + 0.0376
0.60

0.80

1.00 0.96,3.69

1.20

219
f) t x = 3.69 min 1 / 2 → t x =13.62 min
3d 2 3(10 min) 2 (1 min)(1m) 2 −8 m
2
cv = = = 9.178 * 10
4t x 4(13.62 min)(60 sec)(1000mm) 2 sec
g) The strain at the end of the loading is:
∆ (1mm)
εv = = = 0.05 or 5%
L (20mm)
σ v 10 * 10 3 N
Eo = = = 200kPa Vertical Modulus
εv 0.05m 2
Cc
From Coduto (pages 390 and 391) ε v ≈ but Coduto (page 424)
1 + eo
2.30σ z k  1 + e0  σ  k  k
cv =   or cv = z   = E o
γ w  Cc  εz γw  γw
*note: (2.30 is for ln; use 1.0 for log)
 m 2  kN  
  9.178 *10 −8  9 . 81 3 

cv γ w   
sec  
= 
m m
k=  = 4.50 *10 −9
Eo   kN   s
 200 2 
  m  
 

220
**Plastic Settlement-21: The apparent optimum moisture content.
(Revised Oct-09)
Find the Cc and the “apparent” OCR for a superficial clay stratum that is 4m thick. The water is
at the surface. A sample of the clay has a water content of 27.9%, and a specific gravity of 2.65. An
oedometer test showed the following results:

Voids Ratio Vertical Effective Stress


0.736 15
0.733 28
0.73 60
0.675 230
0.638 480
0.6 930

Solution:

Cc = [Ca-Cb]/[log(O’z b)-log(O’z a)] = [0.675-0.600]/[log(930)-log(230)] = 0.123

Se = wGs (Saturation means S = 1)

e = wGs = (0.279)(2.65) = 0.73935 = 0.739

γ = [(Gs + e) γw]/[1 + e] = [(2.65 + 0.739)(9.81)]/[1 + 0.739] = 33.25/1.739 = 19.12

OCR = (98)/(19.12) = 5.13

221
**Plastic Settlement-22: The differential settlement between two buildings.
(Revised Oct-09)
Two tall buildings sit next to each other in the downtown area of Boston. They are separated by a
narrow 5 foot alley and are both 50 stories high (550 feet tall). They also have similar foundations,
which consist of a simple mat foundation. Each mat is a thick reinforced concrete slab, 5 feet thick
and 100 feet by 100 feet in plan view. The total load (dead + live + wind) of each building is
150,000 kips.
The mats are sitting upon a thick prepared stratum of carefully improved soil 40 feet thick, that
has an allowable bearing capacity of 17 ksf. Below the compacted fill stratum lays a medium to
highly plastic clay stratum 38 feet thick. Below the clay stratum is a thick layer of permeable sand.
The water table coincides with the interface between the improved soil and the clay. The dry unit
weight of improved soil is 110 pcf, whereas the in-situ unit weight of the clay is 121 pcf.
The clay has a specific gravity of 2.68. Also a moisture content of 32%, a PI of 52% and a PL of
12% before building “A” was built in 1975. The clay has a consolidation coefficient of 10-4 in2/sec.
The second building “B” was finished by early 1995. Assume that each building’s mat rotates as a
rigid plate.
How much do you predict will building ‘A” drift towards building “B” by early 1996, in inches?
(Note: The drift of a building is its horizontal movement at the edge of the roof level due to lateral
loads such as wind or earthquakes, or the differential settlement due to unequal pressures.)

Given: P = 150,000 kips


qa = 17 ksf;

For clay: Gs = 2.68,


w = 32%,
P.I = 52%,
P.L = 12%,
γ =121 pcf,
cv = 10-4 in2/sec

For the improved soil: γd =110 pcf

222
Diagram 1
PLAN VIEW PLAN VIEW
A B

BUILDING A BUILDING B

50 Stories = 550 ft 50 Stories = 550 ft


Completed 1975 Completed 1975

150,000 kips 150,000 kips

A B

D C F E

Fill

Clay

Sand

223
Solution:
Initial Condition: Geostatic Stresses
σ’zo = ∑Hγ – u = (40)(121) – (40)(62.4) = 2,344 lb/ ft2
Induced Stresses:
To solve the induced stresses we will use the Boussinesq’s Method:

(σ z )induced =
[( (
q 2BLz f B 2 + L2 + z 2f )
1/ 2
)] × B 2
+ L2 + 2 z 2f −1
+ Π − sin −
(
2 BLz f B2 + L2 + 2 z 2f )
1/ 2

( ( )
4 Π z 2f B 2 + L2 + z 2f + B 2 L2 ) B2 + L2 + z 2f ( )
z 2f B 2 + L2 + z 2f + B 2 L2

p 150,000 × 10 3
q= = = 15,000 lbs
A (100 )(100 )
zf = 58 ft (at mid-clay)
Replacing the values of q, B, L and zf into the Boussinesq’s equation yields:
(σz )induced = 8008.65 lb/ ft2 , 1059.79 lb/ ft2
Exists differential stresses at the corners of the foundation.
Stresses due to fill:
(σz )fill = H fill γ fill = (110) (38) = 4,180 lb/ ft2
• The total final stresses at corner “C”
(σ’zf )C = σ’zo + (σz) induced + (σz) fill
= 2344 lb/ ft2+ 8008.65 lb/ ft2 + 4180 lb/ ft2 = 14,532.65 lb/ ft2 ≈ 14,533 lb/ ft2
• The total final stresses at corner “D”
(σ’zf )D = σ’zo + (σz) induced + (σz) fill
= 2344 lb/ ft2+ 1059.79 lb/ ft2 + 4180 lb/ ft2 = 7,583.79 lb/ ft2 ≈ 7,584 lb/ ft2
(Note: Both buildings A and B will experience the same final stress due to fill, loadings and geostatic
factors).

Time Factor
Determining the time factor for building A:
Tv = _4Cvt_
H2dr
For 21 years: t = 662,256,000 sec

Tv A =
( )
4 10 -4 in 2 / sec (662,256,000 sec )
= 165.56
(40)2

Degree of Consolidation

U = [1-10 –(0.085+Tv)/0.933] x 100%

The degree of consolidation at TvA = 165.56 is:


224
U = [1-10 –(0.085+166)/0.933] x 100% =100%

By early 1996, 21 yrs after the load was applied, the soils are completely consolidated.

Therefore,

δc
U=
(δ c )ult

δ c = (δ c )ult

Assuming normally consolidated soils

Cc σz 
δc = ∑ H log  f  (Eqn 1-1)
1 + e0 σz 
 0 

1 + eo σ’zo

Cc = 0.009 (LL – 10)

LL = PL + PL = 52 + 12 = 64 %

Therefore, Cc = 0.009 (64 – 10) = 0.486

Gsγ w
e0 = −1
γd

γd =
γ
=
121
= 91.67 ∴ e0 =
(2.68 )(62.4 ) - 1 = 0.82
1+ w 1 + 0.32 91.67

Recall that uniform stress does not exist beneath the corners of the mat foundation. Therefore, we will
have differential settlement.

At corner C:

H= 40 ft (σ’zf )C = 14,533 lb/ ft2 σ’zo = 2,344 lb/ ft2

Replacing values of H, (σ’zf )C , σ’zo, Cc and eo into (Eqn 1-1):

δc =
0.486
(40 ft ) log  14,533  = 8.46 ft = 101.55 in
1 + 0.82  2,344 

At corner D:

H= 40 ft (σ’zf )D = 7584 lb/ ft2 σ’zo = 2,344 lb/ ft2

225
Replacing values of H, (σ’zf )D , σ’zo, Cc and eo into (Eqn 1-1):

δc =
0.486
(40 ft ) log  7,584  = 5,45 ft = 65.35 in
1 + 0.82  2,344 

The ∆δc = 8.46 -5.45 = 3.01 ft

Diagram 2 A
DRIFT = x
A
550 ft '
?

D C

5.45 ft
8.46 ft
D' ?
3.01 ft
C'

100 ft

∆ = tan -1 (3.01/ 100) =1.72º

Drift = x = 550 tan 1.72 = 16.52 ft = 200 inches.

226
**Plastic Settlement-23: Settlement of a bridge pier.
(Revision: Aug-08)
Estimate the average settlement from primary consolidation of the clay stratum under the center
of the bridge pier.

Solution:

Stress at mid-clay stratum:

kN kN
3 m x 1 9 .6 2 3
= 5 8 .9
m m2
kN kN kN
7 m x 9 .8 0 3
= 6 8 .6 for a total ρ o = 1 4 6 .7
m m2 m2
kN kN
2 m x 9 .6 0 3
= 1 9 .2
m m2

Determine eo : From chart pc ~ 150 < po normally consolidated, eo = 0.81.

Determine ∆σ or ∆p :

227
4m 5m
m = = 0.28 6 a nd n= = 0.5 00 ∴ I 4 = 0.07 1
10 m 10m
Q  28 x10 3 kN  kN
∆σ = 4 I4 = 4 ( 0.07 1 )   = 9 9.5 2
A  8 m x 10 m  m

Determine the settlement ∆Η :

 C   p + ∆p   0.31   146.7 + 99.5 


∆Η = Η  c  log10  o  = 4m   log10   = 0.15 m
 1 + eo   po   1 + 0.81   1 46 .7 

228
Chapter 12
Shear Strength of Soils
Symbols for Shear Strength of Soils

c→ The cohesion of a soil particle.


cu→
p→
p’→
q→
q’→
qu→ Ultimate shear strength of a soil.
u → Pore water pressure.
u c→
ud→
σ→ The normal axial stress.
σ1→ stress
σ1’→ stress
σ3’→ stress
σ3→ Confirming pressure
σd →
τ→ The shear stress.
τf→ The normal shear stress at a failure.
ф→ The angle of internal friction of the soil.
φ→ The angle of inclination of the plane of failure caused by the failure shear stress.
σ’→ Effective stress.

229
*Shear strength–01: Maximum shear on the failure plane.
(Revision: Oct-08)
A consolidated un-drained triaxial test was performed on a specimen of saturated clay with a
kg
chamber pressure σ 3 = 2.0 2 . At failure,
cm
kg kg
σ 1 − σ 3 = 2.8 2 , u = 1.8 2 and the failure plane angle θ = 57° .
cm cm
Calculate (1) the normal stress σ and (2) shear stress τ on the failure surface and (3) the
maximum shear stress on the specimen.

Solution:
kg
σ 1 = 2.8 + 2.0 = 4.8
cm2
kg
σ 3 = 2.0
cm 2

σ1 − σ 3  4.8 − 2  kg
Shear stress τ= sin 2θ =   sin114° = 1.27
2  2  cm 2

 σ1 + σ 3   σ1 − σ 3   4.8 + 2   4.8 − 2  kg
Normal stress σ = +  cos 2θ =  +  cos114° = 2.83
 2   2   2   2  cm 2

σ1 − σ 3  4.8 − 2  kg
Maximum shear τ MAX = sin 2θ =   = 1.4 at θ = 45°
2  2  cm 2

230
*Shear strength–02: Why is the maximum shear not the failure shear?
(Revised Oct-09)
kg
Using the results of the previous Problem 01, and φ = 24°, c ' = 0.80 , show why the sample
cm 2
failed at 57 grades instead of the plane of maximum shear stress.

Solution:

On failure plane

kg
σ ' = (σ − u ) = 2.83 − 1.8 = 1.03
cm 2
kg
S57° = c '+ σ ' tan φ = 0.8 + (1.03 tan 24°) = 1.27
cm 2

Compare that to τ = 1.27 kg/cm from the previous problem, and note that they are equal, and so for
both, S 57 ° = τ 57 ° failed. .

Now at the plane of maximum shear stress θ = 4 5 °

 4.8 + 2   4.8 − 2  kg
σ = +  cos 90° = 3.4 2
 2   2  cm
kg
σ ' = 3.4 − 1.8 = 1.6 2
cm
kg
s45° = c '+ σ ' tan φ = 0.8 + (1.60 tan 24°) = 1.51
cm 2

The shear strength at 45º is much larger than at 57º, therefore failure does not occur.

231
*Shear strength–03: Find the maximum principal stress σ1.
(Revised Oct-09)
Continuing with the data from the two previous problems, the same soil specimen is now loaded
kg
slowly to failure in a drained test, that is u = 0, with σ 3 = 2.0 2 . What will be the major principal
cm
stress at failure?

Solution:

kg
a) Analytically, in a drained test u = 0; at failure σ '3 = σ 3 = 2 , on the failure plane θ = 57° .
cm 2

 σ ' + 2   σ '1 − 2  
s = c '+ σ ' tan φ = 0.80 +  1 +  cos114° tan 24° = (1.426 ) + ( 0.132 ) σ '1
 2   2  
σ ' −2 
and τ =  1  sin114° = ( 0.457 ) σ '1 − ( 0.914 )
 2 
kg kg kg
At failure s = τ ∴ σ '1 = 7.31 2 ; σ θ = 3.6 2 and τ θ = 2.38 2
cm cm cm

b) Graphically,

232
*Shear strength–04: Find the effective principal stress.
(Revised Oct-09)
A drained triaxial test on a normally consolidated clay showed that the failure plane makes an
angle of 58˚ with the horizontal. If the sample was tested with a chamber confining pressure of
103.5 kN/m2, what was the major principal stress at failure?

Solution:

φ φ
θ = 45° + ∴ 58° = 45° + ∴ φ = 26°
2 2
Using the equation that relates the major principal stress σ1 to the minor principal stress σ3, and with c =
0 (the value of cohesion for a normally consolidated clay),

 φ1   2 26   kN
σ 1 ' = σ 3 ' tan 2  45° +  = (103.5 )  tan  45° +   = 265 2
 2   2  m

233
*Shear strength–05: Using the p-q diagram.
(Revised Oct-09)
Triaxial tests performed on samples from our Miami Pamlico formation aeolian sand, showed the
peak stresses listed below. Plot these values on a p-q diagram to find the value of the internal angle
of friction.
σ1 = 76 psi for σ3 = 15 psi p = 45.5, q = 30.5 psi (1)
σ1 = 148 psi for σ3 = 30 psi p = 89.0, q = 59.0 psi (2)
σ1 = 312 psi for σ3 = 60 psi p = 186.0, q = 126.0 psi (3)
σ1 = 605 psi for σ3 = 120 psi p = 362.5, q = 242.5 psi (4)

Solution:

Remember that p = (σ1 + σ3)/2 and q = (σ1 – σ3)/2

q (psi)

300
4
α = 34°

200

100
2

1
p(psi)
100 200 300 400

From the p-q diagram:

q4 2 4 2 .5
ta n α ≈ = = 0 .6 6 8
p4 3 6 2 .5
s i n φ = t a n α = 0 .6 6 8 ∴ φ = 42°

234
**Shear strength–06: Consolidated-drained triaxial test.
(Revised Oct-09)
A consolidated-drained triaxial test was conducted on a normally consolidated clay. The results
kN kN
are as follows: σ '3 = 276 2 ( ∆ σ d ) f = 276 2
m m
Determine:
(a) The angle of friction φ;
(b) The angle θ that the failure plane makes with the major principal plane, and
(c) The normal stress σ’ and shear stress τf on the failure plane.

Solution:

For normally consolidated soil the failure envelope equation is:

τ f = σ ' ta n φ because c = 0

For the triaxial test, the effective major and minor principal stresses at failure are as follows:

kN kN
σ '1 = σ 1 = σ 3 + ( ∆ σ d ) f = 276 + 276 = 552 and σ '3 = σ 3 = 276
m2 m2

Part A.

The Mohr circle and the failure envelope are shown in the figure below, from which:

σ '1 − σ '3
AB σ '1 − σ '3 552 − 276
sin φ = = 2 = = = 0 .3 3 3 ∴ φ = 1 9 .4 5 °
OA σ '1 + σ '3 σ '1 + σ '3 5 5 2 + 2 7 6
2

235
Part B.

φ 1 9 .4 5 °
θ = 45° + = 45° + = 5 4 .7 °
2 2

Part C.

σ '1 + σ '3 σ '1 − σ '3


σ' ( on the failure plane ) = + cos 2θ
2 2

σ '1 − σ '3
and τ f = sin 2θ
2

kN kN
Substituting the values of σ '1 = 552 , σ '3 = 276 and θ = 5 4 .7 ° in the preceding
m2 m2
equations,

552 + 276 552 − 276 kN


σ' = + cos 2 ( 54.7 ) = 36 8 2
2 2 m
552 − 276 kN
and τf = sin 2 ( 54 .7 ) = 130 2
2 m

236
**Shear strength–07: Triaxial un-drained tests.
(Revised Oct-09)
Triaxial un-drained tests were performed on clay samples taken from the stratum shown below.
kN
The tests were taken with pore water pressure measurements, and c ' = 20 2 , and φ = 24 °
m

(a) Find the clay shear strength at its mid-stratum, and


(b) Find the effective and total stresses at the same level acting on a vertical face of a soil element.

Solution:
a) For the gravel:

kN
γ sat = γ d + nγ w = (16 + 0.3x10 ) = 19
m3
 kN   kN   kN  kN
∴ For σ 1 = 4m 16 3  + 9m 19 3  = 3.5 17.6 3  = 297 2
 m   m   m  m
 kN  kN
∴ For σ '1 = σ 1 − u = σ 1 − γ w (9 + 3.5) = 297 − 10 3  (12.5m ) = 173 2
 m  m
kN kN kN
∴ S = c '+ σ '1 tan φ ' = 20 2 + 173 3 tan 24° = 96.6 2
m m m

237
b) Since the clay is saturated,

kN
σ '3 = K oσ '1 = 0.5σ '1 = 0.5 (173) = 86
m2
kN kN kN
and σ 3 = σ '3 + u = 86 2
+ (13m + 3.5m )10 3 = 251 2
m m m

238
**Shear strength-08: Consolidated and drained triaxial test.
(Revised Oct-09)
Two similar clay soil samples were pre-consolidated in triaxial equipment with a chamber
pressure of 600 kN/m2. Consolidated-drained triaxial tests were conducted on these two
specimens. The following are the results of the tests:

Specimen 1: Specimen 2:
kN
σ 3 = 100 σ 3 = 50
kN
m2 m2
kN
( ∆σ d ) f = 410.6 (∆σ d ) f = 384.37
kN
m2 m2

Find the values of the cohesion c and the angle of internal friction φ.

Solution:

For Specimen 1, the principal stresses at failure are,

kN kN
σ 3 ' = σ 3 = 100 and σ 1 ' = σ 1 = σ 3 + ( ∆σ d ) f = 100 + 410.6 = 510.6
m2 m2
Similarly, the principal stresses at failure for specimen 2 are

kN kN
σ 3 ' = σ 3 = 50 and σ 1 ' = σ 1 = σ 3 + ( ∆σ d ) f = 50 + 384.4 = 434.4
m2 m2
239
These two samples are over-consolidated. Using the relationship given by equation

 φ1   φ1 
σ 1 ' = σ 3 ' tan 2  45° +  + 2c tan  45° + 
 2  2

Thus, for specimen 1

φ1  φ1 
( 510.6 ) = (100 ) tan 2  45° + 
 + 2c tan  45° + 
 2  2

and for specimen 2

φ1  φ1 
( 434.4 ) = ( 50 ) tan 2  45° + 
 + 2c tan  45° + 
 2  2

φ1 
Subtracting both equations ( 76.2 ) = ( 50 ) ta n 2  45 ° +  ∴ φ1 = 1 2 °
 2 

Substituting φ 1 into the equation,

 12     12  
( 510.6 ) = (100 ) tan 2  45° +    + 2c tan  45° +   
  2    2 
kN
510.6 = 152.5 + 2.47c ∴ c = 145
m2

240
***Shear strength-09: Plots of the progressive failure in a shear-box.
(Revised Oct-09)
A soil test is performed in the shear-box shown below. The test data lists the stresses and
displacements. Assign positive normal stresses to compression and positive shear stresses are
counter-clockwise. Plot the Mohr circles of stress for each stage.

DISPLACEMENTS(mm) σ’ XX σ’YY τxy τyx


Stage
x y (kpa) (kpa) (kpa) (kpa)
a 0 0 30 70 0 0
b 0.30 -0.50 71 70 31.0 -31.3
c (peak) 2.50 -0.60 145 70 43.3 =49.0
d 3.00 -0.82 - - - -
e 10.00 1.50 90.6 70 24.5 -32.0

Solution:
For small displacements, the x and y planes remain perpendicular. Use a compass to locate by trial and
error the center of the Mohr circle. The center of the circle must lie on the σ’ axis, and it must be
equidistant from the two stress points (σ ' xx , τ xy ) and (σ ' yy , τ yx ) .

TABLE OF VALUES
σ 1 '+ σ 3 ' σ 1 '− σ 3 '
S'= τ= τ Change in angle between
STAGE 2 2
S' x + y plan (in degrees)
kPa kPa
a 50 20 0.40 0˚
b 70.5 31.2 0.44 0˚
c (peak) 103 60 0.58 5.25˚
e 70 32 0.46 21˚

241
242
243
**Shear strength-10: Shear strength along a potential failure plane.
(Revision Oct-09)
An engineer is evaluating the stability of the slope in the figure below, and considers that the
potential for a shear failure occurs along the shear surface shown. The soil has an angle
φ ' = 30° and no cohesive strength. Compute the shear strength at point A along this surface when
the groundwater table is at level B, then compute the new shear strength if it rose to level C. The
unit weight of the soil is 120 lb/ft3 above the WT and 123 lb/ft3 below.

Potential shearing surface

C▼

B▼

●A

Solution:
 lb  lb
u = γ z z w =  62.4 3  (20 ft ) = 1248 2
When the groundwater table is at B:  ft  ft
 lb   lb  lb lb
σ ' z = ∑ γH − u =120 3 (26 ft ) + 123 3  (20 ft ) − 1248 2 = 4332 2
 ft   ft  ft ft

The potential shear surface is horizontal, so σ ' = σ ' z

 lb  lb
s = σ ' tan φ ' =  4332 2  tan 30° = 2501 2
 f t  f t

When the groundwater table is at C:

 lb  lb
u = γ z z w =  62.4 3  (32 ft ) = 1997 2
 ft  ft
 lb   lb  lb lb
σ ' z = ∑ γH − u =120 3 (14 ft ) + 123 3  (32 ft ) − 1997 2 = 3619 2
 ft   ft  ft ft

 lb  lb
s = σ ' tan φ ' =  3619 2  tan 30° = 2089 2
 ft  ft

244
***Shear strength-11: Use of the Mohr-Coulomb failure envelope.
(Revised Oct-09)
Samples have been obtained from both soil strata shown in the figure below. A series of shear
strength tests were then performed on both samples and plotted in diagrams below. The c’ and φ’
values obtained from these diagrams are shown in the figure below. Using this data, compute the
shear strength on the horizontal and vertical planes at points A, B, and C.

Solution:

Point A - Horizontal plane:

kN kN kN
σ ' = ∑ γH − u = (17.0 3
)(3.0m) + (17.5 3 )(1.1m) − (9.8 3 )(1.1m)
m m m
σ ' z = 59.8 kPa
s = c' + σ ' tan φ ' = 10kPa + (59.5kPa ) tan 28° = 41.6 kPa

Point A - Vertical plane:

σ ' z = Kσ ' z = (0.54)(59.5 kPa) = 32.1 kPa


s = c' + σ ' tan φ ' = 10 kPa + (32.1 kPa ) tan 28° = 27.1 kPa

245
Using similar computations:

Point B vertical plane s = 57.2 kPa

Point B horizontal plane s = 35.5 kPa

Point C vertical plane s = 68.1 kPa

Point C horizontal plane s = 54.4 kPa

Commentary
At each point the shear strength on a vertical plane is less than that on a horizontal plane because
K < 1. In addition, the shear strength at point B is greater than that at point A, because the
effective strength is greater. The strength at point C is even higher than at point B because it is in
a new strata with different c’, φ’, and K values. Thus, the strength would increase gradually with
depth within each stratum, but change suddenly at the boundary between the two strata.
Draw the shear strength envelope for the ML stratum and then plot the upper half of the Mohr
circle for point A on this diagram. Assume the principal stresses act vertically and horizontally.

Failure envelope and Mohr’s circle

246
***Shear strength-11b: Use of the Mohr-Coulomb failure envelope.
(Revised Oct-09)
Samples have been obtained from both soil strata shown in the figure below. A series of shear
strength tests were then performed on both samples and plotted in diagrams below. The c’ and φ’
values obtained from these diagrams are shown in the figure below. Using this data, compute the
shear strength on the horizontal and vertical planes at points A, B, and C.

Solution:

Point A - Horizontal plane:

kN kN kN
σ ' = ∑ γH − u = (17.0 )(3 . 0 m ) + (17 . 5 )(1 . 1m ) − (9 . 8 )(1.1m)
m3 m3 m3
σ ' z = 59.8 kPa
s = c' + σ ' tan φ ' = 10kPa + (59.5kPa ) tan 28° = 41.6 kPa

Point A - Vertical plane:

σ ' z = Kσ ' z = (0.54)(59.5 kPa) = 32.1 kPa


s = c' + σ ' tan φ ' = 10 kPa + (32.1 kPa ) tan 28° = 27.1 kPa
247
Using similar computations:

Point B vertical plane s = 57.2 kPa

Point B horizontal plane s = 35.5 kPa

Point C vertical plane s = 68.1 kPa

Point C horizontal plane s = 54.4 kPa

Commentary

At each point the shear strength on a vertical plane is less than that on a horizontal plane because

K < 1. In addition, the shear strength at point B is greater than that at point A, because the effective
strength is greater. The strength at point C is even higher than at point B because it is in a new strata
with different c’, φ’, and K values. Thus, the strength would increase gradually with depth within each
stratum, but change suddenly at the boundary between the two strata.

Draw the shear strength envelope for the ML stratum and then plot the upper half of the Mohr circle for
point A on this diagram. Assume the principal stresses act vertically and horizontally.

Failure envelope and Mohr’s circle

248
**Shear strength-12: Triaxial un-drained tests.
(Revised Oct-09)
Triaxial un-drained tests were performed on clay samples taken from the stratum shown below.
The test were taken with pure water pressure measurements and yield a c` = 20 kN/m3, and φ=
24º.Find (1) the clay shear strength at mid-stratum, and (2) the effective and total stresses at that
same level acting on a vertical face of a small element.

H=4
sandy gravel
H= 13m n= 0.30
γ =16 kN/m3
σ1

H= 7m σ3 clay γ =17.6 kN/m3

Impermeable rock stratum


Solution:

For the gravel: γsat = γsat + nγw = [16 + (0.3)10] = 19 kN/m3

For the clay: σ1 = (4m)(16 kN/m3 )+ (9m)(19 kN/m3 )+ (3.5m)(17.6 kN/m3 )= 297 kN/m3

σ1’ = σ1 − u = σ1 − γw (9 + 3.5) = 297 - 10 kN/m3 [12.5m] = 172 kN/m3

S = c’ + σ1’ tan φ = (20 KN/m3 + 172 kN/m3 tan 24ο) = 96.6 NM/m3

Since the clay is saturated, σ3’ = k σ1’ = 0.5(172) = 86 kN/m3

σ3 = σ3’ + u = 86 kN/m3 + (86m + 86m) 10 kN/m3 = 251 kN/m3

τ
φ = 24ο.

S= 96.6 kN/m3
KN 3
C` = 20 /m

σ`

249
**Shear strength-12b: Triaxial un-drained tests.
(Revised Oct-09)
Triaxial un-drained tests were performed on clay samples taken from the stratum shown below.
The test were taken with pure water pressure measurements and yield a c` = 20 kN/m3, and φ=
24º.Find (1) the clay shear strength at mid-stratum, and (2) the effective and total stresses at that
same level acting on a vertical face of a small element.

H=4 Sandy
gravel
H= n= 0.30
σ1 γ =16 kN/m3

H= σ3 Clay γ =17.6 kN/m3


7m σ3

Solution:
For the gravel: γsat = γsat + nγw = [16 + (0.3)10] = 19 kN/m3
For the clay: σ1 = (4m)(16 kN/m3 )+ (9m)(19 kN/m3 )+ (3.5m)(17.6 kN/m3 )= 297 kN/m3
σ1’ = σ1 − u = σ1 − γw (9 + 3.5) = 297 - 10 kN/m3 [12.5m] = 172 kN/m3
S = c’ + σ1’ tan φ = (20 KN/m3 + 172 kN/m3 tan 24ο) = 96.6 NM/m3
Since the clay is saturated, σ3’ = k σ1’ = 0.5(172) = 86 kN/m3
σ3 = σ3’ + u = 86 kN/m3 + (86m + 86m) 10 kN/m3 = 251 kN/m3

τ
φ = 24ο.

S= 96.6 kN/m3
C` = 20 KN/m3

σ`

250
**Shear strength-13: Determine the principal stresses of a sample.
(Revised Oct-09)
A clay layer, 20 feet thick is covered by a 40 foot sandy gravel stratum with a porosity of 30%, and
a dry unit weight of 103 pcf. Tests on the un-drained samples of the clay gave c = 2.9 psi, γSAT =
112 psf and φ' = 24o. Find:
(a) the soil shear strength s = c + σ'tanφ' at the clay's midlevel (point A), and
(b) the effective and total stress acting on the vertical face of a soil element at the clay midlevel
(point A).

d=13'

H1=40'

γd = 103 pcf
n = 30%
H2=20' •A c = 2.9 psi
γsat = 112 pcf
φ' = 24o

Solution:
a) In order to find s, it is required to know the γsat of the sand.

n 0.30 0.30 γ (1+ e) 103(1+0.429)


e= = = = 0.429 and G S = d = = 2.36
1- n 1 - 0.30 0.70 γW 62.4

( G S + e)γ W (2.36 +0.429)62.4


γ sat = = = 122pcf
1+ e 1.429

*Assume that the clay was normally consolidated to find σ' at midlevel in the clay (point A),
that is c = 0.

251
σ’ A = γ h S + γ’ h S + γ’ h C = ( 0.103)(13) + ( 0.122 − 0.0624 )( 27 )  + ( 0.122 − 0.0624 )(10 )  = 3.4ksf
and
s = c + σ’ A tan φ’
lbkip 144in 2 k
s = 2.9 2
x3
x 2
+ 3.44 2 ( tan 24 o )= 0.42ksf +1.53ksfs = 1.95ksf
in 10 lb ft ft
The pore water pressure u is,
u = (10 + 27 )( 0.0624 ) = 2.3ksf
Therefore, the toatl stress is,
σ A = σ A' + u = 3.4 + 2.3 = 5.7ksf

b) To find the stress on the vertical face of the soil element at A, we find θ through a graphical
solution as follows,

φ o
θ = 45 o + = 45 o + 24 = 57 o_2θ = 114 o
2 2

c 0.42
x1 = = = 0.939ksf
tan φ’ tan 24 o

s 1.95
x2 = = = 4.38ksf
tan φ’ tan 24 o

o o
x 3 = s( tan 24 )= 1.95( tan 24 )= 0.868ksf

252
s 1.95
R= = = 2.13ksf
cos φ’ cos 24 o

σΝ3 = x2 + x3 - x1 - R = 4.38 + 0.868 - 0.939 - 2.13= 2.18 ksf

σΝ1 = 2.18 + 2R = 2.18 + 2(2.13) = 6.44 ksf

σ3 = σΝ3 + u = 2.18 + 2.31 = 4.5 ksf

253
**Shear strength-13b: Determine the principal stresses of a sample.
(Revised Oct-09)
A clay layer, 20 feet thick is covered by a 40 foot sandy gravel stratum with a porosity of 30%, and
a dry unit weight of 103 pcf. Tests on the un-drained samples of the clay gave c = 2.9 psi, γSAT =
112 psf and φ' = 24o. Find (1) the soil shear strength s = c + σ'tanφ' at the clay's midlevel (point A),
and (2) the effective and total stress acting on the vertical face of a soil element at the clay midlevel

D=13’

H=40’ Sand
γd= 103 pct
n= 30%

γsat= 103 pct


• A
Clay c = 2.9 psi
H= 20’
φ' = 24o

Rock

(point A).

Solution:

In order to find s, it is required to know the γsat of the sand.

n 0.30 0.30 γ (1+ e) 103(1+0.429)


e= = = = 0.429 and G S = d = = 2.36
1- n 1 - 0.30 0.70 γW 62.4

( G S + e)γ W (2.36 +0.429)62.4


γ sat = = = 122pcf
1+ e 1.429

* Assume that the clay was normally consolidated to find σ' at midlevel in the clay (point A), that is c =
0.

254
σ’ A = γ h S + γ’ h S + γ’ h C = ( 0.103)(13) + ( 0.122 − 0.0624 )( 27 )  + ( 0.122 − 0.0624 )(10 )  = 3.4ksf
and
s = c + σ’ A tan φ’
kip 144in 2
lb k
s = 2.9 2
x
3
x 2
+ 3.44 2 ( tan 24 o )= 0.42ksf +1.53ksfs = 1.95ksf
in 10 lb ft ft
The pore water pressure u is,
u = (10 + 27 )( 0.0624 ) = 2.3ksf
Therefore, the toatl stress is,
σ A = σ A' + u = 3.4 + 2.3 = 5.7ksf

To find the stress on the vertical face of the soil element at A, we find θ through a graphical
solution as follows,

φ o
θ = 45 o + = 45 o + 24 = 57 o_2θ = 114 o
2 2

c 0.42
x1 = = = 0.939ksf
tan φ’ tan 24 o

s 1.95
x2 = = = 4.38ksf
tan φ’ tan 24 o

o o
x 3 = s( tan 24 )= 1.95( tan 24 )= 0.868ksf

255
s 1.95
R= = = 2.13ksf
cos φ’ cos 24 o

σΝ3 = x2 + x3 - x1 - R = 4.38 + 0.868 - 0.939 - 2.13= 2.18 ksf

σΝ1 = 2.18 + 2R = 2.18 + 2(2.13) = 6.44 ksf

σ3 = σΝ3 + u = 2.18 + 2.31 = 4.5 ksf

256
**Shear strength-14: Formula to find the maximum principal stress.
(Revised Oct-09)
Derive the general formula that gives the value of the major principal stress σ1 as a function of the
minor principal stress σ3, the cohesion c and the angle of internal friction φ.

Solution:

h φ
θ = 45° + → 2 θ = 90 + φ
2

d
σ1 − σ 3
ad =
2
φ
ф
2

f
c
o τ3 a τ1
σ3 σ1 45°

From the figure,

ad
sin φ =
fa (1)

σ1 + σ 3
fa = fo + oa = (c) cot φ +
2

tan φ = c → fo
= cot φ → fo = (c)(cot φ)
fo c

Using the properties of the Mohr circle,

(σ 1 − σ 3 ) (σ 1 + σ 3 ) (σ 1 + σ 3 )
oa = σ3 + → oa = → fa = (c) cot φ + (2)
2 2 2

(σ 1 − σ 3 )
σ1 + σ 3 σ1 − σ 3
Introducing (1) into (2): sin φ = 2
(σ + σ 3 )
→ (sin φ)[(c) cot φ + ] =
2 2
(c) cot φ + 1
2
257
→[(c) sin φ cot φ] + [ σ 1 +σ3 σ −σ3
sin φ] = 1 → (c) sin φ cot φ = [σ 1 −σ3
] - [σ 1 +σ3
sin φ]
2 2 2 2

→ (2) [(c) sin φ cot φ] = [σ1 – (sin φ)( σ 1)] – [σ3 + (sin φ)( σ 3)]

cos φ
→ (2) [(c) sin φ ]= σ1 [1 – (sin φ)] – σ3[1 + (sin φ)]
sin φ

2(c) cos φ (1 + sin φ )


+ (σ 3 ) = σ1
(1 − sin φ ) (1 − sin φ )

cos φ φ (1 + sin φ ) φ
Since ≈ tan (45° + ) and ≈ tan2 (45° + )
(1 − sin φ 2 (1 − sin φ ) 2

 ϕ  ϕ
σ 1 = σ 3 tan 2  45° +  + 2 c tan  45 ° + 
 2  2

258
Chapter 13
Slope Stability
Symbols for Slope Stability

259
*Slope-01: Factor of Safety of a straight line slope failure.
(Revision: Oct.-08)
A slope cut to 1.5H:1V will be made in a shale rock stratum that has bedding planes that have an
apparent dip of 16˚ (see the figure below). If the acceptable factor of safety against failure is at
least 2 along the lower-most bedding plane, is this slope stable? Use a unit weight of 20.1 kN/m3,
and bedding strength parameters of c = 22 kPa and φ = 30˚.

Solution:

T he traingule of rock above the potential slip plane has a weight W per unit width,

( 85.0 m )(11.3 m )  20.1 3  = 9, 650


1 kN kN
W =
2  m  m
T he length L of the slip plane is,
85.0 m
L= = 88.4 m
cos16 °
T herefore,
Resisting For ces cL +  (W ) cos α  tan φ
FS = = =
Driving Forces (W ) sin α
 kN   kN  
 22 2  ( 88.4 m ) +   9, 650  cos16 °  tan 30 °
 m   m  
FS = = 2.7 > 2 OK
 kN 
 9, 650  sin 1 6 °
 m 

260
*Slope-02: Same as Slope-01 but with a raising WT.
(Revision: Oct.-08)
In the previous problem the slope appeared to be stable with a factor of safety = 2.7. What
happens to that factor of safety if the water table rises to the level shown below? Use a unit weight
of 20.1 kN/m3, and bedding strength parameters are reduced by the effective parameters of c’ = 15
kPa and φ’ = 20˚.

Solution:

kN
T h e w e ig h t W o f th e r o c k tr ia n g le p e r u n it w id th is s till 9 , 6 5 0
m
T h e l e n g t h L o f t h e s li p p la n e i s s t i l l 8 8 . 4 m .
T h e p o r e w a t e r p r e s s u r e i s b a s e d o n a n e s t i m a t e o f i t s v a lu e a l o n g t h e l e n g t h L ,
a t w a t e r d e p t h z w a b o v e t h e p l a n e t h a t r a n g e f r o m 0 t o 3 . 2 m ; c o n s e r v a t i v e ly ,
 kN 
u = γz w =  9 .8 1
w  (3 . 2 m ) = 3 1 . 4 k P a
 m3 
R e s is tin g F o rc e s c ' L +  (W ) c o s α − u  t a n φ
FS = = =
D riv in g F o rc e s (W ) s i n α
 kN   kN  
15  (8 8 . 4 m )+   9 , 6 5 0 m  c o s 1 6 ° − ( 3 1 .4 k P a )  t a n 2 0 °
 m2    
FS = = 1 .7 6 < 2 NG
 kN 
 9 , 6 5 0  s i n 1 6 °
 m 
T h e c o m p u t e d f a c t o r o f s a f e t y o f 1 .7 6 i s l e s s t h a n t h e m i n i m u m a c c e p t a b l e
v a lu e o f 2 , t h e r e f o r e t h i s d e s i g n i s N O T a c c e p t a b le . N o t i c e t h a t a r i s i n g W T
d e c r e a s e s th e s ta b ility o f th e s lo p e.

261
*Slope-03: Is a river embankment safe with a large crane?
(Revision: Oct.-08)
Determine if the work site shown below is safe, provided you consider the minimum acceptable
factor of safety for the man-made waterfront slope shown below to be 2. Assume the arc radius is
80 feet; the circular lengths are AB = 22 feet and BC = 102 feet. The total weight of the soil per
unit width are Wsoil = 205 kips and Wcrane =70 kips. The site is located in a seismic zone with a
seismic coefficient of 0.15.

20’ 15’ 40’ 15’ 15’

A
45’
Clayey sand
Wcrane SEA
c = 0.2 ksf

B φ = 40°
C
Wsoil 15’
Sandy clay

c = 1.8 ksf

φ = 15°
Solution:

Mr = R[S1(AB)+S2(BC)] = R[(C1’+σ1’tanφ1)AB + (C2’+σ2’tanφ2)BC]

= 80’[(0.2+0.125(8’)(tan40)22’ + (1.8+(0.130 – 0.064)(21)(tan15)(102)] = 80[23k + 221k] = 19,500 k-ft


W (d )
M 0 = Wb1 - WwH (d 1 ) - WWV (b2 ) + Vb3 + ae 2
g
= 205(40)–(1/2)(0.064)(15)² - (0.064)[30(15)+(1/2)(40)(15)]15+70(55)+205(0.15)(50)

Mo = 8,200 - 7.2 –720 +3850 +1540 = 12,900 k-ft

Therefore: FS = Mr/Mo = 19,500/12,900 = 1.51 Not Good!

Removing Crane  Mo =9,050 k-ft

Therefore: FS = Mr/Mo = 19,500/9050 = 2.15  GOOD!

262
*Slope-04: Simple method of slices to find the FS.
(Revision: Oct.-08)
The stability of a slope was analyzed by the method of slices. One of the trial curved surfaces
through the soil mass yielded the shearing and normal components of each slice as listed below.
The curved length of the trial curved surface is 40 feet, the soil parameters are c = 225 lb/ft2 and φ
= 15º. Determine the factor of safety along this trial surface.

Solution:

Slice Shearing Component Normal Component


Number (W sin α) (lb/ft) (W cos α) (lb/ft)
1 -38 306
2 -74 1410
3 124 2380
4 429 3050
5 934 3480
6 1570 3540
7 2000 3210
8 2040 2190
9 766 600
∑ = 7,751 lb/ft ∑ = 20,166 lb/ft

cL + ∑ (W cos α ) tan nφ (225 psf )(40 ft ) + 20,166 plf


FS = = = 1.86 < 2 NG
∑W sin α 7, 751 plf

263
**Slope-05: Method of slices to find the factor of safety of a slope with a WT.
(Revision: Oct.-08)
lb
A 30 ft tall, 1.5H:1V slope is to be built as shown below. The soil is homogeneous, with c ' = 400
ft 2
and φ ' = 29° . The unit weight is 119 pcf above the groundwater table, and 123 pcf below. Using
the ordinary method of slices, compute the factor of safety along the trial circle.

Solution:

Weights:

W1  1 0 .3  lb
= 1 0 .8  119 = 6, 620
b  2  ft
W2  1 0 .3 + 1 2 .5   5 .2  lb
= 9 .4   1 1 9 + 9 .4   1 2 3 = 1 5, 8 0 0
b  2   2  ft
W3  1 2 .5 + 1 4 .6   5 .2 + 1 0 .0  lb
= 1 2 .1   1 1 9 + 1 2 .1   1 2 3 = 3 0, 8 0 0
b  2   2  ft
W4  5 .0   1 2 .9 + 8 .0  lb
= 2 .9   1 7 .0 + 7 .1   1 7 .8 = 1 6 2 0
b  2   2  ft

W5  1 6 .8 + 12 .8   10 .7 + 7.3  lb
= 9.3   11 9 + 9.3   1 23 = 3 9, 90 0
b  2   2  ft
W6  12 .8 + 9 .9   7.3  lb
= 7 .6   1 19 + 7.6   1 2 3 = 2 6, 70 0
b  2   2  ft
W7  9 .9  lb
= 4 .0   1 19 = 2, 4 00
b  2  ft

264
Average pore water pressure at base of each slice:

u1 = 0
 5 .2  lb
u2 =   6 2 .4 = 1 6 0
 2  ft 2

 5 .2 + 1 0 .0  lb
u3 =   6 2 .4 = 4 7 0
 2  ft 2
 1 0 .0 + 1 0 .7  lb
u4 =   6 2 .4 = 6 5 0
 2  f t2
 1 0 .7 + 7 .3  lb
u5 =   6 2 .4 = 5 6 0
 2  ft 2
 7 .3  lb
u6 =   6 2 .4 = 2 3 0
 2  f t2
u7 = 0

c 'l +
W α  lb  φ  lb  W
Slice ( lb) c ' 2  u 2  l ( ft ) W  ( lb )
b ( Deg )  ft  ( Deg )  ft    ( cosα − ul ) tan φ ' b
b
1 6620 -18 400 29 0 11.4 8,000 -2,000
2 15,800 -7 400 29 160 9.5 11,700 -1,900
3 30,800 8 400 29 470 12.2 18,600 4,300
4 39,900 24 400 29 650 13.9 20,800 16,200
5 26,700 38 400 29 560 11.8 12,700 16,400
6 13,700 53 400 29 230 12.6 8,000 10,900
7 24,00 67 400 29 0 10.2 4,600 2,200
Σ = 84, 400 Σ = 46,100

Therefore, the factor of safety is,

W 
c ' l +   ( cos α − ul ) tan φ '
FS = b =
84, 400
= 1.83 < 2 NG
W  46,100
 
b

Note how slices # 1 and 2 have a negative α because they are inclined backwards.

265
**Slope-06: Swedish slip circle solution of a slope stability.
(Revision: Oct.-08)
Using the Swedish slip circle method, compute the factor of safety along the trial circle shown in
the figure below.

Solution:

Divide the slide mass into vertical slices as shown. One of the slice borders should be directly below the
center of the circle (in this case, the border between slices 2 and 3). For convenience of computations,
also draw a slice border wherever the slip surface intersects a new soil stratum and whenever the ground
surface has a break in slope. Then, compute the weight and moment arm for each slide using simplified
computations as follows:

266
Solution:

Weights

W1  2 .0  kN
= 4 .6   1 7 .8 = 8 0
b  2  m
W2  2 .0 + 9 .8  kN
= 7 .0   1 7 .8 = 1 3 0
b  2  m
W3  9 .8 + 1 2 .9  kN
= 2 .9   1 7 .8 = 5 9 0
b  2  m
W4  5 .0   1 2 .9 + 8 .0  kN
: = 2 .9   1 7 .0 + 7 .1   1 7 .8 = 1 6 2 0
b  2   2  m
W5  5 .0 + 1 0 .3   8 .0  kN
= 7 .2   1 7 .0 + 7 .2   1 7 .8 = 1 4 5 0
b  2   2  m
W 6  1 0 .3 + 9 .8  kN
= 0 .8   1 7 .0 = 1 4 0
b  2  m
W7  9 .8  kN
= 5 .1   1 7 .0 = 4 2 0
b  2  m

Moment arms:

4 .6
d 1 = − 7 .0 − = − 8 .5 m
3
− 7 .0
d2 = = − 3 .5 m
2
2 .9
d3 = = 1 .5 m
2
7 .1
d4 = 2 .9 + = 6 .5 m
2
7 .1
d5 = 2 .9 + 7 .1 + = 1 0 .9 m
2
0 .8
d6 = 2 .9 + 7 .1 + 7 .2 + = 1 7 .6 m
2
5 .1
d7 = 2 .9 + 7 .1 + 7 .2 + 0 .8 + = 1 9 .7 m
3

267
Slice Su ( kPa ) θ ( Deg ) Su θ W  kN  d ( m) W 
    d
b  m  b 
1 80 -8.5 -690
2 130 -3.5 -450
3 80 76 6080 390 1.5 890
4 1620 6.5 10,530
5 1450 10.9 15,800
6 140 17.6 2,460
40 30 1200
7 420 19.7 8,280
Σ = 7280 Σ = 36,830

∑Sθ π ( 23.6 )
2
π R2 7, 280
FS = u
= = 1.9 2 < 2 Not Goo d
180 W  180 36, 8 3 0
∑  b  d

268
Chapter 14
Statistical Analysis of Soils
Symbols for the Statistical Analysis of Soils

269
Chapter 15
Lateral Pressures from Soils
Symbols for Lateral Pressures from Soils

Dx → Diameter of the grains distributed (represent % finer by weight).


e →The voids ratio.
GS→ Specific gravity of the solids of a soil.
H→ Maximum depth of excavation or thickness of a soil layer.
hsoil→ depth of the soil.
icritical→ Critical hydraulic gradient.
kH → Horizontal permeability.
kV → Vertical permeability.
u → pore water pressure.
σ’→ Effective stress.
σV’→ Vertical Effective stress.
g’→ Bouyant unit weight of a soil.
γSAT →Saturated unit weight of a soil
γW →Unit weight of water.
VW → Volume of water.
w→ water content.

270
Formulas and Figures for Lateral Stresses.

Figure for symbols used in the Coulomb earth pressures.

Coulomb’s lateral pressure coefficients Ka and Kp.

cos2 (φ - θ)
Ka = 2
 sin(δ + φ )sin(φ − α ) 
cos2θ cos(δ + θ ) 1 + 
 cos( δ + θ )cos(θ − α ) 
cos2 (φ + θ)
Kp = 2
 sin(φ − δ )sin(φ + α ) 
cos2θ cos(δ − θ ) 1 − 
 cos(δ − θ )cos(α − θ ) 

271
Ka for the case of θ = 0º and α = 0º.

Ka for the case where δ = 2/3 φ.

272
Kp for θ = 0º and α = 0º.

Failure modes for flexible walls (sheet-piling).

273
*Lateral-01: A simple wall subjected to an active pressure condition.
(Revision: Sept.-08)
Consider a small 10-foot tall and 3 feet thick concrete retaining wall. The backfill behind the wall
will be from local sandy gravel with a dry unit weight of 115 pcf and an angle of internal friction
of 30 degrees. The wall will not have to retain water.

Estimate, (a) the lateral force on the wall from the backfill in an active pressure condition, (b) its
stability against overturning, and (c) its stability against sliding (use a Factor of Safety ≥ 2).

Solution:
 φ  30° 
(a) The Rankine active earth pressure coefficient is, K a = tan 2  45° −  = tan 2  45° −  = 0.33
 2  2 
The lateral pressure at the bottom of the wall is pa = γ hK a = ( 0.115 kcf )(10 ft )( 0.33) = 0.38 ksf
1
The force against the wall is Fa = ( pa ) h = ( 0.5 )( 0.38)(10 ) = 1.9 kips per foot of wall
2

(b) The stability of the wall against overturning is found by taking moments about the point "O"
at the toe of the wall,

Factor of Safety ( FS ) =
resisting moment
=
( 3')(10 ')(1')( 0.150 kcf )(1.5 ft ) = 1.07 < 2 NG
overturning moment (1.9 kips )(10 / 3 ft )

(c) The stability of the wall against sliding towards the left is found by,
resisting force ( 3')(10 ')(1' )( 0.150 kcf )( tan 30° )
Factor of Safety ( FS ) = = = 1.37 < 2 NG
driving force (1.9 kips )

274
*Lateral–02: Compare the Rankine and Coulomb lateral coefficients.
(Revision: Sept-2008)
(a) Compare the Rankine and Coulomb lateral earth pressure coefficients for a wall that retains a
granular backfill soil with φ = 35°, δ = 12°, θ = 0º and α = 20°. (Note: δ is the angle of friction
between the soil and the backside of the wall; α is the angle of the slope for the backfill behind
the wall and θ is the back of the wall’s angle with respect to the vertical).
(b) What is the passive earth force on the wall at failure if the wall is 10 m high, γ = 18.1 kN/m3
and c = 9 kN/m2?

Solution:
a) Rankine’s active and passive earth pressure coefficients,
φ 35°
K a = tan 2 (45° - ) = tan 2 (45° - ) = 0.271
2 2
φ 35° 1
K p = tan 2 (45° + ) = tan 2 (45° + ) = 3.690 Note that Ka =
2 2 KP
Coulomb’s active and passive earth pressure coefficients,

cos2 (φ - θ) cos2 (35 - 0)


Ka = 2
= 2
= 0.323
 sin(δ + φ )sin(φ − α )   sin(12 + 35)sin(35 − 20) 
cos2θ cos(δ + θ ) 1 +  cos2 0 cos(12 + 0) 1 + 
 cos(δ + θ )cos(θ − α )   cos(12 + 0)cos(0 − 20) 
cos2 (φ + θ) cos2 (35 + 0)
Kp = 2
= 2
= 3.517
 sin(φ − δ )sin(φ + α )   sin(35 − 12)sin(35 + 20) 
cos2θ cos(δ − θ ) 1 −  cos 0 cos(12 − 0) 1 −
2

 cos(δ − θ )cos( α − θ )   cos(12 − 0)cos(20 − 0) 
When α = 0º, θ = 0º and δ = 0º the Coulomb formula becomes identical to Rankine’s.

b) Therefore, the Rankine coefficient is 3.690 versus 3.517 for Coulomb’s. Using these values, the
total passive force Fp on the wall per unit length is,

Rankine ' s Fp = 0.5γ h2 K p + 2ch K p = ( 0.5)(18.1)(10) ( 3.690) + 2 ( 9)(10) 3.690 = 3,685 kN / m2


2

Coulomb ' s Fp = 0.5γ h2 K p + 2ch K p = ( 0.5)(18.1)(10) ( 3.517) + 2 ( 9)(10) 3.517 = 3,520 kN / m2


2

275
*Lateral-03: Passive pressures using the Rankine theory.
(Revision: Sept-08)
Using the Rankine method, find the magnitude and location of the passive pressure force Fp with
respect to the heel of the wall (point B), exerted upon a temporary retaining wall by a large
jacking system (which is not shown in the figure).

Solution:

276
*Lateral-04: The “at-rest” pressure upon an unyielding wall.
(Revision: Sept-08)

Find the lateral “at-rest” force F o on the wall and its location with respect to the top of the wall.
Given: Sand #1 has a unit weight of 105 pcf, c = 0 psf and φ = 30º; Sand #2 has a unit weight of 122
pcf, c = 0 psf and φ = 30º.

Solution:

From Jaky's empirical relation, K o = 1 − sin φ ' = 1 − sin 30° = 0.50


at z = 0 feet σ ' = 0 ksf , because there is no surcharge loading upon the surface of Sand #1.
at z = 10 feet σ 'h = K oσ 'v = ( 0.5 )( 0.105kcf )(10 ft ) = 0.525 ksf
at z = 20 feet σ 'h = ( 0.5 ) ( 0.105 )(10 ) + ( 0.122 − 0.0624 )10  = 0.823 ksf
σ w = γ w h = ( 0.0624 pcf )(10 ft ) = 0.624 ksf
1 1 1
Fo = ∑ f i = F 1 + F 2 + F 3 + F 4 = ( 0.525)(10 ) + ( 0.525 )(10 ) + ( 0.302 )(10 ) + ( 0.624 )(10 )
i 2 2 2
kip
Fo = 2.63 + 5.25 + 1.49 + 3.12 = 12.5
ft

z=
( 2.63)( 6.67 ) + ( 5.25 )(15 ) + (1.49 )(16.67 ) + ( 3.12 )(16.67 ) = 173.1 kip − ft
12.5 kip 12.5 kip
z = 13.8 ft from the top of the wall.

277
*Lateral-05: The contribution of cohesion to reduce the force on the wall.
(Revision: Sept-08)
A 21 foot high retaining wall supports a purely cohesive soil (φ = 0°) with a cohesion of 630 psf and
a unit weight of 113 pcf. Find:
(a) The Rankine active earth pressure on the wall.
(b) Estimate the depth of separation of the clay from the wall, and (c) find the lateral force upon
the wall whilst considering the clay separation.

21 ft

γ zK a - 2c K a

Solution:

a) The coefficient of active earth pressure is,


 φ  0° 
K a = tan 2  45° −  = tan 2  45° −  = tan 2 45° = 1
 2  2
The net active earth pressure pa on the wall is,
pa = σ 3 = γ hK a − 2c K a
= ( 0.113 kcf )( 21 ft )(1) − 2 ( 0.630ksf ) 1 = 2.37 − 1.26 = 1.11 ksf

b) The crack stops where the pressure is zero, pa = 0,


pa = γ hK a - 2c K a ∴ γ hK a = 2c K a
2c K a 2c 2 ( 0.630ksf )
∴ hcrack = = = = 11.2 feet
γ Ka γ K a ( 0.113kcf )(1)
c) The total (Rankine) active earth force upn the wall Fa is,
1
Fa = γ H 2 K a − 2cH K a
2
but there is no contact on the wall where the tension crack exists, therefore
  1
Fa =
1
2
(γ HK a − 2c K a )  H − γ 2cK  = γ H − 2cH K a +

2 2c 2
γ
 a  2
2 ( 0.63ksf )
2
1
Fa = ( 0.113kcf )( 21 ft ) (1) − 2 ( 0.63ksf )( 21 ft )(1) + = 5.48 k / ft of wall
2

2 ( 0.113kcf )

278
**Lateral-06: The effect of a rising WT upon a wall’s stability.
(Revision: Sept-08)
A 4 m wall retains a dry sand backfill with a unit weight of 18.3 kN/m3, an angle of internal
friction of 36˚ and a porosity of 31%. The backfill is fully drained through weep holes.
1) What is the magnitude of the backfill force on a 1 m wide slice of wall if it is not allowed to
deflect?
2) What is the magnitude of the backfill force on the same 1 m wide slice, if the wall does deflect
enough to develop a Rankine active earth pressure condition?
3) What is the new force on the wall, and its location from its heel, if the wall’s weep holes are
clogged and the water table now rises to within 1 m of the ground surface behind the wall?

Solution:

1) No deflection of the wall means the soil is "at rest" and K 0 = 1 - sin φ = 1 - sin 36 ° = 0.41
 kN 
The force Fo = ½ γ d h 2 K o = ½  18.3 3  ( 4 m ) (0.41) = 60 kN per meter of wall
2

 m 
2) When th e wall deflects to the left sufficiently to develop an active pressure condition,
 φ  36 ° 
K a = tan 2  45 ° −  = tan 2  45 ° −  = 0.26
 2  2 
 kN 
The force Fa = ½ γ d h 2 K a = ½  18.3 3  ( 4 m ) (0.26) = 38 kN per meter of wall
2

 m 
3) The buoya nt weight γ ' of the flooded sand is,
kN
γ ' = γ sat − γ w = γ d − nγ w − γ w = 18.3 + (0.31) ( 9.81) − ( 9.81) = 11.5 3
m
The stress at point "a" is σ a =0, and at "b" which is 1 meter below the surface,
kN
σ b = γ d hK a = (18.3 )(1m )(0.26) = 4.8 kN / m 2 ∴ F1 = ½(4.8 kN / m 2 )(1m ) = 2.4 kN / m
m3

279
kN
σ bc = γ d hK a = (18.3 3
)(1m)(0.26) = 4.8 kN / m 2 ∴ F2 = (4.8kN / m 2 )(3m) = 14.4 kN / m
m
kN
σ c = γ ' hK a = (11.5 3 )(3m)(0.26) = 9.0 kN / m 2 ∴ F3 = ½(9.0kN / m 2 )(3m) = 13.5 kN / m
m
The water pressure and force,
kN
σ w = γ w h = (9.81 )(3m) = 29.4 kN / m 2 ∴ F4 = ½(29.4kN / m 2 )(3m) = 44.1 kN / m
m3
4
Therefore R = ∑F
i =1
= 74.4 kN / m

The location of the resultant is y,


F1d1 + F2 d 2 + F3 d 3 + F4 d 4 ( 2.4 )( 3.33m ) + (14.4 )(1.5m ) + (13.5 )(1m ) + ( 44.1)(1)
y= =
R 74.4
y = 1.17 m from the bottom of the wall.

The percent increase in load upon the wall due to flooding is,

∆F =
( 74.4kN − 38kN ) = 96% increase.
38kN

280
*Lateral-07: The effects of soil-wall friction upon the lateral pressure.
(Revision: Sept-08)
A 7.0 m high retaining wall has a horizontal backfill of dry sand with a unit weight of 17.2 kN/m3
and an angle of internal friction φ = 32˚. The wall is cast-in-place concrete, with a friction angle δ
= 20˚. Ignoring the effect of the passive pressure upon the toe of the footing, find the magnitude of
the active earth force upon a length of wall equal to 3.5 m assuming Rankine conditions.

Solution:

The force applied to the wall first requires the coefficient of active earth pressure,
φ 32°
K a = tan 2 (45° − ) = tan 2 (45° − ) = 0.307
2 2
The horizontal force FH per unit width of wall is,
FH = ½γ h 2 K a = ½(17.2kN / m3 )(7 m) 2 (0.307) = 129.5 kN / m
The FH is related to the total force R on the wall as a function of the angle of wall friction δ ,
FH 129.5kN / m
FH = R cos δ ∴ R = = = 138 kN / m
cos δ cos 20°
We are asked what is the total force every 3.5 m,
Total Active Force every 3.5 m = (138 ) (3.5m) = 482kN

FH
δ
FV R

281
*Lateral-08: What happens when the lower stratum is stronger?
(Revision: Sept-08)
Calculate the active force Fa and its location ŷ with respect to the heel of the 6 m wall (point A), for
the worst case (clogged weep holes).

1m
WT worst load case

Medium dense sand γsat = 18.5 kN/m3 3m


φ = 30°
H=6m
Weep holes γ = 21.2 kN/m3
3m
φ = 90°
Porous limestone
A 0

S o lu tio n .
T h e w o r s t a c t i v e p r e s s u r e lo a d o c c u r s w h e n t h e w a t e r ta b le r a i s e s t o th e to p o f t h e w a ll.
ϕ 30°
K a sa nd = ta n 2 ( 4 5 ° − ) = ta n 2 ( 4 5 ° − ) = 0 .3 3 3
2 2
ϕ 90°
K a lim e s to n e = t a n 2 ( 4 5 ° − ) = ta n 2 ( 4 5 ° − ) = 0 ∴ t h e li m e s to n e d o e s n o t lo a d th e w a ll.
2 2
T h e φ = 9 0 ° i s r e a lly a c o m b in a ti o n o f s h e a r a n d c o h e s io n ( " c e m e n t a ti o n " ) .
p 1 = γ ' h1 K a = (γ SAT -γ w ) h1 K a = (1 8 . 5 - 9 . 8 ) ( 3 ) ( 0 . 3 3 ) = 8 .7 k N / m 2

p2 = γ w H = (9 .8 )( 6 m ) = 5 8 .8 k N / m 2

F1 = ½ p 1 h1 = ( 0 . 5 ) ( 8 . 7 ) ( 3 m ) = 1 3 .1 k N / m
F 2 = ½ p 2 H = ( 0 .5 )(5 8 .8 )( 6 m ) = 1 7 6 .4 k N / m
F to ta l = 1 8 9 .5 k N / m

T h e lo c a tio n y =
y1 F1 + y 2 F 2
=
(4 m ) (1 3 . 1 ) + ( 2 m ) (1 7 6 . 4 ) = 2 .1 m fro m A .
F to ta l (1 8 9 . 5 )

h1 +
H F1 Ftotal

p1
h2 F2 ŷ

A
p2

282
*Lateral-09: Strata with different parameters.
(Revised Oct-09)
Draw the pressure diagram on the wall in an active pressure condition, and find the resultant Ftotal
on the wall and its location with respect to the top of the wall.
q = 2.5 ksf
w.t.
a 0.83

c=0
γ = 115 pcf 10’ 1
ф = 30°
H = 20’ 2
b 0.83
+
c=0 0.66 0.18
5
γ = 125 pcf
10’
ф = 40° 3 4

c 1.25
0.66 0.13

Solution:

Step 1

Ka1 = tan2 (45°- 30°/2) = 0.333


Ka 2 = tan2 (45°- 40°/2) = 0.217

Step 2

The stress on the wall at point a is: pa = q Ka 1 = (2.5) (0.333) = 0.83 ksf
The stress at b (within the top stratum) is: pb’+ = (q + γ’h) Ka 1

= [2.5 + (0.115 - 0.0624) (10’)] [0.333] = 1.01 ksf

The stress at b (within bottom stratum) is: pb’ - = (q + γ’h) Ka 2

= [2.5 + (0.115 – 0.0624) (10’)] [0.217] = 0.66 ksf

The stress at point c is: pc’ = [q + (γ’h)1 + (γ’h)2] Ka 2

= [2.5 + (0.115 – 0.0624) (10’) + (0.125 – 0.0624)(10’)] [0.217] = 0.79 ksf

The pressure of the water upon the wall is: pw = γwh = (0.0624) (20’) = 1.25 ksf

Step 3
283
The forces from each area:

F1 = (10’) (0.83) = 8.30 kips/ft

F2 = ½ (10’)(0.18) = 0.90 kips/ft

F3 = (10’) (0.66) = 6.60 kips/ft

F4 = ½ (10’)(0.13) = 0.65 kips/ft

F5 = ½ (1.25) (20’) = 12.5 kips/ft

Ftotal = 29.0 kips/ft

Step 4

The location of forces ŷ is at:

5 ⋅ 8.3 + 20 ⋅ 0.9 + 15 ⋅ 6.6 + 50 ⋅ 0.65 + 40 ⋅ 12.5


yˆ = 3 3 3 = 0.66 ksf
29

The stress at point c is: ŷ = 11.2 feet from top of wall

284
*Lateral-10: The effects of a clay stratum at the surface.

The sheet pile wall shown below is flexible enough to permit the retained soil to develop an active
earth pressure condition. Calculate the magnitude of the resultant Ftotal of the active force above
the point “A” upon the wall. Assume Rankine conditions.

Solution:

Notice that the vertical pressure diagram will always increase in magnitude, but the horizontal pressures
are governed by the Ka coefficient, which may increase or decrease the pressures on the wall.

Surcharge q = 0.84 ksf


-0.25
-0.84
c+ 0

1
3.25’
no water present

20’
Sandy clay
16.75’
c = 500 psf 2
φ = 10 °
b+
b- +0.48
Dense sand +1.29
10’
c=0 3 4
A φ = 40° a
γ = 130 pcf +0.48 +0.77

Lateral load from the surcharge

σc+ = Ka1 q = (0.70)(0.84 kcf) = 0.59 ksf


σc- = -2c K a1 = -2(0.5) 0.70 = -0.84 ksf
∴ ∑ σc = 0.59 - 0.84 = -0.25 ksf

σb+ = Ka 1 γ h – 2c K a 1 + q Ka
= (0.7) (0.11) (20’) – (2) (0.50) 0.70
= 1.29 ksf

285
σb- = Ka 2 γ h – 0 = (0.22) (0.11) (20’) = 0.48 ksf

σa = 0.48 + Ka 2 γ h = 0.48 + (0.22)(0.13)(10’) = 0.48 + 0.29 = 0.77 ksf

Ka 1 = tan2 (45° - φ /2) = tan2 40° = 0.70 F1 = ½ (-0.25)(3.25’) = - 0.41 k/ft (tension).
Ka 2 = tan2 (45° - 40° / 2) = tan2 25° = 0.22 F2 = ½ (1.29)(16.75’) = +10.80 k/ft
F3 = (0.48)(10’) = + 4.80 k/ft
F4 = ½ (0.29)(10’) = + 1.45 k/ft
Ftotal = +16.6 kip/ft

286
**Lateral-11: Anchoring to help support a wall.
(Revision: Sept.-08)
The wall shown below will be used to retain the sides of an excavation for the foundations of a
large building. The engineer has decided to use earth anchors in lieu of braces or rakers to
stabilize the wall.
a) What is the minimum distance x from the anchor to behind the wall?
b) What is your recommended factor of safety for the anchor? What is an economical load for the
anchor?
x

5’

Grouted anchor A
24’

φ = 30°
c = 150 psf
O
.

Solution:

(1) The anchor must be beyond the passive slip plane, or (x) tan 30º = 19’ or x = 33 feet.

(2) Ka = tan2(45º - φ/2) = 0.33 and Kp = tan2(45º + φ/2) = 3.0

The active force upon the wall per unit width Fa is:

Fa = ½γH2Ka -2cH K a = ½(0.105)(24’)2(0.33) - 2(0.15)(24) 0.33 = 5.84 kip/ft with the force
located at ŷ = ⅓(19’) = 6.33’ above point O (note that the tensile portion does not load the wall).

The potential passive failure force (from the anchor) on the wall Fp is:

Fp = ½γH2Kp + 2cH K p = ½ (0.105)(24)2(3) + 2(0.15)(24) 3 = 103 kip/ft

287
The factor of safety should be the same for an active failure as a passive failure. Therefore, a simple
Fp Fp 103 kips
equation could be written as, Fa ( FS ) = or (FS)2 = = = 17.6 ∴ FS = 4.2
( FS ) Fa 5.84 kips

Note that this corresponds to a load in the anchor of (5.84)(4.2) = 24.5 kips/ft (which is the same as
using the passive force = (103)/(4.2) = 24.5 kips/ft). The horizontal spacing of the anchors is not
influenced by this analysis, and depends on cost factors. A common spacing would be 10 feet, which
means A = 245 kips.

288
**Lateral-12: The effect of five strata have upon a wall.
(Revision Oct-09)
Plot the pressure diagram and find the resultant force F and its location under an active pressure
condition.

At h=0’ p1 = q K1a = (2) (0.307) = 0.614 ksf

at h = -6’ ∆p2 = γ1h K1a = (0.110)(6) (0.307) = 0.203 ksf

at h = -8’ ∆p3 = (γ2 - γw)h K2a = (0.125 - 0.0624)(2)(0.333) = 0.417 ksf

at h = -(8+dh)’ = [q + (γ1) 6’ + (γ2 - γw) 2’] K3a – 2c √(K3a from p = γh Ka - 2c√Ka

= [2 + (0.11)6’ + (0.125 – 0.0624)2’](0.704) – 2(0.6)(0.84) = 0.95 ksf

at h= -17’ ∆p4 = (γ3 - γw)h K3a = (0.126-0.0624)(9)(0.704) = 0.403 ksf ∴0.95+0.403 = 1.35 ksf

at h = -(17 + dh)’ = [2 + 0.66 + 0.125 + (0.0626) (9)](1) –2(0.8)(1) = 1.76 ksf

at h = -25’ ∆p5 = (γ4 - γw)h K4a = (0.120 - 0.0624)(8)(1) = 0.46 ksf ∴ 1.76 + 0.46 = 2.22 ksf

at h = -(25 + dh)’ = [2 + 0.66 + 0.125 + 0.572 + 8(0.120 – 0.0624)](0.49) – 2(0.4)(0.7) = 1.13 ksf

at h = -30’ ∆p6 = (γ5-γw)h K5a = (0.120-0.0624)(5) (0.49) = 0.141 ksf ∴1.31+0.14 = 1.45 ksf

289
F1 = (0.614)(6) = 3.68 kips The resultant R is, R = ∑ Fi = 57.1 kips

F2 = 0.5(0.203)(6) = 0.61 kips

F3 = (0.817)(2) = 1.63 kips

F4 = 0.5(0.042)(2) = 0.04 The location of R is…….∑M0 = 0 (about 0)

F5 = (0.95)(9) = 8.55 kips 57.09(y) = (3.68)(27) + (0.61)(26) = (1.63)(23)

F6 = 0.5(0.40(9) = 1.80 kip

F7 = (1.758)(8) = 14.1 kips

F8 = 0.5(0.461)(8) = 1.84 kips

F9 = (1.31)(5) = 6.55 kips

F10 = 0.5(0.141)(5) = 0.35 kips

F11 = 0.51(1.50)(24) = 18.0 kips ∴ y = 611 / 57.1 = 10.7 feet above “0”

57.1 kips

290
**Lateral-13: The stability of a reinforced concrete wall.
(Revised Oct-09)
Calculate the Factor of Safety against, (a) overturning, (b) sliding, and (c) bearing capacity
failures.

0.4
m α = 10ο

0.62m

4
H = 8
m
1
H = 9.58m

1.5 m 0.6 m 3 3.5 m


0.75
m

0.96m

0
yφc

kN
γ1 = 16.8 kN o
3 γ conc = 23.6 φ1 = 32 c1 = 0
m 3
m

kN o kN
γ2 = 17.6 φ2 = 28 c2 = 30
3 2
m m
291
cos β − cos 2 β − cos 2 φ ' cos10o − cos 2 10o − cos 2 32o
Ka = cos α = cos10o = 0.322
cos β + cos 2 β − cos 2 φ ' cos10o + cos 2 10o − cos 2 32o

Fa = (1/2) H2 γ1 Ka = (1/2)(9.58 m)2(16.8 kN/m3)(0.322) = 248 kN/m

Fv = Fa sin10° = (248 kN/m)(0.174) = 43.1 kN/m

Fh = Fa cos10° = (248 kN/m)(0.985) = 244 kN/m

a) The factor of safety against overturning is found by taking moments about point “O”.

The resisting moment against overturning is MR,

MR = 23.6 kN/m3[(0.4m)(8m)(1.90m) + (1/2)(0.2m)(8m)(1.63m) + (0.96m)(5.6m)(2.8m)] (1m)

+ 16.8 kN/m3 [(3.5m)(8m)(3.85m) + (1/2)(0.617m)(3.5m)(4.43m)] (1m)

+ 43.1 kN/m (5.6m)(1m) = 2661 kN-m

and the overturning moment is MO = Fh (1/3) H’ = 244 kN/m (9.58m)(1/3) = 777 kN-m

FSO = MR / MO = 3 .42

b) The factor of safety (FSS) against sliding failure,

K1 = K2 = 2/3

Kp = tan2( 45° + 28°/2 ) = 2.77

Fp = (1/2) γ2 H2 Kp + 2 c2 H Kp

= (0.5)(2.77)(17.6 kN/m3)(1.75 m)2 + (2)(30kN/m2) 2.77 (1.75 m) = 249 kN/m

the driving force = Fh = 244 kN/m

the resisting force = FR = ΣV tan(2/3)(28) + (5.6)(2/3)(30) = 355 kN/m

FSS = Fh / FR = 1.46

c) the factor of safety (FSBC) against a bearing capacity failure,

292
B ΣMr − ΣMo 2661 − 777
e= − = e = 2 .8 − = 0.31 m
2 ΣV 749

ΣV  6e  749  6(0.31) 
1 + B  = 5.6 1 + 5.6  = 178 kN/m
2
qtoe =
B

B’ = B – 2e = 5.6m – 2 (0.31m) = 4.98 m

qu = (1/2) γ2 B’ Nγ Fγds Fγid + C2 Nc Fcds Fcid + q Nq Fqds Fqid

q = γ2 D = (17.6)(1.75) = 30.8 kN/m2

using φ2 = 28° → Nc = 25.8

Nq = 14.7

Nγ = 16.7

Df 1.75
Fcd = 1 + 0.4 = 1 + 0 .4 = 1.14
B' 4.98

 Ph 
ψ = tan −1   = tan (244 / 749) = 18.04°
-1

 ΣV 

 ψ 
Fqi = Fci = 1 −  2 = 0.96
 90 

2
 ψ
2
  18.04 
Fγi = 1 −  = 1 −  = 0.58
 φ   28 

Fγd = 1

2  Df 
Fqd = 1 + 2 tan φ (1 − sin φ )  2
 = 1 + 2 tan28°(1-sin28°) (1.75/5.6)) =
 B 

= 1.08

qu = (1/2)(17.6)(4.98)(16.7)(1)(0.58) + (30)(25.8)(1.14)(0.96) + (30.8)(14.7)(1.08)(0.96) =

= 1740 kN/m2

FSBC = qu / qtoe = 1740 / 178.00 = 9.78

293
***Lateral-14: Derive a formula that provides K and σH as a function of σv.
(Revised Oct-09)
Using the Mohr-Coulomb failure criterion combined with Rankine’s theory, find the coefficient of
active earth pressure Ka as an investigation of the stress conditions in soil at a state of plastic
equilibrium (in other words, when the soil mass is on the verge of failure).

Solution:

The definition of an “active pressure” condition is when σh decreases until it touches point D on the
Mohr-Coulomb failure envelope.

Find σa:
From the figure,
CD CD
sin φ = =
AC AO + OC
σ −σ3  σv −σa 
CD = radius of failure circle =  1 = 
 2   2 
AO = c cot φ
σ +σ3  σv −σ a 
OC =  1 = 
 2   2 
Substituting values into the equation for CD, AO and OC gives:

294
σv −σa 
 
sin φ =  2 
 σv +σ a 
c cot φ +  
  2 
Rearrange the equation to make σa the subject:
This gives:
σ +σa  σ −σ a
2c.cos φ +  v  sin φ = v
 2  2
Τhis then gives:
σa (sin φ + 1) = σv (1 – sin φ) − 2c.cos φ
 1 − sin φ   cos φ 
∴σa = σ v   − 2c 
 1 + sin φ   1 + sin φ 
Solution of the trigonometric expressions:
2α + (90° + φ) = 180°
90° + φ φ
α= = 45° –
2 2
1 + sin φ
= tan (α + φ)
cos φ
φ
= tan (45° − +φ )
2
 φ
= tan  45° + 
 2

 φ
sin  45° + 
=  2
 φ
cos 45° + 
 2

 φ
cos 45° + 
cos φ  2
∴ =
1 + sin φ  φ
sin  45° + 
 2
But for any complementary angles β and (90° - β), cos β = sin (90° - β).

φ φ φ φ
Thus, cos (45° sin (45° - and sin (45° cos (45° 
2 2 2 2

295
 φ
sin  45° − 
cos φ  2  φ
∴ = = tan  45° − 
1 + sin φ  φ  2
cos 45° − 
 2

1 − sin φ  1 − sin φ   1 + sin φ  12 − sin 2 φ cos 2 φ  φ


∴ =   ×   = = = tan 2  45° − 
1 + sin φ  1 + sin φ   1 + sin φ  (1 + sin φ ) 2
(1 + sin φ ) 2
 2

φ φ
σa = σv tan2 (45° - ) - 2c tan (45° - )
2 2 Rankine’s expression
gives the effective
pressure at failure!!!

φ
∴ σa = (γz) Ka - 2c ( Ka ) ½ ; where Ka = tan2 (45° - )
2

Using this equation, the slip planes can be described by the grid of lines shown below:

296
**Lateral-15: The magnitude and location of a seismic load upon a retaining wall.
(Revision: Sept-08)
The reinforced concrete retaining wall shown below will be subjected to a horizontal seismic load
of 0.2 g without a vertical component. Determine,
(a) The magnitude of the active earth force Pa on the wall;
(b) The magnitude of the earthquake active earth force Pae on the wall;
(c) The location of the resultant of both forces.

1m

∆Pae increase due to the earthquake load located at 0.6H


Pae the earthquake load
H=5m Pa lateral load from the soil located at 0.33H

Dense sand γ = 18 kN/m3


φ = 36°
0

Solution.

Calculate the coefficient of active earth pressure (Coulomb) K a using,


φ = 36°, α = 0°, β = 90° and δ = 2 / 3φ = 24°,
sin 2 ( β + φ ')
Ka = 2
 sin (φ '+ δ ) sin (φ '− α ) 
sin β sin ( β − δ ) 1 +
2

 sin ( β − δ ) sin (α + β ) 
sin 2 ( 90° + 36° )
Ka = 2
 sin ( 36° + 24° ) sin ( 36° − 0° ) 
sin ( 90° ) sin ( 90° − 24° ) 1 +
2

 sin ( 90° − 24° ) sin ( 0° + 90° ) 
sin 2 (126° ) 0.654
Ka = 2
= 2
= 0.2346
 sin ( 60° ) sin ( 36° )   ( 0.866 )( 0.588) 
sin ( 66° ) 1 +  ( 0.914 ) 1 + 
 sin ( 66° )   ( 0.914 ) 

297
The active earth force Pa is,

Pa = γ H 2 K a = ( 0.5 ) (18kN / m3 ) ( 5m ) ( 0.2346 ) = 53 kN / m


1 2

2
Calculate the earthquake coefficient of active earth pressure (Coulomb) K ae ,
 k 
kh = 0.2, kV = 0, δ = 2 / 3φ = 2 / 3 ( 36° ) = 24° and θ ' = tan −1  h  = tan −1 ( 0.2 ) = 11.3°
1 − kV 
sin 2 (φ + β − θ ')
∴ K ae = 2

cos θ 'sin 2 β sin ( β − θ '− δ ) 1 +
sin ( φ + δ ) sin ( φ − θ '− α ) 

 sin ( β − δ − θ ') sin (α + β ) 
sin 2 ( 36° + 90° − 11.3° )
K ae = 2
= 0.372
 sin ( 36° + 24° ) sin ( 36° − 0° ) 
sin 2 ( 90° ) sin ( 90° − 24° ) 1 + 
 sin ( 90° − 24° ) sin ( 0° + 90° ) 
The Mononobe-Okabe earthquake active earth force Pae is,

Pae = γ H 2 (1 − kV ) K ae = ( 0.5) (18kN / m3 ) ( 5m ) (1 − 0) ( 0.372 ) = 83.7 kN / m


1 2

2
The earthquake force is ∆Pae = Pae − Pa = 83.7 − 53 = 30.7 kN / m
The location of the resultant earthquake force z is, found by locating the force Pae at a height
0.6H above the base of the wall; the active earth force Pa is obviously ( 0.33) H above the base.

( 0.6 H )( ∆Pae ) + (1/ 3) H ( Pa ) ( 0.6 )( 5m ) ( 30.7kN / m ) + (1/ 3)( 5m ) ( 53kN / m )


2 2

z= = = 2.1 m
Pae (83.7kN / m ) 2

298
**Lateral-16: Seismic loading upon a retaining wall.
(Revision: Aug-08)
The reinforced concrete retaining wall shown below will be designed to a horizontal seismic
loading of 0.2 g. Assume no vertical seismic component (kv=0). Determine,
(a) The weight of the wall Ww under static conditions;
(b) The weight of the wall under seismic conditions, for zero lateral displacement;
(c) The weight of the wall under seismic conditions, for a lateral displacement = 1.5 inches.

1m

H=5m Dense sand γ = 16 kN/m3


φ = 36°
δ = 2/3 φ = 24º

Solution.

299
Chapter 16
Braced Cuts for Excavations
Symbols for Braced Cuts for Excavations

300
*Braced-cuts-01: Forces and moments in the struts of a shored trench.
(Revision: Sept-08)
You have been asked by a contractor to design the internal supports (struts) of a temporary utility
trench, as shown below. In order to design the steel horizontal strut shown, you must first find the
force and moment on one of them, if they are spaced every 4 m horizontally.

Two triaxial laboratory tests were performed on samples of the clayey sand. The first sample had
a confining pressure of 0 kN/m2, and the sample reached failure with a deviator stress of 90
kN/m2. (N.B.: the deviator stress is the additional vertical stress required to reach failure, i.e. s-1 to
s-3). The second sample had its confining stress increased to 30 kN/m2. The deviator stress needed
to attain failure was 160 kN/m2.

Further laboratory tests show that this clayey sand had an in-situ voids ratio of 0.46 at a moisture
of 34% (assume Gs = 2.65). Show all your calculations.

301
Effective Stress Mohr’s Circle for failure Angle

τ (kN/m2)

σ (kN/m2)

From the Mohr’s Circle, we can get that φ2 = 32 o


(G s )(γ W ) (2.65)(9810)
Gs = 2.65 ; γW = 9810 N/m2 
γs = =  γS = γ2 = 17.8 kN/m2
1+ e 1 + 0.46
o

φ K A 1 = tan 2 (45o − 25 ) = 0.406


KA = tan2 (45o − ) 2
2 o
K A 2 = tan 2 ( 45 o − 32 ) = 0.307
2

302
Pa = (q) (KA1) = (90kN/m2) (.406)  36.54 kN/m2

Pb’+ = [KA1 (q + γ1h1)] = [(.406) (15kN/m2 x 3m)]  54.81 kN/m2

Pb’- = [KA2 (q + (γ2-γW) h] = [(.307) (90 + (17.8-9.81) (3)]  34.99 kN/m2

Pc = [(q + γ1h1 + (γ2-γW) h2] KA2 = .307 [90 + (15)(3) + (17.8-9.81)(2)] 46.35 kN/m2

PW = γW h W = (9.81)(2)  19.62 kN/m2

36.54

2m 1
F1
F2
2
18.27

3m 3 F3
F4 F5
34.99 4 5
11.36 19.62
Location of the Forces (with respect to the top datum):

F1: 3m (1/2) = 1.5m


F2: 3m (2/3) = 2.0m
F3: 3m + 2m (1/2) = 4.0m
F4: 3m + 2m (2/3) = 4.33m
F5: 3m + 2m (2/3) = 4.33m
Magnitude of the Forces:
F1 = (Pa)( h1) = (36.54 kN/m2)(3m) = 109.6 kN/m

F2 = (Pb+- Pa)( h1/2) = (54.81-36.54)(3/2) = 27.4 kN/m


303
F3 = (Pb-)( h2) = (34.99 kN/m2) (2m) = 69.98 kN/m

F4 = (Pc - Pb-)( h2/2) = (46.35-34.99)(2/2) = 11.36 kN/m

F5 = (PW ) (h W/2) = (19.62)(2/2) = 19.62 kN/m

∑F = F 1 + F2 + F3 + F4 + F5 = 237.96 kN/m

Ftot = (∑ F )(space b/t struts) = (237.96kN/m)(4m)  951.84 kN

109.6(1.5) + 27.4(2 ) + 69.98(4 ) + 11.36(4.33) + 19.62(4.33)


Located at y f = = 2.66m
237.96

∑M c = 0 Where C is located at the bottom of the trench along with RA

RB is located at the end of the strut.

 RB (3m) - 951.84 kN (2.34m) = 0

 RB = 742.44 kN
RA = 209.40 kN
304
Shear Diagram

742.44 kN

0 kN

-209.40 kN
Moment Diagram
490.0kN-m

0 kN-m 0 kN-m

0.66 m 2.34 m

305
**Braced cuts-02: A 5 m deep excavation with two struts for support.
(Revision: Sept-08)
Design a braced excavation for a large sanitary sewer force-main, which is a reinforced concrete
pipe with a diameter of 3 m. The trench should be 5 m deep and 5 m wide. The phreatic surface is
below bottom of excavation. The SPT for the silty clay is Navg = 20, and γ = 17 kN/m³. Assume φ = 0.

Solution:

Use Stroud’s relation to estimate the un-drained cohesion of the soil (the previous problem provided the
shear strength):

cu = KN = (3.5 kN/m²) (20) = 70 kN/m².

Therefore,

γH
if > 4 the clay is soft to medium
cu
γH
if ≤ 4 the clay is stiff
cu

In this problem,
γH
=
(17 )( 5 ) = 1.21 < 4 ∴ this is a stiff clay
cu ( 70 )
Also, since γH/ cu < 6, the sheet-piling should extend at least 1.5 m below bottom.

306
Step 1. Establish the lateral earth pressure distribution.

Using Peck's (1967) apparent pressure envelope, we must choose the larger of,
  4c  
(1) pa = γ H 1 −  u  
  γ H 
(2) pa = 0.3γ H = 0.3 (17 )( 5 ) = 25.5 kN / m 2

The location of the top strut should be less then the depth of the tensile crack zc. Since φ = 0,

Ka → Ka = 1. therefore σ3 = σa = (γ)(zc)Ka - 2c Ka

therefore zc = 2c/γ = 2(70 kN/m²)/ 17 kN/m³ = 8.2 m >> 0.6 m OK

Step 2: Determine the lateral loads at strut locations and excavation bottom.

Isolation the left portion between the surface and strut #2.

∑ MF’2 = 0 = F1(1.16m)-(0.5)(1.25m)(26)[0.51+1.25/3]-(0.51)(26)[0.51/2] = 0

therefore, F1 = 15.9 kN/m

∑ Fy = 0 = -15.9 + 1/2 (1.25)(26)+(0.51)(26)- F’2 = 0 therefore, F’2 = 13.6 kN/m

Isolating the right portion between strut #2 and the trench bottom, by symmetry

F22 = F12 = 13.6 kN/m ∑ Fy = 0 = - F22 + (3.75-0.51)(26)- F3

therefore, F3 = 70.6 kN/m

307
Step 3: Find the maximum moment Mmax in the sheet-piling.

Finding moments at A, B, & C (that is, the areas under the shear diagram):

MA = ½(0.60)(12.48)(0.60/3) = 0.75 kN-m/m

MB = ½(1.25)(26)(1.25/3)-15.9(0.65) = 3.56 kN-m/m

MC = (2.71)(26)(2.71/2) = 96 kN-m/m

Obviously, Mmax = 96 kN-m/m

Step 4: Select the steel-piling .

Assume fy = 50 ksi = 345 MN/m², therefore σallow = 50%fy = 172 MN/m²

The required section modulus S

S = MMax/ σall = 96 kN-m/ 172,000 kN/m² = 0.00056m³ = 56 m³/ m-105

Choose a PDA-27 section, which provides 57.46 m³/ m-105.

Step 5: Select the horizontal waler at each strut level.

At strut level #1 the load F1 is 16 kN/m. Select the horizontal spacings to be 4 m. (May use 3 m to
reduce steel size, but increases the difficulty of placing the concrete pipes).

Mmax = F1s²/8 = (16)(4)²/8 = 32 kN-m (where s is the spacing)

therefore, Swale at 1 = Mmax/σallow = 32 kN-m/ 172,000 kN/m² = 18.6 m³/ m-105

At strut level #2 the load is 27.2 kN/m; the spacing s is = 4 m.

Mmax= F2s²/8 = (27.2)(4)²/8 = 54.4 kN-m

Therefore, Swale at 2 = Mmax/σallow = 54.4 kN-m/ 172,000 kN/m² = 31.6 m³/ m-105

Notes: 1. The bottom of the trench has the highest lateral load, with 70.6 kN per every meter. Propose to cast a concrete “mud” slab at the
bottom of the trench. Design the thickness of the slab (diaphragm).

2. Wales are commonly channels or WF beams. Design the steel pipe wales and the struts, calculated in Step 6 below.

Step 6: Select the struts.

Level # 1 strut = F1s = (16 kN/m)(4m) = 64 kN

Level # 2 strut = 2 F2s = (27.2 kN/m)(4m) = 109 kN (Design the steel for the struts).

Step 7: Check for possible heave of the excavation bottom.

308
Braced cuts in clay may become unstable if the bottom heaves upward and fails a section of wall.

FSagainst heaving = [cNc(0.84 + 0.16 B/L)]/ γH = (70)(6.4)(0.84)/(17)(5) = 4.4 > 2 O.K.

Step 8: Expected lateral yielding of the sheet-piling and ground settlement behind the wall.

Expect δh from 5 to 10 cms.

δγ from 1 to 5 cms.

309
*Braced cuts-03: Four-struts bracing a 12 m excavation in a soft clay.
(Revision: Sept-08)
A four-strut braced sheet pile installation is designed for an open cut in a clay stratum, as shown
below. The struts are spaced longitudinally (in plan view) at 4.0 m center to center. Assume that
the sheet piles are pinned or hinged at strut levels B and C.

Find: 1. The lateral earth pressure diagram for the braced sheet pile system.
2. The loads on struts A, B, C, and D.

Solution:

From Terzaghi and Peck (1967), a clay is soft, medium or stiff,


γH  4c 
if > 4 the clay is soft to medium then σ a = γ H 1 − u 
cu  γH 
γH
if ≤ 4 the clay is stiff then σ a = 0.2γ H to 0.4γ H
cu

310
qu 96 kN / m 2
Determine the cohesion from Mohr's circle cu = = = 48 kN / m 2
2 2


γH
=
(17.3 kN / m ) (12 m ) = 4.33 > 4
3

∴ this is a soft to medium clay


cu ( 48 kN / m ) 2

Peck (1969) provided a criterion for soft to medium clays,


 4c   (4)(48 kN / m2 ) 
pa = γ H 1 − u  = (17 .3 kN / m 3
)(12 m ) 1 −  = 15.48 kN / m
2

 γH
3
  (17.3 kN / m )(12 m) 
The lateral earth pressure diagram for the braced sheet pile system in soft clays is,

2. In the free body diagram, part (a), ∑M B =0

( 1 2 ) (15.48 kN / m ) ( 3.0 m )( 4.0 m )  1.5 m 3.03 m  + (1.5 m ) (15.48 kN / m ) ( 4.0 m )  1.52 m  − ( F
2 2
A )( 3.0 m ) = 0
∴ FA = 100.6 kN

From ∑H =0
FB1 = 1( 2 ) (1.5 m + 4.5 m ) (15.48 kN / m ) ( 4.0 m ) − 100.6 kN = 85.2 kN
2

In the free body diagram, part (b)

FB 2 = FC1 = 1 ( 2 ) (3.0 m ) (15.48 kN / m ) ( 4.0 m ) = 92 kN


2

In the free body diagram, part (c), ∑M C =0

( FD )( 3.0 m ) − ( 4.5 m ) (15.48 kN / m 2 ) ( 4.0 m ) 


4.5 m 
=0
 2 
∴ FD = 209.0 kN
311
From ∑H =0
FC 2 + FD − ( 4.5 m ) (15.48 kN / m2 ) ( 4.0 m ) = 0

FC 2 = ( 4.5 m ) (15.48 kN / m 2 ) ( 4.0 m ) − 209.0 kN = 69.6 kN

Therefore,
FA = 100.6 kN
FB = 85.2 kN + 92.9 kN = 178.1 kN
FC = 92.9 kN + 69.6 kN = 162.5 kN
FD = 209.0 kN

312
Chapter 17
Bearing Capacity of Soils
Symbols for the Bearing Capacity of Soils

313
Bearing Capacity Factors for General Shear
Angle φ Angle φ Terzaghi Meyerhof Hansen
(Degrees) (Radians) Kpγ Nc Nq Nγ Nc Nq Nγ Nc Nq Nγ
0 0.0000 10.18 5.70 1.00 0.00 5.10 1.00 0.00 5.10 1.00 0.00
1 0.0175 10.61 6.00 1.10 0.08 5.38 1.09 0.00 5.38 1.09 0.00
2 0.0349 11.07 6.30 1.22 0.18 5.63 1.20 0.01 5.63 1.20 0.01
3 0.0524 11.56 6.62 1.35 0.28 5.90 1.31 0.02 5.90 1.31 0.02
4 0.0698 12.07 6.97 1.49 0.39 6.19 1.43 0.04 6.19 1.43 0.05
5 0.0873 12.61 7.34 1.64 0.51 6.49 1.57 0.07 6.49 1.57 0.07
6 0.1047 13.19 7.73 1.81 0.65 6.81 1.72 0.11 6.81 1.72 0.11
7 0.1222 13.80 8.15 2.00 0.80 7.16 1.88 0.15 7.16 1.88 0.16
8 0.1396 14.44 8.60 2.21 0.96 7.53 2.06 0.21 7.53 2.06 0.22
9 0.1571 15.13 9.09 2.44 1.15 7.92 2.25 0.28 7.92 2.25 0.30
10 0.1745 15.87 9.60 2.69 1.35 8.34 2.47 0.37 8.34 2.47 0.39
11 0.1920 16.65 10.16 2.98 1.58 8.80 2.71 0.47 8.80 2.71 0.50
12 0.2094 17.49 10.76 3.29 1.84 9.28 2.97 0.60 9.28 2.97 0.63
13 0.2269 18.38 11.41 3.63 2.12 9.81 3.26 0.74 9.81 3.26 0.78
14 0.2443 19.33 12.11 4.02 2.44 10.37 3.59 0.92 10.37 3.59 0.97
15 0.2618 20.36 12.86 4.45 2.79 10.98 3.94 1.13 10.98 3.94 1.18
16 0.2793 21.46 13.68 4.92 3.19 11.63 4.34 1.37 11.63 4.34 1.43
17 0.2967 22.65 14.56 5.45 3.63 12.34 4.77 1.66 12.34 4.77 1.73
18 0.3142 23.92 15.52 6.04 4.13 13.10 5.26 2.00 13.10 5.26 2.08
19 0.3316 25.30 16.56 6.70 4.70 13.93 5.80 2.40 13.93 5.80 2.48
20 0.3491 26.80 17.69 7.44 5.34 14.83 6.40 2.87 14.83 6.40 2.95
21 0.3665 28.42 18.92 8.26 6.07 15.81 7.07 3.42 15.81 7.07 3.50
22 0.3840 30.18 20.27 9.19 6.89 16.88 7.82 4.07 16.88 7.82 4.13
23 0.4014 32.10 21.75 10.23 7.83 18.05 8.66 4.82 18.05 8.66 4.88
24 0.4189 34.19 23.36 11.40 8.90 19.32 9.60 5.72 19.32 9.60 5.75
25 0.4363 36.49 25.13 12.72 10.12 20.72 10.66 6.77 20.72 10.66 6.76
26 0.4538 39.01 27.09 14.21 11.53 22.25 11.85 8.00 22.25 11.85 7.94
27 0.4712 41.78 29.24 15.90 13.15 23.94 13.20 9.46 23.94 13.20 9.32
28 0.4887 44.85 31.61 17.81 15.03 25.80 14.72 11.19 25.80 14.72 10.94
29 0.5061 48.26 34.24 19.98 17.21 27.86 16.44 13.24 27.86 16.44 12.84
30 0.5236 52.05 37.16 22.46 19.75 30.14 18.40 15.67 30.14 18.40 15.07
31 0.5411 56.29 40.41 25.28 22.71 32.67 20.63 18.56 32.67 20.63 17.69
32 0.5585 61.04 44.04 28.52 26.20 35.49 23.18 22.02 35.49 23.18 20.79
33 0.5760 66.40 48.09 32.23 30.33 38.64 26.09 26.17 38.64 26.09 24.44
34 0.5934 72.48 52.64 36.50 35.23 42.16 29.44 31.15 42.16 29.44 28.77
35 0.6109 79.40 57.75 41.44 41.08 46.12 33.30 37.15 46.12 33.30 33.92
36 0.6283 87.33 63.53 47.16 48.11 50.59 37.75 44.43 50.59 37.75 40.05
37 0.6458 96.49 70.07 53.80 56.62 55.63 42.92 53.27 55.63 42.92 47.38
38 0.6632 107.13 77.50 61.55 67.00 61.35 48.93 64.07 61.35 48.93 56.17
39 0.6807 119.59 85.97 70.61 79.77 67.87 55.96 77.33 67.87 55.96 66.76
40 0.6981 134.31 95.66 81.27 95.61 75.31 64.20 93.69 75.31 64.20 79.54
41 0.7156 151.89 106.81 93.85 115.47 83.86 73.90 113.99 83.86 73.90 95.05
42 0.7330 173.09 119.67 108.75 140.65 93.71 85.37 139.32 93.71 85.37 113.96
43 0.7505 198.99 134.58 126.50 173.00 105.11 99.01 171.14 105.11 99.01 137.10
44 0.7679 231.10 151.95 147.74 215.16 118.37 115.31 211.41 118.37 115.31 165.58
45 0.7854 271.57 172.29 173.29 271.07 133.87 134.87 262.74 133.87 134.87 200.81
46 0.8029 323.57 196.22 204.19 346.66 152.10 158.50 328.73 152.10 158.50 244.65
47 0.8203 391.94 224.55 241.80 451.28 173.64 187.21 414.33 173.64 187.21 299.52
48 0.8378 484.34 258.29 287.85 600.15 199.26 222.30 526.45 199.26 222.30 368.67
49 0.8552 613.53 298.72 344.64 819.31 229.92 265.50 674.92 229.92 265.50 456.40
50 0.8727 801.95 347.51 415.15 1155.97 266.88 319.06 873.86 266.88 319.06 568.57

314
The bearing capacity of a soil is its ability to carry loads without failing in shear. There are four major
methods to predict failure. The fist method was developed by Karl Terzaghi in 1943. Field tests in
Canada by Meyerhof (1963) lead to modification factors. Finally, Brinch Hansen in Denmark (1970)
and Vesic in the USA modified these factor to a greater refinement.
These bearing capacity factors are based on these three authors:

Terzaghi (1943):
For square footings, qult = 1.3c ' N c + qN q + 0.4γ BNγ
For continuous or wall footings qult = c ' N c + qN q + 0.5γ BNγ
where, q = γ D f and the factors are,
a2
where a = e(
0.75π −φ / 2 ) tan φ
Nq =
a cos 2 ( 45° − ϕ / 2 )
N c = ( N q − 1) cot φ
tan φ  K pγ 
Nγ =  − 1
2  cos φ 
2

Meyerhof (1963):
For vertical loads, qult = cN c Fsc Fdc + qN q Fsq Fdq + 0.4γ BNγ Fsγ Fd γ
and for inclined loads, qult = cN c Fic Fdc + qN q Fiq Fdq + 0.4γ BNγ Fiγ Fdγ
and the factors are,
N q = eπ tan φ tan 2 ( 45° − φ / 2 )
N c = ( N q − 1) cot φ
Nγ = ( N q − 1) tan (1.4φ )

Brinch Hansen (1970):


The general equation, qult = cN c Fsc Fdc Fic + qN q Fsq Fdq Fiq + 0.4γ BNγ Fsγ Fd γ Fiγ
and the factors are,
N q = eπ tan φ tan 2 ( 45° − φ / 2 )
N c = ( N q − 1) cot φ
Nγ = 1.5 ( N q − 1) tan φ

315
*Bearing–01: Terzaghi’s bearing capacity formula for a square footing.
(Revision: Sept-08)
The square footing shown below must be designed to carry a 294 kN load. Use Terzaghi’s bearing
capacity formula to determine B of the square footing with a Factor of Safety =3.

W = 294 kN
Df = 1 m γ = 18.15 kN/m3
φ = 35°
W c =0

B
Solution:

Terzaghi's formula for the ultimate bearing capacity qult of a square footing is,
qult = 1.3c ' N c + qN q + 0.4γ BNγ where q = D f γ
The allowable bearing capacity qall with the factor of safety of 3 is,

qall =
qult 1
3 3
(
= 1.3c ' N c + qN q + 0.4γ BNγ )
and qall =
W 294 kN
B2
=
B2
or
294 1
B2 3
(
= 1.3c ' N c + qN q + 0.4γ BNγ )
For φ =35º, N c =57.8, N q =41.4, and Nγ =41.1.
Substituting these values into Terzaghi's equation, we get
294 1
= ( 0 ) + (18.15)(1) (41.4) + (0.4) (18.15) B (41.1) 
B2 3 
294
= 250.5 + 99.5 B
B2
B 3 + 2.52 B 2 − 2.96 = 0 ∴ B = 0.90 m

316
*Bearing–02: Meyerhof’s bearing capacity formula for a square footing.
(Revision: Sept-08)
The square footing shown below must be designed to carry a 294 kN load. Use Meyerhof’s bearing
capacity formula to determine B with a factor of safety =3.

W = 294 kN
Df = 1 m
γ = 18.15 kN/m3
φ = 35°
W c =0

Solution:

Meyerhof's formula for the ultimate bearing capacity qult of a square footing is,
qult = c ' N c Fsc Fdc Fic + qN q Fsq Fdq Fiq + 0.4γ BN γ Fsγ Fd γ Fiγ where q = D f γ
Since the load is vertical, all three inclination factors Fic =Fiq =Fiγ =1.
B 1 B
Fsq = 1 +   tan φ = 1 +   tan 35° = 1.70 and Fsγ = 1 − 0.4   = 1 − 0.4(1) = 0.6
L 1 L
2  Df  2 1 
Fdq = 1 + 2 tan φ (1 − sin φ )   = 1 + 2 ( tan 35° ) (1 − sin 35°)   ≈ 1.25 and Fd γ = 1
 B  B
The allowable bearing capacity qall with the factor of safety of 3 is,

qall =
qult 1 '
3 3
(
= c N c Fsc Fdc + qN q Fsq Fdq + 0.4γ BN γ Fsγ Fd γ ) and qall =
W 294 kN
B2
=
B2
or
294 1 '
B2 3
(
= c N c Fsc Fdc + qN q Fsq Fdq + 0.4γ BN γ Fsγ Fd γ )
For φ = 35º, N c = 46.12, N q = 33.30, and N γ = 37.15.
Substituting these values into Meyerhof''s equation, we get
294 1
= ( 0 ) + (18.15)(1) (33.3) (1.7 )(1.25 ) + (0.4) (18.15) B (37.15 )( 0.6 )(1) 
B2 3 
294
2
= 428.1 + 53.94 B or B 3 + 7.94 B − 5.45 = 0 ∴ B = 0.65 m
B

317
*Bearing–03: Hansen’s bearing capacity formula for a square footing.
(Revision: Sept-08)
The square footing shown below must be designed to carry a 294 kN load. Use Brinch Hansen’s
bearing capacity formula to determine B with a factor of safety =3.

W = 294 kN
Df = 1 m
γ = 18.15 kN/m3
φ = 35°
W c =0

Solution:

Hansen's formula for the ultimate bearing capacity qult of a square footing is,
qult = c ' N c Fsc Fdc Fic + qN q Fsq Fdq Fiq + 0.4γ BN γ Fsγ Fd γ Fiγ where q = D f γ
Since the load is vertical, all three inclination factors Fic =Fiq =Fiγ =1.
B 1 B
Fsq = 1 +   tan φ = 1 +   tan 35° = 1.7 and Fsγ = 1 − 0.4   = 1 − 0.4(1) = 0.6
L 1 L
2  Df  2 1 
Fdq = 1 + 2 tan φ (1 − sin φ )   = 1 + 2 ( tan 35° ) (1 − sin 35°)   ≈ 1.255 and Fdγ = 1
 B  B
The allowable bearing capacity qall with the factor of safety of 3 is,

qall =
qult 1 '
3 3
(
= c N c Fsc Fdc + qN q Fsq Fdq + 0.4γ BN γ Fsγ Fdγ ) and qall =
W 294 kN
B2
=
B2
or
294 1 '
B2 3
(
= c N c Fsc Fdc + qN q Fsq Fdq + 0.4γ BN γ Fsγ Fd γ )
For φ = 35º, N c = 46.12, N q = 33.30, and N γ = 33.92.
Substituting these values into Hansen's equation, we get
294 1
= ( 0 ) + (18.15)(1) (33.3) (1.7 )(1.255 ) + (0.4) (18.15) B (33.92 )( 0.6 )(1) 
B2 3 
294
2
= 429.8 + 49.25 B or B 3 + 8.73B − 5.97 = 0 ∴ B = 0.70 m
B

318
*Bearing–04: Same as #01 but requiring conversion from metric units.
(Revision: Sept-08)
The square footing shown below must be designed to a load of 30,000 kgm. Using a factor of safety
of 3 and using Terzaghi’s method, determine the size B of the square footing.

m = 30,000 kgm
Df = 1 m
ρ = 1,850 kg/m3
φ = 35°
W c =0

Solution:

The soil density ρ = 1,850 kgm / m3 converts to a unit weight via γ = ρ g ( like F = ma ) ,
 kg m   m
 1,850 m3   9.81 s 2 
γ = ρg =    = 18.15 kN / m3 and the load to be supported by the footing is,
(1, 000 N / kN )
( 30, 000 kg m )  9.81
m
 s 2 
W = ma = = 294 kN
(1, 000 N / kN )
Terzaghi's ultimate bearing capacity of a square footing is given by,
qult = 1.3c ' N c + qN q + 0.4γ BN γ

∴ qall =
qult 1
3 3
(
= 1.3c ' N c + qN q + 0.4γ BN γ ) P 294
and qall = 2 = 2
B B
or
294 1
B 2
3
(
= 1.3c ' N c + qN q + 0.4γ BN γ )
For φ = 35º, N c = 57.8, N q = 41.4, and N γ = 41.1,
294 1
2
= ( 0 ) + (18.15)(1) (41.4) + (0.4) (18.15) B (41.1)  ∴ B 3 + 2.52 B 2 − 2.96 = 0
B 3
B = 0.90 m

319
*Bearing–05: General versus local bearing capacity failures.
(Revision: Sept-08)
Using Terzaghi’s method, distinguish between the value of the local shear failure versus the
general shear failure.

Solution:

Terzahi's general bearing capacity failure of a square footing is,


qult = 1.3c ' NC + qN q + 0.4γ BNγ
For φ = 28° N c = 31.6, N q = 17.8, Nγ = 15.0 and q = γ D f = (0.115)(2) = 0.23 ksf
Therefore,
qult = 1.3(0.30)(31.6) + (0.23)(17.8) + 0.4(0.115)(2.5)(15.0) = 18.1 ksf

To find the value of the bearing capacity of a local shear failure, the cohesion and angle
of internal friction are reduced by two-thirds,
2 2
−local = 1.3c ' N c + qN q + 0.4γ BN γ where c' = c = (0.30) = 0.2 ksf
' ' ' '
qult
3 3
2 2
and ϕ ' = (ϕ ) = (28°) = 18.7° which give N c' = 16.2, N q' = 6.5 and Nγ' = 4.52
3 3
∴ qult −local = (1.3) (0.2)(16.2) + (0.23)(6.5) + (0.4)(0.115)(2.5)(4.52) = 6.2 ksf
'

qult − general failure = 18.1 ksf versus qult −local failure = 6.2 ksf ( Almost a three − to − one)

320
*Bearing–06: Comparing the Hansen and Meyerhof bearing capacities.
(Revision: Sept-08)
Compare the results of the Hansen and the Meyerhof bearing capacity formulas to the results of a
field test that took a rectangular footing to failure when the load reached 1,863 kN. Given B = 0.5
m, L = 2.0 m, c = 0, φtriaxial = 42° and γ’ = 9.31 kN/m3 (the WT is at the surface).

Pult = 1,863 kN
▼ WT

Df = 0.5 m

B = 0.5 m

Solution:

Pult 1, 863 kN
qult = = = 1, 863 kPa was the field measured failure load.
BL ( 0.5 m )( 2.0 m )
(1) The Hansen formula predicts an ultimate bearing capacity of,
qult = 0 + qN q Fqs Fqd + 0.5γ BN γ Fγ s Fγ d
Lee ' s adjustment formula is φ ps = 1.5φtriaxial − 17 ° = 1.5 ( 42 ° ) − 17 ° = 46 °
For ϕ = 46 °, N q = 158.5 and N γ = 244.65
B  0.5 
Fqs = 1 +   tan ϕ = 1 +   tan 46 ° = 1.26
L  2 
B  0.5 
Fγ s = 1 − 0.4   = 1 − 0.4   = 0.9
L  2 
D   0.5 
Fqd = 1 + 2 tan ϕ (1 − sin ϕ ) 2  f  = 1 + 2 tan 46 °(1 − sin 46 °) 2   = 1.16
 B   0.5 
Fγ d = 1.0
∴ qult = 0 + (9.31)(0.5)(159)(1.27)(1.16) + (0.5)(9.31)(0.5)(245)(0.9)(1.0)
∴ qult = 1, 485 kPa versus 1, 863 kPa measured ( Hansen underestimates by 20%)

(2) The Meyerhof formula with φ = 46º, N q = 158.5 and N γ = 328.7 3,


qult = 0 + qN q Fqs Fqd + 0.5γ BN γ Fγ s Fγ d
∴ qult = 0 + (9.31) ( 0.5 )(158.5 ) (1.27) (1.16 ) + (0.5)(9.31)(0.5)(328.73)(0.9)(1.0)
∴ qult = 1, 782 kPa versus 1, 863 kPa ( Meyerhof underestimates by 4 % ) .

321
*Bearing–07: Increase a footing’s width if the WT is expected to rise.
(Revision: Sept-08)
Use Meyerhof’s bearing capacity formula (with a factor of safety = 3) to select a footing’s width B
if, (a) the water table is as shown below, and (b) if the water table rises to the ground surface?
The soil has a unit weight of 112 pcf, a moisture of 10%, φ = 25º, a cohesion cu = 240 psf and a
specific gravity of solids of Gs = 2.68.

Solution:

(a) Find γ SAT to determ ine γ ',


γ 112 Ws γ dry
γ dry = = = 101.8 pcf and V s = =
1 + wN 1.10 G sγ w G sγ w
101.8
set V = 1 ft 3 ∴ V s = = 0.61 ft 3 ∴ V v = V − V s = 1 − 0.61 = 0.39 ft 3
2.68 ( 62.4 )
V 
but γ sat = γ dry + nγ w = γ dry +  v  γ w ∴ γ sat = 101.8 + (0.39) ( 62.4 ) = 126.2 pcf
V 
and γ ' = γ sat − γ w = 126.2 − 62.4 = 63.8 pcf
T ry B = 5.7 feet with M eyerhof's equation,
q ult = c ' N c ( Fcs Fcd Fci ) + qN q ( Fqs Fqd Fqi ) + 0.5γ BN γ ( Fγ s Fγ d Fγ i )
w here the load inclination factors Fci , Fqi and Fγ i = 1
 ϕ  25 ° 
For ϕ ≥ 10 ° K p = tan 2  45 ° +  = tan 2  45 ° +  = 2.46, therefore
 2  2 
B  5.7 
Fcs = 1 + (0.2)   K p = 1 + (0.2)   (2.46) = 1.49
L  5.7 
 Df   4 
Fcd = 1 + (0.2)   K p = 1 + (0.2)   2.46 = 1.22
 B   5.7 
 Df   4 
Fqd = Fγ d = 1 + (0.1)   K p = 1 + (0.1)   2.46 = 1.11
 B   5.7 
B 5.7
Fqs = Fγ s = 1 + (0 .1)   K p = 1 + (0.1)( )(2.46) = 1.25
L 5.7
322
The Meyerhof bearing capacity factors for φ = 25° are
N c = 20.7, N q = 10.7, and N γ = 6.77
qult = c ' N c ( Fcs Fcd Fci ) + qN q ( Fqs Fqd Fqi ) + 0.5γ BN γ ( Fγ s Fγ d Fγ i )
qult = (0.24)(20.7)(1.49)(1.22)(1) + (0.112)(4)(10.7)(1.25)(1.11)(1) + (0.5)(0.112)(5.7)(6.67)(1.25)(1.11)(1)
qult = 18.6 ksf
qult 18.6 Q 200
qall = = = 6.2 ksf therefore B 2 = = = 32.25 ft 2 ∴ B = 5.7 ft
FS 3 qall 6.2
Therefore the choice of B = 5.7 ft was a good choice.

(b) When the water table rises to the ground surface, need a larger footing; try B = 7.0 feet.
B 7
Fcd = 1 + 0.2   K p = 1 + 0.2   (2.46) = 1.49
L 7
Fcs = 1.49 same as above
D 4
Fqd = Fγ d = 1 + 0.1   K p = 1 + 0.1   2.46 = 1.09
B 7
Fqs = Fγ s = 1.25 same as above
qult = (0.24)(20.7) (1.49 )(1.18 ) + (0.062) ( 4 )(10.7 )(1.25 )(1.09 ) + (0.5)(0.062)(7) ( 6.67 )(1.09 )(1.25 )
qult = 16.62 ksf
qult 16.62 Q 200
qall = = = 5.54 ksf and B 2 = = = 36.1 ft 2 ∴ B = 6.01 ft
FS 3 qall 5.54
Iterate once more, and find B = 7.5 feet.

323
**Bearing–08: The effect of the WT upon the bearing capacity.
(Revision: Sept-08)
Using the Hansen method, what are the ultimate and allowable bearing capacities for the footing
shown below if you require a factor of safety of at least 2?

Solution:

Always use the effective unit weight of water in the bearing capacity formulas. The average effective
weight γ e of the soil can also be given by the formula:

dw γ'
γ e = (2H − dw ) γ wet + 2 ( H − d w )
2
2
H H
 ϕ  35 ° 
where H = (0.5) B tan  45 ° +  = (0.5)(2.5) tan  45 ° +  = 2.40 m
 2  2 
and d w = depth to the WT below the footing invert = 0.85 m
Set the total volume V = 1 m 3
γ wet 18.10 kN γ dry 16.5
γ dry = = = 16.5 3 and Vs = = = 0.63 m 3
1+ w 1 + 0.10 m G s γ wet (2.68) ( 9.8 )
kN
Vv = 1.0 − V s = 1 − 0.63 = 0.37 m 3 and γ sat = γ dry + nγ wet = 16.5 + (0.37) ( 9.8 ) = 20.1
m3
 0.85 (18.10 )   20.1 − 9.8  kN
∴ γ e = ( (2)(2.40 − 0.85) )  +  ( 2.40 − 0.85 ) = 12.6 3
2

 ( 2.4 )   ( 2.4 ) 
2 2
m

Using Hansen’s method with φ = 35º, the bearing capacity factors are Nq = 33.3 and Nγ = 33.92.

324
B  2 .5 
Fq s = 1 + tan φ = 1 +   tan 3 5 ° = 1 .7 0
L  2 .5 
2  D f   2  1 
= 1 + 2 tan φ (1 − sin φ )   = 1 + 2 tan 3 5 (1 − sin 3 5 )   = 1 .1 0

Fq d
 B   2 .5 
B 2 .5
Fγ s = 1 − 0 .4 = 1 − 0 .4 ( ) = 0 .6
L 2 .5
Fγ d = 1 .0
T h erefo re, th e u ltim ate an d allo w ab le b earin g cap acities are,
q u lt = 0 + q N q ( F q s F q d ) + 0 .5 γ e B N γ ( Fγ s Fγ d )
q u lt = (1 8 .1) (1 .0 )( 3 3 )(1 .7 0 )(1 .1 0 ) + (0 .5) (1 2 .6 )( 2 .5 )( 3 4 )( 0 .6 )(1 )
q u lt = 1, 4 9 7 kP a
1, 4 9 7
q a ll = = 7 4 9 kP a
2

325
*Bearing–09: Finding the gross load capacity.
(Revision: Sept-08)
Use the Hansen formula to determine the gross normal load N on the column shown below using a
factor of safety of 3.

γ =18.1 kN/m3
0.61 m φ =32°
c=0

0.61 m
γsat = 21.07 kN/m3

1.22 m

Solution:

The Hansen formula for a footing is,


qult = cN c Fcs Fcd + qN q Fqs Fqd + 0.5γ BN y Fys Fyd
The inclination factors Fci , Fqi , and Fγ i are all equal to 1 because the load is vertical.
For φ = 32° , N c = 35.49, N q = 23.18 and N γ = 20.79 and B / L = 1
 Nq 
Fcs = 1 +   = 1 + ( 23.20 / 35.50 ) = 1.65
 Nc 
Fqs = 1 + tan φ = 1 + 0.62 = 1.62
B
Fys = 1 − 0.4   = 1 − 0.4 = 0.60
L
2  Df 
Fqd = 1 + 2 tan φ (1 − sin φ )   = 1 + (2)(0.62)(0.22)(1) = 1.273 for D f / B ≤ 1
 B 
Fyd = 1
 (1 − Fqd )   1 − 1.273  
Fcd = Fqd −   = 1.273 −    = 1.292
 N q tan φ   23.20 × 0.62  

326
The WT is located above the footing, therefore,
q = ( 0.61m ) (18.1 kN / m3 ) + ( 0.61m )( 21.07 − 9.81) = 17.9 kN / m 2
∴ qult = (17.9)(1.62)(1.273)(23.20) + ( 0.5 )( 0.6 )( 21.07 − 9.81)(1.22 )( 20.8 )(1) = 981 kN / m 2
Therefore,
 q   981kN / m 
2
qall =  ult  =   = 327 kN / m
2

 3   3 
Hence, the total gross load N is,
N = qall B 2 = (327 kN / m 2 )(1.22m) 2 = 487 kN

327
**Bearing–10: The effect of an eccentric load upon bearing capacity.
(Revision: Sept-08)
A rectangular footing measures 5 feet by 2.5 feet. Determine the gross ultimate load Qult applied
eccentrically upon the footing, and the ultimate bearing capacity of the soil qult, given that γ = 115
pcf, c = 0 and φ = 30°.

Solution:

The effective width footing width B ' = B - 2 e x = ( 2.5 ) - 2 ( 0.2 ) = 2.1 ft


and the effective length L ' = L - 2 e y = ( 5 ) - 2(0.4) = 4.2 ft.
M eyerhof's ultimate bearing capacity formula with c = 0 is,
q ult = 0 + qN q Fqs Fqd + 0.5γ B ′N γ F ys Fγ d
For φ = 30  , N q = 18.4 and N γ = 15.67
 B'   2.1 
Fqs = 1 +  '  tan φ = 1 +   ( 0.58 ) = 1.29
 L   4.2 
2  2 
Fqd = 1 + 2 ( tan 30 ° )(1 − sin 30 ° )   = 1.275
 2.1 
 B′   2.1 
F ys = 1 − 0.4   = 1 − 0.4   = 0.8
 L′   4.2 
F yd = 1
q ult = ( 2)(0.115 ) (18.4) (1.29 )(1.275 ) + (0.5)(0.11 5)(2.1)(15.67) ( 0.8 )(1 ) = 8.47 ksf
Henc e , Q ult = q ult ( B ′L ′ ) = ( 8.47 )( 2.1)(4.2 ) = 74.73 kips

328
**Bearing–11: The effect of an inclined load upon the bearing capacity.
(Revision: Sept-08)
A square 8’ x 8’ footing is loaded with an axial load of 400 kips and Mx = 200 ft-kips, My = 120 ft-
kips. Un-drained triaxial tests (the soil is not saturated) gave φ = 33º and c = 200 psf. The footing
depth Df = 6.0 feet, the soil unit weight is 115 pcf, and the WT was not found.
Use the Hansen equation with the Meyerhof reduction factors and a FS = 3 to find the

Vertical axial load = 400 kips

Mx = 200 ft-kips

My = 120 ft-kips

Solution:
My 120 ft − k M x 200 ft − k
Eccentricities ex = = = 0.3 feet and ey = = = 0.5 feet
Q 400 Q 400
∴ Br = B − 2ey = 8 '− 1' = 7 feet and Lr = L − 2ex = 8 '− 0.6 ' = 7.4 feet (ie. Lr > Br)

Adjusting the φ from triaxial (φtr ) to a plane-strain value (φ ps ) via Lee’s formulation,

φ ps ≅ 1.1φtr = 1.1( 32.7°) = 36°

 36° 
N q = eπ tan 36° tan 2  45° +  = 37.8
 2 

Nc = (N q − 1) cot φ = ( 36.8 ) cot 36° = 50.6

Nγ = ( N q − 1) tan (1.4φ ) = ( 36.8 ) tan 50.4° = 44.4


Nγ = 1.5 ( N q − 1) tan φ = 1.5 ( 36.8 ) tan 36° = 40.1

B  7
∴ Sc = 1 + 0.2 K p  r  = 1 + 0.2 ( 3.85 ) = 1.73
 Lr  7.4

and
D
d c = 1 + 0.2 K p 
 Br

 = 1 + 0.2

( )6
3.85   = 1.34
7
Since φ > 10° , S q = Sγ ≅ 1.0 and d q = dγ = 1.0 .

Hansen’s qult = 0.5γ BN γ Sγ d γ iγ g γ bγ + cN c S c d c ic g c bc + qq N q S q d q iq g q bq


329
Also i = g = b = 1.0 for this problem, since α = 0 = i (inclination factor f / load Q with t vertical) η = 4
g (ground factor with t inclined ground on side of footing)
b (base factor with t inclined ground under the footing)

qult = 0.5 ( 0.115 )( 7 )( 40.1)(1) + ( 0.200 )( 50.6 )(1.73 )(1.34 ) + ( 0.115 )( 6 )( 37.8 )(1) =
qult = 16.1 + 23.5 + 26.1 = 65.7 ksf

qult 65.7
q all = = = 21.9 ksf
FS 3
1 1
 e 2  0.3  2
R ex = 1−  x  = 1−   = 0.81
 B   8 
1 1
 e 2  0.5  2
Rey =1−  y  = 1−   = 0.75
 B   8 

Qall = qall ( B 2 ) ( R e x ) ( R e y ) = 21.9 ( 8 x8 )( 0.81)( 0.75 ) = 851 kips

Qall  851 
qall = =  = 13.3 ksf
B 2  64 

 400 
(The contact load qo = 13   = 6.1 ksf )
 851 

330
**Bearing-12: Interpretation of borings to estimate a bearing capacity.
(Revision: Sept-08)
Use the boring logs show below to recommend an allowable soil pressure qall for the footings
located in the vicinity of elevation 284, boring No. 2?
The building is a four-story (five on the low side) office building with column loads around 160
kips. State your reasons.

Topsoil Fine brown silty sand - small gravel Fine to medium brown silty
sand -some small to medium
gravel
Brown silty clay Fine brown silty sand - trace of coarse sand

Boring No.5
Boring No.2 Boring No.3 Boring No.4 Elevation
296.6
295 Elevation Got
295.0 Firmer 7
Elevation Elevation 5 Got
290.6 292.8 13
Firmer
4 14
290 Elevation
288.0 Dark 25
6 7 brown 71 29
Got
Sandy 10 Firmer Hard
8 16 46
285
Got 71
Got Firmer Got
Firmer 25 51
27 Firmer
47 Hard
Got Firmer 69
280 22 36 6 in. 38
boulder
38 62
Cohesive 34
Got 34 67

275 firmer 39

32 Notes: 1. All elevations are in accordance with plot furnished by architect.


Hard 2. Borings were made using standard procedures with 2-in. -OD split spoon.
3. Figures to the right of each boring log indicate the numbeer of blows required
69 to drive the 2-in.-OD split spoon 12 in. using a 140 lb weight falling 30 in.
270
4.No water encountered in any of the borings.
74

Solution:

It is presumed that all the building’s footings will be placed at roughly elevation 284 or thereabouts.
This is fine for the building area covered by borings # 3, 4 and 5 because they have good SPT values.

Meyerhof has proposed formulas for the allowable bearing capacity adjusted so that the settlement is
limited to 1-inch. These formulas are:

N
qall = ( K D ) for B ≤ 4 ft
4

331
N  B +1
2

qall =   ( K D ) for B> 4ft


6 B 

 Df 
where K D = 1 + 0.33   ≤ 1.33
 B 

47 + 51 + 71
For the silty sand use N= = 56.33 ≃ 56 (#3, 4, and 5)
3

Let’s assume B=4.5 ft and Df =0

56  4.5 + 1 
2
qall =   (1) = 13.9 ksf This suggests that a B = 4.5 feet is excessive since
6  45 

Q 160kips
q0 = = = 7.9 ≪ qall = 13.9 ksf
B2 20.25sf

Assume B < 4 ft, say B~ 3.5 ft , and use formula

N 56  0.33Df 
qall= ( KD ) kd =1+ 0.33Df /B 1 +  and Df = 0 qall = 14 ksf
4 4  B 

Q 160 kips
∴ qo = = = 13.06 ≅ 13 ksf ≤ 14 ksf OK
B 2
(3.5)2

For footings in area of borings # 1 and #2, they will be deeper by 1-story (ie. for 5-story building). That
places the shallow foundation at elevation 274 ft. This area will have bearing in the same strata. N= 32
and using B= 3.50’ and Df = 4.5’

N
kd =1+0.33Df /B ≤ 1.33
32  0.33 x 4.5 
qall= kd Kd = 1 +  = 1.33
4 4  3.50 

qall = 10.64ksf < 13 ksf NOT GOOD

Q
Let’s use B= 3.90 feet qall = 10.64 ksf q0 =  2  = 10.51 ksf ≤ 10.64 ksf
B 

Use B = 3.90 feet.

332
Chapter 18
Shallow Foundations
Symbols for Shallow Foundations

333
Properties of Reinforcing Steel (British and SI units).

334
*Footings–01: Analyze a simple square footing.
(Revision: Sept-08)
Design a square reinforced concrete footing for a column 15”x15” with 4 # 8 rebars, and,
D L = 100 kips f c' = 3, 000 psi
LL = 120 kips f y = 60, 000 psi
q allowable = 4 ksf from qult = 10 ksf and FS = 2.5

Solution:
1) Footing size for service loads:
Q 220 kips
B= = = 7.42 feet therefore use B = 7.5 feet.
qa 4 ksf

2) Check ultimate parameters: that is the actual soil pressure qo under Qult,
QULT = 1.2 DL + 1.6 LL = 1.2 (100 ) + 1.6 (120 ) = 120 + 192 = 312 kips

QU 312
∴ qO = = = 5.5 ksf < 10 ksf for qult …GOOD
( 7.5 ft )
2 2
B

3) Compute the allowable concrete shear strength vc ( allowable )

vall = 4φ f 'C where ϕ = 0.75 for shear and torsion ,

vC = vall = 4 ( 0.75 ) 3, 000 psi = 164 psi ≈ 23.7 ksf

4) Find d, the effective depth, (in this case two-way shear governs) in feet.

15”

B = 7.5 feet

15” + d

335
    2  qo 
 w − (B − w )  = 0
q qo
d 2  vc + o  + d  vc +
2

 4   4  4
5.5  15   2  15    5.5 
2
2 5.5  
d  23.7 +  + d  23.7 +   − 7.5 −     =0
 4   4  12    12    4 
d 2 + 1.25d − 3.0 = 0

+1.22 ft
Which yield to two solutions for d = and using the modified formula equation 2a:
−2.47 ft

When the column has a rectangular area bxc, the formula is,

 BLqo   15 15   ( 7.5 ) 2 ( 5.5 ) 


4d + 2 ( b + c ) d − 
2
 = 4d + 2  +  d − 
2
=0
 vc   12 12   23.7 
d 2 + 1.25d − 3.26 = 0 which yields d = +0.39 ft
Use the largest d = 1.22 feet = 14.6 inches; round-out to d = 15 inches. It is not necessary to check for
wide-beam shear on a square footing.

5) Compute the area of steel AS for flexure.


  15  
 7.5 −  12  
 B−w  
Unit strip of the cantilever arm =  =L ∴ L= = 3.13 ft
 2  2

B = 7.5 ft

15”
1 ft

q0
336
The cantilever moment:

 k  
( ) ( )
2
  5.5  3.13 ft 12 in 
 q o L2   
2
ft   = 359 in − kips
MU = =
 2  2

d −a
M U = φ AS f y   where φ = 0.9 for tension
 2 
AS f y AS ( 60 )
a= = = 1.96 AS
0.85 f 'C b 0.85 ( 3 )(12 in )

Substituting a into the M U equation above,

d −a MU d −a
M U = ( 0.9 ) AS f y   ∴ = AS  
 2  ( 0.9 ) f y  2 
359 in − kip  A 
= AS  15 in − 1.96 S 
( 0.9 )( 60 ksi )  2 
15 AS − 0.98 AS 2 − 5.99 = 0
As2 − 15.3 As + 6.11 = 0
AS = 0.41 in 2 per foot of footing
The total steel required across the footing is AS = 7.5 ft (0.41 in2/ft) = 3.08 in2 (Check ACI 10.5.1) for
minimum steel and ACI 7.12 for temperature and shrinkage,
AS 0.41
ρ= = = 0.0023 > 0.0018
bd 12 (15 )
3 f c'
ρ min = = 0.0018bh or
fy
200
ρ min = = 0.0018
fy

Therefore, AS = 7.5 (0.0023)(12)(15) = 3.1 in2 or 0.59 in2 per foot of footing

For B = 90” (7.5’) use 6 # 7 bars ( AS = 3.18 in2 ) @ 12 inches on center

or 5 # 8 bars ( AS = 3.95 in2 ) @ 12 inches on center

Check for Development length Ld (ACI-318-08.12.3), and the embedment length of the dowels.

337
338
*Footings–02: Add a moment to the load on a footing.
(Revision: Sept-08)
The footing shown below is a square footing with the dimensions and loads as shown.
a) Compute the load’s eccentricity e.
b) Check the bearing pressure at ultimate load.
c) Calculate the wide beam shear.

M
P = 165 kips P
M = 20 ft-kips
DL = 100 kips
LL = 65 kips d 3’-d

15”

7 feet
d) Determine the flexural moment for a strip of footing 1 foot wide.
qall = 4 ksf ( with a FS = 1.5) , f 'C = 3 ksi , f y = 60 ksi . P = DL + LL

Solution:
M 20
a) Compute the load eccentricity: e = = = 0.121 ft
P 165

 P  6 e   165   6 ( 0.121' ) 
q =   1 ± =  1 ± =
 A  B   7 ' x7 '   7' 
q max = 3.72 ksf and q min = 3.02 ksf

b) Check bearing pressure at ultimate load: Qu = 1.2(100) + 1.6(65) = 224 kips

Q   224  Q   224 
qall = qmax  u  = 3.72   = 5.05 ksf and qall = qmin  u  = 3.02   = 4.1 ksf
 P   165   P   165 
Then consider qall = 5.1 ksf < qall (FS) = 4 (1.5) = 6 ksf, GOOD.

339
5.1 + 4.1
c) Calculate the allowable one-way (wide beam) shear; assume a d = 1 ft; q = = 4.6
2
B c 7 1
VU = q ( − T − ) = 4.6( − 1 − ) = 9.2 kips Equation (1)
2 2 2 2

The allowable shear = 2φ f c' = 110 psi


Vu (9.2)(1000) Equation (2)
The required d = = = 9.3 inches ≈ 12 in assumed
2ϕ f c' b (0.75)(110)(12)

3’ - d

4.1 ksf
5.1 ksf

d) Determine the flexural moment for a strip 1 foot wide:

x 3
 5 .9 x − 0 .1 5 7 x 2  ft − k ip
M = ∫ Vdx =
0
∫0  2 d x = 2 5 .9
 ft

340
*Footings–03: Find the thickness T and the As of the previous problem.
(Review:
Find, 1) The soil pressure under the footing for the given loads,
2) The footing thickness T, and
3) The flexural steel reinforcement As.

M
N

Given: f y = 60 ksi , f 'C = 3 ksi , qa = 4 ksf , DL = 65 kips , LL = 100 kips , M = 20 ft − kips

N = DL + LL

The actual soil pressure qo (versus the allowable soil bearing capacity):

7’

c=3.5
7’ My
x

bd 3 ( 7 ' )( 7 ' )
3
N Mc
qo = ± where I= = = 200 ft 4
A I 12 12

qo =
165k
±
( 20k . ft )( 3.5 ft ) =
( 7 ' x7 ' ) 200 ft 4
qo max = 3.7 ksf and qo min = 3.0 ksf < 4 ksf allowable GOOD

341
My

3.0 ksf
3.7 ksf

b) The ultimate load on the soil:

Qu = 1.2 D L + 1.6 LL = 1.2 ( 65 kips ) + 1.6 (100 kips ) = 238 kips


M u = 1.6 M LL = 1.6 ( 20 ft .kips ) = 32 ft ⋅ kips
Qu M u c 238 kips ( 32 ft .kips )( 3.5 ft )
qu = ± = ± =
A I ( 7 ' )( 7 ' ) 200 ft 4
qu max = 5.9 ksf and qu min = 4.7 ksf

My

4.7 ksf
5.9 ksf

342
a) Determine the thickness of the footing T.

d
T

3”
3” minimum cover
Check wide-beam shear: Assume d =12”, (controls for rectangular footing).

Q My
7’

w =12”
12”
Critical
d 2’
Section

Vu
d
From ∑f y = 0 , the ultimate shear is
Vu = 82, 600 lbs , but Vu = φ 2 f 'c bd ,

Vu 82, 600 lbs


d = = = 10.6 in < 12 in assumed.
φ2 f 'c b ( 0.85 )( 2 ) 3000 ( 84 in )

Check for punching shear (controls for square footings, such as this one).

d/2

c =
12”

Critical Section
ACI 11.11.1.2

b0 /4

7’

343
 d 
Vu = φ 4 f 'c bo d , where bo is the perimeter of the critical section; bo = 4  c + 2  
 2 

∴ d =
Vu
=
( 5.9 ksf ( 7 ft x 7 ft − 2 ft x 2 ft )1000 )
= 14.8 in assumed;
φ 4 f 'c b ( 0.85 )( 4 ) ( 3000 ) ( 4 )( 24 in )

must increase d.
Recalculate by trying d = 14.0 in.

∴ d =
Vu
=
( 5.9 ksf ( 7 ft x 7 ft − 2.17 ft x 2.17 ft )1000 )
= 13.5 in
φ 4 f 'c b ( 0.85 )( 4 ) ( 3000 ) ( 4 )( 26 in )
d = 13.5 in < 14.0 in, therefore is Good!!
The footing thickness T = 14 in + 1 bar diameter (1”) + 3 in (cover) = 18 in.

Footing dimensions: 7.5 feet x 7.5 feet x 18 in.


Finding the flexural reinforcement As:

q Bl 2 ( 5.9 ksf )( 7 ft )( 3 ft )
2

Mu = s = = 186 kip . ft
2 2
As f y 0.85 f 'c Ba 0.85 ( 3000 )( 7 ft )( a )
a= ( where b = B) then As = = = 0.3a
0.85 f 'c b fy 60, 000

 a
The ultimate moment Mu is given by: M u = φ As f y  d −  with Mu = 186 kip-ft , and φ = 0.9 .
 2

 kip   in 2   a
∴ 186 kip . ft = 0.9 ( 0.3 a ft )  60 2 
2
144 2 
1.16 −  ft =
 in   ft   2
a = 0.07 ft and ∴ As = 0.3 a = 0.021

As A 0.021 ft 2
Percentage of steel p= = s = = 0.00257 > pmin = 0.0018
bd Bd ( 7 ft )(1.167 ft )
344
As min = 0.021 ft 2 = 3.024 in 2 Therefore, use 6 # 7 bars at 12 inches.

 fy 
e) Check development length, Ld (ACI 12.2.2) Ld = 0.04 Ab   > 0.0004d b f y
 f' 
 c 

 0.44 ( 60, 000 ) 


Ld = 0.04 Ab   > 0.0004 ( 0.75 )( 60, 000 )
 3000 
Ld = 19.3 in > 18 in

c  12in 
Actual Ld ( provided ) = B− − ( cover ) =  7 ft  − 6 in − 3 in = 75 in
2  1 ft 
f) Check bearing strength on concrete (ACI 318-02 10.15)

 A   A2 
Bearing strength = 0.85φ f 'c A g  2  where   ≤ 2
 A 
 g   Ag 
Ag is the area of the column and A2 is the area of the footing.

 A   3 kips 
0.85φ f 'c Ag  2  = 0.85 ( 0.70 )   (144 in ) ( 2 ) = 514 kips
2
Then
A   in 2

 g 
But, the factored column load U = 261 kips < 514 kips. Good !
g) Dowels to column:
Since bearing strength is adequate, a minimum area of dowels should be provided across the interference
of the column and footing (ACI 318-02 15.8.2.1)
Minimum area of steel = 0.005 (area of column) = 0.005 (144 in2) = 0.72 in2.
Use 4 # 4 bars AS = 0.20 in 2 ( 4 ) = 0.80 in 2
h) Final design

345
*Footings–04: Find the dimensions B x L of a rectangular footing.
(Revision: Sept-08)
Find the footing dimensions B x L to carry a moment induced by winds of 800 kN-m.

Solution:

Select a test value for B x L. Set B x L = B2 and check the increase in soil pressure due to wind

N (800 kN + 800 kN )
B2 = = = 8 m2 ∴ B = 2.82 m
qa  kN 
 200 2 
 m 
M 800
e= = = 0.5 m ∴ L ≥ 6 ( 0.5 ) = 3 m
N 1600

If L = 3 m, try for qmax ≅ 2qavg iterate by trying footing: 2.5 m x 4 m = B x L.

N 1600 kN
q avg = = = 160 kPa
A 10 m 2
N 1600  6 ( 0.5 ) 
q max = = 1 +  = 280 kPa
 6e  10  4 
BL  1 + 
 L 

Note that qmax exceeds qavg by 33 %, therefore increase the area.

Try using 2.75 m x 4.5 m dimensions;

346
N 1600 kN
q avg = = = 130 kPa
A 2.75 ( 4.5 ) m 2
N  6 ( 0.5 ) 
q max = = 130 1 +  = 217 kPa
 6e   4.5 
BL  1 + 
 L 

Iterate again, with B = 3.0 m and L = 5.0 m

N 1 6 0 0 kN
q a vg = = = 1 0 7 kP a
A 3 m (5 m )
N 1600  6 ( 0 .5 ) 
q m ax = = 1 +  = 1 7 1 kP a GOOD
 6e  15  5 
BL 1 + 
 L 

The footing dimensions are B = 3 m by L = 5 m.

347
*Footings–05: Design the steel for the previous problem.
(Revision: Sept-08)

Design the previous footing using f ' c = 21 MPa and f y = 415 MPa .

Solution.
1) Check the ultimate pressures:

N u = 1.2 ( DL ) + 1.6 ( LL ) = 1.2 ( 800 ) + 1.6 ( 800 ) = 2240 kN

M 800
e= = = 0.5 m
N 1600

Nu  2240   6 ( 0.5 ) 
qmax = =  1 +  = 239 kPa
 6e   15   5 
A 1 + 
 L
N  2240   6 ( 0.5 ) 
qmin = =  1 −  = 60 kPa
 6e   15   5 
A 1 − 
 L

q max = 239 kPa and q min = 60 kPa

348
2240 kN
qavg = = 149 kPa < qall = 200 kPa Good
15 m 2

2a) Calculate footing depth T based on punching shear, for a f 'c = 21 MPa , and

Vc = 1.29 MPa
.

Using the simplified equation for a square footing:

4T 2 + 2 ( 0.5 m + 0.5 m ) T −
(15x 149 )
=0
2240
T 2 + 0.5T − 0.25 = 0 ∴ T ≅ 0.50 m

2b) Calculate footing depth T based on wide beam shear:

The shear is calculated from the outer edge of footing ( x = 0 ) inwards towards a distance d
from the column ( x = 2.225 – d ) :

dv = qdx
2.25 − T
  40.2 x 2  
x x
V = ∫ qdx = ∫ ( 266 − 40.2 x ) dx =  266 x −  
0 0   2   0
V = 598 − 266T − 20.1 ( 2.25 − T ) =
2

Vc 1290
V = = ∴ T 2 + 40.8T − 24.7 = 0 T = 0.60 m
2T 2T

We will use the highest of (2a) or (2b), therefore T = 0.60 m.

349
As f y 415 As
3) a) Find AS ( Longitudinal ): a= = = 23.3 As
0.85 f 'c b 0.85 ( 21)(1)
2.25
2.25 2.25
 40.2 x 2   266 x 2 40.2 x 3 
Mu = ∫ Vdx = ∫
0 0


266 x −
2


dx =  2 − 6 
 0
= 597 kN .m

 a  a  Mu
M u = φ As f y  d −  ∴ As  d −  =
 2  2  φ fy
 23.3  597
As  0.6 − = = As 2 − 0.0515 As + 1.37 x10 − 2 = 0
 2  0.9 ( 415 )
cm 2
As = 28.2
m
As 0.00282
Check p= = 0.0047 > 0.002 GOOD
bd 1 ( 0.6 )
3) b) Find AS (transverse): Use the high average.

q= ( q ave + q max ) = (149 + 266 ) = 20.37 kPa


  ( 3 − 0.5 )  
2
 ( B − 0.5 ) 
2

q  215   
  2
wl 2  2  =    = 168 kN .m
Mu = =
2 2 2

 a M cm 2
and As  d −  = u ∴ As 2 − 0.0515 As + 0.000386 = 0 ∴ As = 7.61
 2  φ fy m

As 7.61 x 10 −4 m 2
Check for ρ= = = 0.00126 < 0.002
bT 1 ( 0.6 )

m2 cm 2
∴ Use minimum As = 0.002 ( b )(T ) = 0.002 (1)( 0.6 ) = 0.0012 = 12
m m
4) Check footing:
cm 2 1m 2
Longitudinal steel: 28.2 (3 m) = 84.6 cm2 2
= 8 .4 6 x 1 0 − 3 m
m 1 0, 0 0 0 c m
Therefore use 17 # 25 mm bars at 17.6 cms o − c

350
Sketch:

351
*Footings–06: Design a continuous footing for a pre-cast warehouse wall.
(Revision: Sept-08)
Design a continuous footing for the warehouse wall with the loads shown below:

Roof

Precast concrete wall


Overhead Crane

12 in
Floor Slab

4 ft

Solution:
1) Assume an initial footing thickness T = 12”:
From ACI 7.7.2, the minimum cover is 3” from the steel to the footing invert. Assume # 4 bars.
Therefore, d = 12” - 3” - 0.5” / 2 = 8.75”

12 in
T=12 in

352
b =12 in

a
d
T

Find the ultimate soil pressure qult.

The actual soil pressure from the structure is,

qo = qall – qconcrete – qsoil-above footing

qo = 2 ksf – (1 ft )( 0.15 k/ ft3 ) – ( 3 ft )( 0.110 k/ ft3 ) = 2 – 0.15 – 0.33 = 1.52 ksf

Estimate B = Q/ qo = 4.2 k/ 1.52 ksf = 2.76 ft. per unit length. Therefore, assume B = 3 ft.

Ultimate load, U = 1.2 DL +1.6 LL = 1.2 ( 3 ) + 1.6 ( 1.2 ) = 4.0 + 2.0 = 6.0 kips

Therefore, the soil pressure at ultimate loads qu is:

qu = U / ( B )( 1 ) = 6.0 kips/ ( 3 ft )( 1 ft ) = 2.0 ksf < qult = 4 ksf  GOOD

2) Check the shear strength of the footing:

The critical section for shear section occurs at a distance d from the face of the wall.

(ACI 15.5 & ACI 11.11.1 ).

qo = 2.1 ksf B = 3 ft
Vu = (12 − d ) qo (1)

12” 12”
b =12” 12 in - d
in d

Vu
T

qo = 2.1 ksf
353
The ultimate shear Vu = (12” – 8.75”) ( 1 ft / 12 in ) x 2.1 k/ ft2 = 0.57 kips / ft of wall

The concrete shear strength must be: Vu ≤ φ Vc = φ 2 f' c bd

Vu = 0.85 ⋅ 2 ⋅ 3000 ⋅ 12in ⋅ 8.75in = 9.8 kips / ft of wall >> 0.57 kips / ft

Since Vu << ØVc , we can reduce the thickness of the footing T from 1.0 ft to say 0.85 ft (~10”).

Therefore, d = 10” – 3” – 0.5 / 2 = 6.5” > 6” = dmin from ACI 15.4

Rechecking, Vu = 0.85 ⋅ 2 ⋅ 3000 ⋅ 12in ⋅ 6.5in = 7.3 kips / ft of wall → GOOD

Therefore, the total thickness T = d + bar diameter + 3” = 10” + 1” + 3” = 14”

3) Design the flexural reinforcement, As ( ACI 15.4):

M u = qo ℓ 2 / 2 where ℓ = 12”

Mu = (2.1 ksf x 1 ft2 )/ 2 = 1.05 kip-ft / ft of wall

But, a = Asfy / [ 0.85 f’c ( b ) ] = [ (60 ksi ) As ) ] / [ 0.85 ( 3 ksi )( 12 in ) ] = 1.96 As (inches)

and, Mn = Asfy ( d – a/2) = As ( 60 k / in2 ) ( d – a / 2 ) = 60 As ( 6.5 in – 9.8 As )

But, Mu = Ø Mn = 0.9 Mn

Therefore, 1.05 kips-in / in = 0.9 (60 k / in2 ) As ( 6.5 in – 9.8 As )

53 As2 – 351As – 1.05 = 0

Two possible answers: As ( 1 ) = 6.6 in2 per ft. of wall

As ( 2 ) = 0.003 in2 per ft. of wall

The percentages of steel with As ( 1 ) & As ( 2 ) ( ACI 7.12.2.1 )

354
ρ1 = As(1) / bd = 6.6 in2 / (12 in )( 6.5 in) = 0.085

ρ2 = As(2) / bd = 0.003 in2 / (12 in )( 6.5 in) = 0.0004 < 0.0018 minimum steel

The maximum steel percentages allowed ρmax = 0.75 ρb, where

ρb = ( 0.85 f’c / fy ) β ( 87,000 / ( 87,000 + fy ) )

= ( 0.85 ( 3 ) / 60 ) 0.85 ( 87,000 / ( 87,000 + 60,000 ) = 0.021

therefore,

ρmax = 0.75 ρb = 0.75 ( 0.021 ) = 0.016

Note that ρ1 = 0.085 > ρmax = 0.016 therefore, use ρmin = 0.0018

Therefore,

As = ρmin b d = ( 0.0018 )( 12 in )( 6.5 in )

= 0.14 in2 per ft. of wall  use 1 # 4 every ft. of wall ( As = 0.20 in2 )

4) Check the development length, Ld ( ACI 12.2 ):

fy 60,000
Ld = 0.04 Ab (But not less than 0.0004 d b f y ) = 0.04 ⋅ 0.20 ⋅ = 8.8"
f' c 3000
Ld = 8.8” or 12”  Clearly, 12” controls.
Presently we have 12” – 3” cover = 9” < 12”. Therefore, we are missing 3” on each side.
Increase the footing width B by 6” to B = 3.5 ft.
(Note that increasing B, reduces qo, and the design could be further optimized.)

355
Therefore,

B = 3.5 ft
T = 13”
As = 1 # 4 @ 12” along the wall

Use minimum steel in longitudinal direction, to offset shrinkage and temperature effects ( ACI
7.12 ):
As = ( 0.0018 )( b )( d ) = 0.0018 ( 42 in )( 6.5 in ) = 0.49 in2
Provide 3 # 4 bars at 12” o.c. ( As = 0.60 in2 )

356
**Footings–07: Design the footings of a large billboard sign.
(Revised Oct-09)
Design a spread footing for the billboard sign shown below using FBC-2004 and ASCE 7- 02.
Ignore the torsion and the wind load on the column, and the water table.
Given: γ = 150 pcf c = 150 pcf φ = 20° V = 146 mph

32'

20'

P = 10 k
24" STEEL COLUMN 40 ' HIGH
WITH 1" THICK WALLS

20'

Y
X
D

solution:
STEP #1: Find the wind load as per ASCE 7-02, assuming an Exposure C, Category I.
qz = 0.00256 Kz (IV)2
Kz = 0.98
I = 1.05
F = qz Gh Cf Af ∴ qz = 52 psf
V = 146 mph
Gh = 1.26
Cf = 1.2
M 32
The sign shape factor is = = 1.6
N 20
(34 psf) (1.26) (1.2) (32 ft x 20 ft)
Therefore: F= = 32.4 kips
1000

357
.

Step #2: Calculate loads on footing


Weight of steel column = γs L A = 0.49
Mx = 10 kips x 15’ = 150 k-ft
My = 32.4 k x 30’ = 972 k-ft
Mz = 32.4 k x 15’ = 486 k-ft
Total (normal) load N = 10 k + 5 k = 15 kips

Step #3: Calculate the footing’s bearing capacity using Hansen’s formula.
c (cohesion) = 0.150 ksf
q = γDf = (embedment pressure) = (0.130 ksf)(3 ft) = 0.39 ksf
B = (footing width – initial assumptions) = 5 ft
L = (footing length – initial assumptions) = 15 ft
Nq (factor for embedment at φ = 20°) = eπ tan φ tan2(45+φ/2) = 6.40
Nc (factor for cohesion at φ = 20°) = (Nq – 1)cot φ = 14.83
Nγ (factor for width at φ = 20°) = 1.5 (Nq – 1)tan φ = 2.95
Fsq = (shape factor for embedment) = 1.0 + (B/L) sin φ = 1.11
Fsc = (shape factor for cohesion) = 1.0 + (Nq / Nc) (B/L) = 1.14
Fsγ = (shape factor for width) = 1.0 – 0.4 (B/L) = 0.867
Fdq = (depth factor for embedment) = 1 + 2 tan φ(1 – sin φ)2 (Df/B) = 1.19
Fdc = (depth factor for cohesion) = 1.0 + 0.4 (Df / B) = 1.24
Fdγ = (depth factor for width) = 1.0

 1-H 
Fic = (inclination factor) = 0.5 -  
 (Af Ca) 
where ca = (0.6 to 1.0) c
Fiq = [ 1 – (0.5 H) / (V +Af Ca cot φ)]d where 2 ≤ d ≤ 5
Fiγ = [1 – (0.7 H) / (V +Af Ca cot φ)]α
qult = c’ Nc Fsc Fdc Fic +q Nq Fsq Fdq Fiq + 0.5 γ B’ Nγ Fsγ Fdγ Fiγ

358
Step #4: Assume:
B = 10’
L = 50’
D = 3’
B/L = 0.2
FS = 3.0
SC = 1.0 + (0.431 x 0.2) = 1.09
DC = 1.0 + (0.4)(3/10) = 1.12
Q = (130)x3 = 390
SQ = 1.0 + 0.2 sin 20 = 1.07
SJ = 1.0 – 0.4(0.2) = 0.92
DQ = 1 + (0.315)(3/10) = 1.09

qmax,min =
P  - 6P e 
+  2 y + 
 - 6P e x 
where P = 15 kips +
( (3x10x50x0.150) ) = 240 kips
2 
BL  BL   BL  FTG WT
My 972
ex = = = 4 .0 5 '
P 2 40
Mx 150
ey = = = 0.6 25 '
P 24 0
 240kips   - 6(240)(0.625)   - 6(240)(4.05) 
qmax,min =  +  +  = 0.89 ksf < 2.5 ksf GOOD
 500   102 x 50   10 x 502 
Step #5: Check out (long direction),
MOT = 32.4 Kips (30+3) = 1069 kip-ft
MR = 5 Kips x 25’ + 225 Kips x 25’ = 5750 kip-ft
F.S. = 5750 / 1069 = 5.4 >> 1.5 GOOD
Check Sliding RS = Σ V tan f + CB = 240 tan f + 150 (10) = 1587 kips >> 32.4 kips
#3
LOAD COMBINATION = 0.75 (1.2D + 1.6L + 1.7W)
FACTORED LOADS: Pu = 1.05 x 240 Kips = 252 kips
Mu-x = 1.05 x 150 kip-ft = 158 kip-ft
Mu-y = 1.275 x 972 kip-ft = 1239 kip-ft
 336   6 x 336 x 0.625   6 x 336 x 4.9 
∴ Q MAX,MIN =  ± ±  = 0.989 ksf, 0.019 ksf
 10 x 50   10 2 x 50   10 x 50
2

Check beam shear: D = 36” – 4” = 32”
VU = 0.019x(50 – 21.33) + 0.15(50-21.33)(0.989 – 0.019) = 14.4 kip per feet
Punching shear will not govern by observation.
359
#3 Design for flexure in long direction
f’C = 3000 PSI and fY = 60000 PSI
A = AS x FY / 0.85 F’C B AS = MU x 12 / F FY (D – A/2)
A = 1.64 in. AS = 0.83 in2
AS  0.83 
R= =   = 0.0022 > 0.0018 OK
BD  12 x 32 
USE # 7 @ 8” O/C, As = 0.90 in2 OK
 1.64 
132 - 
M U = ( 0.9 )( 60 ) 
2 
Therefore; = 126 kip-ft > 116 kip-ft GOOD
12
Use a footing 10 feet x 50 feet x 3 feet thick with # 7 bars @ 8” on-center top and bottom, each
way.

NOTE: In lieu of such a large and expensive footing, a short drilled shaft would be an
efficient and inexpensive foundation. That alternative will be covered in the drilled shaft
section later in this course.

360
Chapter 19
Combined Footings
Symbols for Combined Footings

361
**Combined Footing-01: Rectangular footing with an eccentric column.
(Revision: Sept-08)
Find a suitable rectangular combined footing for the conditions shown below.
Given f 'c = 3 ksi f y = 60 ksi qa = 2.5 ksf (qult = 10 ksf) and use ACI 318-05.

Column #1 Column #2
12 in x 12 in 16 in x 16 in = 1.33 ft x 1.33 ft
4 # 7 bars 6 # 8 bars
DL = 80 kips DL = 120 kips
LL = 60 kips LL = 110 kips
140 kips 230 kips
Edge of column #1 is at the Property Line (PL); spacing between columns is 20 ft on-center.

Solution:
STEP 1: Convert column loads to ultimate; then, convert qall to qult.
P1 = 140 P1u = 1.2 ( 80 ) + 1.6 ( 60 ) = 192 kips
P1 = 230 P2 u = 1.2 (120 ) + 1.6 (110 ) = 320 kips

∑ = 370 = 512 kips

U ltimate U 512 U 512


Ratio: = = = 1.38 qult = ( q all ) = = 1.38 ( 2.5 ) = 3.45 ksf
Service S 370 S 370

362
STEP 2: Determine footing dimensions L and B

∑M o =0 R x = P1 x1 + P2 x2 where the moments are taken about the left corner.

512 x = 192 ( 0.5 ) + 320 ( 20.5 )


x = 12.98 ft , say : 13 ft Thus , L = 2 x = 26 ft
The rounding to 26 ft will prevent complete closure of the moment diagram, but negligibly.

B=
( P1u + P2 u ) = 512
= 5.70 ft Use 5 ft 9 in. For construction; B = 5.75 feet.
Lq u 26 ( 3.45 )

kip
q ' = 3.45 ( 5.70 ) = 19.66
ft
STEP 3: Draw shear (V) and moment (M) diagrams. The column loads are treated as concentrated
loads acting at the centers of the columns. The shear and moments diagrams are shown below.

363
STEP 4: Determine the footing depth T based on one-way (wide-beam) and two-way (punching) shear,
as shown in figure below. Note that the punching analysis reflects on a three-sided section for column 1
and four-sided section for column 2. We shall first determine d via a wide-beam analysis, and then
check for punching shear. From the shear diagram, the maximum shear is near column 2 at a distance d
from the face of column 2.
V
V = 204.4 − 19.66 ( d ) , Also, from vc = , Bdvc = 204.44 − 19.66 ( d )
Bd

ACI 318-05 φ = 0.75

In our case, vc = 2φ f 'c = 2 ( 0.75 ) 3000 = 82.16 psi = 11.8 ksf ,

Hence, ( 5.73 )(13.4 ) d = 204.40 − 19.66 d . Solving d = 2.11 ft = 25.30 in

Using d = 25.30 in we check diagonal tension at column 1.

V=column load less upward soil pressure = 192 − Aqu = 219 − A ( 3.45 )

 d
A =  12 +  (12 + d ) = (12 + 12.6 )(12 + 25.3 ) = 917.5 in 2 = 6.37 ft 2
 2
 d
Hence, V= 192 - (6.37)(3.45) =167.5 kips, Perimeter = 2 12 +  + (12 + d ) = 86.50 in = 7.2 ft
 2
167.5
vc = = 11.02 ksi < 4φ f 'c = 23.7 ksf OK
( 7.2 )( 2.11)

( 41.3)  ( 41.3 ) 
2

At column 2 A= = 11.84 ft 2 and Perimeter =   4 = 13.76 ft


144  12 
V = 320 − 11.84 ( 3.45 ) = 279.20 kips

305.76
vc = = 9.61 ksi < 4φ f 'c = 26.8 ksf OK
(13.76 )( 2.11)

364
STEP 5: Determine the longitudinal reinforcement steel, As:

For f y = 60 ksi , f c' = 3 ksi , M u = 842 k − ft , B = 5.75 ft = 69 in ; d = 25.3 in


M u (12, 000) (842)(12, 000)
R= ' 2
= = 0.0763
f c bd (3000)(69)(25.3) 2
From this value w ≈ 0.08
wf c'
ρ = = 0.0039 > 0.0033 < ρ max GOOD
fy
As = ρ Bd = (0.0039)(69)(25.3) = 6.81 in 2

Use 11 # 8 bars (As = 8.63 in2) at 6 in o.c. across top of footing (at approximately 4 in from each
Side). Provided that 1/3 of the bars extend the full length of the footing, the bars could be cut off as
dictated by the moment requirements (moment diagram). However, the saving is not worth the effort
(engineering, fabrication, placing, etc.). Thus, typically, all bars will run the full length of the footing.
200
Based on , As min = ( 0.0033 )( 25.3 )(12 ) = 1.00 in 2 . For the positive moment,
fy
+As = 1.07 x 5.25 = 5.75 in2. Use 8 # 8 bars; As = 6.28 in2 > 5.75 in2; this is larger by the required
positive moment. Place these bars at 8.5 in. c.c. and say 4 in. from bottom.
Also, based on the moment diagram, running 1/3 of As (three bars) the full length of the footing satisfies
both code and moment requirements. The other five bars could be cut off at, say, half length and placed
on the right half (column 2 side) of the footing.

STEP 6: Determine As in the short direction. Reference is made to the figure below. The widths s1 =
12 + 0.75(25.3) = 31.00 in. = 2.58 ft and s2 = 16 + 1.5(25.3) = 54.0 in. = 4.5 ft.

s1 s2

B 1 2

l1 l2

3.15
2.58 ft 15.77 ft 4.50 ft ft

365
Column #1
P 192
q'= = = 12.94 ksf
B s1 ( 5.75 )( 2.58 )

l1 =
( 5.68 − 1 ) 2.37 ft
2
2  12 
M 1 = (12.94 )( 2.37 )   = 436.0 k − in
 2 
Placing the transverse steel above the positive longitudinal steel, d = (25.92 – 1) = 25.92 in
461.92 0.334 in 2 0.895 in 2
Thus, As ( 25.92 − 0.98 As ) = Solving As = = width.
( 0.9 )( 60 ) ft s1
Based on pmin As = 2.75 in 2 (for 0.0033)

s1 s2

B 1 2

l1 l2

3.15
2.68 ft 15.47 ft 4.70 ft ft
See the figure above to determine transverse steel reinforcement for the footing.
Thus, use 5 # 7 bars, at 8 inches c.c. As = 3.00 in 2 > 2.75 in 2

0.04 ( 0.60 )( 60, 000 )


ld = = 26.3 in ld available = 2.34 ( 2 ) = 28 in
3000
For column 2: Transverse + As .
355
s2 = 4.70 ft ; q' = = 13.30 psf
( 4.70 )( 5.68 )
l2 =
( 5.75 − 1.33) = 2.21 ft
2
(13.30 )( 2.21) (12 ) = 389.7
2

M2 = k − in
2
Using d = 25.92 in (transverse bars on top of longitudinal)

366
375.71 4.82 in 2
As ( 25.92 − 0.92 As ) = = 4.82 in 2 Based on pmin As = width.
0.9 ( 60 ) s2

Use nine # 7 bars at 6.5 in. c.c. As = 5.40 in 2


Development length is same as above.
Based on pmin As in 15.47 ft − sec tion = 15.87 in 2 , Thus use 27 # 7 bars at 6.75 in. c.c.
STEP 7: Evaluate dowel steel.
Column 1
Allowable f c = 0.85 ( 0.65 ) f 'c = 1.65 ksi (ACI 10.15)
P = 12 x 12 x 1.65 = 238 > 192 kips (bearing capacity of the column)
Thus, dowel size, should be one size smaller than column bars; use 4-#6 As = 1.76 in 2
Column 2:
In our case,
3000
f c' = ( 0.85)( 0.65)(16 )(16 ) = 424 ksi > 320 ksi GOOD
1000
Use four # 6 bars; As = 1.76 in 2 > (16 )(16 )( 0.005) = 1.28 in 2 GOOD

STEP 8: Make a drawing showing design details.

367
An alternative design is of course, a strap footing,

Typical configuration of a strap footing.

368
**Combined Footing-02: Combined footing on the property-line.
(Revision: Sept-08)
Design the combined rectangular footing shown below using the assumptions given.
f 'c = 3.5 ksi; f y = 50 ksi; soil qa = 5 ksf , SF=3, qultimate = 15 ksf ;

Solution:

STEP 1: Convert column loads to ultimate; convert qa to qultimate .

P1 = 255k P1 ult = 1.4 (80k ) + 1.7 (175k ) = 410k

P2 = 330k P2 ult = 1.4 (130k ) + 1.7 (200k ) = 522k

∑ = 585 k
= 932

369
STEP 2: Determine the footing dimensions L and B:
R x = P1 x1 + P2 x2 or 932k x = 410k (0.75') + 522 k (18'+ 0.75')

x = 10.8 feet say 11 feet and L = 2 x = 2 (11 ft ) = 22 feet


Note: Rounding the 10.8’ to 11’ will prevent the closure of the moment
diagram, but thi is acceptable with hand calculations. A computer
program is better.
P1ult + P2ult 410k + 522 k
B= = = 5.31 feet B = 5'− 4" in construction
Lqo (22 ')(7.97 ksf )

k
Finally q '(soil reaction) = qo B = 7.97 x 5.31 ft = 42.3 k ft along L.
ft 2

STEP 3: Draw the shear V and moment M diagrams.

370
STEP 4: Determine the footing depth D.
Shear at a distance d from the face of column 2, V = 341k − (42.3k ft ) (d )
V
But also the shear stress Vc in concrete is Vc = ∴ BdVc = 341k − (42.3d ) k
Bd
But Vc = 2∅ f 'c = 2 (0.85) 3500 = 100.6 psi = 14.5 ksf (for wide beam shear)

∴ (5.31 ft ) d (14.5ksf ) = 341k − 42.3d ∴ d = 2.86 ft = 34.3 inches


Using d = 34.3” check diagonal tension at column # 1

V = Column load less upward soil pressure


V = 410 k − Aqult = 410k − A(7.97)
d
but A = (18"+ ) (18"+ d ) = (18"+ 17.1") (18"+ 34.3")
2
A = 1836 in 2 = 12.75 ft 2
Hence V = 410 k − (12.75 ft 2 ) (7.97 ksf ) = 308.4k
d
Perimeter = 2 (18"+ ) + (18"+ d ) = 2 (35.1") (52.3")
2
= 122.5" = 10.2 ft
V 308.4k for diagonal
∴ Vc = = = 10.6 ksf < 4φ f 'c tension shear
( perim) d (10.2 ') (2.86 ')
4 (0.85) 3500
201 psi
29 ksf OK
Checking around column # 2
Area A = (24"+ d ) 2 = 3400in 2
= 23.6 ft 2
Perimeter = 4 (24"+ d ) = 4 (58.3") = 233.2"
= 19.4 ft
Shear V = 522 - (23.6 ft ) (7.97 ksf ) = 334 k
k 2

V 334k
∴ Vc = = = 6.0 ksf < 4∅ f 'c
( perim) d (19.4 ') (2.86 ')
29 ksf OK

371
STEP 5: Determine the reinforcing steel As (longitudinal).
a M As f y As (50 ksi ) 1
As (d − ) = but a= = = 1.40 As
2 ∅f y B 0.85 f 'c 0.85 (3.5 ksi ) 12"

∴ As (34.3"- 0.70 As ) = [(1646 k - ft ) (12 in ft )] [(0.9) (50) (5.31')]

As = 2.5 in 2 ft width

for balanced steel


As 2.5in 2 200
ρ= = = 0.006 >
db (34.3") (12") fy
200
>
50000
> 0.004 OK
also < ρ max = 0.016

AS = 2.5 in 2 x B

in 2
total = 2.5 (5.31 ft ) = 13.3 in 2 of steel
ft
Use 11 #10 bars (1.27 in 2 each x 11 = 14 in 2 ) at 6” spacing center-to-center.

372
**Combined Footing-03: Design a combined footing.
(Revised Nov-09)
Design the combined footing shown below using (ACI 318-02), given f’c = 3 ksi and fy = 60 ksi,
DL = 140 k DL = 180 k
LL = 90 k LL = 110 k

Solution:

Compute the footing dimensions:

P1 = 140k + 90k = 230k


P2 = 180k + 110k = 290k

The location of the resultant is,

P2 L c − c ( 2 9 0 ) (1 6 )
X = = = 8 .9 2 fe e t
P1 + P2 290 + 230

373
The footing length L is,

L c1 13
L = 2( + x) = 2( + 8 .9 2 ) = 1 8 . 9 3 ' ∴ u s e L = 1 9 fe e t
2 12

P1 + P2 230 k + 290 k
The footing area is, A= = = 148.6 ft 2
qall 3.5 ksf

A 148.6 ft 2
The footing width is, B = = = 7.82 feet; use 8 feet
L 19 ft

Therefore footing size is 8 feet x 19 feet.

2) Convert the allowable soil pressure, qall to an equivalent ultimate load value, qu :

Under column #1, P1 = 1.2 DL + 1.6 LL = 1.2(140 K ) + 1.6(90 K ) = 312 K

Under column #2, P2 = 1.2 DL + 1.6 LL = 1.2(180 K ) + 1.6(110 K ) = 392 K

Overall factor = ( P1 + P2 ) ∑ N = (312 K + 392 K ) 520 K = 1.35

That is, P = 1.35( DL + LL)

∴ qult = (3.5 ksf )(1.35) = 4.725 ksf

3) Find the shears and moments along the footing, and draw their diagram.

∑P u = Pu1 + Pu 2 = 312 kips + 392 kips = 704 kips

Vu 704 kips kips


The load per foot of the beam (w) = = = 37.05
L 19 ft ft

Check: qult ( B ) = (4.725)(8 ft ) = 37.8 k / ft > 37.05 k / ft GOOD

w = 37.05 k/ft

1.0833 ft
x

374
Note: These two values would be exact if we corrected qult to 5.28 ksf.

Integrating from x = 0 to x = 1.083 ft

312 k kip
q = 37.8 − = −250.3
1.083 ft ft

1.083
V=∫ qdx = 250.3 x + c0 , where c0 = 0 since V = 0 at X = 0
0

∴ V = ( −250.3)(1.083) = −271 kips a very minor difference

250.3 X 2
M = ∫ Vdx = + c1 , where c1 = 0, since M = 0 at X = 0
2
 250.3 
∴M =   (1.083) = −146.7 kip-ft
2

 2 

Integrating from 1.083' < Χ ≤ 15.9 '

37.05 kips
q=
ft
V = ∫ qdx = 37.05 − 312

M = ∫ Vdx = 37.05( x 2 2) − 312( x − 0.54)

The maximum moment, and its location is found by setting the shear equal to zero in terms: V = 0

∴ V = 37.5 x − 312 = 0 ∴ x = 8.42 feet

(8.42) 2
∴ M max =(37.05)[ ] − (312)(8.42 − 0.542) = −1145 kip − ft
2

Similarly, integrate

from 15.9 ft < x ≤ 17.17 ft , and from 17.17 ft < x ≤ 18.93 ft

375
The resulting shear and moment diagrams are shown below.

376
Find the depth d for wide beam shear, at the location of the largest shear (that is, at the interior of
column #2, V max = 277.5 kips). The allowable concrete stress VC is given by ACI 318-02- 114.1,

vC = 2∅ f c ' = (2)(0.75) 3000 = 82.16 psi = 11.83 ksf where φ = 0.75


Vbeam = Vmax − wd = 277.5 − 37.05d
Vconcrete = BdvC = 8d (11.83 ksf ) = 94.64d
Equating both: Vbeam =Vconcrete ∴ 277.5=94.64d + 37.05d
∴ d = 2.10 feet = 25.3 inches; therefore h = d + 3 + bar diameter = 25.3 + 3 + 1 = 29.3 use 30 inches
Check the punching shear: vc = 4∅ f c' = (4)(0.75) 3000 = 164.3 psi = 23.66 ksf

At column #1, the perimeter p in shear is,

2.1
perimeter p = 2(1.08 + ) + (1.08 + 2.1) = 7.5 feet
2
Vshear = pdVc = 7.5(2.1)(23.66) = 373 kips provided > 349 kip needed = P1 GOOD
capacity

Similarly, at column #2, the perimeter in shear is,

perimeter p = 4(1.25 + 2.1) = 13.4 feet


Vshear = (13.4)(2.1)(23.66) = 666 kips provided > 392 kips needed = Pu 2 GOOD
capacity
concrete

377
5) Compute the area As of steel required.

Longitudinal steel will be required both at the top of the footing between columns (positive moment
steel), and at the bottom in the cantilevered part. Also required is transverse steel perpendicular to
longitudinal axis, plus shrinkage and temperature steel.

Negative moment (flexural steel):

M u (max) = −1082 kip − ft for a footing width of B = 8 ft = 96 inches and d = 25.3 inches
M u (12, 000) (1082)(12,000)
R= ' 2
= = 0.07 which gives w ≃ 0.008
f c bd (3000)(96)(25.3)2
wf c' (0.008)(3)
ρ= = = 0.0004 < ρ min (0.0033) therefore use ρ min = 0.0033
fy (60)
As = ρ min bd = (0.0033)(96)(25.3) = 8.01 in 2
Use 11#8 bars at 6" o − c (this provides 11x 0.79 = 8.69 in 2 > 8.01 in 2 required )
For temperature steel As = 0.002 Bh = 0.002(96)(30) = 5.76 in 2

These top bars must be hooked (see ACI 12.1, 2) at column #1’s end; remember to use 3”cover.

Positive steel (at column #2)

83.9 kip − ft
MU = = 10.5 kip -ft ft which requires As = 0.09 in 2 per foot
8 ft
This area of steel is obviously too low, therefore use same as above 11#8, etc.

Design for shrinkage steel requirements and spacing of bars (ACI 7.4.3).

Pu 1 + Pu 2 312 + 392
Transverse steel (short direction) q u lt = = = 4 .6 3 k s f
BL (8 )(1 9 )

d  25.3
For column #1 (a=13” wide): a +   = 13 + = 25.65 in = 2.14 ft
2 2

d for steel = 25.3 + 0.56 –1.69 (for #9 bars) ∴ d = 24.2 inches

The length of cantilever Lc (to calculate bending moment) is,

Lc =
(8 ft − 1.083 ft )
= 3.46 ft and ∴ M =
qL2 4.63 (3.46) 2
= = 27.7 ft -kips
2 2 2

The required area of steel As ,

378
M (12, 000) (27.7)(12, 000)
R= ' 2
= = 0.0018 which gives a w ≃ 0.003
f c bd (3000)(96)(25.3) 2
wf c' (0.003)(3)
ρ= = = 0.00015 << 0.002 of ρ min therefore use As min
fy (60)
As min = ρ min bd = (0.002)(96)(25.3) = 4.85 in 2
Use # 6 bars at 12 inches on center both top and bottom.

7) Sketch of the completed design:

379
**Combined Footing-04: Optimization via a trapezoidal footing.
(Revised Nov-09)
Design a trapezoid-shaped footing, with columns at ends of footing as shown below, where f’c = 21
MPa, fy = 414 MPa, qall = 120 kPa and L = 7 m.

7m

a b
1 2

Column #1 (0.3 m x 0.3 m) Column #2 (0.4 m x 0.4 m)


4 # 25-mm bars 6 # 25-mm bars
DL = 355 kN DL = 535 kN
LL = 265 kN LL = 490 kN

Solution:
Step 1. Convert P1 and P2 to ultimate loads P1u and P2u, and qall to qult:
P1 = 620 P1u=1.4(355) + 1.7(625) = 947
P2 = 1025 P2u=1.4(535) + 1.7(490) = 1582

∑ = 1645 = 2529 kN

ultimate.loads 2529
Ratio = = = 1.54
actual.loads 1645
qult = qall x 1.54 = 184.5 kPa
Step 2. Determine a and b (the two footing widths):
[(947 )(0.15) + (1582)(6.8)] = 4.31 m
∑M O =0 x=
2529
For the trapezoid,

A=
(a + b )L = (a + b )7 = 3.5(a + b )
2 2

Also, A =
∑ P = 2529 = 13.7 m
qu 184.5

380
Thus, 13.71 = 3.5(a + b) or (a + b) = 3.92 m (1)

 7  
 3 (a + 2b )
 L  (a + 2b )  
From x =  or 4.31 =  
 3  (a + b ) (a + b )
Simplifying, 0.8471a – 0.1529b = 0 (2)

Simultaneous solution of Equations 1 and 2 yields,


b = 3.32 m; a = 0.60 m
Check via substituting in the expression for A or x ; for A,
3.5(3.32 + 0.6) = 13.71; OK
Step 3. Draw the shear V and moment M diagrams on the next sheet.

381
Step 4. Determine d. First, we’ll evaluate wide-beam shear at the narrow end, and then
check for diagonal tension at column 2. From the shear equation,
V = 121(x − 0.15) + 35.8(x − 0.15) − 930
2

At d from the inner face of column 1 , x = 0.3 + d. Hence,


V = 121(d + 0.15) + 35.8(d + 0.15) − 930
2

Or V = 35.8d 2 + 131.74d − 911 (3)


V
Also, vc = 660 KPa =
bd
b at a distance d from inner face of column 1 is

b = 0 .6 +
(d + 0.3)(3.32 − 0.6) = 0.3885d + 0.716
7
V V
660 = =
( )
Thus,
(0.3885d + 0.716)d 0.3885d 2 + 0.716d
or V = 256.4d 2 + 472.56d (4)
Equating equations #3 and #4,
(
256.4d 2 + 472.56d = 35.8d 2 + 131.74d − 911 )
or 292.2d 2 + 604.3d − 911 = 0 which yields, d = 1.01 m
Check vc ; at x = 1.01 + 0.3 = 1.31 m
b = 0.392 + 0.716 = 1.108
V = 121(1.16 ) + 35.8(1.16 ) − 930 = −741.5
2

741.5
vc = = 662 which is close to 660 kPa OK
(1.108)(1.01)
By inspection, required d at the large end is less than 1.01 m.
Step 5. Determine the flexural steel area based on moments at several points,
 a M
Use As  d −  =
 2  0 .9 f y b

As f y 400 As
a= = = 22.4 As
0.85 f ' c b 0.85(21)(1)
at x = 1.5 m, we have
 22.4 As  1115 3
As1.01 − = = 2.65 × 10 −
 2  (0.9 )(400 )(1000(1.18))
Or
As − 0.09018 As + 0.000234 = 0
2

382
7m

0.6 3.32

2.5
3.5
3.32
Possible Arrangement of As
Solving
cm 2
As = 0.0036m 2 = 36 width
m
Checking percent steel,
As 0.0036
ρ= = = 0.00356 > ρ min ; OK < ρmax
bd (1)(1.01)
Thus total As at x = 1.5m is ( As )(b ) = (36)(1.18) = 42 cm2/m
Similarly for points at x = 2.5 m; 3.84 m; 5m. The results are summarized below

As cm2
x (m) Width b M Per 1 m For Width min.* No. of
(m) Kn-m Width b 25-mm
Bars
1.5 1.18 1115 36 42 42 8
2.5 1.57 1695 31 48.5 48.5 10
3.84 2.09 2005 max 27.3 57 57 12
5 2.54 1725 19 48.5 51 1

* As based on ρmin is probably debatable, particularly near the wide end, since b varies so greatly. Thus
using 12-25-mm bars are deemed as satisfactory (no cutoffs) for the 4 m on “right” side.

Step 6. Ast in the transverse direction. The effective “transverse beam width “ s2 can be

approximated as before.

s 2 = 0.4 + 0.75(d ) = 1.16m

383
 3.095 
L2 =   − 0.4 = 1.147
 2 

qL2  P  (1.147 )2
M = =  , etc.
2 A  2
 strip 

s1 7m s2

2.87 m 3.32 m

By inspection , As’ ≤ As, based on ρmin

Thus, Asmin = (5-25-mm bars)

Asmin = (0.002)(1.01) = 20 cm2 / m

Practically, its advisable to space these bars at about 0.25 m even spacing for the entire 7 m length, with
varying length, for approximately 3 m near the wide end, to about 0.4 m near the narrow end. These bars
would be tied to the +As at about 0.2 m from the bottom of excavation. Other configurations of
placement are, of course, possible.

Step 7 Four dowels, 25-mm bars are recommended for each column (see Example 7.1)

Step 8. Detail the drawing. The following figure shows a suitable layout. Because of the irregular
shape, some practical improvisation is frequently a practical necessity.

384
4 25-mm bars 4 25-mm bars

12 25-mm bars placed as shown

25-mm bars at 0.25 m c.c. (varied lengths)


● ● ● ● ● ● ● ● ● ● ● ● ● ● ● ● ● ● ● ●

Fig. 7.11 Illustration of final design details for footing in Example 7.2

385
**Combined Footing-05: Combined footing.
(Revised Nov-09)
Design a combined footing for the following conditions:
f ’c = 3,750 psi, fy = 20,000 psi and qall = 8.5 ksf.

Dead and Live Loads


DL = 1050k + DL = 1210k = 1375k
LL = 325 + LL = 615 = 1825k
Total load = 1375k + TL= 1825k = 3200k

Solution:

Locate centroid: x = 1825 * 24’ = 13.5’ + 2.5 =


3200
16.0’
Length: 2 * 16.0’ = 32.0’
Clearance: 32.0’ – 26.5’ = 5.5’

3200
Width = Combined Load: 32 x 8.5 = 11.8 ≈ 12.0’

Length x qallow

Required depth: d = 6018 = 42”


0.298x12

(3” added for cover + 1” margin for error)


d = 46”

V= 1175000 = 175 psi


12(12) 7 * (54)
8
Longitudinal Steel:

Bottom steel + As = 6018 = 90.85 in2


(1.44)(54)

ΣO = 1175000 = 104.26 in2.


280( 7 ) * 54
8

Top steel - As = 1010 = 12.3 in2


1.44(57)
386
Combined Footing
Transverse steel: M = 5’ x 32’ x 8.33 ksf x 2.5 = 3330 kip-ft.

As = 3330 = 41.6 in2 x 1825 = 23.8 in2


1.44(55) 3200
1375
3200
(5)(32)(8330)
ΣO = = 100 in
280( 7 ) * 55
8
Use 15- #11 bars in left column at 5.5” c/c.
12- #11 bars in right column at 10” c/c.
Web reinforcing: Vc = 0.075(12)(12) ( 78 ) 55 = 515 k
V’ = V – Vc = 1175 – 515 = 660 k
6 sets of #6 stirrups Av = 5.28 in2

Avfujd 5.28(16)( 7 ) * 55
Spacing = s = = 8 = 6 in.
V' 660
For 12” spacing: V’ = 330 + 515 = 845 kips
For 24” spacing: V’ = 165 + 515 = 680 kips

387
**Combined Footing–06: Strap footing (or grade beam) between two columns.
(Revision: Sept-08)
Design a strap-footing given f’c = 3.5 ksi, fy = 60 ksi, and qult = 3.85 ksf.
Column #1 Column #2
12 in x 12 in with 4 # 7 bars 16 in x 16 in = 1.33 ft x 1.33 ft with 6 # 8 bars
DL = 80 kips DL = 120 kips
LL = 60 kips LL = 110 kips

Solution:

P1 = 1.2 DL + 1.6 LL = 1.2( 80 kips) + 1.6( 60 kips) = 192 kips.

P2 = 1.2 DL + 1.6 LL = 1.2(120 kips) + 1.6(110 kips) = 320 kips.

The left edge of column #1 is placed at the property line (P.L.).

P1 = 140 kips and P1u = 192 kips

P2 = 230 kips and P2u = 320 kips

∑ →= 370 kips →= 512 kips


Ratio = 512 / 370 = 1.38 and qult = 3.85 ksf
Try e = 3 feet. From Σ M2 =0 , we have (192)(25) – R1(22) = 0, R1 = 218.2 kips
From Σ M1 = 0, 3220(22) – (192)(3) – R2(22) = 0, R2 = 293.8 kips
From Σ Fy = 0, 218.2 + 293.8 = 512 kips.
388
Step 2:
L1 = 2(0.5 + e) = 2(0.5 + 3) = 7 feet
B1 = R1 / (L1)(qult) = 218.2 / (7)(3.85) = 8 feet
L1
= 0.875
B1
The ratio appears reasonable. If a specific clearance between footings is required, then
adjustment of L1 and L2 would be in order. If we select to make footing # 2 a square,
R2 293.8
B=L= = = 8.73 feet Round out to 9 feet.
qu 3.85

Step 3: Draw the shear and moment diagrams.

389
Step 4:

Design the strap using the figure above and V = 23.6 kips and M = 493 k-ft. While a deeper strap is more
efficient than a wider but shallower one, a too narrow strap may pose some practical or functional
concerns (for example, limited space for reinforcement, lateral or torsional stiffness, etc). The
recommended range for strap footing widths b is,

 L' 
2 fe e t ≤ b ≥   fe e t
 10 

In addition, the effective depth d, should be 50 to 100 % larger than theoretically needed, in order to
reduce the number of reinforcing bars, provide spacing and minimize the effects of an uneven
excavation and perhaps even eliminate the need for stirrups.

Let L’ = 14 feet, the distance between footings. In this problem L’/10 = 1.4 ft; hence use b = 2 ft.

V = vc bd where vc = 2φ f 'c = (2)(0.75) 3500 = 88.7 psi = 12.8 ksf

Thus, from 23.60 = (12.7 ksf)(2 ft) d, d = 0.92 feet. From experience use d = 24 in.

With M u = 493 kpi − ft from the moment diagram,


M u (12, 000) (493)(12, 000)
R= ' 2
= = 0.122 which gives w = 0.149
f c bd (3500)(24)(24) 2
wf c' (0.149)(3.5)
ρ= = = 0.0087 therefore As = ρ bd = (0.0087)(24)(24) = 5.0 in 2
fy (60)
Use 7 # 8 bars ( which provide As = 5.5 in 2 ) at 3.25 inches o − c, extended to the footings.

Step 5. Design of the footings using the figure below. In footing #1, for b = 8 feet and

Determine the section thickness d for the strap.

With vc = 12.7 ksf, (8 ft)(12.70 ksf)d = 161.2 – 30.8 d (see the shear diagram).

Solving, d = 1.22 feet = 14.6 inches.

390
Check diagonal tension with V = 192 kips

1+0.75d =23
in

3.83 ft
40 in

8 ft #2
#1 12 in + d = 28
9 ft

12 in + d/2=20

7.0 14.0 9.0


 d
where A=  12+  (12+d ) = (12 + 7.3)(12+14.6 ) = 513.4 in 2 = 3.56 ft 2
 2

Perimeter = 2 (12 + 7.3) + (12 + 14.6) = 65 in = 5.43 feet

Hence, V = 192 - 18.73 = 173.3 kips

V 173.3
and vc = = = 26.14 > 4φ f 'c =(4)(0.75) 3500 = 25.55 ksf Not Good
pd (5.43)(1.22)

Therefore, increase d to 16 inches. Therefore, A = 2 (20)(28) = 1120 in2 = 7.78 ft2

Perimeter = 2 (12 + 8) + (12 + 16) = 68 in = 5.67 feet

Hence, V = 192 – (7.78)(3.85) = 184 kips

V 184
and vc = = = 24.4 < 25.55 ksf GOOD
pd (5.43)(1.33)

The transverse steel. In the transverse direction, a disproportionate percentage of the column load P1 is
carried by the left side of the footing #1. Hence, concentrate more of the transverse steel in that segment
of the footing (as done for similar cases of rectangular and trapezoidal footings). On the other hand,
footing #2 resembles a spread footing condition, and therefore will be treated that way. Thus, for footing
#1, assume the shaded width s, as 12 in. + 0.75d = 2 feet, as shown in the figure.

391
192
q= = 10.66 ksf the critical section for flexure at the face of the column.
(2)(8)
10.66(3.5) 2
M= = 65.3 k-ft
2
M (12, 000) (65.3)(12, 000)
R= ' 2
= = 0.0091 which gives w = 0.011
f c bd (3500)(8 x12)(16)2
wf c' (0.011)(3.5)
ρ= = = 0.00064 < 0.0033 = ρ min therefore use ρ min
fy (60)
As = ρ min bd = (0.0033)(96)(16) = 5.0 in 2

Since the reinforcing from the strap is the same, add 3 rebars (#8s) at each side of the footing as shown
in the final sketch of the footings with the strap.

Footing #2. When comparing column loads, the d = 1 ft appears low for footing #2 when d = 16 inches
was needed for footing 1. Check diagonal tension using d = 16 in. similar to footing # 1.

A = (16 + 16)2 = 1024 in2 = 7.11 ft2

Perimeter = 4 (16 +16) = 128 inches = 10.67 feet

V = 320 – 7.11(3.85) = 292.6 kips

V (297.6 kip )
vc = = = 20.6 ksf < 4φ f c'=(4)(0.75) 3500=25.55 ksf Good
pd (10.67 ft )(1.33 ft )

Use d = 16 inches = 1.33 feet for both footings.

As = ρ min bd = (0.0033)(108)(16) = 5.7 in 2

The maximum moment is 502 k-ft and occurs in footing #1 versus the moment 493 k-ft that occurs at
footing-strap juncture. Therefore extend the seven # 8 bars into footing #1 and add six more #8s (three
on each side) to carry the slightly larger moment.

In the transverse direction,

392
V 292.6
vc = = = 20.61 ksf < 4φ f c'=(4)(0.75) 3500=25.55 kips Good
pd (10.67)(1.33)
qult ( L' )2 (3.85 ksf)(3.83 ft)2
M= = = 28.3 k-ft
2 2
M (12, 000) (28.3)(12,000)
R= ' 2
= = 0.0035 which gives w = 0.004
f c bd (3500)(8 x12)(16)2
wf c' (0.004)(3.5)
ρ= = = 0.0002 < 0.0033 = ρ min therefore use ρ min
fy (60)
As = ρ min bd = (0.0033)(108)(16) = 5.7 in 2

In the longitudinal (axial direction), M = 502 k-ft,

M (12, 000) (502)(12,000)


R= ' 2
= = 0.062 which gives w = 0.071
f c bd (3500)(8 x12)(16)2
wf c' (0.071)(3.5)
ρ= = = 0.0042 > 0.0033 = ρ min
fy (60)
As = ρ bd = (0.0042)(108)(16) = 7.25 in 2

Use 13 # 8 bars (As = 10.20 in2) and extend the 7 # 8 bars from the strap the full length (7 feet) and add
3 # 8 bars at approximately 16 inches c.c. on each side of strap bars.

393
394
Chapter 20
Mat Foundations
Symbols for Mat Foundations

395
*Mat Foundations–01: Ultimate bearing capacity in a pure cohesive soil.
(Revision: Sept-08)
Determine the ultimate bearing capacity of a mat foundation measuring 45 feet long by 30 feet
wide placed 6.5 feet below the surface and resting upon a saturated clay stratum with cu = 1,950
lb/ft2 and φ = 0º.

Solution:
Mat foundations in purely cohesive soils have the following ultimate bearing capacity:

0.195 B 0.4 D f
qult(net) = 5.14 cu (1+ )(1+ )
L B
0.195 ( 30 ft ) 0.40 ( 6.5 ft )
qult(net) = 5.14(1.95 ksf ) [1+ ] [1 + ] = 12 ksf
( 45 ft ) ( 30 ft )

396
*Mat Foundations–02: Ultimate bearing capacity in a granular soil.
(Revision: Sept-08)
What will be the net allowable bearing capacity of a mat foundation 15 m long by 10 m wide,
embedded 2 m into a dry sand stratum with a corrected SPT to 55% efficiency N55 = 10? It is
desired that the allowable settlement is ∆Hall = 30 mm.

Solution:

The allowable bearing capacity of a mat foundation in granular soils was proposed by Meyerhof (with a
Factor of Safety of 3) to be based on the SPT corrected to a 55% efficiency as,

0.33D f ∆H all 0.33 ( 2 ft ) 30 mm


qall = 12.5 N 55 [1 + ][ ] = 12.5 N55 [1 + ][ ] = 151 kN / m 2
B 25.4 mm (10 ft ) 25.4 mm
An alternate formula is,
∆H all 30 mm
qall = 15.9 N 55 [ ] = 15.9 N 55 [ ] = 188 kN / m2
25.4 mm 25.4 mm
Since qall = 151 kN / m 2 is the smaller of the two, choose this one for the answer.

397
*Mat Foundations–03: Find the depth Df for a fully compensated mat.
(Revision: Sept-08)
The mat shown above is 30 m wide by 40 m long. The live and dead load on the mat is 200 MN.
Find the depth Df for a fully compensated foundation placed upon a soft clay with a unit weight γ
= 18.75 kN/m3.

Df Q

Solution:
The net soil pressure q under the mat is the load from the building Q over the entire
mat, minus the weight of the soil excavated D f γ ,
Q
q= − Df γ
A
When the mat is fully compensated, the weight of the soil W excavated is equal to the
weight of the newly imposed building Q, in other words q = 0 and therefore,
Q [200x103 kN ]
Df = = =9m
Aγ [(30 m)(40 m)(18.75 kN / m3 )]

398
*Mat Foundations–04: The consolidation settlement of a mat foundation.
(Revision: Sept-08)
The mat foundation shown below is 30 m wide by 40 m long. The total dead plus live load on the
mat is 200 MN. Estimate the consolidation settlement at the center of the foundation; Cc and eo of
the normally consolidated clay are 0.28 and 0.9 respectively.

Solution:
The net load per unit area q is,
( 200, 000kips ) − 2m 15.7 kN / m3 = 135 kN / m 2
( )( )
Q
q= − Df γ =
A ( 30m )( 40m )
The pressure at mid-clay (depth of 18 m below the mat) is found via Boussinesq as,
z 18m z 18m L 40m
m= = = 0.6 n = = = 0.45 and = = 1.33 ∴∆pm = 0.66q
B 30m B 40m B 30m
∆pm = 0.66q = ( 0.66 ) (135 kN / m 2 ) = 89 kN / m 2
The in-situ stress at mid-clay layer before the mat foundation is built is,
po = ( 4m ) (15.7 kN / m3 ) + (13m )(19.1 − 9.81) ( kN / m3 ) + ( 3m )(18.6 − 9.81) ( kN / m3 ) = 210kN / m 2
The consolidation (plastic) settlement is,
Cc H  p + ∆pm  ( 0.28 )( 6, 000mm )  ( 210 ) + ( 89 ) 
∆H = log10  o  = log10   = 136 mm
1 + eo  po  1 + ( 0.9 )  ( 210 ) 

399
*Mat Foundations–05: Settlement of a rigid mat.
(Revision: Sept-08)
A building is to be supported by a rigid reinforced concrete mat foundation, whose dimensions are
20 m wide by 50 m long. The load on the mat is to be uniformly distributed with a magnitude of 65
kPa. The mat rests on a deep deposit of saturated clay with an approximate un-drained Young’s
modulus Eu = 40 MPa and a Poisson ratio ν = 0.4. Estimate the immediate settlement at the center
and corner of the mat.

Solution:

Since the mat foundation is stiff, use the rigid factor Cs is found from the L x B ratio,

L 50
= = 2.5, which by interpolation in the chart provides a Cs = 1.20.
B 20

One of the possible equations for immediate settlement ∆Hi is this one,

1 - µ2 2 1 - (0.4)2
∆Hi = C s q B ( ) = (1.20)(65 kN/m )(20 m) [ ] = 0.033 m
Eu 40x103 kN / m 2

Since the mat is assumed to be rigid, the surface settlement at both the center and at the corners of
the mat, are the same, which is 33 mm.

400
*Mat Foundations–06: Mat foundation carries a large transfer girder.
(Revision: Sept-08)
A transfer girder carries a load of 9,000 kips. It is supported by two square reinforced concrete
columns, 30 in x 30 in. The columns are to be supported by a mat foundation upon a soil that has
an allowable bearing capacity qa = 4 ksf.
Using a load factor of 1.5, f’c = 4 ksi, concrete shear vc = 31 ksi and fy = 60 ksi, design the width and
length B by B of the mat and its thickness T.

Solution:

9000
The square mat has dimensions BxB, B2qa = Q therefore B = = 47.4 feet
2

The effective depth d of the mat can be found using the approximate formula,

BLq
4d2 + 2(b + c)d = where b = c = 30 in = 2.5 feet, and the column load = 2,000 kips
vc

BLq = column factored load = 1.5(2,000) = 3,000 kips

3000
Therefore, 4d2 + 2(2.5 + 2.5)d = or d2 + 2.5d = 24.193 or d = 3.8 feet
31

The thickness T = d + 0.4 (for cover and reinforcing) = 4.25 feet = 50 inches

401
**Mat Foundations–07: Design a small mat for an office building.
(Revision: Sept-08)
A small office building with the column loads shown below is founded 3 m deep into a sand
stratum with a unit weight of 18 kN/m3. The foundation is the mat shown below. All the columns
are 0.5 m x 0.5 m. The concrete strength is f ’c = 20.7 MN/m2 and the steel yield strength is fy =
413.7 MN/m2. Determine their reinforcement requirements in the y-direction only.

402
Solution:

Step 1:

Find the soil pressures, the location of the soil reaction’s resultant and the eccentricities in the x and y
directions.

The service load = (400 × 2) + (500 × 2) + 450 + 350 + (1500 × 4) + (1200 × 2) = 11, 000 kN
The moments of inertia of the mat in the x and y-directions are,

xy 3 16.5 × 21.53
IX = = = 13, 670 m 4
12 12
x 3 y 16.53 x 21.5
IY = = = 8, 048 m 4
12 12
To find the eccentricity in x and y directions, take moments about the axes.

For the eccentricity about the y’-axis, take ∑M y' =0


(11,000)x’ = (8 m)(500+1500+1500+500)+(16 m)(450+1200+1200+350)
16.5
x’ = 7.56 m, which translated to the mat’s centroid gives ex = − 7.56 = 0.69 m
2

For the eccentricity about the x’-axis, take ∑M x' =0


(11,000)y’ = (7 m)(1500+1500+1200) + (14 m)(1500+1500+1200) +
(21)(400+500+450)
21.5
y’ = 10.60 m, translated to the mat’s centroid gives ey = − 10.60 = −0.15 m
2

Step 2.

Find the soil reaction pressures.

Let us factor the applied loads: 1.7(Service Loads) = 1.7(11,000 kN) = 18,700 kN
The two eccentricities ex and ey create moments about the centroid. The soil reaction is no longer
uniform, and varies linearly between the columns. These moments are:
Mx = R ey = (18,700 kN)(0.15 m) = 2,805 kN-m
My = R ex = (18,700 kN)(0.69 m) = 12,903 kN-m
The soil reaction pressure at any point under the mat is found from the relation:
R M yx Mx y 18, 700 12, 903( x) 2,805 ( y )
q= ± ± = ± ±
A Iy Ix 16.5 × 21.5 8, 048 13, 665
403
Therefore, q = 52.7 ± 1.6 x ± 0.21 y kN / m 2

Step 3.

Using the equation for q, prepare a table of its value at points A through J.

POINT R/A x 1.6x y 0.21y q


2
(kN/m ) (m) (m) (m) (m) (kN/m2)

A 52.7 -8 -12.8 10.5 2.21 42.11


B 52.7 0 0 10.5 2.21 54.91
C 52.7 8 12.8 10.5 2.21 67.71
D 52.7 8 -8.16 -10.5 -2.21 63.29
E 52.7 0 0 -10.5 -2.21 50.50
F 52.7 -8 -12.8 -10.5 -2.21 37.7
G 52.7 -4 -6.4 10.5 2.21 48.51
H 52.7 -4 -6.4 -10.5 -2.21 44.10
I 52.7 4 6.4 10.5 2.21 61.31
J 52.7 4 6.4 -10.5 -2.21 56.9

Step 4.

Determine the effective depth d and the thickness T of the mat.

a) Check a critical edge column (for example, one of the 1.5 MN at the left edge):
U = factored column load = 1.2(1.0) + 1.6(0.5) = 2 MN or 1.7(1.5) =2.55 MN
b0 = critical perimeter = 2(0.5 m + d/2) + (0.5 + d) = (1 + d) + 0.5 + d = 1.5 + 2d
Using φVC ≥ Vu (from ACI 318-05) and fc’ = 20.7 MN/m2 (3 ksi),
φ (0.34) f '  ( b d ) = (0.85)(0.34) 20.7 × (1.5 + 2d )(d ) ≥ U = 2 MN
 c  o

d2 + 0.75 d – 0.76 ≥ 0

− b ± b 2 − 4ac −0.75 ± (0.75)2 − 4(1)(−0.76)


d= = = 0.57 m
2a 2(1)
b) Check the largest corner column (the 0.45 MN at top right corner):
d = 0.36 m (This does not control).
c) Check the most critical internal column (the 1.5 MN):

404
b0 = 4(0.5 + d) = 2 + 4d
0.85(0.34)( 20.7 )(2 + 4d )(d ) = 2 MN
5.26d 2 + 2.63d − 2 = 0
d = 0.415m
∴ use d = 23 inches or 585 mm
and T = 23+3+1 = 27 in or 686 cm

Step 5.

Find the average soil reaction for each strip:

Strip AGHF (W = 4.25 m)

q A + qG 42.11 + 48.5 kN
q1 = = = 45.31 2
2 2 m

qH + qF 44.1 + 37.7 kN
q2 = = = 41 2
2 2 m

Strip GIJH (W = 8 m)

48.51 + 54.91 + 61.31 kN


q1 = = 54.91 2
3 m

56.9 + 50.5 + 44.1 kN


q2 = = 50.5 2
3 m

Strip ICDJ (W = 4.25 m)

61.31 + 67.71 kN
q1 = = 64.51 2
2 m

63.29 + 56.9 kN
q2 = = 60.1 2
2 m

Soil reaction AGHF = ½ (45.31+41)(4.25)(21.5) = 3943 kN

Soil reaction GIJH = ½ (54.91+50.5)(8)(21.5) = 9065 kN

Soil reaction ICDH = ½ (64.51+60.1)(4.25)(21.5) = 5693 kN

∑ Fy = 3943 + 9065 + 56903 = 18,700 kN


Strip GIJH.

405
Q1 = 1.7(500) = 850 kN

Q2 = 1.7(1500) = 2550 kN

Q3 = 1.7(1500) = 2550 kN

Q4 = 1.7(500) = 850 kN

54.91x8 m = 43.93 kN/m 50.50x8 m = 404 kN/m


427.52 kN/m 415.76 kN/m
433.4 kN/m
421.64 kN/m 409.9 kN/m

Step 6.

Find the maximum positive moments for each span at midpoints a, b & c.

( 439.3 + 433.4 ) (3.52 ) + M


∑ Ma = 0 ∑ M A = (850 × 3.5 m) − 2 2
A =0

∴ M A = −302.36 kN-m/m

(439.3 + 415.76) 10.5 (


2
)
∑M b =0 ∑ M b = (850 ×10.5m) + ( 2550 × 3.5m ) − 2 2
+ Mb = 0

∴ M b = 5718 kN-m/m

Step 7.

Calculate maximum negative moment at d, column B, see page 449:

(439.3 + 427.52) 7 ( )
2

∑ M d = (850 × 7m) − 2 2
+ Md = 0

∴ M d = 4668.5 kN-m/m

Step 8.

Design the strip for flexure:

406
d = 23 inches = 685 mm , f `c = 3 ksi (20.7 MN / m 2 ), f y = 60 ksi (413.7 MN / m 2 )

As f y Mu
a= AS =
0.85 f `c b φ ⋅ f y (d − a 2)

say Mu = 5718 kN-m/ 8 m = 715 kN-m/m = 161 k-ft/ft

say a = 3.3 in , As required = 1.68 in2/ft

Try #9 @ 6” o.c. As = (1)(12/6) = 2 in2 > 1.68 in2

∴ Use #9 @ 6” bottom, As required = 1.68 in2

ρmin = 200/60000 = 0.0033 As-min= 0.0033(23)(12) = 0.91 in2 < 1.68 in2 Good

As-min Bottom → 1.68 in2

Top → 1.3 in2

Negative moment:

Mu = 4668.5 / 8 m = 584 KN-m/m = 131.6 k-ft/ft

Say a = 2.54 in , As required = 1.3 in2/ft

Try #9 @ 9” o.c. As = (1)(12/9) = 1.33 in2 > 1.30 in2 Good

∴ Use #9 @ 9” top.

Use top and bottom reinforcing throughout the mat in the y-direction.

Step 9. Sketch the mat’s cross-sections and reinforcement.

407
Chapter 21
Deep Foundations - Single Piles
Symbols for Single Piles of Deep Foundations

408
*Single-Pile–01: Pile capacity in a cohesive soil.
(Revision: Oct.-08)
A concrete pile 20 m long with a cross section of 381 mm x 381 mm is fully embedded in a
saturated clay stratum. The clay has γsat = 18.5 kN/m3, φ=0º and cu = 70 kN/m2. The water table lies
below the tip of the pile. Determine the allowable capacity of the pile for a FS = 3 using the α-
method.
Solution:
The ultimate capacity of the pile Qult is given by the simple formula,
Qult = Qpoint + Qshaft = Ap q p + ( cu ) ( perimeter ) L = Ap ( cu ) N c + α ( cu ) ( perimeter ) L

Notice that the value of the cohesion is reduced by the "α " factor found in the graph below,
∴ Qult = ( 0.38m ) (70 kN / m 2 )(10.97) + ( 0.75 ) (70 kN / m 2 )4(0.38m)(20m) = 1,890 kN
2

The allowable capacity is,


Qult 1,890kN
Qall = = = 630 kN
3 3

409
*Single-Pile–02: Pile capacity in granular soils.
(Revision: Oct.-8)
Find the ultimate load of a slightly tapered precast pre-stressed concrete pile, given
Pile length L = 50 ft
Width of pile b = 16 in
Bearing capacity factor Nq* = 55
Unit weight of sand γ = 110 lb/ft3
Effective sand’s friction angle φ = 30°
Effective earth coefficient K = 1.3
Soil-pile friction angle δ = 0.8φ

Solution:
The ultimate load Qult is found from the formula,
Qult = Q p + Qshaft = Ap q N q * +

Qult = b 2 ( D f h )( N q *) = ( ft )2 (0.110kcf ) ( 50 ft )  (55) = 538 kips


16
12
b) Ultimate frictional resistance
Unit skin friction at depth z =0, fs(0) = 0 ksf
Unit skin friction at depth z = L’
Where L’ = 15D = 15(16 in)/12 = 20 ft
fs(L’) = Kσ’o tanδ = (1.3)(2.2 ksf)(tan24°) = 1.27 ksf
Where σ’o = γL’ = (110 lb/ft3)(20 ft)/1000 = 2.2 ksf
Frictional resistance from z = 0 to z = L’ = 20 ft
Qs(0-L’) = pL’fs(avg) = (5.33 ft)(20 ft)(0.635 ksf) = 67.7 kips
Where p = 4b = 4(16 in)/12 = 5.33 ft
fs(avg) = (fs(L’) - fs(0))/2 = (1.27 ksf – 0 ksf)/2 = 0.635 ksf
Frictional resistance from z = L’ = 20 ft to z = L = 50 ft
Qs(L’-L) = p(L-L’)fs(L’) = (5.33 ft)(50 ft -20 ft)(1.27 ksf) = 203 kips
Total frictional resistance
Qs = Qs(0-L’) + Qs(L’-L) = 67.7 kips + 203 kips = 271 kips
The total pile capacity = 56 + 68 + 203 = 327 kips.

410
*Single-Pile–02b: The friction of a concrete pile in granular soils.
(Revised Nov-09)
Find the length of a 14” square pile required to carry a 50 ton load embedded in a firm sand (N =
25).
Given: Design load Qdesign = 100 kips
Side length of the square pile D = 1.167 ft
Atmospheric pressure Pa = 2 ksi
Avg. corrected SPT at 10D above and 4D below pile point (N1)60 = 25
Average standard penetration resistance ( N 1)60 = 25
Factor of safety FS = 3

Solution:
Ultimate load Qult = Qdesign FS = (100)(3) = 300 kips
Length of the pile is found from Meyerhof’s method,
Qult = 4Pa(N1)60Ap + 0.02Pa( N 1)60As
300 = 4(2)(25)(1.36) + 0.02(2)(25)(4.67)L = 272 + 4.67L
L = (300 – 272) / 4.67 = 6 feet
where Ap = D2 = (1.167)2 = 1.36 ft2 and As = 4DL = (4)(1.167)L = 4.67L

411
*Single-Pile–03: The capacity of a driven concrete pile.
(Revision: Sept.-08)
Find the ultimate and the design (allowable) capacities of a prestressed precast concrete pile
(PSPC) driven with a 72 blows/feet criterion. The piles are 30 feet long and have 14” square cross
sections. The hammer available is a LinkBelt 520 with a ram weight of 5,070 lbs and a drop height
of 5.2 feet. The hammer efficiency is 0.9, and the contractor intends to use scrap plywood in place
of cushion blocks (hence, n = 0.4).
Given: Concrete pile length L = 30 ft
Width of pile b = 14 inches
Weight of the ram Wram = 5,070 lbs
Height of the fall of the ram h = 5.2 feet
Hammer efficiency e = 0.9
Penetration of pile per hammer blow s = 0.17 inches / blow
Coefficient of restitution n = 0.4
Elastic compression of the pile cap sc = 0.37 in
Elastic compression of the pile itself sp = 0.18 in
Elastic compression of the soil ss = 0.10 in
Factor of safety FS = 3

Solution:
The ultimate load capacity Qult of a driven pile is found through Hilley's formula,

Qult =
12Wram h eeff (W
ram + n 2W pile )
s + 0.5 ( sc + s p + s ) (W
s ram + W pile )
The weight of the pile W pile is,
2
 14in 
W pile = b Lγ concrete =   ( 30 ft )( 0.150kcf ) = 6.13kips
2

 12in / ft 
12 ( 5.07 kips )( 5.2 ft ) ( 0.9 ) ( 5.07 k ) + ( 0.4 )2 ( 6.13k ) 
∴ Qult =   = 311 kips
( 0.17in ) + 0.5 ( 0.37in ) + ( 0.18in ) + ( 0.10in )  ( 5.07k ) + ( 6.13k ) 
The service load is commonly a factor of 2 to 3 from the ultimate pile capacity:
Qult 311 kips
∴ Qservice = = = 104 kip (52tons)
3 3

412
*Single-Pile–04: The driving criterion for a concrete pile.
(Revised Nov-09)
A highway bridge will be founded on 14” x 14” square pre-stressed pre-cast (PSPC) concrete piles
at mid-stream. Establish the driving criterion for a 50 ton design capacity for these end bearing
piles using MKT S-14 pile driving hammer with a plywood cushion. Given:
Design load Qdesign = Qall = 100 kips
Concrete pile length L = 50 ft
Width of pile b = 14 in
Factor of safety FS = 3
Weight of the ram Wr = 14 kips
Hammer energy HE = 37.5 ft-kips
Height of fall of the ram h = HE / Wram = 37.5 /14 = 2.67 ft
Hammer efficiency e = 0.9
Coefficient of restitution for plywood n = 0.4
Elastic compression of the pile cap sc = 0.37 in
Elastic compression of the pile itself sp = 0.006L = 0.006(50) = 0.30 in
Elastic compression of the soil ss = 0.10 in

Solution:
Ultimate load: Qult = Qall FS = (100)(3) = 300 kips
Weight of the pile Wp = b2Lwc = (14 in/12 in/ft)2(50 ft)(150 lb/ft3) = 10.21 kips
Hiley’s dynamic pile driving formula is,
12 Wr he W + n 2W p
Qult = ×X r
s + 1/2(S c + S p + S s ) Wr + W p

12 (14 )(2.67 )(0.9 ) 14 + 0.4 2 (10.21)


300 = ×X
s + 1/2(0.37 + 0.30 + 0.10 ) 14 + 10.21

300(s + 0.385) = (403.7)(0.646)

s = 0.484 in / blow ⇒ 0.484 / 12 = 0.0403 ft / blow


The pile driving criterion is, 1 / s = 1 / 0.0403 = 24.79 ⇒ 25 blows / foot

413
*Single-Pile–05: The Meyerhof and Vesic methods for pile capacity.
(Revised Nov-09)
For the pile described below, find: a) the ultimate point load using the Meyerhof method, b) the
ultimate point load using the Vesic method, c) an approximate ultimate point load method, d) the
ultimate shaft friction resistance, and e) the total design load.

Given: Pile length in sand L = 25 m


Diameter of pile D = 0.305 m
Atmospheric pressure Pa = 100 kN/m2
Bearing capacity factor Nq* = 150
Bearing capacity factor Nσ* = 71
Reduced rigidity of the soil Irr = 50
Dry unit weight of sand from 0 to 5 m γdry = 16 kN/m3
Effective soil friction angle from 0 to 5 m φ = 32°
Cohesion of soil supporting pile tip at 5 m c’ = 0
Water table WT = 5 m
Unit weight of water γw = 9.81 kN/m3
Unit weight of saturated sand from 5 to 10 m γsat = 18.4 kN/m3
Effective soil friction angle from 5 to 10 m φ = 32°
Cohesion of soil supporting pile tip at 10 m c’ = 0
Unit weight of saturated sand from 10 to 25 m γsat = 19.9 kN/m3
Effective soil friction angle from 10 to 25 m φ = 37°
Cohesion of soil supporting pile tip at 25 m c’ = 0
Effective earth coefficient K = 1.4
Soil-pile friction angle δ = 0.7φ
Factor of safety FS = 4

Solution:
a) Ultimate point load using Meyerhof’s method
Qp1 = Apqp = (0.073 m2)(5,652 kN/m2) = 413 kN
Where Ap = πD2/4 = 3.14(0.305 m)2/4 = 0.073 m2
N q∗ = 150 for φ =37 and pA = 100 kN/m 2 (atmospheric pressure)

qp = 0.5paNq*tanφ’ = 0.5(100)(150)tan37° = 5,652 kN/m2

414
b) Ultimate point load using Vesic’s method
Qp2 = Apqp = (0.073 m2)(11,644 kN/m2) = 850 kN
Where Ap = πD2/4 = 3.14(0.305 m)2/4 = 0.073 m2 Nσ∗ = 71 for I R = 50
qp = σo’Nσ* = 164(71) 11,644 kN/m2
and σo’ = [(1+2Ko)/3]q’ = {[1+2(0.398)]/3}(274) = 164 kN/m2
and Ko = 1 - sinφ = 1 - sin37° = 0.398
q’ = (5-0) γdry + (10-5)(γsat-γw) + (25-10)(γsat-γw)
= (5)(16) + (5)(18.4-9.81) + (15)(19.9-9.81) = 274 kN/m2
c) Approximate ultimate point load: (Average between the Meyerhof and Vesic methods)
Qp = (Qp1 + Qp2)/2 = (413 + 850)/2 = 631 kN
d) Ultimate frictional resistance
Unit skin friction at depth z =0, fs(0) = 0 kN/m2
Unit skin friction at depth z = L’
Where L’ = 15D = 15(0.305) = 4.575 m
Use L’ = 5 m
fs(L’) = Kσ’o tanδ = (1.4)(80 kN/m2)(tan22.4°) = 46.16 kN/m2
Where σ’o = γdryL’ = (16 kN/m3)(5 m) = 80 kN/m2
δ = 0.7φ = 0.7(32°) = 22.4°
Frictional resistance from z = 0 to z = L’ = 5 m
Qs(0-L’) = pL’fs(avg) = (0.958 m)(5 m)(23.08 kN/m2) = 110.55 kN
Where p = πD = 3.14(0.305 m) = 0.958 m
fs(avg) = (fs(L’) - fs(0))/2 = (46.16 – 0)/2 = 23.08 kN/m2
Frictional resistance from z = L’ = 5 m to z = 10 m
Qs(L’-L) = p(10-L’)fs(L’) = (0.958 m)(10 m-5 m)(46.16 kN/m2) = 221.1 kN
Frictional resistance from z = 10 m to z = L = 25 m
Qs(L’-L) = p(L-10)fs(L’) = (0.958 m)(25 m-10 m)(46.16 kN/m2) = 663.32 kN
Total frictional resistance
Qs = Qs(0-L’) + Qs(L’-10) + Qs(10-L) = 110.55 kN + 221.1 kN + 663.32 kN
Qs = 995 kN
e) Total design load
Qall = (Qp + Qs)/FS = (631 kN + 995 kN)/4 = 406 kN

415
*Single-Pile–06: Tension or pull-out capacity in granular soils.
(Revised Nov-09)
Find the allowable pullout capacity for a 10.4 m long by 406 mm square concrete pile embedded in
a sand stratum.
Given: Pile length in sand L = 10.4 m
Side of square pile D = 0.406 m
Unit weight of sand γ = 15.8 kN/m3
Effective soil friction angle φ = 30°
Effective earth coefficient K = 1.4
Soil-pile friction angle δ = 0.7φ
Factor of safety FS = 4

Solution:
Unit skin friction at depth z =0, fs(0) = 0 kN/m2
Unit skin friction at depth z = L’ where L’ = 8D = 8(0.406) = 3.25 m
fs(L’) = Kσ’o tanδ = (1.4)(51.35 kN/m2)(tan 21°) = 27.59 kN/m2
Where σ’o = γL’ = (15.8 kN/m3)(3.25 m) = 51.35 kN/m2
δ = 0.7φ = 0.7(30°) = 21°
Frictional resistance from z = 0 to z = L’ = 3.25 m
Qs(0-L’) = pL’fs(avg) = (1.624 m)(3.25 m)(13.8 kN/m2) = 72.84 kN
Where p = 4D = 4(0.406 m) = 1.624 m
fs(avg) = (fs(L’) - fs(0))/2 = (27.59 – 0)/2 = 13.8 kN/m2
Frictional resistance from z = L’ = 3.25 m to z = 10.4 m
Qs(L’-L) = p(10.4-L’)fs(L’) = (1.624 m)(10.4 m-3.25 m)(27.59 kN/m2) = 320.36 kN
Total pullout capacity Qs = Qs(0-L’) + Qs(L’-L) = 72.84 kN + 320.36 kN = 393.2 kN
Allowable pullout capacity Qall = Qs/FS = 393.2 kN / 4 = 98.3 kN

416
*Single-Pile–07: Tension (uplift) capacity of a pile in granular soils.
(Revised Nov-09)

A square concrete pile (406 mm x 406 mm) is driven into a sand stratum that has a γ = 15.8
kN/m3, a relative density of 70% and a φ = 30°30°. If the pile has reached refusal at a depth of 10.4
feet long, estimate its allowable pullout capacity T using a FS = 4.

Solution:
L
The criterion that   = 8, means that Lcr = 8 (0.406 m) = 3.25 m
 D  cr
Since L > Lcr the ultimate pullout capacity Tult is provided by the formula,
1
Tult =   (perimeter)(γ)(Lcr) 2 (Kw ) tanδ + (perimeter)(γ)(Lcr)(Kw) tanδ (L - Lcr)
2
δ 
Dr =   = 0.70, in other words δ = 0.70(φ) = 0.70(30°) = 21° and assume Kw =1.4
φ 
Therefore,
1
Tult =   (0.406 m x 4)(15.8 kN/m 3 )(3.25 m)2 (1.4) tan21˚
2
+ (0.406 m x 4)(15.8 kN/m 3 )(3.25 m)(1.4) tan 21˚ (10.4 m – 3.25 m)
= 73 + 320 = 393 kN
Tult  393 
The allowable pullout capacity is Tall = =  = 98 kN
FS  4 

417
***Single-Pile–08: The settlement of a drilled shaft in cohesive soils.
(Revised Nov-09)
To illustrate the calculation of the settlement of a single pile, consider the case of a large bored pile
in clay. The pile has been tested by Whitaker and Cooke (1966) and is denoted as Pile F in the
graphs shown here.
The pile details are as follows:
L = 39.9 ft (12.2 m) [shaft length = 36.5 ft (11.1 m)]
D = 2.0 ft (0.61 m)
db = 4.0 ft (1.2m)
Ep = 3.0 X 106 lb/sq in. (19.3 X 104 tons/sq ft, 20.67 x 106 kN/m2)
The soil details are as follows:
Along shaft, c u= 1.2 tons/sq ft (129 kN/m2)
At base, c u = 1.4 tons/sq ft (150 kN/m2)
Average Es along shaft = 10,500 lb/sq in (675 tons/sq ft, 72, 400 kN/m2)
vs = 0.5 (assumed undrained conditions)

Solution:
From the above data,
L/d = 19.5, db/d = 2.0 K = 3 X 106/1.05 X 104 = 285

Determination of I and β
From Fig. 21.1, Io = 0.085 (for L/d = 19.5 and db/d = 2.0)
From Fig. 21.2, RK = 1.35 (for K = 285)
From Fig. 21.3, Rv = 1.0
Treating the pile as a floating pile in a deep soil layer, Rh = 1.0
PI
From ρ = , I = 0.085 X 1.35 X 1.0 X 1.0 = 0.115
Es d
From Fig. 21.4, βo = 0.215
where I = IoRKRhRv
ρ = settlement of pile head
P = applied axial load
Io = settlement-influence factor for incomp pile in semi-infinite mass, for vs = 0.5.
RK = correction factor for pile compressibility
From Fig. 21.5, CK = 0.76
From Fig. 21.6, Cv = 1.0
β = 0.215 X 0.76 X 1 = 0.164

418
Fig. 21.2-Compressibility correction factor for
settlement, RK.
Fig. 21.1 - Settlement-influence factor Io.

Fig. 21.3-Poisson’s ratio correction factor for settlement, Rv.

419
Fig. 21.5 - Compressibility correction factor for
Fig. 21.4 - Proportion of base load, βo. base load, CK.

Fig. 21.6-Poisson’s ratio correction factor for settlement, Rv.

420
Determination of Ultimate Shaft and Base Resistance
Assuming ca/cu = 0.33,
Psu = π X 2.0 X 36.5 X 1.2 X 0.33 = 92 tons (920 kN)
Pbu = 9.0 X 1.4 X 12.6 = 158 tons (1580 kN)

Determination of Overall Load-Settlement Curve


P SU 92
The total load at ultimate shaft yield is PYl = = = 110 tons (1100 kN)
1 − β 0.836

 I 
(PYl ) =
0.115 * 110 * 12
The settlement at ultimate yield shaft is ρ Y 1=  = 0.112 in.
 E sd  2.0 * 675
The settlement at ultimate failure of pile [Pu = 92 + 158 = 250 tons (2500 kN)] is
ρu = Psu + Pbu ρu = 1.088 in. (27.6 mm)

 I  Pbu   P β  L 0.115 158


ρu =    +  Pbu − su  = * *
 β  
 E sd (1 − β ) Ap E p 2.0 * 675 0.165
 0.165  39.5 * 12
12 +  158 − 92 * * = 1.088
 0.835  3.14 * 19.3 * 10

The load-settlement curve is thus drawn


as two straight lines, the first joining the
origin to the point P = 110 tons, ρ =
0.112 in., and the second joining the latter
point to the point P = 250 tons, ρ = 1.088
in. This computed curve is compared
with the measured curve in Fig. 21.7

Fig. 21.7 – Comparisons between observed and predicted


load-settlement curves. Tests of Whitaker and Cooke (1966)
421
***Single-Pile–09: The performance of steel pipe piles.
(Revised Nov-09)
Piles are being considered for a power plant, specifically a closed-ended steel pipe pile, 24.4 m long
and 0.32 m in diameter, driven into a fine sand grading to coarse and having an average standard
penetration value, N, of about 16. The water table was 3.4 m below the surface. On the basis of
the available data, the following values were adopted:
Bulk unit weight above water table = 17.3 kN/m3.
Submerged unit weight below water table = 7.8 kN/m3.
Angle of internal friction angle prior to installation:
φ′1 = 25° (0 – 2.4 m)
φ′1 = 32° (2.4 – 18.3 m)
φ′1 = 30° (18.3 –20.8 m)
φ′1 = 33° (> 20.8 m)

Solution:
(Reference to Poulos and Davis).
3
First, find the ultimate skin resistance (given by φ = φ′1 +10), as follows:
4
φ = 28.75° (0 – 2.4 m)
φ = 34° (2.4m – 18.4 m)
φ = 32.5° (18.3 – 20.8 m)
φ = 34.75° (>20.8 m)

The values of Ks tan φ′a are 1.00 (0 –2.4 m), 1.30 (2.4 – 18.3 m), 1.18 (18.3 – 20.8 m), 1.35 (20.8). If it
is assumed that the critical depth is less than 2.4 m below the surface, then for φ = 28. 75°, zc/d = 5.0;
that is, zc = 5.0 x 0.32 = 1.56 m. Thus, the assumption is justified. At the critical depth, the effective
overburden stress is :

σ′vc = 1.56 x 17.3 = 26.99 kN/m2

Because the pile has uniform diameter, Fw = 1. For the ultimate base resistance, the value of φ given by
φ ′ + 40
φ= is 36.5°. The value of Nq is 98.
2

L
The formula for the ultimate load capacity of a single pile Pu = ∫ Fw Cσ ν′ K S tan φ a′ + Abσ vb
′ Nq −W
0

422
 0 + 26.99   
  × 1.56 + 26.99 × (2.4 − 1.56 ) 
 2   
 0.32 2
Pu = π × 0.32× 1.00 + 26.99 × (18.3 − 2.4 ) × 1.30 + 26.99  + 26.99 × π × × 98
× (20.8 − 18.3) × 1.18 + 26.99 × (24.4 − 20.8) 4
 
× 1.35 

Pu = 816 + 213 = 1029 kN (115.6 tons)

This compares well with the measured value of 1,112 kN (125 tons) from a load test.
The second type of pile (B) considered was a Raymond Standard Pile, 10 m long, with a head
diameter of 0.55 m and a tip diameter of 0.20 m. The pile taper w= 1°, so Fw = 3.35 (0-2.4 m), and
Fω = 4.1(2.4-18.3 m). Assuming again that the critical depth is above 2.4 m, zc / d = 5.0 as before,
and taking an average value of d of 0.51, zc = 2.55 m, that is, greater than 2.4 m. However, the
difference is negligible and hence zc will be taken as 2.55 m. At this level,
σ ν′c = 2.55 × 17.3 = 44.12 kN/m2
At 2.4 m,
σ ν′ = 2.4 × 17.3 = 41.52 kN/m2

φ ′ + 40
Since the pile tip is founded in the second stratum, φ from is 36° and the corresponding value of
2
Nq is 88. Substituting into Pu and using, for simplicity, the mean diameter of the pile in the upper 2.4 m
and the lower 7.6 m,

L
Pu = ∫ Fw Cσ ν′ K S tan φ a′ + Abσ vb
′ Nq −W
0

 0 + 41.52   (41.52 + 44.12 ) 


Pu =   × π × 0.51 × 3.35 × 1.00 × 2.4 +  × 0.15 + 44.12 × (10 − 2.55)
 2   2 
2
0.20
× π × 0.33 × 4.1 × 1.30 + 44.12 × π × × 88
4

Pu = 2243 kN (252.2 tons)

The measured ultimate load for this pile was 2,400 kN (270 tons).

423
Fig 21.8 – Values of zc/d and Ks tan φ’a for piles in sand.

424
Figure 21.9 - Pile taper factor Fω (after Nordlund, 1963)

Figure 21.10 - Relationship between Nq and Ø (after Berezantzev et al., 1961)

425
Chapter 22
Deep Foundations - Pile Groups and Caps
Symbols for Pile Groups and Caps of Deep Foundations

426
**Pile-caps–01: Design a pile cap for a 9-pile cluster.
(Revision: Oct-08)
Design a pile cap footing to support an 18” square column subjected to a live load reaction of 180
kips and a dead load reaction of 160 kips at service loads. The testing laboratory recommends an
ultimate pile load of 70 kips per pile, and a service pile load of 42 kips per pile. The vertical steel in
the column consists of 12 No.7 bars. Use ƒ‫׳‬c = 3000 psi, ƒy = 40,000 psi, and 12” diameter piles.

Solution.

Since the footing weight will be about 3 kips/pile, the net service load per pile is 42.0-3.0 = 39.0 kips/
pile. The number of piles required in N=W/P = 340/39 = 8.7, or 9 piles. Use a pile pattern as shown in
Fig. 22.1. The net ultimate load is used to design the footing; thus Wu = (1.4) (160) + (1.7) (180) = 530
kips, and the load per pile is Pu = 530/9 = 58.9, say 59.0 kips/pile, which is less than the maximum
ultimate load, 70 kips/ pile. Punching shear around a single pile often governs the footing depth
determination, except in cases in which the loads are small. In this case, it will be shown that beam
shear governs. Referring to Fig. 22.2, we calculate the punching shear stress. After several trials,
assume d = 19.5″. The shear perimeter is bo = π(12 + d) = 99.0″. The permissible shear force around
the pile will be,

Vc = 4√f‫׳‬c bod = 4√3000 (99) (19.5) / 1000 = 423 kips

Since the actual shear force is the nominal pile reaction, Pn = Pu/φ = 59.0/0.85 = 69.4 kips < 423 kips,
the pile will not punch through the pile cap (footing).

Fig 22.2
Perimeter shear (punching shear) must now be checked around
the column in a similar manner. In this case, all of the nominal
pile reactions outside of the critical section plus any partial
reactions outside of the critical section will contribute to
punching shear for the column. Refer to Fig. 22.3. Assuming
Fig. 22.1 No. 6 bars will be used, clearance above the pile butts will be 3″
and embedment of the piles will be 6″. The total dept required will be 28.75″. For practical reasons use
29″; this furnishes an effective depth d = 19.625″. Thus c = a + b = 18.0 + 19.625 = 37.625″ and bo =
4(37.625) = 150.0″. Hence, Vou = 472 kips on 8 piles outside of the critical section as shown on Fig.
22.3. The permissible punching shear force (βc = 18/18 < 2) is given by (6.12) as:

Vc = 4 √3000 (150) (19.625) / 1000 = 644.9 kips

427
The force to be resisted is Vn = Vou/φ = 472/0.85 = 555.3 kips; therefore the pile cap (footing) is
satisfactory for punching shear. Beam shear must now be checked. Refer to Fig. 22.4. Three piles exist
beyond the critical section, so Vu = (3) (59.0) = 177.0 kips. Since b = B= 8′-6″ = 102″, the permissible
beam shear (one-way shear) force on the critical section is

Vc = 2√ƒ′c bd = 2 √3000 (102) (19.625) / 1000 = 219.3 kips

Fig. 22.3 Fig. 22.4


The force to be resisted is the nominal shear force, Vn = Vu / φ = 177/ 0.85 = 208.2 kips. Hence the
footing is satisfactory for beam shear. The bending moment about the face of the column must now be
investigated. Refer to Fig. 22.4

Mu = (177) (27/12) = 398.3 ft-kips

φ Ru = Mu / bd2 = 398.3 x 12,000 / (102)(19.625)2 = 121.67 psi

When ƒ ′c = 3000 psi and ƒy = 40,000 psi, the steel ration required is less than the minimum steel ration,
ρmin = 200 / ƒy = 0.005. Further, if the steel ration required is increased by 1/3, it will still be less than
ρmin. It would appear that 4/3 times the required steel ration would satisfy the 1983 ACI Code.
However, the Code does not permit un-reinforced (plain concrete) pile caps. Since any section having
less than minimum reinforcement is usually considered to be un-reinforced, the minimum are of steel
will be provided. Thus,

As = (200/ƒy) bd = (200/ 40,000) (102) (19.925) = 10.0in.2

Use seventeen No. 7 bars (As = 10.2 in.2).

The 1983 ACI Code is not explicit concerning minimum steel for
footings. Hence, some structural engineers use 0.002bh for
minimum steel area if ƒy 40,000 psi and 0.0018bh if ƒy = 60,000
psi. This corresponds to temperature and shrinkage reinforcement
requirements. The assumed footing weight must finally be
checked. The total weight is

WF = (8.5) (8.5) (29) (12.5) / 1000 = 26.2 kips

And the weight per pile is 26.2/ 9 = 2.91 kips / pile. The assumed
weight of 3.0 kips / pile is most satisfactory. The final details are
shown in fig. 22.5. Fig. 22.5

428
**Pile cap-02: Pile group efficiency and lateral load capacity.
(Revised Nov-09)

A pile group is constructed in a sand stratum (plan shown below) with piles of φ =18 in and a
center-to-center spacing of 36 inches.
Find the pile group efficiency η using the Converse-Labarre formula;
Piles are acting in friction only; select a hammer to drive them;
Determine the allowable load-bearing capacity of the pile group (for a FS = 4);
Determine the allowable horizontal load on the pile cap, and the collective bending moment
capacity.

Solution:
a) The Converse-Labarre group efficiency η is,

 (n − 1)m + ( m − 1)n  d
η = 1− ξ   where ξ = arctan ( in degrees )
 90mn  s
d  18" 
∴ ξ = arctan = arctan   = 26.6°
s  36" 
 (3 − 1) ( 3) + (3 − 1) ( 3) 
η = 1 − ( 26.6 )   = 1 − ( 26.6 )( 0.015 ) = 0.61 or 61%
 90 ( 3)( 3) 

b) If pre-cast pre-stressed concrete piles are used, assume a pile/hammer ratio,

Wp 2 1 1 π  k 
= ∴ Wr = W p = [ (1.5 ft ) ( 20 ft ) 1.5 3  = 2.9 kips
2

Wr 1 2 2 4  ft 

429
Choose a hammer from a list of available hammers.

Vulcan INI00
McKiernan-Terry DE30
LinkBelt 312
Foster K13
Berminghammer B225
Mitsubishi M14
c) QW = QP + QS = QS (for friction piles only)

20
QW = Q S = π D (C u + κγ tan φ ∫ Hdh ) use κ = 1
0

H2 20 2
Q w = π D (1)(0.125) tan φ = π D (0.125) tan 38 ° = 92 kips
2 2

Q u group Q group
but η = = 0 . 61 = u
Q u SINGLE × NO . PILES 92 kips × 9

Q u group 505
∴ Q u group = 505 kips but Q all = =
FS 4

∴ Q all = 126 kips

d) For granular soils

38 °
H u = 1 .15γ soil L2 DK P = 1 .15 ( 0 .125 )( 20 ) 2 (1 .5 ) tan 2 ( 45 ° + ) =362.5 kips
2

H all =90 kips

2 2
Mu = H u L = (362 .5 )( 20 ) = 4833 k-ft
3 3

M all = 1,210 k-ft

430
***Pile caps–03: Settlement of the combined pile-cap and pile cluster.
(Revised Nov-09)
A square group of 25 piles extends between depths of 1 m and 13 m in a deposit of stiff clay 25 m
thick over-laying rock. The piles are 0.6 m in diameter and are spaced at 2 m on centers in the
group. The un-drained shear strength of the clay at pile base level is 170 kN/m2 and the average
value of un-drained strength over the depth of the piles is 105 kN/m2. The adhesion coefficient α is
0.45, Eu is 65 MN/m2 and mv is 0.07 m2/MN. The pore pressure coefficient A is 0.24. If the total load
on the pile group is 12,000 kN, determine the load factor and the total settlement.

Solution:

1. At the base level, cu = 170 kN/m2. Therefore, qf = 9 (cu) = 9 · 170 = 1530 kN/m2

2. Over the depth of the piles, cu = 105 kN/m2.

Therefore, fs = α (cu) = 0.45 · 105 = 47 kN/m2

3. For a single pile the ultimate load is given by

Qf = (Ab) qf + (As) fs = [(π/4) x 0.62 x 1530] + (π x 0.6 x12 x 47) = 432 + 1063 = 1495 kN

4. The ultimate load on the group assuming single pile failure and a group efficiency of 100% , →
QGROUP = 25 x 1495 = 37,375 kN

431
5. The width of the group is 8.6 m. Therefore the ultimate load on the group assuming a block failure,
and taking full un-drained strength on the perimeter:

Qult = (8.62 x 1530) + (4 x 8.6 x 12 x 105) = 113 159 + 43 344 = 156,500 kN

Hence the load factor is 37,375 / 12,000 = 3.1. Even if the re-molded strength were taken on the
perimeter there would be no likelihood of block failure. However, settlement is likely to be the limiting
criterion. Referring to the figure below, the equivalent raft is located 8 m (2/3 x 12 m) below the top of
the piles. The width of the equivalent raft is 12.6 m. The load on the equivalent raft (12 000 kN) is
spread at a slope of 1:2 to the underlying clay

Equivalent raft concept

The pressure on the equivalent raft is: q = 12 000 / 12.62 = 76 kN/m2

The immediate settlement is determined using the graphs for the coefficients for vertical displacements,
shown below. Now,

H/B = 16/12.6 = 1.3

D/B = 9/12.6 = 0.7

L/B = 1

432
Coefficients for vertical displacement. (Reproduced from N. Janbu,
L. Bjerrum and B. Kjaernsli (1956) Norwegian Geotechnical Institute
Publication No. 16, by permission.)

433
Therefore µ1 = 0.50 and µ0 = 0.80; thus

∆i = (µ0 µ1 ) (qB/Eµ) = 0.80 x 0.50 x [(76 x 12.6)/65] = 6 mm

To calculate the consolidation settlement, the clay below the equivalent raft will be divided into four
sublayers each of thickness H = 4m. The pressure increment (∆σ) at the center of each sublayer is equal
to the load of 12,000 kN divided by the spread area and is shown in the Table below. The settlement
coefficient is obtained from the last graph that is shown below. The diameter of a circle having the same
area as the equivalent raft is 14.2 m. Thus H/B = 16/14.2 = 1.1. Then from the graph with the settlement
coefficients for A = 0.24 and H/B = 1.1, the value of µ is 0.52 and the consolidation settlement is:

∆c = µsoed = 0.52 x 36.9 = 19 mm

Layer Z (m) Area (m2) ∆σ (kN/m2) mv∆σH (mm)


1 2 14. 62 56.3 15.8
2 6 18. 62 34.7 9.7
3 10 22. 62 23.5 6.6
4 14 26. 62 17 4.8
Soed = 36.9

The total settlement is: ∆ = ∆i + ∆c = 6 + 19 = 25 mm

Settlement coefficient. Reproduced from R.F Scott (1963).


434
Chapter 23
Deep Foundations: Lateral Loads
Symbols for Lateral Loads on Deep Foundations

435
**Lateral loads on piles-01: Find the lateral load capacity of a steel pile.
(Revision: Oct-08)
Determine the lateral load capacity Qg of a steel H-pile (HP 250 x 0.834) fully embedded to a depth
of 25 m in very dense submerged sand. The top end of the pile is allowed to deflect laterally 8 mm.
For simplicity assume that there is no moment applied to the top of the pile (that is, Mg = 0).

Solution:

The subgrade modulus ks is a description of the reaction of the soil mass to vertical loads. The modulus
of horizontal subgrade reaction nh is a function of ks at any depth z,

k z = nh z

nh modulus of horizontal subgrade reaction


Type of soil lb/in3 kN/m3
Dry or moist sand
- loose 6.5 to 8.0 1,800 to 2,200
- medium 20 to 25 5,500 to 7,000
- dense 55 to 65 15,000 to 18,000
Submerged sand
- loose 3.5 to 5.0 1,000 to 1,400
- medium 12 to 18 3,500 to 4,500
- dense 32 to 45 9,000 to 12,000

436
From this table and the soil conditions noted above, choose nh = 12 MN/m3 for the modulus. Now
choose the parameters for the steel H-pile,

For future reference, this is the same table in British units,

437
For the steel HP 250 x 0.834 pile, the moment of inertia about the strong axis is Ip = 123 x 10-6 m4, its
modulus of elasticity is Ep = 207 x 106 kN/m2, the steel’s yield strength is Fy = 248 MN/m2 and the pile
depth d1 = 0.254 m.

The characteristic length T of a pile-soil system is given by,

T=
EpI p
= 5
( 207 x10 )(123 x10 ) = 1.16 m
6 −6

(12 , 000 )
5
nh

Therefore, the ratio L / T = 25 m / 1.16 m = 21.6 > 5, so this is a long pile.

The formula for the pile’s top end lateral deflection ∆ at any depth z is given by,

Q gT 3 M gT 2
∆ = Az + Bz but M g = 0
EpIp EpIp

In this problem we are given this value of ∆ = 8 mm, and we want to find the allowable lateral load Qg,
at a depth z = 0, where the coefficient Az is taken from a table of coefficients kz = nh z for long piles.

438
The magnitude of the lateral load Qg limited by the displacement condition only is,

∆(E p I p ) ( 0.008 m ) ( 207 x106 kN / m 2 )(123 x10−6 m 4 )


∴ Qg = = = 54 kN
AzT 3 ( 2.435 )(1.16 )
3

Since the value of the allowable lateral load Qg found above is based on the limiting displacement
conditions only, and ignores that the pile has a moment capacity, that moment capacity at any depth z is
found through,

M z = Am Q gT

The table above shows that the maximum value of Am at any depth is 0.772. The maximum allowable
moment that the pile can carry is,

(123 x10 −6
)
m4
( ) ( 0.254 m / 2 ) = 240 kN − m
Ip
M max = Fy = 248 MN / m 2

d1 / 2

∴ Qg =
M max
=
( 240 kN − m ) = 268 kN
AmT ( 0.772 )(1.16 m )

439
This last value of Qg emanating from the moment capacity is much larger than the value of Qg = 54 kN
found for the deflection criterion.

Therefore use, Qg = 54 kN

440
**Lateral loads on piles-02: Find the lateral load capacity of a steel pile. (US Units)
(Revised Nov-09)
Determine the lateral load capacity Qg of a steel H-pile (HP 10 x 57) fully embedded to a depth of
82 feet in very dense submerged sand. The top end of the pile is allowed to deflect laterally .31
inches. For simplicity assume that there is no moment applied to the top of the pile (that is, Mg =
0).

Figure 1:

Solution:

Step 1: The sub-grade modulus (ks) is a description of the reaction of the soil mass to vertical loads. The
modulus of horizontal sub-grade reaction nh is a function of ks at any depth z,

kz = nh z

From Table 1 for the soil conditions noted, choose nh = 45 lb/in3 for the modulus.

Table 1 - nh = modulus of horizontal sub-grade reaction


441
Now choose the parameters for the steel H-pile,

Table 2: H-pile parameters (British Units)

For the steel HP 10 x 57 pile:

the moment of inertia about the strong axis is Ip = 294 in4, (SI: 123 x 10-6 m4)
its modulus of elasticity is Ep = 3.01 x 107 lbf/in2, (SI: 207 x 106 kN/m2)
the steel’s yield strength is Fy= 3.60 x 104 lbf/in2, (SI: Fy = 248 MN/m)
and the pile depth d1 = 10 in. (SI: 0.254m)
The characteristic length T of a pile-soil system is given by,
Ep I p (3.01e7lbf / in 2 )(294in 4 )
T =5 =5
np (45lb / in3 )

= 5 198680414in5 = 45.67in = 3.81 ft


Therefore, the ratio
L / T = 82 ft / 3.81 ft = 21.5 > 5, so this is a long pile.
The formula for the pile’s top end lateral deflection _ at any
depth z is given by,

442
In this problem we are given this value of ∆ = 0.31 inches, and we want to find the allowable lateral
load Qg, at a depth z = 0, where the coefficient Az is taken from a table of coefficients kz = nh z for long
piles.

The magnitude of the lateral load Qg limited by the displacement condition only is,
∆( E p I p ) (0.31in)(3.01e7lbf / in 2 )(294in 4 )
∴ Qg = = = 12139lbf
AzT 3 (4090)(3.81)3
Since the value of the allowable lateral load Qg found above is based on the limiting displacement
conditions only, and ignores that the pile has a moment capacity, that moment capacity at any depth z is
found through,

The table above shows that the maximum value of Am at any depth is 0.772. The maximum allowable
moment that the pile can carry is,
Working backwards from Qg = 268 kN = 60248 lbf then Am = 0.768

443
SI Units:

British Units:
Ip (294in 4 )
M max = Fy = (3.60e 4lbf / in 2 )
d1 / 2 (10in) / 2
= 2,116,800lbf − in = 176,400lbf − ft
M max (176,400lbf − ft )
∴ Qg = = = 60,000lbf
AmT (.772)(3.81 ft )
This last value of Qg emanating from the moment capacity is much larger than the value of Qg = 12139
lbf found for the deflection criterion.
Therefore use, Qg = 12139 lbsf

444
**Lateral loads on piles-03: The Broms method for pile lateral loads.
(Revised Nov-09)
For the previous problem, use the Broms method to determine the lateral load capacity Qg of a
flexible steel H-pile (HP 250 x 0.834) fully embedded to a depth of 25 m in very dense submerged
sand. The top end of the pile is free and allowed to only deflect 8 mm. The unit weight of the soil is
γ = 18 kN/m3 and the angle of internal friction is φ = 35°.

Solution:

The moment at steel’s yield failure is,

(123 x10
) 248 x10
−6
m4
( F ) ( )
Ip
M y = S( Fy ) = = 3
kN / m 2 = 240 kN − m
d1 /2 ( 0.254 m ) / 2
y

Therefore, the unitless ratio


My
=
( 240 kN − m ) = 926
D γ Kp 35° 
( 
)
4
( 0.250 m )
4
18 kN / m tan  45° −
3 2

 2 

Broms’ graphs are used to determine the ultimate lateral resistance for long piles in sand shown in (a), or
in clays shown in (b). We use (a):

445
Using Broms’ graph (a) for granular soils, the ratio of 926 and a ratio of e / D = 0 yields an ultimate
lateral resistance ratio of approximately 140, therefore,

35° 

Qultimate = 140 K p D 3γ = 140 tan 2  45° −
 2  (
3
)
( 0.250 m ) 18 kN / m 3 = 581 kN
and the deflection η of the head is given by,
nh 12 MN / m 3
η= 5 = = 0.86 m −1
( 207 x10 )(123 x10 )
5
6 −6
EpI p

η L = ( 0.86 m −1 ) ( 25 m ) = 21.5 and using


e
= 0 which is a free-headed pile.
L

Using Broms’ graph for estimating the deflection of the pile head in sands (a) and in clays (b), we find
for sands a value of 0.15,

Broms’ graphical solutions for estimating the deflection of the pile head in granular soils is shown
below, and for cohesive soils below that one,

446
447
Using figure (a) for the granular soils, the ratio yields,

( ) (n )
3/ 5
∆ EpIp
2/5

≈ 0.15 by interpolation
h

Qg L

( ) (n )
3/ 5 2/ 5
∆ EpI p
∴ Qg =
h

0.15 L
( 0.008 m ) ( 207 x106 )(123 x10−6 ) (12 , 000 )
3/ 5 2/ 5

Qg = = 40 kN
0.15 ( 25 m )

This value 40 kN is much smaller than the 581 kN found above, therefore,

The lateral capacity of the pile Qg = 40 k.

448
***Lateral loads on piles-03: Lateral load capacity of a mooring bollard.
(Revised Nov-09)
Design a steel sheet-pile cellular dolphin for a mooring bollard that will be subjected to an
equivalent horizontal load of 60 kips. The live load upon the dolphin will be 250 psf, and assume
that there are negligible wave forces because it is located in a shielded area. The mooring
geometry limits the diameter of the dolphin to 20 feet or less. Choose a FS = 2.5. The site
conditions are shown below.

Solution:

Step 1: Determine the Required Depth of Embedment D of the steel sheet-pile cell.

For φ = 30° Ka = 0.33 and Kp = 3.0 and Kp – Ka = 2.67

449
Apply the factor of safety to the K p − K a factor, ( )
(K − Ka ) ( 3.0 − 0.33) = ( 2.67 ) = 1.07
(K p − Ka ) FS
reduced
=
p

2.5 2.5
actual
=

Using Tschebotarioff's analysis, the forces E1 and E 2 are,


E1 = area ( abc ) 2d
E 2 = area ( cfg ) 2d
where d is the cell diameter = 20 feet
In granualar soils, the maximum pressure pmax is
( )
pmax = K p − K a γ D = (1.07 ) γ D

The embedment depth D is usually estimated


through trial and error. The depth (ac) is estimated
by having the force E1 be a bit larger than the
mooring force P. Now locate the point (f) so that
the force E2 will satisfy equilibrium, or

∑F H =0 P + E2 = E1
Taking moments about the center of the circular base yields,
∑M a =0 P h = ( E1 + E2 )( 0.318) d tan (δ ) + E2e2 − E1e1
and with δ = 20° ,
P h=E2 ( 0.12d + e2 ) − E1 ( e1 − 0.12d )
First try D = 20 feet. This yields ac = 16 feet and cf = 4 feet
Therefore,
1
E1 = (16' )(16' )( 0.065 kcf )( 40' ) = 333 kips
2
1
E2 = ( 4' )( 2.67' )( 0.065 kcf )( 20 )( 40' ) = 278 kips
2
Check equilibrium, ∑ FH = 0 P + E2 = E1 ∴ 60 kips + 278 kips ≈ 333 kips GOOD
∑M a = 0 P h=E2 ( 0.12d + e2 ) − E1 ( e1 − 0.12d )
where e1 = (16' ) = 10.7 feet and e2 = 16' + 23 ( 4' ) = 18.7 feet
2
3

∴ ( 60 kips )( 42 ft ) = ( 278 kips ) ( 0.12 )( 20' ) + 18.7'  − ( 333 kips ) 10.7' − ( 0.12 )( 20' ) 
∴ 2 , 520 kips − ft < ( 5 , 866 − 2 , 764 ) kips − ft = 3,102 kips − ft
∴ the D = 20-foot embedment is conservative (could be reduced).

450
Step 2: Check the cell’s Stability against the Over-turning Moment.

The forces F1 and F2 are ,


F1 = E1 tan δ = E1 tan ( 23 ϕ ) = ( 333 kips )( 0.36 ) = 121 kips
F2 = E2 tan δ = E 2 tan ( 23 ϕ ) = ( 278 kips )( 0.36 ) = 101 kips
and the distance f = ( 0.318 ) d = ( 0.318 )( 20' ) = 6.36 feet

Find the total weight of the cell above a plane throuogh point A above,
π ( 20' )
2
π D2
W = h= ( 2' )( 0.150 kcf ) + ( 4' )( 0.106 kcf ) + ( 55' )( 0.065 kcf )  = 1, 350 kips
4 4
Note that W ≡ R in the figure.

The total moment about point A is,


∑M A = ( 62' )( 60 k ) − ( 9.3' )( 333 k ) − ( 6.36' )(121 k ) − ( 6.36' )(101 k ) + (1.33' )( 278 k )
∑M A = −400 kip − ft
Therefore, the reactive force R has an eccentricity e of,
M −400 kip − ft
M A = Re ∴ e = A = = 0.25 ft
R 1, 350 kip
This eccentricity is negligible, and clearly the resultant falls in the middle third.
The cell is stable with respect to the over-turning moment.

451
Step 3: Check the cell’s Stability against Horizontal Sliding.

It is clear from the figure that the cell will tilt before it will slide. Therefore, this analysis is unnecessary.

Step 4: Check the Bearing Capacity of the soil beneath the cell.

The ultimate bearing capacity of a soil under a cellular dolphin is given by the formula shown below.
The bearing capacity factors are functions of the angle of internal friction φ = 30°, Nq = 22.5 and Nγ =
20.

The ultimate bearing capacity of the soil beneath the cell is,
1 1
qultimate = qN q + ( 0.6 ) γ dN γ = γ D f N q + ( 0.6 ) γ dN γ
2 2
1
qultimate = ( 0.065 )( 20' )( 22.5 ) + ( 0.6 )( 0.065 )( 20' )( 20 ) = 37 ksf
2
The total load pressure ptotal from the cell is,
ptotal = ( 2' )( 0.150 pcf ) + ( 4' )( 0.106 pcf ) + ( 55' )( 0.065 pcf ) + ( 0.250 psf ) = 4.6 ksf

qultimate 37 ksf
The Factor of Safety is FS BearingCapacity = = = 8 GOOD
ptotal 4.6 ksf

Step 5: Check the steel Sheet-piling inter-lock Tension.

The maximum internal pressure at ground level is,

Use an "at-rest" lateral pressure rather than an "active" pressure,


Ko = 1− tanϕ = 1− tan( 30°) = 0.50
Therefore, the pressure pa is given by,
pa = Ko ∑( γ z ) = ( 0.50) ( 2' )(150 pcf ) + ( 4' )( 0.106 pcf ) + ( 35' )( 65 pcf ) + 250 = 1,625 psf
The tension is,
1 1
T = pa D = (1,625 psf )( 20' ) = 16, 250 lb / feet = 1,350 lb / inch
2 2

Use a PSA-23 steel sheet-pile. The wall requires 46 pieces.


The actual diameter is 19 feet 6.25 inches.

452
Chapter 24
Reinforced Concrete Retaining Walls and
Bridge Abutments
Symbols for Reinforced Concrete Retaining Walls

453
**RC Retaining Walls–01: Design a RC wall for a sloped backfill.
(Revision: Oct-08)
Design a reinforced concrete wall with a backfill γ = 125 pcf, an allowable soil bearing capacity of
qall = 3 ksf, and a friction at the base of φ = 30º. Design the wall and check for it’s stability under
working loads. (Note: All loads, shears and moments are per linear ft. of retaining wall).

3
1

1.83'
1'
1

3 2

18.33'
15'
16'-6"

Pv
Ph

4
6.11'
2'

6
1'-6"

5
3' 1'-6" 5'-6"
10'

Solution:
Step 1: Find the lateral pressures and forces.
The active lateral earth pressure K a is,
 φ  30° 
K a = tan 2  45° −  = tan 2  45° −  = tan ( 30° ) = 0.33
2

 2  2 
The presure at the base (invert) of the wall footing is,
pb = γ HK a = ( 0.120kcf )(18.33 ft )( 0.33) = 0.733ksf
The forces on the wall (per unit length) are,
Pv = ½(12)(18.33)² = 2.0 kips
Ph = ½(39)(18.33)² = 6.6 kips

Step 2: Stability analysis for sliding and overturning.

454
Moment About A

Area Area Force Arm Moment


(kip) (ft) (kip-ft)

1 ½ x 5.5x 1.83 = 5.03 x 0.125 = 0.63 1.83 1.2


2 5.5 x 15.0 = 82.5 x 0.125 = 10.31 2.75 28.4
3 1.0 x 15.0 = 15.0 x 0.150 = 2.25 6.00 13.5
4 ½ x 0.5 x 15.0 = 3.75 x 0.150 = 0.56 6.67 3.7
5 10.0 x 1.5 = 15.0 x 0.150 = 2.25 5.00 11.3
6 3.0 x 2.0 = 6.0 x 0.125 = 0.75 8.50 6.4
Pv 2.00
Ph ΣH = 6.60 6.11 40.4

ΣV=18.75 ΣM=104.9

Location of Resultant

From point A, 104.9 = 5.6 ft


18.75
then e = 5.6 – 10 = 0.6 ft o.k. < B
2 6
Soil Pressure at Toe of Base

qmax = 18.75 (1 + 6 x 0.6 ) = 1.875 (1 + 0.36) = 2.55 ksf OK < 3 ksf


10 10

Check F against Sliding

Shear available along base = 18.75 kips x 0.58 = 10.9 kips

Passive force at toe

cos δ
Use S = 2/3 (30º) = 20º , Pp = = 5.8 kips
1/2 γ H 2

Pp =
(5.8 )(0.125 )(3.5 )2 = 4.7 kips
(2 )(0.940 )
10.9 10.9 + 4.7
Min F = = 1.7 kips , Max F = = 2.4 kips OK without Key
66 66

455
Step 2: Design parameters.

Load Factors

Stem – Use 1.7 Ph

Base (toe and heel) – distribute ΣV uniformly over front B/3

Concrete and Steel Data

Capacity reduction factors: 0.90(flexure); 0.85(shear)

F’c = 3,000psi x 0.85 = 2,550 psi (for stress block)

Vc = 2 √3,000 = 110 psi

fy = 40,000 psi ; ρmin = 0.005 ; ρmax = 0.0278 ; ρshrinkage = 0.002

ld = 0.04 Ab (40,000) = 29.2 Ab (bottom bars) x 1.4 = 40.9 (top bars)


√3,000

Step 3: Design the stem of the wall.

Vertical Reinforcement
12"
Ph = 1.7 ½ 39 (15)² = 7.46

M = 7.46 x 5 x 12 = 448 kip-in

Use 6” batter on front, then t = 12 + 6 = 18”

Use d = 18 – 4 = 14”

Assume arm = d – a/2 = 13”

T = 448/13 = 34.5 kip


15'

Pv As = 34.5 / (40 x 0.90) = 0.96 in²/ft

Ph At bottom of wall Use #6 @ 5” ctrs

As = 1.06 in²/ft

ρ=1.06/(14 x 12) = 0.0063 >0.005 and <0.0278 OK


5'

14"

456
Check compressive stress block:

C = T = 34.5 / (2.55 x 12 x 0.90) = αmin = 1,25 in OK < 2.0

Check Shear:

7.46 / (14 x 12 x 0.85) = 52psi OK < 110 psi

Moment computations for steel cutoff:

At bottom : M = 448 kips

At 10’ level : M = 448 (10/15)³ = 133 kip-in

At 5’ level : M = 448 (5/15)³ = 17 kip-in

Resisting moment of steel :

#6 @ 5” ctrs. ; As = 1.06 in/ft

At bottom : Mr = 1.06 (40) (0.90) (13) = 496 kip-in

At top : Mr = 106 (40) (0.90) (7.5) = 286 kip-in


286 kip-in
143 kip-in

2"
0

#6 @ 10"ctrs
17 kip-in

8'-10"

5'
133 kip-in

Ld = 40.9 (0.44) = 1' - 6"

10'
#6 @ 5"ctrs
6'

top of base
15'
0
248 kip-in

448 kip-in
496 kip-in

14" plus std. hook

457
Compute level of cutoff for #6 @ 10” ctrs. :

As = 0.53 in²/ft ; ρ = 0.53 / (±11 x 12) = 0.004 > 0.002 < 0.005

Therefore, cutoff and develop bars above level where:

M = 0.53 (40) (±11) (0.90) = 158 kip-in


1.33

M = 158kip-ft @ ± 10’-6” level

Developmental length = 1’-6”

Level of Cutoff = 9’-0”

Step 4: Design the toe of the base of the wall.

Distribute ΣV over front B/3. Assume t = 18” , d =14”

ΣV = 18.75 kips = qmax = 5.63 ksf


(10 / 3)

wt. of soil over toe = 2.0 (0.125) = 0.25 ksf

wt of concrete base = 1.5 (0.150) = 0.23 ksf

Net toe pressure for design = qmax - wt. of soil over toe - wt of concrete base

= 5.63ksf - 0.25ksf - 0.23 ksf = 5.15 ksf

front face of stem


3'
1'-6"

d =14"

V = 5.15 x (3.0) = 15.45 kips

5.15 kip M = 15.45 x (3/2) x 12 =278 kip-in

Assume arm = 13 in

T = 278/13 = 21.4 kips = 0.59 in²/ft


(40 x 0.9)

Check ρ = 0.59 = 0.0035 > 002, shrinkage OK


458
14 x 12

Ρ< 0.005, so increase As by 1/3 , then As = 0.59 x 1.33 = 0.80 in²/ft

use #6 @ 6” ctrs. As = 0.88 in²/ft

Compressive stress block and shear OK by inspection after stem computations

Development length : Extend full base width, therefore ld OK

459
Step 5: Design the heel of the base of the wall.

At Stem:

Weight of soil above heel at back face of stem = 15.0 (0.125) = 1.88 ksf

Weight of concrete base = 1.5 (0.150) = 0.23 ksf

Net pressure for design = 1.88 + 0.23 = 2.11 ksf

At Back:

Weight of soil above hell at back = 16.83 (0.125) = 2.10 ksf

Weight of concrete base = 0.23 ksf

Net pressure for design = 2.33 ksf

2.33 ksf
2.11 ksf
V1 = ½ (2.11) (5.5) = 5.80 kips

V2 = ½ (2.33) (5.5) = 6.41 kips


1'-6"

d =14"
Total V = 5.80 + 6.41 = 12.21 kips

5'-6" M1= 5.8 (1/3) (5.5) (12) = 128 kip-in

M2 = 6.41 (2/3) (5.5) (12)=282 kip-in

Total M = 128 + 282 = 410 kip-in

Assume arm = 13”

T= 410/13 = 31.5 kip

As = 31.5kip = 0.88 in²/ft


(40 x 0.9)

Use #6 @ 6” ctrs. As = 0.88 in²/ft ρ > 0.002 and >0.005 and < 0.0278 OK

Compressive stress block and shear OK

Development length: Extend full base width, therefore ld OK

Horizontal shrinkage steel in stem:

460
Required: 0.002 (±15) (12) = 0.36 in²/ft of height

Use #4 @ 9” ctrs. front As = 0.27 in²/ft

Use #4 @ 18” ctrs back As = 0.13 in²/ft

Total As = 0.27 + 0.13 = 0.40 in²/ft

Horizontal shrinkage steel in base:

Use #4 @ 12 ctrs. top and bottom As = 0.40 in²/ft

Step 6: Finished sketch of the wall.

3
1

12"

#4@ 9" front face


#4@18" back face
Provide 3" clear to
Vert. bars, back face

all bars except 112 "


clear to bars in
front face of stem
#6 @ 10"

#4 @ 2'-0"
16'-6"

6" drains
@ 10'-0"
Vert. bars,
back face
#6 @ 5"

#6 @ 6"
2'

3' 1'-6" 5'-6"


1'-6"

#6 @ 6" 2"x6" key #4 @ 12"

461
**RC Retaining Walls–02: Design a RC bridge abutment wall.
(Revised Nov-09)
Design a bridge abutment for a backfill of γ = 110 lb/ft3, an equivalent surcharge of 3 feet, an
allowable soil pressure of 3.5 kips/ft2, and an allowable shear between soil and base of 45% of the
vertical shear.

9"

L v =5.26 k
3'-0"

4
L h =0.24 k 3.88'
12" 12"

5 6
3
12'-0"

12" 1
7
5'-6"

6'-0"
2'-6" 12" 3'-6"
4'-0"
1'-6"

8
A
7'-0"
Note:

All loads and moments are per lineal foot of abutment.

Solution:
Phs = 30(3.0)(12) = 1.08 kips

Ph = 1/2(30)(122) = 2.16 kips

462
Stability Computations:

Moments about A

Area Force Arm Moment


1 2.5 x 10.5 x 0.110 = 2.89 1.25 3.61
2 1.0 x 5.5 x 0.110 = 0.61 3.00 1.83
3 1/2 x 1.0 x 1.0 x 0.110 = 0.06 2.83 0.15
4 0.75 x 3.0 x 0.150 = 0.34 2.88 0.98
5 2.0 x 1.0 x 0.150 = 0.30 3.50 1.05
6 1/2 x 1.0 x 1.0 x 0.150 = 0.08 3.17 0.25
7 1.0 x 6.5 x 0.150 = 0.97 4.00 3.88
8 7.0 x 1.5 x 0.150 = 1.58 3.50 5.53

Lv 5.26 3.88 20.40


Lh 0.24 9.00 2.16
Phs 1.08 6.00 6.48
Ph 2.16 4.00 8.64
SH = 3.48 kips SV = 12.09 kips S MA= 54.96 ft-kips

Location of Resultant

54.96 7 .0 B
From point A, = 4.55’ then e = 4.55 - = 1.05’ < ok
12.09 2 6

Soil Pressure at Base:

12.09  6 × 1.05 
At toe qmax = 1 + 
7  7 

qmax = 1.73 (1 + 0.9) = 3.3 k/ft2 < 3.5 ok

At heel qmin = 1.73 (1 - 0.9) = 0.2 k/ft2

Sliding:

Shear available along base = 12.09 k x 0.45 = 5.43 kips

5.43
Factor of safety = = 1.6 > 1.5 ok
3.48

463
**RC Retaining Walls–02: Design a RC bridge abutment wall.
(Revised Nov-09)
Design a bridge abutment for a backfill of _ = 110 lb/ft3, an equivalent surcharge of ___ feet, an
allowable soil pressure of 3.5 kips/ft2, and an allowable shear between soil and base of 45% of the
vertical shear.

All loads and moments are per lineal foot of abutment.

Solution:

Phs = 30(3.0)(12) = 1.08 kips

Ph = 1/2(30)(122) = 2.16 kips

464
Stability Computations:

Moments about A

Area Force Arm Moment


1 2.5x10.5x0.11 = 2.89 1.25 3.61
2 1x5.5x0.11 = 0.61 3 1.83
3 1/2x1x1x0.11 = 0.06 2.83 0.15
4 0.75x3x0.15 = 0.34 2.88 0.98
5 2x1x0.15 = 0.3 3.5 1.05
6 .5x1x1x0.15 = 0.08 3.17 0.25
7 1x6.5x.15 = 0.97 4 3.88
8 7x1.5x0.15 = 1.58 3.5 5.53

Lv 5.26 3.88 20.4


Lh 0.24 9 2.16
Phs 1.08 6 6.48
Ph 2.16 4 8.64
H 3.48kips
V 12.09kips
MA 54.96kips

465
Location of Resultant

54.96 7.0 B
From point A, = 4.55 ft then e = 4.55 - = 1.05 ft < OK
12.09 2 6

Soil Pressure at Base:

Sliding:

Design the stem:

466
Vertical Reinforcement

Ph + Phs = 8.64 +6.48 = 15.12

M = 15.12 x 12/3 x 12 = 726 kip – in

Use 6” batter on front, then t = 9 + 6 = 15 “

Assume arm 15 – 4 = 11”

T = 726/11 = 66 kip

As = 66/ 40 x 0.9 = 1.83 in^2/ft

At bottom of wall Use #5 @ 2in =1.86in^2/ft or #9 at 6” c.c. = 2.00in^2/ft

p = 2.00/ 14 x 9 = 0.015 >.005 < 0.0278 OK

Check compressive stress block

C = T = 66kip / 2.55 x 12 x 0.9 = 2.39 >2 NOT GOOD

Check Shear

15.12 / 14x9 x .75 = 160psi > 110 psi. Need Shear Design for stirrups

Resisting Moment

Top : 2 x 40 x .9 x 9 = 648 kip –in

Bottom : 2 x 40 x.9 x 14 = 1008 kip – in

Horizontal shrinkage in stem

Required = 0.002 x 14 x 9 = 0.336in^2/ft

Use #4@9” c.c front As = 0.27in^2/ft

Use #4@18” c.c back As = 0.13in^2/ft

Total As = 0.27 + 0.13 = 0.40

467
Chapter 25
Steel Sheet Pile Retaining Walls
Symbols for Steel Sheet Pile Retaining Walls

468
**Sheet-pile Wall-01: Free-Earth for cantilevered walls in granular soils.
(Updated: 9 April 2008)
Select an appropriate steel sheet-pile section and its total length L to retain a medium sand backfill
for the conditions shown below without an anchoring system.

γ = 115 pcf
γ ' = 65 pcf
φ = 35° therefore
K a = 0.27 and K p = 3.70

Solution:
Step 1. Determine the pressures on the cantilevered wall.

Step 2. Determine the depth of embedment D.

The procedure will follow the following steps:

a) From statics, find the dimension z shown in the figure of previous page,
b) Assume a value for D,
c) Calculate z,
d) Use z to check if ΣM = 0; adjust D for convergence.

469
(1) ΣFH = 0
Area ( BAA1 ) + Area ( AA1 A2 F ) + Area ( ECJ ) − Area ( EA1 A2 ) = 0
1 D z D
Hp A1 + ( p A1 + p A 2 )   + ( pE + pJ )   − ( pE + p A 2 )   = 0
2 2 2 2
Solving for z ,

z=
( pE − pA1 ) D − Hp1
p E + pJ
( 2) ΣM = 0 about any po int, say F ,
 D2   z2   D2   D2 
( H ) pA1  D +  + ( pA1 )   + ( pE + pJ )   − ( pE + pA2 )   + ( p A2 − pA1 )   = 0
1 H
2  3   2   6  6   6 

p A1 = γ HK a = (115 )(14 )( 0.27 ) = 435 psf


p A 2 = p A1 + γ DK a
= ( 435 ) + ( 65 )( 0.27 ) D = 435 + 17.6 D
( )
pE = γ D K p − K a − p A1 = 65 D ( 6.29 ) − 435
= 409 D − 435
(
pJ = γ D K p − K a + γ HK p )
= 65 D ( 6.29 ) + (115 )(14 )( 6.56 )
= 409 D + 10 , 560

Try D = 10.5 feet ,


p A1 = 435 psf ,
p A 2 = 620 psf ,
pJ = 14 , 860 psf
pE = 3 , 860 psf

therefore, z =
( 3,860 − 435)(10.5) − (14 )( 435) = 1.60 feet
14 , 860 + 3 , 860
(10.5 ) (10.5 )
2 2
1
∑ M F = 2 (14 )( 435)(10.5 + 4.67 ) + ( 435) 2 + ( 620 − 435) 6
(1.60 ) (10.5)
2 2

+ ( 3 , 860 + 14 , 860 )
− ( 3 , 860 + 620 ) = −730 ft − lb
6 6
This value is close to zero, therefore adopt D = 10.5 feet .
Increase D by 30% as a factor of safety ,∴ D = 13.5 feet .

470
Step 3. Determine the maximum moment Mmax (point of zero shear).

p A1 435
y= = = 1.06 feet
γ ( K p − Ka )
'
( 65)( 6.29 )
1 1
P1 = p A1 H = ( 435 )(14 ) = 3 , 040 lb
2 2
1 1
P2 = p A1 y = ( 435 )(1.0 ) = 218 lb
2 2
also,
1 '
2
( )
γ K p − K a x 2 = P1 + P2

2 ( P1 + P2 ) 2 ( 3 , 040 + 218 )
∴ x2 = = = 16 ∴ x = 4.0 feet
γ ( K p − Ka )
'
( 65)( 6.29 )
1
( )
P3 = γ ' K p − K a ( 4 ) = 3 , 280 lb
2
2

M max = P1l1 + P2 l2 − P3 l3
where ,
H
l1 = + y+ x
3
2y
l2 = +x
3
x
l3 =
3
 14   2 (1.0 )   4 .0 
M max = P1l1 + P2 l2 − P3l3 = ( 3 , 040 )  + 1.0 + 4.0  + ( 218 )  + 4.0  − ( 3 , 280 )  
 3   3   3 
M max = 29 , 300 + 1, 030 − 4 , 360 = 26 , 000 ft − lb = 312 in − kips

471
472
If instead , an Ex − Ten − 45 steel sec tion is used , f s = 29 ksi
M max ( 26 , 000 )(12 )
The required sec tion mod ulus S = = = 10.8 in3
fs ( 29 , 000 )
∴ Use a MP − 116 that provides a S = 10.7 in3 at a lower cos t .
This is shown plotted with the red lines.

Step 5. Determine the sheet-pile length L.

The total sheet-pile length L = H + D = 14.0 + 13.5 = 27.5 feet

473
**Sheet-pile Wall-02: Free-Earth for cantilevered walls in cohesive soils.
(Updated: 9 April 2008)
Determine the required length L of steel sheet-piles for the site conditions shown below for an
unanchored wall.

γ = 120 pcf
γ ' = 60 pcf
c = 750 psf
φ = 0°

Solution:
Step 1. Determine all the pressures
on the cantilevered wall.

Step 2. Apply a Factor of Safety =


1.5 to the cohesive strength of the
soil.

750 750
F S = 1 .5 = ∴ c red u ced = = 50 0 p sf
c red u ced 1 .5
q u = 2 c = 2 (5 0 0 ) = 1 , 0 0 0 p s f
T h e fin a l s tr e n g th ( lo n g − te r m s tr e n g th o f th e s o il ) b e c o m e s ,
c = 0
φ = 27°
T h e c r itic a l h e ig h t H c is ,
4c 4 (5 0 0 )
H = = = 1 6 .7 fe e t > 1 4 fe e t
c
γ (1 2 0 )

474
Step 3. Determine the wall pressures.

2c 2 ( 500 )
H o = po int of zero pressure = = = 8.3 feet
γ (120 )
H − H o = 14.0 − 8.3 = 5.7 feet
1 ) γ H − 2c = (120 )(14.0 ) − 2 ( 500 ) = 680 psf
2 ) 4c − γ H = 4 ( 500 ) − (120 )(14.0 ) = 320 psf
3 ) 4c + γ H = 4 ( 500 ) + (120 )(14.0 ) = 3 , 680 psf

Step 4. Determine the depth of embedment D.


The procedure will follow the following steps:
a) From statics, find the dimension z shown in the figure of slide #12,
b) Assume a value for D,
c) Calculate z,
d) Use z to check if ΣM = 0; adjust D for convergence.

(1) ΣFH = 0
Area ( O' A' A ) + Area ( CJE ) − Area ( BAFE ) = 0
1 8c ( z )
( γ H − 2c )( H − H o ) + − ( 4c − γ H ) D = 0
2 2
Solving for z ,
2 D ( 4c − γ H ) − ( γ H − 2c )( H − H o )
z=
8c
( 2) ΣM = 0 about any po int, say F ,
1  H − Ho  8cz 2 D2
∑ MF = ( γ H − 2c )( H − H o )  D +  + − ( 4 c − γ H ) =0
2  3  6 2

The method of solution is as follows,


1) Assume a depth of penetration D,
2) Calculate the value of z (from ΣFH = 0),
3) Substitute z into the ΣMF = 0 equation and check to see if it is zero,
4) Adjust D and reiterate if necessary.

475
2 D ( 4c − γ H ) − ( γ H − 2c )( H − H o ) 2 ( 320 ) D − 5.7 ( 680 )
z= = = 0.16 D − 0.97
8c 8 ( 500 )
and
1  H − H o  8cz 2 D2
∑ M F = 2 (γ H − 2c )( H − H o )  D + 3  + 6 − ( 4c − γ H ) 2 = 0
=
( 680 )( 5.7 )( D + 1.90 ) 8 ( 500 ) z 2 320 D 2
+ = =0
2 6 2
1938 ( D + 1.90 ) + 666.7 z 2 − 160 D 2 = 0
160 D 2 − 1938 ( D + 1.90 )
∴ z2 =
666.7

Try D = 14.0 feet ,


therefore , z = ( 0.16 )(14.0 ) − ( 0.97 ) = 1.27 feet
Check ∑ M F =0

(160 )(14.0 ) − (1,938)(14.0 + 1.90 ) = 0.82


2

z 2
=
666.7
z = 0.9 feet < 1.27 feet ∴ not satisfactory

Try D = 14.5 feet ,


therefore , z = ( 0.16 )(14.5 ) − ( 0.97 ) = 1.35 feet
Check ∑ M F =0

(160 )(14.5) − (1, 938)(14.5 + 1.90 ) = 2.79


2

z 2
=
666.7
z = 1.65 feet > 1.27 feet ∴ acceptable.

Step 5. Determine the required sheet-pile length L.

The total sheet-pile length L is, L = H + D = 14 + 14.5 = 28.5 feet.

476
** Sheet-pile Wall–03: Free-Earth Method f/anchored walls in granular soils.
(Updated: 9 April 2008).
Select an appropriate steel sheet-pile section and its total length L to retain a sand backfill for the
conditions shown below without an anchoring system.

Sand backfill :
γ = 110 pcf
γ ' = 60 pcf
φ = 34° ∴ K a = 0.28

In − situ medium sand :


γ ' = 65 pcf
φ ' = 34.5° ∴ K 'a = 0.26 and K 'p = 3.85

Solution:
Step 1. Determine the pressures on the wall.

pB = γ H1 K a = (110 )(10 )( 0.28 ) = 308 psf


pC 1 = pB + γ H w K a = ( 308 ) + ( 60 )( 26 )( 0.28 ) = 308 + 437 = 745 psf
pC 2 = [γ H1 + γ H w ] K a' = (110 )(10 ) + ( 60 )( 26 )  ( 0.26 ) = 692 psf

( )
pE = γ D1 K p − K a = ( 65 ) D1 ( 6.37 ) = 414 D1
Determine the pressure on the wall from the surcharge,
a ) above point C : psurcharg e = 300 K a = ( 300 )( 0.28 ) = 84 psf
b ) below point C : psurcharg e = 300 K a' = ( 300 )( 0.26 ) = 78 psf
Determine the pressure on the wall due to the line load ,
K 4 .0
using Lecture #18 theory, m = = = 0.111
H 36
pH = ( 0.55 ) Q = ( 0.55 )( 2 , 000 ) = 1,100 lbs
with its resultant at = ( 0.60 ) H = ( 0.60 )( 36 ) = 21.6 feet from the dredge line
Locating the point y,
y=
pC 2
=
( 692 ) = 1.67 feet
γ '(K − K
'
p
'
a ) ( 65)( 6.37 )
477
The resultant forces from the pressures are,

1 1
P1 = H 1 pB = (10 )( 308 ) = 1, 540 plf
2 2
P2 = H w pB = ( 26 )( 308 ) = 8 , 008 plf
1 1
P3 = H w ( pC 1 − pB ) = ( 26 )( 745 − 308 ) = 5 , 681 plf
2 2
1 1
P4 = ypC 2 = (1.67 )( 692 ) = 578 plf
2 2
Psurcharg e ( + C ) = ( H 1 + H w ) psurcharg e = ( 36 )( 84 ) = 3 , 024 plf
Psurcharg e ( − C ) = Dpsurcharg e = ( D1 + 1.67 )( 78 ) = 78 D1 + 130 plf
1 1
P5 = pDE D1 = ( 414 D1 ) D1 = 207 D12
2 2

Step 2. Determine the depth of penetration D.


Solve for D using ∑M ap = 0,
Force (lb) Arm (ft) Moment (ft-lb)
1) 1,540 - 2.33 -3,588
2) 8,008 +14.00 +112,112
3) 5,681 +18.33 +104,133
4) 578 +26.56 +15,930
Sur(+C) 3,024 +9.00 +27,216
 D + 1.67 + 27.0 
Sur(-C) 78D1 +130 +  1  39D12 +2,236D1 +3,619
 2 
Line-load 1,100 +5.40 +5,940
 2D 
Passive 207D12 -  27+1.67+ 1  - 138D13 + 5 , 935 D12
 3 
( )
5) ∑ ( +M ) = 39 D12 + 2 , 236 D1 + 268 , 950
∑ ( -M ) = 138 D + 5 , 935 D
3
1 1
2
+ 3 , 588
but ∑ ( +M ) = ∑ ( − M )
39 D12 + 2 , 236 D1 + 268 , 950 = 138 D13 + 5 , 935 D12 + 3, 588
D13 + 42.72 D12 − 16.20 D1 − 1, 923 = 0 ∴ D1 = 6.50 feet

The total depth of penetration D = D1 + y = 6.50 + 1.67 = 8.2 feet.


Apply a Factor of Safety of about 30% to the calculated depth D = 11.0 feet.

478
Step 3. Determine the tie rod’s tension T.
T = Active forces – Passive forces
= 1,540 + 8,008 + 5,681 + 578 + 3,024 + (78)(6.5) + 1,100 – (207)(6.5)2 = 11,870 lb
Apply a Factor of Safety of about 30% to the tie rod tension, so T = 16,000 lb.

Step 4. Determine the maximum moment Mmax in the sheet-pile.


Let the point of zero shear be at a distance “x” below the low water level.

1
(16.8) x 2
11,870 = 1, 540 + 1,100 + 308 x +
2
∴ x + 37 x − 1,100 = 0 ∴ x = 20 feet
2

The moments are,


(1 ) − (1, 540 )( 20 + 3.33) = − 35 , 900

( 308)( 20 )
2

(2) - = − 61, 600


2
(16.8)( 20 ) ( 20 ) =
2

(3) - − 22 , 400
2
(4) + ( 11,870 )( 20 + 1.0 ) = + 249 , 000
( 5 ) - ( 1,100 )( 20 + 10 − 14.4 ) = − 17 ,150
Maximum moment = +112,000 ft-lb = 112 ft-kip = 1,340 in-kip

Step 5. Select the sheet-pile section.


Selecting the USS Ex-Ten 50 steel provides an allowable stress = 32 ksi. The required section modulus S
is,
M max 1, 340 in − kips
S= = 2
= 42 in3
f all 32 kips / in

479
Initially, assume that there is no moment reduction due to the flexibility of the steel sheet-piling.
Therefore, select a section MZ-38, that has a S = 46.8 in3 / foot of wall.

480
Step 6. Apply Rowe’s Theory of Moment Reduction.
(1) Select the appropriate moment reduction curve (see next slide #12) corresponding to the relative
density of the sand. For this case, the curve for medium compact and coarse grained sand is used.

Calculate the ratio P =


(H + D )4 for the sheet-pile sections being considered.
EI
M design
Obtain the ratio,
M max imum
From the curves (next slide) calculate the design moment and the stress for the sheet pile that was
selected on page 472.

( H + D) ( 36 + 11)(12 ) 
4 4

=
3 , 373
P= =
( 30 )(10 ) I
6
EI I

481
Pile Sections
M Z-38 M Z-32 M Z-27
3 3
Section m odulus S (per foot) 46.8 in 38.3 in 30.2 in 3
M om . of Inertia I (per foot) 280.8 in 4 220.4 in 4 184.2 in 4
3 , 373
P = 12.0 15.3 18.3
I
R atio = M de sign / M m ax . 0.73 0.67 0.62
M design = R atio( M m ax . ) 980 in-kip 900 in-kip 833 in-kip
= Ratio (1,340)
M 980 in-kip 900 in-kip 833 in-kip
Stress = design
S 46 . 8 in 3 38 . 3 in 3 30 . 2 in 3
21.0 ksi 23.5 ksi 27.6 ksi

Based on Rowe’s Theory of Moment Reduction, the following sheet-pile selections may be used,

MZ-38 Regular carbon grade steel, fs = 21.0 ksi < 25 ksi

MZ-32 Regular carbon grade steel, fs = 23.5 ksi < 25 ksi

MZ-27 Ex-Ten 45 grade steel, fs = 27.6 ksi < 29 ksi

Step 7. Establish the length of the steel sheet-pile L.

L = H1 + Hw + D = 10 feet + 26 feet + 11 feet = 47 feet.

482
** Sheet-pile Wall–04: Free-Earth Method anchored walls in cohesive soils.
(Updated: 9 April 2008).
Find the anchor force T and the steel sheet-pile length L for these conditions,

Sand backfill
γ =115 pcf and γ ' =60 pcf
φ =32°∴ K a = 0.33

In-situ soft clay


γ =120 pcf and γ '=65 pcf
c=400 psf
φ =0°∴ K a = 1.0

In − situ medium soft clay


γ '=65 pcf
c=850 psf
φ =0°∴ K a = 1.0

Solution:
Step 1. Introduce a Factor of Safety for the cohesion.
The shear strength of the cohesion is typically reduced through a Factor of Safety of between 1.5 to 2.0;
let us use FS = 1.7,

cmeasured 850 psf


creduced = = = 500 psf
FS 1.7
Step 2. Determine the pressures on the wall.

p1 = γ HK a = (115 )( 6 )( 0.33) = 228 psf


pB −a = γ ( H − H B ) K a = (115 )( 6 )( 0.33) + ( 60 )( 2.5 )( 0.33) = 228 + 50 = 278 psf
pB −b = γ ( H − H B ) − 2c = (115 )( 6 ) + ( 60 )( 2.5 ) − ( 2 )( 400 ) = 40 psf
pC −c = pB −b + γ H B = 40 + ( 65 )(11.5 ) = 788 psf
pC −d = γ H − 2c = (115 )( 6 ) + ( 60 )( 2.5 ) + ( 65 )(11.5 ) − ( 2 )( 500 ) = 588 psf
pC − f = 2c = 2 ( 500 ) = 1, 000 psf
pC − net = pC − f − pC −d = 1, 000 − 588 = 412 psf

483
Step 3. Find the resultant forces on the wall.

(115)( 6 ) ( 0.33) = 684 plf


2
1
P1 = γ H 2 K a =
2 2
1 1
P2 = ( p1 + pB − a )( H a − H B ) = ( 228 + 278 )( 2.5 ) = 633 plf
2 2
1 1
P3 = ( pB − b + pC −c )( H B ) = ( 40 + 788 )(11.5 ) = 4 , 760 plf
2 2
and the c.g. at 8.5 ft + 7.48 ft = 15.98 ft from point A.
P4 = 412 D

Step 4. Find the depth of embedment D.


From statics, the following conditions must be satisfied,
(1) ∑ F = P + P + P − P −T = 0
H 1 2 3 4

( 2 ) ∑ M ( about the anchor tie rod ) = 0


D 
∴ ∑M ap = ( 684 )(1) − ( 633)(1 + 1.29 ) − ( 4 , 760 )(15.98 − 5.0 ) + 412 D  + 15  = 0
2 

Solving the two equations,


D 2 + 30 D − 257.4 = 0
∴ D = 7.0 feet

The required length L of the sheet-piling is,


L= H + D = 20 + 7 = 27 feet

Step 5. Find the tensile force in the anchor tie rod T.

Using the equation for ∑F H = 0,


T = 6 , 077 − ( 412 )( 7 ) ≈ 3 , 200 lb per foot
If the anchors are spaced every 20 feet, each anchor carries,
A = T s = ( 3,200 )( 20 ) = 64 , 000 lbs = 64 kips

484
Chapter 26
MSE (Mechanically Stabilized Earth) Walls
Symbols for Mechanically Stabilized Earth Walls

485
**MSE Walls-01: Design the length L of geotextiles for a 16 ft wall.
(Revision: Oct-08)
Determine the length L of a geotextile-reinforcing for the 16 foot high temporary retaining wall
shown below. Also determine the required vertical spacing of the reinforcing layers Sv and the
required lap length ll. The geotextile chosen has an allowable strength of σG of 80 lb/inch, and the
available granular backfill has a unit weight of γ = 110 lb/ft3 and the angle of internal friction is φ
= 36°.
Comment: These geotextile walls are usually used for temporary civil works, such as detour
roadways, temporary abutments or excavation walls, etc. If the wall must become permanent,
then the face is stabilized and the plastic geotextile is protected from ultra-violet light through a
layer of shotcrete.

16 feet

Solution:
The active pressure on the wall is,
σ a = K aσ v = K a ( γ z ) where K a = tan2 ( 45° − ϕ / 2 ) = tan2 ( 45° − 36° / 2 ) = 0.26
Step 1: Find the vertical spacing SV of each layer of geotextile, where the Factor of Safety (FS) is
generally chosen between 1.3 to 1.5 for temporary walls. Permanent walls should use at least FS ≥ 2. For
this problem choose FS = 1.5.
σG σG
SV = =
σ a FS ( γ zK a ) FS
At a depth of z = 8 feet from the top,

SV =
σG
=
( 80 x12 lb / ft ) = 2.8 feet ≈ 34 inches
(γ zK a ) FS (110 )(8)( 0.26 )(1.5 )
At a depth of z = 12 feet from the top,

SV =
σG
=
( 80 x12 lb / ft ) = 1.9 feet ≈ 22 inches
(γ zK a ) FS (110 )(12 )( 0.26 )(1.5)
At a depth of z = 16 feet from the top,

SV =
σG
=
( 80 x12 lb / ft ) = 1.4 feet ≈ 17 inches
(γ zK a ) FS (110 )(16 )( 0.26 )(1.5)
486
Notice how the spacing becomes denser the deeper we go below the surface; choose SV = 20 inches from
z = 0 to z = 8 feet; below z = 8 feet use SV = 16 inches throughout, as shown in the figure.

Step 2: Find the length of each layer of geotextile L, which is composed of two parts, lR which is the
length of the geotextile within the Rankine failure zone, and le which is the effective length of the
geotextile beyond the failure zone (see the first figure, page 408). Again use FS = 1.5, and the angle φF
is the soil-to-geotextile angle of friction, which is usually assumed to be 2/3 φ of the soil. Other values
can be used, and a few are shown in this table,

Therefore, the length L of the geotextile layer is,

L = l R + le =
( H − z ) + SV K a ( FS ) = (H − z) S ( 0.26 )(1.5 )
+ V
tan ( 45° − φ / 2 ) 2 tan ϕ F tan ( 45° − 36° / 2 ) 2 ( 0.445 )
L = ( 0.51)( H − z ) + 0.438 SV
487
From this equation, we can now prepare a table with the required lengths.

z SV (0.51)(H-z) (0.438SV) L
inches feet feet feet feet feet
16 1.33 1.67 7.48 0.731 8.21
56 4.67 1.67 5.78 0.731 6.51
76 6.34 1.67 4.93 0.731 5.66
96 8.00 1.67 4.08 0.731 4.81
112 9.34 1.33 3.40 0.582 3.99
144 12.00 1.33 2.04 0.582 2.66
176 14.67 1.33 0.68 0.582 1.26

Based on this table, use L = 8.5 feet for z ≤ 8 feet and L = 4.0 feet for z > 8 feet.
Step 3: Find lap length ll for the geotextile, but never smaller than 3 feet,
SV σ a ( FS ) SV ( 0.26 )(1.5 )
ll = = = 0.219 SV
4σ v tan ϕ F 4 tan ( 23 ) ( 36° ) 
For a depth of z=16 inches,
 20 inches 
l l = 0.219 SV = ( 0.219 )   = 0.365 feet < 3 feet
 12 in / ft 
Therefore, use ll = 3 feet

Comment:

These MSE problems commonly


use Rankine’s active pressure
coefficient. However, the actual
value of K must depend on the
degree of restraint of the type of
reinforcement, as shown in this
figure:

488

You might also like